10.04.2020 Views

BỘ CHUYÊN ĐỀ DHBB XÂY DỰNG HỆ THỐNG CÂU HỎI VÀ BÀI TẬP NHÓM VIIB VÀ VIIIB H05, H07, H12, H14

https://app.box.com/s/ctb1vwvz59rnm6dteucgaxlzc55mk4mh

https://app.box.com/s/ctb1vwvz59rnm6dteucgaxlzc55mk4mh

SHOW MORE
SHOW LESS

You also want an ePaper? Increase the reach of your titles

YUMPU automatically turns print PDFs into web optimized ePapers that Google loves.

MỤC LỤC

Trang

MỞ ĐẦU

I. Lí do chọn đề tài 2

II. Mục đích nghiên cứu 2

III. Nhiệm vụ và nội dung của đề tài 3

IV. Phương pháp nghiên cứu 3

V. Điểm mới của đề tài 3

NỘI DUNG

Chương I: Giới thiệu kim loại nhóm VIIB, VIIIB 4

- I.1 Nhóm VIIB 4

- I.2. Nhóm VIIIB 4

Chương II: Hệ thống câu hỏi trắc nghiệm nhóm VIIB, VIIIB 5

Chương III: Hệ thống bài tập nhóm VIIB, VIIIB 26

III.1. Cấu tạo nguyên tử, phân tử, từ tính 26

III.2. Sơ đồ phản ứng, dãy chuyển hóa kim loại nhóm VIIB, VIIIB 28

III.3. Tinh thể của kim loại nhóm VIIB, VIIIB 38

III.4. Bài toán liên quan đến phản ứng hạt nhân 48

III.5. Cân bằng trong dung dịch điện li 52

III.6. Phản ứng oxh-k và bài tập phần điện hóa 55

III.7. Bài tập về phức của các kim loại nhóm VIIB, VIIIB 69

III.8. Các bài tập tính toán khác 81

KẾT LUẬN VÀ KIẾN NGHỊ 90

Tài liệu tham khảo 91

Phát hành PDF bởi Ths Nguyễn Thanh Tú

Đăng ký Word doc qua Zalo 0905779594 Email thanhtuqn88@gmail.com

1


MỞ ĐẦU

I. Lí do chọn đề tài

Hiện nay, các nền giáo dục trên thế giới đang có những bước tiến lớn với nhiều thành tựu về

mọi mặt. Hầu hết các quốc gia đều nhận thức được sự cần thiết và cấp bách phải đầu tư cho giáo

dục.

Hệ thống các trường THPT chuyên đã đóng góp quan trọng trong việc phát hiện, bồi dưỡng

học sinh năng khiếu, tạo nguồn nhân lực chất lượng cao cho đất nước, đào tạo đội ngũ học sinh có

kiến thức, có năng lực tự học, tự nghiên cứu, đạt nhiều thành tích cao góp phần nâng cao chất

lượng giáo dục phổ thông. Tuy nhiên, trong thực tế giảng dạy ở các trường chuyên, việc dạy và học

gặp một số khó khăn:

- Đã có tài liệu giáo khoa dành riêng cho học sinh chuyên hóa, nhưng nội dung kiến thức lí

thuyết chưa đủ để trang bị cho học sinh, chưa đáp ứng được yêu cầu của các kì thi học sinh giỏi các

cấp.

- Tài liệu tham khảo về mặt lí thuyết thường được sử dụng là các tài liệu ở bậc đại học, cao

đẳng đã được biên soạn, xuất bản từ lâu. Khi áp dụng những tài liệu này cho học sinh phổ thông trở

thành rất rộng. Giáo viên và học sinh thường không đủ thời gian nghiên cứu do đó khó xác định

được nội dung chính cần tập trung là vấn đề gì.

- Trong các tài liệu giáo khoa chuyên hóa lượng bài tập rất ít, nếu chỉ làm các bài trong đó

thì HS không đủ sức để thi vì đề thi khu vực, HSGQG, Quốc Tế hằng năm thường rộng và sâu hơn

nhiều. Nhiều đề thi vượt quá chương trình.

- Tài liệu tham khảo, phần bài tập vận dụng các kiến thức lí thuyết còn thiếu, không đồng

bộ, chưa có sách bài tập dành riêng cho học sinh chuyên Hóa. Bộ Giáo Dục và Đào tạo chưa xây

dựng được chương trình chính thức cho học sinh chuyên nên để dạy cho học sinh, mỗi năm tự thân

mỗi giáo viên dạy trường chuyên phải tự lập kế hoạch giảng dạy, mất nhiều thời gian và công sức

cập nhật thông tin từ mạng internet, trao đổi với đồng nghiệp, tự nghiên cứu tài liệu, tự soạn giáo

trình phù hợp.

Mặt khác, Hóa học lại là một bộ môn khoa học lí thuyết và thực nghiệm, rất quan trọng,

nhiều mảng kiến thức rộng lớn như Hóa hữu cơ, hóa vô cơ, nhiệt động học, động hóa học, hóa dung

dịch,... Đặc biệt là những kiến thức giành cho học sinh chuyên hóa, học sinh giỏi cấp khu vực, cấp

Quốc Gia, Quốc tế. Xuất phát từ thực tiễn đó, là giáo viên trường chuyên, chúng tôi rất mong có

được một nguồn tài liệu có giá trị và phù hợp để giáo viên giảng dạy, bồi dưỡng học sinh giỏi các

cấp và cũng để cho học sinh có được tài liệu học tập, tham khảo. Do vậy chúng tôi đã chọn một vấn

đề của Hóa vô cơ, đó là : “Hệ thống hóa câu hỏi và bài tập nhóm VIIB, VIIIB”.

II. Mục đích nghiên cứu

Phát hành PDF bởi Ths Nguyễn Thanh Tú

Đăng ký Word doc qua Zalo 0905779594 Email thanhtuqn88@gmail.com

2


Sơ lược lí thuyết, sưu tầm, lựa chọn, phân loại và xây dựng hệ thống bài tập mở rộng và

nâng cao về kim loại nhóm VIIB, VIIIB để làm tài liệu phục vụ cho giáo viên trường chuyên giảng

dạy, ôn luyện, bồi dưỡng học sinh giỏi các cấp và làm tài liệu học tập cho học sinh đặc biệt cho học

sinh chuyên. Ngoài ra còn là tài liệu tham khảo mở rộng và nâng cao cho giáo viên môn hóa học và

học sinh yêu thích môn hóa học nói chung.

III. Nhiệm vụ và nội dung của đề tài

1- Nghiên cứu lí thuyết về kim loại nhóm VIIB, VIIIB trong chương trình hóa học vô cơ của đại

học và đưa vào có chọn lọc nội dung giảng dạy phần kim loại chuyển tiếp ở trường chuyên.

2- Thống kê, phân loại các bài tập trong tài liệu giáo khoa, sách bài tập cho học sinh, trong các tài

liệu tham khảo có nội dung liên quan đến kim loại nhóm VIIB, VIIIB, từ đó phân tích việc vận

dụng nội dung lí thuyết cấu trúc, liên kết, tính chất của kim loại nhóm VIIB, VIIIB trong giảng dạy

hoá học ở các trường chuyên.

3- Đưa ra các bài kim loại nhóm VIIB, VIIIB trong các đề thi Olympic Quốc gia các nước và

Olympic Quốc tế để thấy được mức độ yêu cầu vận dụng cơ sở lí thuyết ngày càng cao của các đề

thi, từ đó đặt ra nhiệm vụ cho các giáo viên phải có khả năng tự bồi dưỡng nâng cao trình độ để

không những trang bị được kiến thức cơ bản, nâng cao cần thiết cho các em mà còn phải biết dạy

cách học, dạy bản chất vấn đề để giúp học sinh học có hiệu quả nhất.

IV. Phương pháp nghiên cứu

- Nghiên cứu thực tiễn dạy học và bồi dưỡng học sinh giỏi hóa học ở trường THPT chuyên

- Nghiên cứu các tài liệu về phương pháp dạy học hóa học, các tài liệu về bồi dưỡng học sinh giỏi,

các đề thi học sinh giỏi, . . .

- Thu thập tài liệu và truy cập thông tin trên internet có liên quan đến đề tài.

- Đọc, nghiên cứu và xử lý các tài liệu.

V. Điểm mới của đề tài

Đề tài xây dựng hệ thống bài tập mở rộng và nâng cao đầy đủ, có phân loại rõ ràng các câu

hỏi trắc nghiệm khách quan ôn tập kiến thức sau khi học lí thuyết, các dạng bài tập về kim loại

nhóm VIIB, VIIIB để làm tài liệu phục vụ cho giáo viên trường chuyên giảng dạy, ôn luyện, bồi

dưỡng học sinh giỏi các cấp và làm tài liệu học tập cho học sinh đặc biệt cho học sinh chuyên về

Hóa vô cơ. Ngoài ra còn là tài liệu tham khảo mở rộng và nâng cao cho giáo viên môn hóa học và

học sinh yêu thích môn hóa học nói chung.

.

Phát hành PDF bởi Ths Nguyễn Thanh Tú

Đăng ký Word doc qua Zalo 0905779594 Email thanhtuqn88@gmail.com

3


NỘI DUNG

CHƯƠNG I: GIỚI THIỆU KIM LOẠI NHÓM VIIB, VIIIB

I.1. Nhóm VIIB

Nhóm VIIB gồm các nguyên tố: mangan (Mn), tecneti (Tc) và reni (Re).

I.2. Nhóm VIIIB

Nhóm VIIIB gồm 9 nguyên tố họ d ở các chu kỳ 4, 5, 6 thuộc bảng tuần hoàn.

Khi so sánh tính chất lý học và hoá học cơ bản của các nguyên tố nhóm VIIIB, người ta thấy

các nguyên tố sắt, coban, niken có tính chất tương tự nhau, nên xếp chúng thành họ sắt; các

nguyên tố còn lại có tính chất giống nhau theo chiều thẳng đứng, nên được xếp chung thành họ

platin.

Điều này cho thấy tính chất của các nguyên tố nhóm VIIIB không đồng nhất như các nhóm

khác thuộc bảng tuần hoàn. Nó được xem như những bộ ba chuyển tiếp giữa các nguyên tố nhóm

VIIB (Mn, Tc, Re) và nhóm IB (Cu, Ag, Au).

HỌ PLATIN

43 44 45 46 47

Tc Ruteni Rodi Paladi Ag

75 76 77 78 79

Re Osmi Iridi Platin Au

Phần cấu tạo, tính chất vật lí và hóa học của các kim loại nhóm VIIB, VIIIB tác giả xin không đề

cập, chúng ta có thể tham khảo rất đầy đủ ở sách Hóa học vô cơ cơ bản, tập 3 - các nguyên tố

chuyển tiếp và các tài liệu khác.

4

Phát hành PDF bởi Ths Nguyễn Thanh Tú

Đăng ký Word doc qua Zalo 0905779594 Email thanhtuqn88@gmail.com


CHƯƠNG II: HỆ THỐNG CÂU HỎI TRẮC NGHIỆM NHÓM VIIB, VIIIB

Câu 1: Theo mô hình VSERP thì dạng hình học của ion [Fe(CN) 6 ] - là:

A. Cấu trúc bát diện đều. B. Cấu trúc hình chóp vuông.

C. Cấu trúc lưỡng tháp tam giác. C. Cấu trúc tứ diện đều.

Đáp án: A

Hướng dẫn:

Để xác định dạng hình học của một phân tử ta dựa vào công thức VSERP dạng AX m E n

trong đó m là số nguyên tử X liên kết với nguyên tử trung tâm A bằng những liên kết σ tương

ứng và n là số cặp electron không liên kết hay cặp electron tự do E. Khi đó tổng m+ n xác định

dạng hình học của phân tử. Chẳng hạn:

m + n = 2 → phân tử phẳng, tương ứng với trạng thái lai hoá của nguyên tử trung tâm A là sp.

M + n = 3 → phân tử phẳng tam giác, tương ứng với trạng thái lai hoá của nguyên tử trung tâm A

là sp 2 .

m + n =4 → phân tử tứ diện, tương ứng với trạng thái lai hoá của nguyên tử trung tâm A là sp 3 .

m + n = 5 → phân tử tháp đôi đáy tam giác, tương ứng với trạng thái lai hoá của nguyên tử trung

tâm A là sp 3 d.

m + n = 6 → phân tử tháp đôi đáy vuông (bát diện), tương ứng với trạng thái lai hoá của nguyên

tử trung tâm A là sp 3 d 2 .

m + n = 7 → phân tử tháp đôi đáy ngũ giác, tương ứng với trạng thái lai hoá của nguyên tử trung

tâm A là sp 3 d 2 .

Như vậy, ion [Fe(CN) 6 ] - có công thức VSERP là AX 6 tương ứng với m = 6, n= 0 và tổng

m + n = 6 → Dạng hình học tương ứng là bát diện đều.

Câu 2: Theo mô hình VSERP thì trạng thái lai hoá của nguyên tử Ni trong phức Ni(CO) 5 là:

A. sp 3 d 2 B. sp 3 d C. sp 3 D. sp 3 d 3

Đáp án: B

Hướng dẫn:

Ni(CO) 5 có công thức VSERP là AX 5 E 0 → m + n =5 → trạng thái lai hoá của nguyên tử trung

tâm là sp 3 d.

Câu 3: Theo mô hình VSERP thì trạng thái lai hoá của nguyên tử Co trong phân tử phức

[Co(NO 2 ) 6 ] 3- là:

A. sp 3 d 2 B. sp 3 d C. sp 3 D. sp 3 d 3

Phát hành PDF bởi Ths Nguyễn Thanh Tú

Đăng ký Word doc qua Zalo 0905779594 Email thanhtuqn88@gmail.com

5


Đáp án: A

Hướng dẫn:

[Co(NO 2 ) 6 ] 3- có công thức VSERP là AX 6 → m + n = 6 → trạng thái lai hoá của Co là sp 3 d 2 .

Câu 4: Trong ion [PtCl 4 ] 2- thì trạng thái lai hoá của Pt là:

A. sp B. sp 3 d C. sp 3 d 2 D. sp 3 d 3

Đáp án: C

Hướng dẫn

Theo VSERP, ion [PtCl 4 ] 2- có công thức AX 4 E 2 → m + n = 6 → Trạng thái lai hoá của Pt là sp 3 d 2 .

Câu 5: Ion hay phân tử nào sau đây có trạng thái lai hoá d 2 sp 3 :

A. Ni(CO) 5 B. [Co(NO 2 ) 6 ] 3- C PH 3 D CHCl 3

Đáp án: B

Câu 6: Cho ion phức chất sau: [Ni(CN) 4 ] 2- . Nhận định nào sau đây là đúng:

A. [Ni(CN) 4 ] 2- là ion phức có cấu trúc vuông phẳng và nghịch từ.

B. [Ni(CN) 4 ] 2- là ion phức có cấu trúc vuông phẳng và thuận từ.

C. [Ni(CN) 4 ] 2- là ion có cấu trúc bát diện đều và thuận từ.

D. [Ni(CN) 4 ] 2- là ion có cấu trúc bát diện đều và nghịch từ.

Đáp án: A

Hướng dẫn: Ni 2+ : [Ar]3d 8 CN - CN - CN - CN -

Vì sự tương tác giữa ion Ni 2+ và ion CN - là tương tác mạnh nên trong trường hợp này khi tạo

thành phức chất xảy ra sự ghép đôi 2 electron độc thân của Ni 2+ (sự dồn e) và từ đó xuất hiện

AO3d trống có khả năng nhận cặp electron tự do của ion CN - . Vì số phối tử là 4 nên ở đây có sự

lai hoá dsp 2 (lai hoá trong) làm cho phức chất có cấu trúc vuông phẳng và do không còn electron

độc thân nên phức có tính nghịch từ.

Câu 7: Công thức của phèn sắt – amoni là:

A. (NH 4 ) 2 SO 4 .Fe 2 (SO 4 ) 3 .24H 2 O B. (NH 4 ) 2 SO 4 .FeSO 4 .12H 2 O

C. (NH 4 ) 2 SO 4 .Fe 2 (SO 4 ) 3 .12H 2 O D. (NH 4 ) 2 SO 4 .FeSO 4 .24H 2 O

Đáp án: A

Câu 8: Phát biểu nào sau đây đúng?

A. Sắt không tạo ra hợp chất tương ứng với bậc oxi hoá cao nhất là +8.

B. Khi cho muối sắt(III) sunfat tác dụng với kalixianua sản phẩm thu được là kaliferixianua.

C. Khi cho muối sắt(III) tác dụng với xoda thu được kết tủa là sắt(II) cacbonat.

6

Phát hành PDF bởi Ths Nguyễn Thanh Tú

Đăng ký Word doc qua Zalo 0905779594 Email thanhtuqn88@gmail.com


D. Khi axit hoá dung dịch muối sắt(III) thì màu nâu của dung dịch đậm dần.

Đáp án: A

Hướng dẫn

Câu A: Đúng. Cấu hình electron ở trạng thái cơ bản có 4 electron độc thân, 2obitan đã ghép đôi ở

3d và 4s. Muốn tạo ra bậc oxi hoá +8 cần kích thích electron từ obitan 3d sang 4p còn trống (sau

khi electron 4s đã kích thích sang 4p) nhưng năng lượng ở obitan 3d và 4p khác nhau khá lớn,

nên sắt không có khả năng tạo ra hợp chất có bậc oxi hoá +8.

Câu B: Sai. Vì trong dung dịch nước KCN có môi trường kiềm do KCN thuỷ phân tạo ra kết tủa

Fe(OH) 3 (Tt = 3,2.10 -38 ), mặt khác phản ứng tạo phức [Fe(CN) 6 ] 3+ lại diễn ra chậm.

Câu C: Sai. Vì muối cacbonat Fe(III) bị thuỷ phân hoàn toàn:

2Fe 3+ + 3CO 2- 3 + 3H 2 O → 2Fe(OH) 3 + 3CO 2

Câu D: Sai. Màu của dung dịch là màu của phản ứng thuỷ phân:

Fe 3+ + HOH ⇌ [Fe(OH)] 2+ + H + Fe 3+ + 2HOH ⇌ [Fe(OH) 2 ] + + 2H +

Fe 3+ + 3HOH ⇌ [Fe(OH) 3 ] + 3H +

Khi thêm axit, cân bằng thuỷ phân chuyển sang trái nên màu nhạt dần.

Câu 9: Muối nào sau đây không tồn tại trong thực tế:

A. FeCl 3 B. FeI 3 C. Fe 2 (SO 4 ) 3 D. Fe(ClO 4 ) 3

Đáp án: B

Hướng dẫn: Ion I - có tính khử mạnh, nên trong tinh thể ion Fe 3+ đã oxi hoá ion I - thành iot:

2Fe 3+ + 2I - → 2Fe 2+ + I 2 .Vì vậy trong thực tế không tồn tại muối FeI 3 .

Câu 10: Điều kiện nào sau đây để một kim loại A có thể khử được ion sắt(III) trong dung dịch để

tạo thành sắt.

A. Thế điện cực của A ≤ -0, 44V. B. A không phản ứng với nước để tạo ra môi trường kiềm.

C. Lượng chất A phải lấy dư. D. Cả 3 đáp án trên.

Đáp án: D

Câu 11: Dãy kim loại nào sau đây đều khử được ion Fe 3+ thành Fe:

A. Mg, Al, Zn B. Na, Mg, Cr C. Ag, Zn, Sc D. Cu, Mn, Al

Đáp án: A

Hướng dẫn: Theo điều kiện ở câu 10 thì các kim loại: Mg, Al, Zn thoả mãn.

Câu 12: Cặp ion nào sau đây có thể tồn tại đồng thời trong 1 dung dịch:

A. Fe 2+ và Sn 2+ B. Fe 3+ và Sn 2+ C. Fe 2+ -

và MnO 4 D. Fe 2+ 2-

và Cr 2 O 7

Đáp án: A

Hướng dẫn:

Ion Fe 2+ và Sn 2+ đều có tính khử nên cùng tồn tại trong dung dịch.

Ion Fe 3+ có tính oxi hoá bị ion Sn 2+ khử, nên không cùng tồn tại trong dung dịch:

Phát hành PDF bởi Ths Nguyễn Thanh Tú

Đăng ký Word doc qua Zalo 0905779594 Email thanhtuqn88@gmail.com

7


2Fe 3+ + Sn 2+ → 2Fe 2+ + Sn 4+

Ion Fe 2+ có tính khử nên không cùng tồn tại với ion MnO - 4 có tính oxi hoá:

5Fe 2+ + MnO - 4 + 8H + → 5Fe 3+ + Mn 2+ + 4H 2 O

Ion Fe 2+ bị ion Cr 2 O 2- 7 oxi hoá nên không cùng tồn tại.

6Fe 2+ + Cr 2 O 2- 7 + 14H + → 6Fe 3+ + 2Cr 3+ + 7H 2 O

Câu 13: Phát biểu nào sau đây đúng?

A. Fe 3+ và Cr 2 O 2- 7 có thể tồn tại đồng thời trong dung dịch.

B. Trong thực tế có sự tồn tại của muối sắt(III) iotua.

C. Sắt phản ứng mãnh liệt với clo khi đốt nóng tạo thành sắt(II) clorua.

D. Ion Fe 3+ và SO 2- 3 có thể tồn tại đồng thời trong một dung dịch.

Đáp án: A

Hướng dẫn: Ion Fe 3+ và ion Cr 2 O 2- 7 đều có tính oxi hoá nên cùng tồn tại trong dung dịch.

Câu 14: Khi nhiệt phân muối Mohr: (NH 4 ) 2 SO 4 .FeSO 4 .6H 2 O thu được hỗn hợp khí E gồm:

A. N 2 , NH 3 , SO 2 , O 2 B. N 2 , SO 2 , O 2 , N 2 O C. N 2 O, NH 3 , SO 2 , O 2 D. NO 2 , N 2 O, SO 2 , O 2

Đáp án: A

Hướng dẫn: (NH 4 ) 2 SO 4 .FeSO 4 .6H 2 O bị nhiệt phân theo phương trình:

12[(NH 4 ) 2 SO 4 .FeSO 4 .6H 2 O] → 6Fe 2 O 3 + 4N 2 + 16NH 3 + 24SO 2 + 3O 2 + 96H2O

Câu 15: Cho phản ứng: FeO + HNO 3 → Fe(NO 3 ) 3 + NO + H 2 O. Trong phương trình phản ứng

trên, khi hệ số của FeO là 3 thì hệ số của HNO 3 là:

A. 6 B.8 C. 4 D. 10

Đáp án: D

Hướng dẫn: Thay thẳng hệ số 3 vào FeO, rồi cân bằng theo thứ tự Fe, N, H, O.

Ta được: 3FeO + 10HNO 3 → 3Fe(NO 3 ) 3 + NO + 5H 2 O

Câu 16: Khi nhiệt phân muối Mohr có công thức: (NH 4 ) 2 SO 4 .FeSO 4 .6H 2 O được hỗn hợp khí.

Dẫn hỗn hợp này qua dung dịch thuốc thử Nessler thấy xuất hiện kết tủa nâu đỏ. Chứng tỏ trong

hỗn hợp này có:

A. NH 3 B. N 2 O C. NO 2 D.H 2

Đáp án: A

Hướng dẫn: Nessler là thuốc thử đặc trưng để nhận biết sự có mặt của NH 3 hay vết NH + 4 trong

dung dịch. Dung dịch nessler có màu vàng nâu khi có mặt của NH 3 hay vết NH + 4 xảy ra phản

ứng giữa chúng tạo kết tủa nâu đỏ.

2K 2 [HgI 4 ] + 3KOH + NH 3 → [HOHg.NHHgI] + 7KI + 2H 2 O

Câu 17: Sản phẩm của phản ứng: Fe + HF là:

A. FeF 6 + H 2 B. FeF 3 + H 2 C. FeF 2 + H 2 D. Fe 2 F 5 + H 2

Đáp án: A

Phát hành PDF bởi Ths Nguyễn Thanh Tú

Đăng ký Word doc qua Zalo 0905779594 Email thanhtuqn88@gmail.com

8


Câu 18: Sản phẩm của phản ứng: K 4 [Co(CN) 6 ] + O 2 + HCl là:

A. K 3 [Co(CN) 6 ] + KCl + H 2 O B. K 3 [Co(CN) 4 ] + KCl + H 2 O

C. K 2 [Co(CN) 6 ] + KCl + H 2 O D. K 2 [Co(CN) 4 ] + KCl + H 2 O

Đáp án: A

Câu 19: Hợp chất nào sau đây thuộc loại Claste:

A. Co 2 (CO) 8 B. K[Au(CN) 2 ] C. [Ni(NH 3 ) 4 ]Cl 2 D. K 3 [Fe(CN) 6 ]

Đáp án: A

Hướng dẫn: Claste là hợp chất mà trong phân tử có tồn tại liên kết hoá học giữa các nguyên tử

của kim loại chuyển tiếp. Trong phân tử Co 2 (CO) 8 là hợp chất hai nhân, nguyên tử coban tạo ra 6

kiên kết σ, trong đó có 4 liên kết σ tạo ra bởi 4 cặp electron của 4 phân tử CO đặt vào 4 obitan tự

do của coban; liên kết σ thứ 5 tạo ra từ một cặp electron d của coban đặt vào obitan π của phân tử

CO. Liên kết σ tạo ra giữa 2 nguyên tử coban do sự ghép đôi bởi 2 electron độc thân của 2

nguyên tử coban.

Câu 20: Khi cho dung dịch FeSO 4 tiếp xúc với khí NO tạo ra chất lỏng màu nâu đỏ đó là:

A. [Fe(NO)]SO 4 B. [Fe(NO) 4 ]SO 4 C. [Fe(NO) 5 ]SO 4 D. [Fe(NO) 6 ]SO 4

Đáp án: A

Hướng dẫn: FeSO 4 + NO → [Fe(NO)]SO 4 (màu nâu đỏ)

Câu 21: Chất nào sau đây được dùng trong các ẩm kế để xác định độ ẩm của không khí:

A. CoCl 2 .6H 2 O B. K 3 [Co(CN) 6 ] C. Co 2 (CO) 8 D. K 3 [Co(OH) 6 ]

Đáp án: A

Hướng dẫn

Muối CoCl 2 .6H 2 O bị mất một phần nước kết tinh kèm theo sự thay đổi màu sắc rõ rệt:

Khi cho nước tác dụng lên coban clorua khan, quá trình xảy ra ngược lại, vì sự thay đổi màu sắc

đó nên được dùng trong các ẩm kế để xác định độ ẩm của không khí.

Câu 22: Cho các phát biểu sau:

(1) Điều có 1e hoặc 2e ở lớp ngoài cùng, do đó đều có tính chất của kim loại, màu sắc từ xám

đến trắng; nhiệt độ nóng chảy, nhiệt độ sôi cao, thể tích nguyên tử thấp.

(2) Tất cả đều có khả năng hấp phụ H 2 trên bề mặt gây ra hoạt tính cao của hidro (hidro hoạt

động).

(3) Tất cả đều có tác dụng xúc tác cho các phản ứng hữu cơ hoặc vô cơ.

(4) Không có khuynh hướng tạo phức chất với NH 3 , CO.

(5) Có khả năng tạo ra nhiều hợp chất có hoá trị khác nhau và có thể chuyển từ trạng thái hoá trị

này đến trạng thái hoá trị khác.

Phát hành PDF bởi Ths Nguyễn Thanh Tú

Đăng ký Word doc qua Zalo 0905779594 Email thanhtuqn88@gmail.com

9


(6) Đều tạo ra hợp chất có màu.

(7) Hidroxit của chúng đều có tính bazơ yếu hoặc có tính axit, tính lưỡng tính.

(8) Có ái lực electron mạnh với oxi, giảm dần từ trái sang phải.

Số phát biểu đúng về tính chất của các nguyên tố nhóm VIIIB:

A. 5 B. 6 C. 7 D. 8

Đáp án: B

Hướng dẫn:

Phát biểu 4: Sai. Các nguyên tố nhóm VIIIB đều có khuynh hướng tạo phức đặc trưng nhất là

phản ứng tạo phức với NH 3 và CO.

Phát biểu 8: Sai. Các nguyên tố nhóm VIIIB có ái lực yếu với oxi, giảm dần từ trái sang phải,

nhưng lại có ái lực lớn với lưu huỳnh tăng dần từ trái sang phải.

Câu 23: Quặng nào sau đây giàu sắt nhất?

A. Xiderit B. Manhetit C. Hematit đỏ D. Pirit sắt

Đáp án: B

Câu 24: Trong các hợp chất cacbonyl thì Fe, Co, Ni đều mang số oxi hoá bao nhiêu?

A. 0 B. 6 C. 3 D. 2

Đáp án: A

Câu 25: Cho một mẫu quặng sắt (loại bỏ các tạp chất không chứa sắt) vào dung dịch HNO 3 đặc,

nóng thấy thoát ra khí NO 2 (duy nhất). Cho dung dịch BaCl 2 vào dung dịch sau phản ứng không

thấy có kết tủa.Quặng đã đem hoà tan thuộc loại:

A. Pirit. B. Xiderit. C. Hematit. D. Manhetit.

Đáp án: D

Câu 26: Thực hiện các thí nghiệm sau:

1. Đốt dây sắt trong khí clo.

2. Đốt nóng hỗn hợp bột Fe và S (trong điều kiện không có oxi).

3. Cho FeO vào dung dịch HNO 3 (loãng, dư).

4. Cho Fe vào dung dịch Fe 2 (SO 4 ) 3 .

5. Cho Fe vào dung dịch H 2 SO 4 (loãng, dư).

Có bao nhiêu thí nghiệm tạo ra muối sắt(II)?

A. 2 B. 1 C. 4 D. 3

Đáp án: D

Hướng dẫn: Các thí nghiệm 2, 4, 5 tạo ra muối sắt(II).

Câu 27: Niken thường được sử dụng làm chất xúc tác cho quá trình hidro hoá các chất hữu cơ

vì:

A. Niken có khả năng phản ứng mạnh với hidro. B. Niken có khả năng hấp thụ mạnh hidro.

Phát hành PDF bởi Ths Nguyễn Thanh Tú

Đăng ký Word doc qua Zalo 0905779594 Email thanhtuqn88@gmail.com

10


C. Niken bị thụ động trong hidro. D. Niken không phản ứng với hidro.

Đáp án: B

Câu 28: Phát biểu nào sau đây không đúng?

A. Các KL chuyển tiếp họ d có khả năng tạo phức dễ hơn so với các KL chuyển tiếp họ f.

B. Fe(CO) 5 vừa có tính oxi hoá, vừa có tính khử.

C. Có thể dùng dung dịch amoniac để điều chế: Ni(OH) 2 , Fe(OH) 3 và Mn(OH) 2 .

D. Nguyên tắc để điều chế Fe, Co, Ni là dùng dòng điện hoặc dùng chất khử để khử các oxit kim

loại ở nhiệt độ cao.

Đáp án: C

Hướng dẫn:

Phát biểu C: không đúng.

Không dùng dung dịch amoniac để điều chế Ni(OH) 2 , vì kết tủa tủa này tan trong NH 3 dư.

Ni(OH) 2 + 6NH 3 → [Ni(NH 3 ) 6 ](OH) 2

Câu 29: Công thức của kaliferixianua là:

A. K 3 [Fe(CN) 6 ] B. K 3 [Fe(CN) 4 ] C. K 3 [Fe(CN) 2 ] D. K 3 [Fe(CN)]

Đáp án: A

Câu 30: Để bảo quản dung dịch FeSO 4 trong phòng thí nghiệm, người ta ngâm vào dung dịch đó

một đinh sắt đã làm sạch. Chọn cách giải thích đúng cho việc làm trên:

A. Để sắt khử muối sắt(III) thành muối sắt(II): Fe + Fe 2 (SO 4 ) 3 → 3FeSO 4

B. Để sắt tác dụng hết O 2 hòa tan: 2Fe + O 2 → 2FeO

C. Để Fe tác dụng hết với H 2 SO 4 dư khi điều chế FeSO 4 bằng phản ứng:

Fe + H 2 SO 4 loãng → FeSO 4 + H 2

D. Để Fe tác dụng với các tạp chất trong dung dịch, chẳng hạn với tạp chất là CuSO 4 :

Fe + CuSO 4 → FeSO 4 + Cu

Đáp án: A

Câu 31: Trường hợp nào sau đây không xảy ra phản ứng hoá học?

A. Sục khí Cl 2 vào dung dịch FeCl 2 B. Sục khí H 2 S vào dung dịch CuCl 2

C. Sục H 2 S vào dung dịch FeCl 2 D. Cho Fe vào dung dịch H 2 SO 4 đặc, nguội.

Đáp án: D

Câu 32: Cho phản ứng hóa học: Zn + Fe 2 (SO 4 ) 3 → 2FeSO 4 + ZnSO 4 .

Trong phản ứng trên xảy ra

A. Sự khử Zn và sự oxi hóa Fe 3+ B. Sự khử Zn và sự oxi hóa Fe 2+

C. Sự oxi hóa Zn và sự khử Fe 2+ D. Sự oxi hóa Zn và sự khử Fe 3+

Đáp án: D

Câu 33: Trong dung dịch nước ion Fe 2+ tồn tại ở dạng:

Phát hành PDF bởi Ths Nguyễn Thanh Tú

Đăng ký Word doc qua Zalo 0905779594 Email thanhtuqn88@gmail.com

11


A. [Fe(H 2 O) 6 ] 2+ B. [Fe(H 2 O) 4 ] 2+ C. [Fe(H 2 O) 2 ] 2+ D. [Fe(H 2 O)] 2+

Đáp án: A

Câu 34: Số dạng thù hình của Fe là:

A. 1 B. 2 C. 3 D. 4

Đáp án: D

Câu 35: Khi nhiệt phân phèn sắt – amoni có công thức: (NH 4 ) 2 SO 4 .Fe 2 (SO 4 ) 3 .24H 2 O thu được

hỗn hợp khí. Khi dẫn hỗn hợp khí qua dung dịch nước brom thấy màu vàng của dung dịch brom

bị biến mất. Khí làm mất màu dung dịch brom là:

A. NH 3 B. SO 2 C. N 2 D. O 2

Đáp án: B

Hướng dẫn: Phèn sắt – amoni bị nhiệt phân theo phương trình:

6[(NH 4 ) 2 SO 4 .Fe 2 (SO 4 ) 3 .24H 2 O] → 6Fe 2 O 3 + 2N 2 + 8NH 3 + 24SO 2 + 9O 2 + 36H 2 O

Khí làm mất màu dung dịch brom chính là SO 2 : SO 2 + Br 2 + 2H 2 O → 2HBr + H 2 SO 4

Câu 36: Cho cân bằng: FeCl 3 + 3KSCN ⇌ Fe(SCN) 3 + 3KCl

(vàng nhạt)

(đỏ đậm)

Phát biểu nào sau đây đúng?

A. Khi thêm vào hệ một ít tinh thể KCl thấy màu của dung dịch đậm dần.

B. Khi thêm vào hệ một ít tinh thể KCl thấy màu của dung dịch nhạt dần.

C. Khi thêm vào hệ một ít tinh thể FeCl 3 thấy màu của dung dịch nhạt dần.

D. Khi tăng áp suất của hệ thấy màu của dung dịch đậm dần.

Đáp án: B

Hướng dẫn

- Theo nguyên lí chuyển dịch cân bằng Lơ Stơliê, khi cân bằng đã được thiết lập nếu ta thêm vào

hệ một ít tinh thể KCl thì cân bằng sẽ chuyển dịch theo chiều nghịch tức là chiều làm giảm nồng

độ KCl vì vậy mà màu của dung dịch nhạt đi. Ngược lại khi thêm vào hệ FeCl 3 thì cân bằng sẽ

chuyển dịch theo chiều thuận nên màu của dung dịch đậm dần.

- Sự chuyển dịch của cân bằng trên không phụ thuộc vào áp suất.

Câu 37: Trong thiên nhiên có đến hơn 200 loại quặng khác nhau có chứa coban trong đó có

cobantit. Công thức quặng cobantit là:

A. CoAsS B. Co 3 S 4 C. Co 3 O 4 D. CoAs 2

Đáp án: A

Câu 38: Trong nước ngầm thường có chứa Fe(HCO 3 ) 2 và FeSO 4 . Hàm lượng sắt trong nước cao

thường làm cho nước sinh hoạt có mùi tanh, để lâu có màu vàng gây ảnh hưởng đến sức khoẻ và

sinh hoạt của con người. Người ta đề xuất 3 phương pháp sau:

1. Dùng giàn phun mưa hoặc bể tràn cho nước ngầm được tiếp xúc nhiều với không khí rồi lắng

Phát hành PDF bởi Ths Nguyễn Thanh Tú

Đăng ký Word doc qua Zalo 0905779594 Email thanhtuqn88@gmail.com

12


lọc.

2. Sục khí clo vào bể nước ngầm với liều lượng thích hợp.

3. Sục không khí giàu oxi vào bể nước ngầm.

Phương pháp được chọn để loại sắt ra khỏi nước ngầm là:

A. Phương pháp 3 B. Phương pháp 1 C. Phương pháp 2 D. Cả 3 phương pháp

Đáp án: B

Câu 39: Dãy kim loại nào sau đây đều có cấu trúc mạng lập phương tâm diện:

A. Ni, Pd, Pt, Ir B. Ni, Pd, Fe, Ir C. Ni, Pd, Pt, Co D. Ni, Os, Pt, Ir

Đáp án: A

Hướng dẫn

- Fe có cấu trúc lập phương tâm khối.

- Os, Co có cấu trúc lục phương.

Câu 40: Những phương pháp nào sau đây có thể điều chế được Fe?

(I) Dùng CO khử FeO

(II) Dùng H 2 khử Fe x O y

(III)Dùng Zn tác dụng với dung dịch FeCl 2 (IV) Dùng Ca tác dụng với dung dịch FeCl 2

A. I, II, III B. I, II, IV C. I, III, IV D. II, III, IV

Đáp án: A

Hướng dẫn: Nguyên tắc để điều chế Fe, Co, Ni là dùng dòng điện hoặc dùng chất khử để khử

các oxit kim loại ở nhiệt độ cao.

Câu 41: Cho sơ đồ chuyển hoá: Fe 3 O 4 + dung dịch HI (dư) → X + Y + H 2 O

Biết X và Y là sản phẩm cuối cùng của quá trình chuyển hoá. Các chất X và Y là

A. Fe và I 2 B. FeI 3 và FeI 2 C. FeI 2 và I 2 D. FeI 3 và I 2

Đáp án: C

Câu 42: Cho cân bằng: 2FeCl 2 + Cl 2 ⇌ 2FeCl 3 .

Phát biểu nào sau đây đúng:

A. Khi thêm một ít dung dịch KMnO 4 , cân bằng chuyển dịch theo chiều thuận.

B. Sục một ít khí H 2 S vào hệ, cân bằng chuyển dịch theo chiều thuận.

C. Sục khí clo vào hệ, cân bằng chuyển dịch theo chiều nghịch.

D. Khi thêm một ít dung dịch FeCl 3 , cân bằng chuyển dịch theo chiều thuận.

Đáp án: A

Hướng dẫn

- Phát biểu A đúng. Khi thêm MnO - 4 vào thì thế tăng lên, nghĩa là có sự chuyển Cl -

→ Cl 2 theo sơ đồ phản ứng: MnO - 4 + Cl - → Cl 2 + … Phản ứng làm tăng nồng độ Cl 2 nên theo

nguyên lí chuyển dịch cân bằng Lơ Stơliê, cân bằng chuyển dịch theo chiều giảm nồng độ Cl 2 tức

là chuyển dịch theo chiều thuận.

Phát hành PDF bởi Ths Nguyễn Thanh Tú

Đăng ký Word doc qua Zalo 0905779594 Email thanhtuqn88@gmail.com

13


- Phát biểu B sai. Tương tự khi thêm H 2 S là chất khử, nên thế của hệ giảm xuống, có sự chuyển

dịch tăng nồng độ Cl - nên cân bằng chuyển dịch theo chiều nghịch.

- Theo nguyên lí chuyển dịch cân bằng, khi thêm Cl 2 thì cân bằng chuyển dịch theo chiều thuận

và ngược lại khi thêm FeCl 3 , cân bằng chuyển dịch theo chiều nghịch → Phát biểu C, D đều sai

Phát hành PDF bởi Ths Nguyễn Thanh Tú

Đăng ký Word doc qua Zalo 0905779594 Email thanhtuqn88@gmail.com

14


Câu 43: Để bảo quản dung dịch Fe 2 (SO 4 ) 3 tránh hiện tượng thuỷ phân người ta thường nhỏ vào

ít giọt dung dịch:

A. H 2 SO 4 B. NaOH C. NH 3 C. BaCl 2

Đáp án: A

Hướng dẫn: Trong dung dịch Fe 2 (SO 4 ) 3 có cân bằng: Fe 3+ + HOH ⇌ [Fe(OH)] 2+ + H +

Thêm H 2 SO 4 tức là thêm H + , điều này làm tăng nồng độ H + trong dung dịch làm cân bằng chuyển

dịch theo chiều nghịch. Hay nói cách khác, thêm H 2 SO 4 vào dung dịch Fe 2 (SO 4 ) 3 nhằm hạn chế

khả năng thuỷ phân của ion Fe 3+ .

Câu 44: Phương pháp điều chế sắt trong công nghiệp là:

A. Điện phân dung dịch FeCl 2 B. Khử Fe 2 O 3 bằng CO ở nhiệt độ cao

C. Khử Fe 2 O 3 bằng Al D. Khử Fe 2 O 3 bằng H 2 ở nhiệt độ cao

Đáp án: B

Câu 45: Sục H 2 S vào dung dịch FeCl 3 , hiện tượng quan sát được là:

A. Dung dịch trong suốt. B. Kết tủa trắng.

C. Khí màu vàng thoát ra. D. Dung dịch mất màu vàng, có hiện tượng vẫn đục.

Đáp án: D

Hướng dẫn: Khi sục H 2 S vào dung dịch FeCl 3 có phản ứng: H 2 S + Fe 3+ → Fe 2+ + S + H 2 O

Câu 46: Có thể thu được kết tủa FeS bằng cách cho dung dịch FeSO 4 tác dụng với dung dịch

hay chất nào sau đây?

A. H 2 S B. Na 2 S C. CuS D. FeS 2

Đáp án: B

Câu 47: Hiện tượng xảy ra khi nhúng một thanh sắt vào một cốc đựng axit H 2 SO 4 đặc, nguội

một thời gian, sau đó nhúng vào cốc đựng H 2 SO 4 loãng:

A. Thanh sắt bị ăn mòn trong H 2 SO 4 loãng, không tan trong H 2 SO 4 đặc nguội.

B. Thanh sắt bị ăn mòn trong H 2 SO 4 đặc, nguội, không tan trong H 2 SO 4 loãng.

C. Trong cả hai trường hợp thanh sắt đều bị ăn mòn.

D. Trong cả hai trường hợp thanh sắt đều không bị ăn mòn.

Đáp án: D

Câu 48: Quá trình nào sau đây không xảy ra sự ăn mòn điện hoá?

A. Vật bằng Al – Cu để trong không khí ẩm.

B. Cho một vật bằng sắt vào dung dịch H 2 SO 4 loãng cho thêm vài giọt dung dịch CuSO 4 .

C. Phần vỏ tàu bằng Fe nối với tấm Zn để trong nước biển.

D. Nung vật bằng sắt rồi nhúng vào H 2 O.

Đáp án: D

Phát hành PDF bởi Ths Nguyễn Thanh Tú

Đăng ký Word doc qua Zalo 0905779594 Email thanhtuqn88@gmail.com

15


Hướng dẫn: Ăn mòn điện hoá học là quá trình oxi hoá – khử trong đó kim loại bị ăn mòn do tác

dụng của dung dịch chất điện li và tạo nên dòng electron chuyển dời từ cực âm sang cực dương.

Điều kiện xảy ra sự ăn mòn điện hoá:

- Các điện cực phải khác nhau về bản chất, có thể là cặp hai kim loại khác nhau hoặc cặp kim loại

với phi kim…

- Các điện cực phải tiếp xúc trực tiếp hoặc gián tiếp với nhau qua dây dẫn.

- Các điện cực cùng tiếp xúc với một dung dịch chất điện li.

→ Thiếu một trong ba điều kiện trên sẽ không xảy ra sự ăn mòn điện hoá.

Câu 49: Cho các cặp oxi hoá - khử được sắp xếp theo chiều tăng dần tính oxi hoá của dạng oxi

hóa như sau: Fe 2+ /Fe, Cu 2+ /Cu, Fe 3+ /Fe 2+ . Phát biểu nào sau đây là đúng?

A. Fe 2+ oxi hóa được Cu thành Cu 2+ . B. Cu 2+ oxi hoá được Fe 2+ thành Fe 3+ .

C. Fe 3+ oxi hóa được Cu thành Cu 2+ . D. Cu khử được Fe 3+ thành Fe.

Đáp án: C

Câu 50: Sản phẩm tạo thành của phản ứng giữa Fe 3 O 4 với H 2 SO 4 đặc, nóng là:

A. FeSO 4 , Fe 2 (SO 4 ) 3 , H 2 O B. Fe 2 (SO 4 ) 3 , H 2 O

C. FeSO 4 , H 2 O D. Fe 2 (SO 4 ) 3 , SO 2 , H 2 O

Đáp án: D

Câu 51: Phát biểu nào sau đây không đúng?

A. Tính sắt từ là điểm khác biệt của Fe, Co, Ni so với hầu hết các kim loại khác.

B. Fe, Co, Ni có thể tác dụng trực tiếp với CO tạo thành các cacbonyl kim loại.

C. Osmi là kim loại có tỉ khối đứng đầu trong tất cả các kim loại.

D. Ion Fe 2+ và ion Fe(CN) 3- 6 có thể cùng tồn tại trong một dung dịch.

Đáp án: D

Hướng dẫn: Ion Fe 2+ và ion Fe(CN) 3- 6 không cùng tồn tại do phản ứng tạo phức:

Fe 2+ + Fe(CN) 3- 6 → [Fe[Fe(CN) 6 ] 2 ] -

Câu 52: Phát biểu nào sau đây không đúng:

A. Các kim loại Fe, Co, Ni bền với flo và không bị flo ăn mòn.

B. Người ta dùng nước vôi trong để xử lí nước thải nhà máy có chứa các ion kim loại như Fe 3+ ,

Cu 2+ và Pb 2+

C. FeS có khả năng tan được trong HCl trong khi CuS thì không có khả năng đó.

D. Sắt không có khả năng đẩy H 2 ra khỏi nước.

Đáp án: D

Hướng dẫn

- Mặc dù flo có tính oxi hoá mạnh nhưng các kim loại Fe, Co, Ni bền không bị flo ăn mòn. Lí do

Phát hành PDF bởi Ths Nguyễn Thanh Tú

Đăng ký Word doc qua Zalo 0905779594 Email thanhtuqn88@gmail.com

16


là khi phản ứng tạo một lớp muối florua mỏng bền ngăn không cho kim loại phản ứng tiếp với

flo.→ phát biểu A đúng.

- Phát biểu B đúng. Nước vôi trong có khả năng tạo kết tủa với các ion Fe 3+ , Cu 2+ và Pb 2+ tạo

thành các hidroxit tương ứng: Fe(OH) 3 , Cu(OH) 2 , Pb(OH) 3 . Các hidroxit này không tan tách ra

khỏi nước thải.

- CuS là muối suafua không tan trong cả nước và axit → phát biểu C đúng.

- Thế oxh-k của Fe 2+ /Fe = -0,44 < 0, nên có khả năng đẩy H ra khỏi H O → Phát biểu D sai

Câu 53: [Pt(NH 3 ) 2 ]Cl 2 là hợp chất dùng để chống bệnh ung thư. Trong hợp chất này Pt có số oxi

hoá:

A. 0 B. +1 C. +2 D. +3

Đáp án: C

Câu 54: Trong các phức chất sau, phức chất nào thuận từ:

A. [Ni(CN) 4 ] 2- B. [NiCl 4 ] 2- C. [Ni(CO) 4 ] D. [Co(NO 2 ) 6 ] 3-

Đáp án: B

Câu 55: Dãy gồm các chất hoặc dung dịch đều phản ứng được với dung dịch FeCl 2 là:

A. Khí Cl 2 , dung dịch Na 2 S, dung dịch HNO 3 .

B. Bột Mg, dung dịch NaNO 3 , dung dịch HCl.

C. Bột Mg, dung dịch BaCl 2 , dung dịch HNO 3 .

D. Khí Cl 2 , dung dịch Na 2 CO 3 , dung dịch HCl.

Đáp án: A

Câu 56: Gọi X là nhóm kim loại tác dụng được với dung dịch HCl và Y là nhóm kim loại tác

dụng được với dung dịch Fe(NO 3 ) 2 . Hãy cho biết nhóm kim loại X và Y nào dưới đây phù hợp

với quy ước trên?

A. Mg, Zn và Sn, Ni B. Mg, Ag và Zn, Cu

C. Fe, Cu và Mg, Zn D. Sn, Ni và Al, Mg

Đáp án: D

Câu 57: Nhúng một lá Fe nhỏ vào dung dịch dư chứa một trong những chất sau: FeCl 3 , AlCl 3 ,

CuSO 4 , Pb(NO 3 ) 2 , NaCl, HNO 3 , H 2 SO 4 (đặc,nóng), NH 4 NO 3 . Số trường hợp phản ứng chỉ tạo ra

muối Fe(II) là:

A.3 B.4 C.5 D.6

Đáp án: A

Câu 58: Cho các chất sau: Cl 2 (1), I 2 (2), dung dịch HNO 3 loãng (3), dung dịch H 2 SO 4 đặc, nguội

(4), dung dịch AgNO 3 (5), dung dịch NH 4 NO 3 (6). Với hoá chất nào trong các hoá chất trên thì Fe

tác dụng tạo ra sản phẩm là hợp chất Fe(III)?

Phát hành PDF bởi Ths Nguyễn Thanh Tú

Đăng ký Word doc qua Zalo 0905779594 Email thanhtuqn88@gmail.com

17


A. (1), (2), (3), (4), (5), (6) B. (1), (2), (3), (4), (5)

C. (1), (3), (5) D. (1), (2), (4), (6)

Đáp án: C

Câu 59: Dãy nào sau đây bao gồm các chất đều tác dụng được với dung dịch FeCl 3 :

A. AgNO 3 , KI, NH 3 , H 2 S B. AgNO 3 , Br 2 , NH 3 C. NaOH, Mg, KCl D. KI, Br 2 , NH 3

Đáp án: D

Câu 60: Khi cho từ từ dung dịch Fe(NO 3 ) 3 vào dung dịch Na 2 CO 3 thu được kết tủa A. Kết tủa

A là:

A. FeCO 3 B. Fe(OH) 3 C. Fe(OH) 2 D. Fe 2 (CO 3 ) 3

Đáp án: B

Hướng dẫn: Muối cacbonat Fe(III) bị thuỷ phân hoàn toàn:

2Fe 3+ + 3CO 3 2- + 3H 2 O → 2Fe(OH) 3 + 3CO 2

Câu 61: Cho một dung dịch NH 3 vào dung dịch X chứa hai muối AlCl 3 và FeSO 4 được kết tủa

A. Nung A được chất rắn B. Cho khí H 2 dư đi qua B nung nóng được chất rắn C. Thành phần

chất rắn C gồm:

A. Al và Fe 2 O 3 B. Al 2 O 3 và Fe

C. Fe, FeO, Fe 2 O 3 và Fe 3 O 4 D. Al 2 O 3 , FeO

Đáp án: B

Câu 62: Cho các phản ứng sau:

Fe + 2Fe(NO 3 ) 3 → 3Fe(NO 3 ) 2 AgNO 3 + Fe(NO 3 ) 2 → Fe(NO 3 ) 3 + Ag

Dãy sắp xếp theo thứ tự tăng dần tính oxi hoá của các ion kim loại là:

A. Ag + , Fe 3+ , Fe 2+ B. Fe 2+ , Ag + , Fe 3+ C. Fe 2+ , Fe 3+ , Ag + D. Ag + , Fe 2+ , Fe 3+

Đáp án: C

Câu 63: Hỗn hợp X gồm Fe 3 O 4 và Al có tỉ lệ mol tương ứng 1:3. Thực hiện phản ứng nhiệt

nhôm X (không có không khí) đến khi phản ứng xảy ra hoàn toàn thu được hỗn hợp gồm:

A. Al, Fe, Fe 3 O 4 và Al 2 O 3 B Al 2 O 3 , Fe và Fe 3 O 4

C. Al 2 O 3 và Fe D. Al, Fe và Al 2 O 3

Đáp án: D

Câu 64: Cho hỗn hợp gồm Fe 2 O 3 , ZnO và Cu tác dụng với dung dịch HCl (dư) thu được dung

dịch Y và phần không tan Z. Cho Y tác dụng với dung dịch NaOH (loãng, dư) thu được kết tủa

là:

A. Fe(OH) 3 B. Fe(OH) 2 và Cu(OH) 2

C. Fe(OH) 2 , Cu(OH) 2 và Zn(OH) 2 D. Fe(OH) 3 và Zn(OH) 2

Đáp án: A

Phát hành PDF bởi Ths Nguyễn Thanh Tú

Đăng ký Word doc qua Zalo 0905779594 Email thanhtuqn88@gmail.com

18


Câu 65: Cho hỗn hợp Fe và Zn vào dung dịch AgNO 3 đến khi các phản ứng xảy ra hoàn toàn,

thu được dung dịch X gồm hai muối và chất rắn Y gồm hai kim loại. Hai muối trong X là:

A. Fe(NO 3 ) 3 và Zn(NO 3 ) 2 B, Zn(NO 3 ) 2 và Fe(NO 3 ) 2

C AgNO 3 và Zn(NO3) 2 D. Fe(NO 3 ) 2 và AgNO 3

Đáp án: B

Câu 66: Cho Fe tác dụng dung dịch AgNO 3 dư thì sau phản ứng sản phẩm thu được:

A. Fe(NO 3 ) 3 , Ag B. Fe(NO 3 ) 2 , Ag

C. Fe(NO 3 ) 2 , Fe(NO 3 ) 3 , Ag D. Fe(NO 3 ) 2 , Fe(NO 3 ) 3 , Fe

Đáp án: A

Câu 67: Trong các chất sau: Fe, FeSO 4 , Fe 2 (SO 4 ) 3 , chất nào có tính khử, chất nào có cả tính oxi

hoá và tính khử? Cho kết quả theo thứ tự là:

A. Fe, FeSO 4 B. FeSO 4 , Fe 2 (SO 4 ) 3

C. Fe, Fe 2 (SO 4 ) 3 D.FeSO 4 , Fe

Đáp án: A

Câu 68: Xét khả năng khử của Fe 2+ trong môi trường nước và kiềm mạnh thì:

A. Khả năng khử của Fe 2+ trong nước mạnh hơn trong kiềm.

B. Khả năng khử của Fe 2+ trong kiềm mạnh hơn trong nước.

C. Trong cả hai môi trường tính khử của Fe 2+ là tương đương nhau.

D. Không xác định được

0

0

Cho biết: E 2 =-0,44 V; E 3 = -0,04V, Tt

Fe(OH)

=1,65.10 -15 ; Tt

2

Fe(OH)

=10 -38

3

Đáp án: B

Fe

/ Fe

Fe

/ Fe

Hướng dẫn

Fe 3+ → Fe 2+ → Fe

Fe 3+ + 3e → Fe E 1 = - 0,04V

Fe 2+ + 2e → Fe E 2 = - 0,44V

Fe 3+ + 1e → Fe 2+ E 3

3E E 2E 0

E E 3E 2E 0,76V 0

E 0,76V

1 3 1 3 2 3 1 2

3 2

Fe / Fe Fe / Fe

0,76

0,059

Fe H O

12,84

lg K 12,84 K 2

10 K '

( 2 )

Trong OH - : Fe(OH) 2 ⇌ Fe 2+ + 2OH - Tt 1

Fe 2+ ⇌ Fe 3+ + 1e K’

Fe 3+ + 3OH - ⇌ Fe(OH) 3 (Tt 2 ) -1

Tổ hợp 3 cân bằng ta có: Fe(OH) 2 + OH - ⇌ Fe(OH) 3 + 1e K X = Tt 1 .K’.(Tt 2 ) -1

→ K X = 1,65.10 -15 .10 -12,84 .(3,8.10 -38 ) -1 = 6,28.10 9

Phát hành PDF bởi Ths Nguyễn Thanh Tú

Đăng ký Word doc qua Zalo 0905779594 Email thanhtuqn88@gmail.com

19


→ K X ≫ K’ nên Fe 2+ trong OH - khử mạnh hơn trong nước.

Câu 69: Cho hai cốc chứa các dung dịch với nồng độ của các ion như sau:

Cốc 1: [Fe 3+ ] = 0,2M và [Fe 2+ ] = 0,1M; Cốc 2: [Fe 3+ ] = 0,1M và [Fe 2+ ] = 0,2M

Nhúng vào hai dung dịch hai thanh platin và nối hai dung dịch bằng một cầu muối. Sức điện

động của pin có giá trị gần nhất là:

A. 0,05 B. 0,1 C. 0,15 D. 0,2

Đáp án: A

Hướng dẫn

3+

0

[Fe ]

Thế của mỗi điện cực được tính bằng phương trình Nernst: E E 0,059lg Fe

3 2

/ Fe

[ 2

Fe ]

Sức điện động của pin: E = E 1 – E 2 = 0,0355V

Câu 70: Cho phản ứng: 2FeF 3 + 2I - ⇌ 2Fe 2+ + I 2 + 6F -

E 0

3 2

Fe / Fe = 0,77V, 0

E 0,54V

. Quá trình Fe 3+ + 3F - ⇌ FeF

I2

/2I 3 ; β = 1012,0

Bỏ qua quá trình tạo phức hidroxo của Fe 3+ , Fe 2+ . Phản ứng xảy ra theo chiều nào?

A. Chiều thuận B. Chiều nghịch C. Phản ứng đạt cân bằng D. Ý kiến khác

Đáp án: B

Hướng dẫn Ta có các quá trình:

FeF 3 → Fe 3+ + 3F - β -1 = 10 -12,06

E

0,059

1

Fe 3+ + 1e → Fe 2+ K1 10

12,060,77

(1) Fe 3+ + 1e → Fe 2+ 0,059

K 2

10 10

(2) I 2 + 2e → 2I - 0,54

0,059

K3 10 10

Tổ hợp (1) và (2): 2FeF 3 + 2I - → 2Fe 2+ + I 2 + 6F - K = K 2 2 .K -1 3 = 10 -17,25 .

→ K quá bé nên phản ứng không thể xảy ra theo chiều thuận, mà chỉ xảy ra theo chiều nghịch.

Câu 71: Cho dung dịch A gồm Cu(NO 3 ) 2 , Fe(NO 3 ) 2 đều có nồng độ 0,1M. Phát biểu nào sau đây

không đúng?

A. Dung dịch A có môi trường bazơ (pH > 7)

B. Nếu cho từ từ dung dịch NH 3 vào dung dịch A cho đến dư thì có kết tủa đỏ nâu tạo thành và

dung dịch có màu xanh thẫm.

C. Cho từ từ dung dịch HCl vào dung dịch A thì thấy có khí không màu thoát ra sau đó hoá nâu

ngoài không khí.

D. Cho thanh Mg vào dung dịch A thấy thanh Mg tan và màu của dung dịch thay đổi chuyển

9,1525

0,99

Phát hành PDF bởi Ths Nguyễn Thanh Tú

Đăng ký Word doc qua Zalo 0905779594 Email thanhtuqn88@gmail.com

20


sang không màu.

Đáp án: A

Hướng dẫn:

Câu A: Cu 2+ + H 2 O ⇌ Cu(OH) + + H +

Fe 2+ + H 2 O ⇌ Fe(OH) + + H +

→ Dung dịch A có môi trường axit, pH< 7

Câu B: Đúng

Cu 2+ + 2NH 3 + 2H 2 O +

Cu(OH) 2 + 2NH 4

Fe 2+ + 3NH 3 + 3H 2 O +

Fe(OH) 2 + 3NH 4

→ Câu A sai

Cu(OH) 2 + 4NH 3

[Cu(NH 3 ) 4 ](OH) 2

Câu C: Đúng. Fe 2+ + H + + NO - 3 → Fe 3+ + NO + H 2 O

NO+1/2O 2

NO 2

Câu D: Đúng. Mg + Cu 2+ → Mg 2+ + Cu 2+

Mg + Fe 2+ → Mg 2+ + Fe

Câu 72 Cho pin: Fe | FeSO 4 || Hg 2 SO 4 (r), SO 2- 4 | Hg (l)

Biết tại 25 o C sức điện động của pin ở điều kiện chuẩn là E o = 0,98 V .Thế điện cực tiêu chuẩn

của Fe 2+ /Fe bằng – 0,440 V và cặp Hg 2+ 2 /Hg bằng 0,792V. Tích số tan của Hg 2 SO 4 có giá trị gần

nhất là:A. 3.10 -9 B. 4.10 -9 C. 5.10 -9 D. 6.10 -9

Đáp án: A

Hướng dẫn:

E

0 0

E E

0

E E

0

E 0,54V

pin

pin

0,059 0,059 T

E E Hg E

2 2 [SO ]

0 0 2

0

Hg2SO4

2

lg[

2

]= 2

lg

Hg 2

2 / Hg Hg2

/ Hg

4

[SO ]=1M T 2,868.10

2

9

4

Hg2SO4

Câu 73: Cho phản ứng: KMnO 4 + FeCl 2 + HCl → MnCl 2 + KCl + FeCl 3 + H 2 O

Biết thể tích dung dịch thay đổi không đáng kể. Hằng số cân bằng của phản ứng ở 25 o C. Cho

0

0

0

E 3

Fe = -0,0376V, E 2

/ Fe

Fe = - 0,44V, E 2 1,52V

/ Fe

MnO4

, H / Mn

A.10 75 B. 10 52 C. 10 65 D. 10 72

Đáp án: C

Hướng dẫn:

Fe 3+ 0

+ 3e → Fe (1)

G 1

Fe 2+ + 2e → Fe (2)

Fe 3+ + e → Fe 2+ (3)

0

G 2

0

G 3

Phát hành PDF bởi Ths Nguyễn Thanh Tú

Đăng ký Word doc qua Zalo 0905779594 Email thanhtuqn88@gmail.com

21


(3)=(1)-(2)

= -

0

G 3

0

G 1

0

G 2

FE 3FE 2FE E 3E 2E

=

0 0 0 0 0 0

3 1 2 3 1 2

=3. (-0,0376) – 2(-0,44)=0,77V

Khi phản ứng đạt đến trạng thái cân bằng thì E = 0, do đó:

0,059 [Fe ] .[Mn ] 0,059

E E lg E lg K 0

5 [Fe ] .[H ] .[MnO ] 5

3+ 5 2+

0 0

2+ 5 + 8

4

0

5E

65

lg K 65 K 10

0,059

Câu 74. Cho các thế khử chuẩn: E o Ag+/Ag= 0,8 V; E o Fe3+/Fe2+= 0,77 V. Ở 25 o C trộn các

dung dịch Fe(NO 3 ) 3 0,3 M, AgNO 3 0,03 M, Fe(NO 3 ) 2 0,03 M theo tỉ lệ thể tích bằng nhau rồi

thêm một ít Ag kim loại vào. Phản ứng xảy ra trong dung dịch là:

A. Fe 2+ + Ag + → Fe 3+ + Ag B. Fe 3+ + Ag → Fe 2+ + Ag +

C. Fe 3+ + Ag ⇌ Fe 2+ + Ag + D. Không xảy ra phản ứng.

Đáp án: B

Câu 75: Hoà tan 200 ml FeCl 3 2M vào 300ml dung dịch KI 2M. Nếu xem quá trình hoà

0

3 2

tan của các halogenua là phản ứng 1 chiều. Biết E Fe / Fe = 0,77V, 0

E 0,54V

. Nồng độ các ion

I2

/2I

sau khi trộn

A. [Fe 2+ ] = 0,8M; [K + ] = 1,2M; [Cl - ] = 2,4M; [I - 3 ] = 0,4M

B. [Fe 2+ ] = 0,8M; [K + ] = 0,8 M; [Cl - ] = 2,4M

C. [Fe 2+ ] = 0,8M; [K + ] = 0,8M; [Cl - ] = 1,6M; [I - 3 ] = 0,4M

D. [Fe 2+ ] = 0,8M; [K + ] = 0,8M; [Cl - ] = 2,4M; [I - 3 ] = 0,4M

Đáp án: A

Hướng dẫn

Vì E 0

> 0

3 2 E

nên FeCl 3 tác dụng được với KI

Fe

/ Fe

I 2 /2I

2FeCl 3 + 2KI → 2FeCl 2 + I 2 + 2KCl

0,4 0,4 0,4 0,2 0,4

KI + I 2 → KI 3

0,2 0,2 0,2

Theo phương trình phản ứng ta có: [FeCl 2 ] = [KCl] = 0,8M; [KI 3 ] = 0,4M

Khi đó sau khi trộn: [Fe 2+ ] = 0,8M; [K + ] = 1,2M; [Cl - ] = 2,4M; [I - 3 ] = 0,4M

Câu 76: Cho một hợp chất của sắt vào dung dịch H 2 SO 4 loãng dư, thu được dung dịch. Dung

dịch này vừa tác dụng với KMnO 4 , vừa tác dụng

với Cu. Vậy hợp chất đó là:

A. FeO B. Fe 2 O 3 C. Fe(OH) 2 D. Fe 3 O 4

Phát hành PDF bởi Ths Nguyễn Thanh Tú

Đăng ký Word doc qua Zalo 0905779594 Email thanhtuqn88@gmail.com

22


Đáp án: D

Câu 77: Người ta chuẩn độ 40 ml dung dịch Fe 2+ bằng dung dịch chuẩn Cr 2 O 7 2- 0,02 M. Điểm

tương đương đạt được khi cho vào 40ml dung dịch chuẩn Cr 2 O 2- . Biết rằng trong dung dịch, pH

luôn bằng không trong suốt quá trình chuẩn độ. Nồng độ mol Fe 2+ trong dung dịch đệm chuẩn độ

là:

A. 0,01M B. 0,06M C. 0,12M D. 0,08M

Đáp án: B

Hướng dẫn:

Phương trình phản ứng: Fe 2+ +

2-

Cr 2 O 7 + 14H + ⇌ Fe 3+ + 2Cr 3+ + 7H 2

Nồng độ

2-

Cr 2 O 7 ban đầu = (40.0,2)/80=0,01

Từ phương trình phản ứng → nồng độ ban đầu của Fe 2+ = 6.0,01 = 0,06M

Dùng đề bài sau để trả lời cho câu hỏi 78 và 79:

Có 1 pin sau ở 25 o C: Pt| Fe 3+ 0, 1M, Fe 2+ 0,2M || Fe 3+ 0,2M, Fe 2+ 0,1M| Pt.

Câu 78: ∆G của phản ứng xảy ra trong pin:

A. 3473 J B. -3473 J C. 1737 J D. – 1737 J

Đáp án: A

Hướng dẫn

3+

0

[Fe ]

E

0,059lg 3

2

E

Fe / Fe

3

/

2

[ 2

Fe

Fe

Fe ]

Khi đó: E 1 = 0,77 + 0,059.lg(0,2/0,1)= 0,788 (V)

E 2 = 0,77 + 0,059.lg(0,1/0,2) = 0,752 (V)

7

Phát hành PDF bởi Ths Nguyễn Thanh Tú

Đăng ký Word doc qua Zalo 0905779594 Email thanhtuqn88@gmail.com

23


Khi đó: ∆G = -nFE = -96500.(0,788 – 0,752) = - 3473 (J)

Câu 79: Nồng độ các ion Fe 2+ và Fe 3+ ở các điện cực khi cân bằng là:

A. [Fe 3+ ] = 0, 15 và [Fe 2+ ] = 0, 05 B. [Fe 3+ ] = 0, 05 và [Fe 2+ ] = 0, 15

C. [Fe 3+ ] = 0, 15 và [Fe 2+ ] = 0, 10 D. [Fe 3+ ] = 0, 15 và [Fe 2+ ] = 0, 15

Đáp án: D

Hướng dẫn

Khi cân bằng ta có:

[Fe 3+ ] ở cực dương = [Fe 3+ ] ở cực âm 0,2 – x = 0,1 + x

[Fe 2+ ] ở cực dương = [Fe 2+ ] ở cực âm 0,1 + x = 0,2 – x

3+

[Fe ] 0, 2 x

Ở cực dương:

2

[ Fe ] 0,1 x

3+

[Fe ] 0,1

x

2

[ Fe ] 0,2 x

Suy ra 0, 2 x 0,1 x

0,1

x 0, 2 x

x 0,05. Vậy [Fe 3+ ]=0,15; [Fe 2+ ]=0,15

Câu 80: Biết rằng nếu ion có nồng độ bằng 10 -6 M thì coi như đã tách hết. Khoảng pH

thích hợp để tách một trong hai ion Mg 2+ hoặc Fe 3+ ra khỏi dung dịch là:

A. 6 < pH< 39 B. 0 < pH< 11 C. 3< pH< 10 D. 0 < pH< 6

Đáp án: C

Hướng dẫn:

Để tạo kết tủa Mg(OH) 2 : [OH - ] = 10 -4 → [H + ] = 10 -10 → pH = 10

Để tạo kết tủa Fe(OH) 3 : [Fe 3+ ] > 10 -6 → [OH] 3 < 10 -33 → [H + ] > 10 -3 → pH > 3

Vậy để tách Fe 3+ ra khỏi dung dịch: 3< pH< 10

Câu 81: Trộn hai thể tích bằng nhau của hai dung dịch SnCl 2 0,1M và FeCl 3

0,1M. Nồng độ ion Fe 3+ khi cân bằng ở 25 o C.

A. 1,58.10 -12 B. 1,58.10 -8 C. 1,32.10 -8 D. 1,32.10 -12

Đáp án: A

Hướng dẫn

Sn 2+ + 2Fe 3+ → Sn 4+ + 2Fe 2+

[] 0,05 - x 0,05 – 2x x 2x

2(0,77 0,15)

lg K 21 K 10

0,059

21

K rất lớn và nồng độ Fe 3+ cho phản ứng nhỏ hơn nhiều so với Sn 2+ → phản ứng gần như

hoàn toàn: 2x≈0,05

[Fe 2+ ] = 0,05; [Sn 4+ ] = 0,025M; [Sn 2+ ] = 0,025M; [Fe 3+ ] = εM

Phát hành PDF bởi Ths Nguyễn Thanh Tú

Đăng ký Word doc qua Zalo 0905779594 Email thanhtuqn88@gmail.com

24


2

0,025.(0,05) 21 0,025

3+ -12

2 2

K 10 [Fe ]=1,58.10 M

0,025.

Câu 82: Trộn hai thể tích bằng nhau của hai dung dịch SnCl 2 0,1M và FeCl 3

0,1M. Nồng độ ion Fe 3+ khi cân bằng ở 25 o C.

A. 1,58.10 -12 B. 1,58.10 -8 C. 1,32.10 -8 D. 1,32.10 -12

Đáp án: A

Câu 83: Nhúng một sợi Ag vào dung dịch Fe 2 (SO 4 ) 3 2,5.10 -2 M. Thế oxi hoá khử

của cặp Fe 3+ /Fe 2+ khi cân bằng ở 25 o C là:

A. 0,36 B. 0,44 C. 0,72 D. 0,54

Đáp án: C

Câu 84: Dung dịch A gồm FeSO 4 0,020M; Fe 2 (SO 4 ) 3 . Lấy chính xác 25,00 ml dung

dịch A, khử Fe 3+ thành Fe 2+ ; chuẩn độ Fe 2+ trong hỗn hợp (ở điều kiện thích hợp) hết

11,78 ml K 2 Cr 2 O 7 0,0180M. Nồng độ mol/lít của Fe 2 (SO 4 ) 3 trong dung dịch A có giá

trị gần nhất là:

A. 0,015 B. 0,02 C. 0,01 D. 0,025

Đáp án: A

Câu 85: Khi bắt đầu có kết tủa Fe(OH) 3 thì nồng độ của FeCl 3 đạt giá trị gần nhất

là:

A. 0,05 B. 0,1 C. 0,15 D. 0,2

Đáp án: A

Dùng đề bài sau để trả lời cho câu hỏi từ 86 đến 88:

Sắt dạng (Fe ) kết tinh trong mạng lập phương tâm khối, nguyên tử có bán kính r =

1,24 Å. Cho Fe = 56

Câu 86: Cạnh a của tế bào sơ đẳng:

A. 2,6 Å B. 2,85 Å C. 2,9 Å D. 3,05 Å

Đáp án: B

Câu 87: Tỉ khối của Fe theo g/cm 3 có giá trị gần nhất là:

A. 8 g/cm 3 B. 8,5 g/cm 3 C. 9 g/cm 3 D. 9,5 g/cm 3

Đáp án: A

Câu 88: Khoảng cách ngắn nhất giữa hai nguyên tử Fe có giá trị gần nhất

là:

A. 3,5 Å B. 3,1 Å C. 2,9 Å D. 2,7 Å

Đáp án: C

Phát hành PDF bởi Ths Nguyễn Thanh Tú

Đăng ký Word doc qua Zalo 0905779594 Email thanhtuqn88@gmail.com

25


Dùng đề bài sau để trả lời câu hỏi 89 và 90:

Cho một pin thiết lập ở 25 o C: (-) Ni| NiSO 4 0,2M || CuSO 4 0,4M| Cu (+)

Câu 89: Phản ứng hoá học xảy ra trong pin khi pin hoạt động:

A. Ni + Cu 2+ → Cu + Ni 2+ B. Cu + Ni 2+ → Ni + Cu 2+

C. Ni + Cu 2+ Cu + Ni 2+ D. Không có phản ứng xảy ra.

Đáp án: A

Hướng dẫn:

Ở cực âm: Ni → Ni 2+ + 2e

Ở cực dương: Cu 2+ + 2e → Cu

→ phản ứng xảy ra trong pin: Ni + Cu 2+ → Ni 2+ + Cu

Câu 90: Phát biểu nào sau đây đúng?

A. Dòng điện trong dây dẫn chạy từ cực dương (cực Cu) sang cực âm (cực Ni)

B. Dòng điện trong dây dẫn chạy từ cực âm (cực Ni) sang cực dương (cực Cu)

C. Dòng electron dịch chuyển từ cực dương (cực Cu) sang cực âm (cực Ni)

D. Trong pin điện ở cực Cu xảy ra quá trình oxi hoá và ở cực Ni xảy ra quá trình

khử.

Đáp án: A

Hướng dẫn: Trong một pin điện, khi pin hoạt động thì dòng điện trong dây dẫn chạy từ

cực dương (cực Cu) sang cực âm (cực Ni), ngược lại dòng electron chuyển từ cực Ni

sang cực Cu.

Trong một pin điện, ở cực dương (catot) xảy ra quá trình khử, ở cực âm (anot) xảy ra

quá trình oxi hoá.

CHƯƠNG III: HỆ THỐNG BÀI TẬP NHÓM VIIB, VIIIB

III.1 Cấu tạo nguyên tử, phân tử, từ tính

Bài 1: Số oxi hóa cao nhất của nguyên tố Mn, Tc là bao nhiêu?

Phát hành PDF bởi Ths Nguyễn Thanh Tú

Đăng ký Word doc qua Zalo 0905779594 Email thanhtuqn88@gmail.com

26


Ở trạng thái oxi hóa cao nhất, những chất này thuận từ hay nghịch từ? Nhận xét chung về

từ tính của những chất đó. Giải thích nguyên nhân?

Hướng dẫn:

Mn thuộc nhóm VIIB, số oxi hóa max là +7

Mn : [Ar]3d 5 4s 2

7

Mn

: [Ar] : nghịch từ

Tc thuộc nhóm VIIB, số oxi hóa max là +7

Tc : [Kr]4d 5 5s 2

7

Tc

: [Kr] : nghịch từ

Số oxi hóa cao nhất của các nguyên tố chuyển tiếp bằng số thứ tự của nhóm. Tất cả các

chất trên khi đạt tới trạng thái oxi hóa cao nhất đều có tính nghịch từ. Sở dĩ như vậy vì ở

trạng thái oxi hóa lớn nhất, cấu hình electron của các ion đó đều là những khí hiếm, tất cả

các lớp electron (hoặc chí ít tất cả các phân lớp electron) đều đầy đủ, không có electron

độc thân.

Bài 2: a) Vì sao ion của các nguyên tố chuyển tiếp thường thuận từ, còn ion của các

nguyên tố thuộc nhóm A thường nghịch từ.

b) Vì sao dung dịch ion của các nguyên tố chuyển tiếp thường có màu, còn đối với các

nhóm A thường không màu?

Hướng dẫn

a) Ion của các nguyên tố chuyển tiếp (trừ trạng thái oxi hóa cao nhất) thường có một hay

nhiều electron độc thân nên thường có tính thuận từ. Ion của các nguyên tố thuộc nhóm A

thường nghịch từ vì cấu hình e của chúng là những khí hiếm, không có electron độc thân.

b) Thường những chất thuận từ là những chất có màu

Bài 3

1. Có thể viết cấu hình electron của cation Fe 2+ ở trạng thái cơ bản theo các cách sau:

Cách 1: Fe 2+ : 1s 2 2s 2 2p 6 3s 2 3p 6 3d 6

Cách 2: Fe 2+ : 1s 2 2s 2 2p 6 3s 2 3p 6 3d 4 4s 2

Cách 3: Fe 2+ : 1s 2 2s 2 2p 6 3s 2 3p 6 3d 5 4s 1

Áp dụng phương pháp gần đúng Slater tính tổng năng lượng các electron của Fe 2+ ứng

với mỗi cách viết (theo đơn vị eV). Cách viết nào phù hợp với thực tế ? Tại sao?

Hướng dẫn

Phát hành PDF bởi Ths Nguyễn Thanh Tú

Đăng ký Word doc qua Zalo 0905779594 Email thanhtuqn88@gmail.com

27


Theo phương pháp gần đúng Slater, năng lượng của electron được xác định bằng công

thức:

2

(Z b)

n 13,6 (n

* )

2

Trong đó: b là hằng số chắn; n * là số lượng tử chính hiệu dụng.

- Với cách viết 1: Fe 2+ [Ar]3d 6

2

(26 0,3)

1s 13,6 8982,664eV

2

1

2

(26 7.0,35 2.0,85)

2s, 2p

13,6 1623, 2365eV

2

2

2

(26 7.0,35 8.0,85 2.1)

3s,3p 13,6 328,7611eV

2

3

2

(265.0,35 18.1)

3d 13,6 59,0277 eV

2

3

E1 21s 82s,2p 83s,3p 6 3d

= - 33935,475 eV

- Với cách viết 2: Fe 2+ [Ar]3d 4 4s 2

1s, 2s,2p,

3s,3p

có kết quả tương tự như trên. Ngoài ra:

2

(263.0,35 18.1)

3d 13,6 72,9904 eV

2

3

2

(26 1.0,35 12.0,85 10.1)

4s 13,6 29,507eV

2

3,7

E2 21s 82s,2p 83s,3p 43d 2 4s

= - 33932,2844 eV

- Với cách viết 3: Fe 2+ [Ar]3d 5 4s 1

1s, 2s,2p,

3s,3p

có kết quả tương tự như trên. Ngoài ra:

2

(26

4.0,35 18.1)

3d 13,6 65,824eV

2

3

2

(26 13.0,85 10.1)

4s 13,6 24,3414eV

2

3,7

E3 21s 82s, 2p

83s,3p 53d

4s

= - 33934,7702 eV

E 1 thấp (âm) hơn so với E 2 và E 3 , do đó cách viết 1 ứng với trạng thái bền hơn. Kết

quả thu được phù hợp với thực tế là ở trạng thái cơ bản cation Fe 2+ có cấu hình

electron [Ar]3d 6 .

Phát hành PDF bởi Ths Nguyễn Thanh Tú

Đăng ký Word doc qua Zalo 0905779594 Email thanhtuqn88@gmail.com

28


Bài 4 Có thể viết cấu hình electron của Ni 2+ là:

Cách 1: Ni 2+ [1s 2 2s 2 2p 6 3s 2 3p 6 3d 8 ].

Cách 2: Ni 2+ [1s 2 2s 2 2p 6 3s 2 3p 6 3d 6 4s 2 ].

Áp dụng phương pháp gần đúng Slater, tính năng lượng electron của Ni 2+ với mỗi cách

viết trên (theo đơn vị eV). Cách viết nào phù hợp với thực tế? Tại sao?

Hướng dẫn

Cách 1: Ni 2+ [ Ar] 3d 8

Hằng số chắn b 1s = 0,3.1 = 0,3

nên E 1s = -13,6. Z

= -13,6. (280,3)

2

= -10435,144 (eV)

n

1

b 2s= b 2p = 7.0,35 + 2.0,85 = 4,15

= (284,15) E 2s E 2p = -13,6.

2

2 2 = - 1933,9965 (eV)

b 3s = b 3p = 7.0,35 + 8.0,85 + 2.1 = 11,25

(2811,25) E 3s = E 3p = -13,6.

2

3 2 = - 423,961 (eV)

b 3d = 7.0,35 + 18.1 = 20,45

(2820,45) E 3d = -13,6.

2

3 2 = - 86,1371( eV)

Vậy E 2+ Ni = 2E 1s + 8 E 2s + 8E 3s + 8E 3d = - 40423,045 (eV)

Cách 2: Ni 2+ [ Ar] 3d 6 4s 2

Tính hằng số chắn và năng lượng của 1s, 2s 2p, 3s 3p cho kết quả giống cách 1

=

b 3d 5.0,35 + 18.1 = 19,75

(2819,75) E 3d = -13,6.

2

3 2 = -102,85 (eV)

b 4s = 1.0,35 + 14.0,85 + 10.1 = 22,25

(2822,25) 2

E 4s = -13,6.

3,7 2 = -32,845 (eV)

2+

Vậy E Ni = 2E 1s + 8 E 2s + 8E 3s + 6E 3d + 2E 4s = - 40416,738 (eV)

So sánh 2 giá trị E 2+ Ni trên thấy giá trị E ở cách 1 nhỏ hơn, do vậy ứng với trạng thái bền

hơn và cấu hình e ở cách 1 phù hợp với thực tế hơn.

III.2 Sơ đồ phản ứng, dãy chuyển hóa kim loại nhóm VIIB, VIIIB

Phát hành PDF bởi Ths Nguyễn Thanh Tú

Đăng ký Word doc qua Zalo 0905779594 Email thanhtuqn88@gmail.com

29


Bài 1: Viết phương trình hóa học của Fe 2+ (aq) và Fe 3+ (aq) với

a) OH - (aq) b) NCS - (aq) c) [Fe(CN) 6 ] 4- (aq) d) [Fe(CN) 6 ] 3- (aq)

Dùng ion nào ở trên để nhận biết Fe 2+ (aq), nhận biết Fe 3+ (aq), để phân biệt Fe 2+ (aq) và

Fe 3+ (aq)

Hướng dẫn:

a) Fe 2+ (aq) + 2OH - (aq) Fe(OH) 2 (r) màu xanh nhạt

Fe 3+ (aq) + 3OH - (aq) Fe(OH) 3 (r) màu nâu đỏ

b) Fe 2+ (aq) + NCS - (aq) không có phản ứng

Fe 3+ (aq) + NCS - (aq) [Fe(NCS)] 2+ (aq) màu đỏ thẫm

Phản ứng phân biệt Fe 2+ (aq) và Fe 3+ (aq)

c) Fe 2+ (aq) + [Fe(CN) 6 ] 4- (aq) [Fe 2 (II)(CN) 6 ] 2- (r) kết tủa trắng

Fe 3+ (aq) + [Fe(CN) 6 ] 4- (aq) [Fe(III)Fe(II)(CN) 6 ] - (r) kết tủa xanh da trời đậm

(phản ứng nhận biết Fe 3+ (aq))

(xanh Phổ)

d) Fe 2+ (aq) + [Fe(CN) 6 ] 3- (aq) [Fe(III)Fe(II)(CN) 6 ] - (r) kết tủa xanh Phổ

(phản ứng nhận biết Fe 2+ (aq))

Fe 3+ (aq) + [Fe(CN) 6 ] 3- (aq) [Fe 2 (III)(CN) 6 ] (aq) dung dịch màu nâu

* Kết luận: Dùng ion [Fe(CN) 6 ] 4- để nhận biết Fe 3+ (aq)

Dùng ion [Fe(CN) 6 ] 3- để nhận biết Fe 2+ (aq)

Dùng ion NCS - để phân biệt Fe 2+ (aq) và Fe 3+ (aq)

Bài 2 Viết các phương trình phản ứng sau:

a. Khử Mn (III) oxit bằng CO, đun nóng để điều chế Mn 3 O 4 .

b. Hòa tan Cr 2 O 3 vào dung dịch phức Fe(CN) 6 3- trong môi trường kiềm.

Hướng dẫn

Bài 3.

3Mn 2 O 3 + CO → 2Mn 3 O 4 + CO 2 .

Cr 2 O 3 + 6Fe(CN) 6 3- + 10 OH - → 2CrO 4 2- + 6 Fe(CN) 6 4- + 5H 2 O.

FeCl 2 (dd)

KCN

A (dd)

FeSO 4

Fe 2 (SO 4 ) 3

AgNO 3

KMnO 4 , H +

B kết tủa trắng

C kết tủa xanh đậm

D kết tủa trắng

FeCl 2

E (dd)

Pb(OH) 2 , KOH

Cho sơ đồ chuyển

hóa:

G kết tủa xanh

A + F kết tủa nâu

Phát hành PDF bởi Ths Nguyễn Thanh Tú

Đăng ký Word doc qua Zalo 0905779594 Email thanhtuqn88@gmail.com

30


1. Viết phương trình ion của các phản ứng xảy ra theo sơ đồ trên.

2. Hãy cho biết từ tính của hợp chất A, dùng thuyết lai hóa để giải thích.

Hướng dẫn :

1. Các phương trình phản ứng:

Fe 2+ + 6 CN - 4-

[Fe(CN)

6

]

4-

[Fe(CN)

6

] + 2 Fe 2+ Fe 2 [Fe(CN) 6 ] trắng (B)

3

4-

[Fe(CN)

6

] + 4 Fe 3+ Fe 4 [Fe(CN) 6 ] 3 xanh đậm

4-

[Fe(CN)

6

] + 4 Ag + Ag 4 [Fe(CN) 6 ] trắng

5

2

4-

[Fe(CN)

6

] +

3-

[Fe(CN)

6

] + 3 Fe 2+

Hoặc K + +

2

-

MnO

4

+ 8 H + Mn 2+ + 4 H 2 O + 5

3-

[Fe(CN)

6

] + Fe 2+

Fe 3 [Fe(CN) 6 ] 2 xanh

3-

[Fe(CN)

6

] + Pb(OH)

2

+ 2 OH - 2

PbO 2 nâu

2. Cấu hình electron của Fe 2+ là [Ar]3d 6 4s 0 4p 0 4d 0

KFe[Fe(CN) 6 ] xanh

3-

[Fe(CN)

6

]

4-

[Fe(CN)

6

] + 2 H 2 O +

(A)

(C)

(D)

(E)

(G)

(F)

3d 6 4s 0 4p 0 4d 0

Vì CN - là phối tử trường mạnh, do đó khi tạo phức với Fe 2+ , 4 electron độc thân trên

4 obitan 3d của Fe(II) bị ghép đôi, giải phóng 2 obitan 3d trống. Hai obitan này lai hóa

với 1 obitan 4s và 3 obitan 4p, tạo thành 6 obitan lai hóa d 2 sp 3 hướng về 6 đỉnh của hình

bát diện đều. Mỗi obitan lai hóa này xen phủ với một obitan tự do có hai electron của CN -

4-

, tạo ra 6 liên kết cho nhận, hình thành phức [Fe(CN)

6

] lai hóa trong, có cấu trúc bát diện.

Phức này nghịch từ vì có tổng spin bằng không:

CN - CN - CN - CN - CN - CN -

↓ ↓ ↓ ↓ ↓ ↓

d 2 sp 3

Bài 4 Cho sơ đồ chuyển hóa như hình dưới đây

A3

(4)

(5)

A4

(7)

Phát hành PDF bởi Ths Nguyễn Thanh Tú

Đăng ký Word doc qua Zalo 0905779594 Email thanhtuqn88@gmail.com

31


A1

(2)

A2

(3)

(6)

A5

(1)

(10)

A6

(8)

(9)

Các hợp chất A1, A2, A3, A4, A5 và A6 đều chứa nguyên tố kim loại A và nguyên tố

oxi, số oxi hóa của A tăng dần từ +2 đến +7 trong các hợp chất từ A1, A2, A3 đến A4.

Hợp chất A2 chỉ gồm 2 nguyên tố, phần trăm khối lượng của oxi trong A2 là 36,78%

1. Xác định công thức phân tử của các chất từ A1 đến A6. Viết phương trình hóa học của

các phản ứng xảy ra trong sơ đồ trên.

2. Trong PTN, dung dịch của A4 (thường được sử dụng trong các phép chuẩn độ oxi hóa

– khử). Giải thích (bằng các phương trình hóa học) tại sao:

a) Khi thực hiện phép chuẩn độ này, người ta cho dung dịch của A4 vào buret, chất khử

vào bình tam giác mà không làm ngược lại.

b) Dung dịch của A4 được bảo quản trong các bình tối màu.

3. Cho dung dịch của A4 tác dụng với AgNO 3 , thu được kết tủa màu đỏ X1. Cho BaCl 2

vừa đủ vào dung dịch bão hòa của X1, thu được kết tủa trắng X2 và dung dịch của X3.

Khi cho H 2 SO 4 loãng vào dung dịch của X3 thu được kết tủa trắng X4 và dung dịch của

X5. Đun nóng dung dịch của X5 thì thu được kết tủa A2. Còn nếu cho A4 tác dụng với

H 2 SO 4 đậm đặc thì thu được oxit X6 là một chất oxi hóa rất mạnh. Xác định CTPT các

chất từ X1 đến X6 và viết phương trình hóa học của các phản ứng xảy ra trong các thí

nghiệm trên. Biết các hợp chất X1, X3, X5 và X6 đều chứa nguyên tố kim loại A.

Hướng dẫn:

1. - Gọi số oxi hóa của A trong hợp chất oxit A2 là n.

Ta có: 63, 22 n

36,78 2

A 16

Với n = 4 A= 55 (Mn)

- Dựa vào số oxi hóa ta có: A1:

2

MnO

A2:

4

MnO 2

6

A3: K MnO

2 4

A4:

7

K MnO 4

- Dựa vào sơ đồ chuyển hóa ta có: A5: Mn 2 O 7 A6: HMnO 4

Phát hành PDF bởi Ths Nguyễn Thanh Tú

Đăng ký Word doc qua Zalo 0905779594 Email thanhtuqn88@gmail.com

32


t

(1) MnO + 1/2O 0

2 MnO 2

(2) 2MnO 2 + 4KOH + O 2

0

t

2K 2 MnO 4 + 2H 2 O

(3) K 2 MnO 4 + 2Fe(OH) 2 + 2H 2 O MnO 2 + 2Fe(OH) 3 + 2KOH

3K 2 MnO 4 + 2H 2 O 2KMnO 4 + MnO 2 + 4KOH

(4) 4KMnO 4 + 4KOH 4K 2 MnO 4 + O 2 + 2H 2 O

(5) 2K 2 MnO 4 + Cl 2 2KMnO 4 + 2KCl

(6) 2KMnO 4 + K 2 SO 3 + H 2 O 2MnO 2 + 3K 2 SO 4 + 2KOH

2KMnO 4

0

t

K 2 MnO 4 + MnO 2 + O 2

(7) 2KMnO 4 + H 2 SO 4 đặc Mn 2 O 7 + K 2 SO 4 + H 2 O

t

(8) 2Mn 2 O 0

7 4MnO 2 + 3O 2

(9) Mn 2 O 7 + H 2 O 2HMnO 4

t

(10) 4HMnO 0

4 4MnO 2 + 3O 2 + 2H 2 O

2. - Chất cần chuẩn độ là chất khử nếu làm ngược lại thì cho kết quả sai số nhiều vì chất

khử sẽ còn lại trên buret.

- KMnO 4 dễ bị phân hủy bởi ánh sáng nên cần chứa trong bình sẫm màu.

3. KMnO 4 + AgNO 3 KNO 3 + AgMnO 4

(A4)

(X1)

2AgMnO 4 + BaCl 2 Ba(MnO 4 ) 2 + 2AgCl

(X3) (X2)

Ba(MnO 4 ) 2 + H 2 SO 4 loãng BaSO 4 + 2HMnO 4

(X4) (X5)

t

4HMnO 0

4 4MnO 2 + 3O 2 + 2H 2 O

(A2)

2KMnO 4 + H 2 SO 4 đặc Mn 2 O 7 + K 2 SO 4 + H 2 O

(A4)

(X6)

Bài 5:

Hai muối A và B đều chứa nguyên tố X với cùng số oxid hóa. Trong muối A, nguyên tố

này thuộc phần cation, còn trong muối B thì thuộc anion. Muối A là bột tinh thể màu tím

nhạt, trong dung dịch có màu vàng nhạt. Muối B có màu đỏ.

Trộn lẫn 0.4444 gam A và 0.3621 gam B (phản ứng 1), thu được dung dịch màu xanh lục,

và cuối cùng có kết tủa màu xanh lục C tách ra. Thêm dung dịch KOH 40 % vào kết tủa

Phát hành PDF bởi Ths Nguyễn Thanh Tú

Đăng ký Word doc qua Zalo 0905779594 Email thanhtuqn88@gmail.com

33


xanh lục rồi khuấy đều (phản ứng 2), thu được huyền phù màu nâu - là chất D kết tủa.

Sau khi li tâm, dung dịch E nằm trên chất D có màu tím - do trong đó có chứa chất F.

Acid hóa dung dịch E bởi nitric acid thì xuất hiện bọt khí (phản ứng 3a) và tạo thành

dung dịch keo màu xanh dương của chất G (phản ứng 3b). Chất G bị hòa tan khi thêm

KF vào (phản ứng 4), tạo thành hỗn hợp đẳng mol của hai chất H và I - đều chứa nguyên

tố X nhưng có số oxid hóa khác nhau.

Muối B có thể nhận được bởi phản ứng của I với hydrogen peroxide (phản ứng 5).

Từ dung dịch kiềm của chất F với barium hydroxide, có thể tách ra muối J kết tủa, là chất

tương đồng với barium sulfate (phản ứng 6). Khi acid hóa dung dịch F bởi hydrochloric

acid thì có khí mùi khó chịu thoát ra (phản ứng 7).

Muối A bị phân hủy khi đun nóng, tạo thành khí màu nâu (phản ứng 8).

Tên gọi thông thường của G có chứa tên một thành phố.

Chất А В I J

Hàm

lượng 13.82 % 16.96 % 13.22 % 21.72 %

của Х

1) Xác định nguyên tố X và các chất A - D, F - J.

2) Viết các phương trình phản ứng đã mô tả (8 phương trình).

3) Giải thích sự khác nhau về màu sắc của A ở dạng rắn và dung dịch

4) Có thể cô lập A từ dung dịch nước bằng cách nào?

5) Đề xuất 2 hướng khả thi để thu được chất F.

Hướng dẫn:

1.

A Fe(NO 3 ) 3 .9H 2 O B K 3 [Fe(CN) 6 ] C Fe[Fe(CN) 6 ] D Fe(OH) 3 hoặc

Fe 2 O 3 .xH 2 O

F G K[FeFe(CN) 6 ] H K 3 [FeF 6 ] hoặc I K 4 [Fe(CN) 6 ]·3H 2 O J BaFeO 4

K 2 FeO 4

K 2 [FeF 5 ]

2. Viết các phương trình phản ứng đã mô tả:

(1) Fe(NO 3 ) 3 + K 3 [Fe(CN) 6 ] 3KNO 3 + [FeFe(CN) 6 ]

(A) (B) (C)

(2) 3Fe[Fe(CN) 6 ] + 14KOH 3K 4 [Fe(CN) 6 ] + K 2 FeO 4 + 2Fe(OH) 3 + 4H 2 O

Phát hành PDF bởi Ths Nguyễn Thanh Tú

Đăng ký Word doc qua Zalo 0905779594 Email thanhtuqn88@gmail.com

34


(C) (F) (D)

(3) 4K 2 FeO 4 + 20HNO 3 8KNO 3 + 4Fe(NO 3 ) 3 + 10H 2 O + 3O 2

Fe(NO 3 ) 3 + K 4 [Fe(CN) 6 ] 3KNO 3 + K[FeFe(CN) 6 ]

(G)

(4) K[FeFe(CN) 6 ] + 6KF K 3 [FeF 6 ] + K 4 [Fe(CN) 6 ]

(G) (H) (I)

(5) 2K 4 [Fe(CN) 6 ] + H 2 O 2 2K 3 [Fe(CN) 6 ] + 2KOH

(I)

(B)

(6) K 2 FeO 4 + Ba(OH) 2 BaFeO 4 + 2KOH

(F)

(J)

(7) 2K 2 FeO 4 + 16HCl 4KCl + 2FeCl 3 + 3Cl 2 + 8H 2 O

t

(8) 4Fe(NO 3 ) 3 .9H 2 O o

2Fe 2 O 3 + 36H 2 O + 12NO 2 + 3O 2

3) Màu của muối A được xác định bởi cation [Fe(H 2 O) 6 ] 3+ - bị thủy phân trong dung dịch

nước làm biến đổi màu sắc: [Fe(H 2 O) 6 ] 3+ + H 2 O [Fe(OH)(H 2 O) 5 ] 2+ + H 3 O +

4) Để thu được muối A thì quá trình kết tinh phải được thực hiện với dung dịch axit

HNO 3 để chuyển cân bằng thủy phân theo chiều sang trái.

5) Hướng 1: Oxi hóa pha rắn

8KO 2 + 2Fe 2 O 3

2KO 2 + Fe

o

t

4K 2 FeO 4 + 3O 2

o

t

K 2 FeO 4

o

t

Fe + 2KOH + 3KNO 2 K 2 FeO 4 + 3KNO 2 + H2O

Hướng 2: Oxi hóa huyền phù của Fe(OH) 3 hoặc Fe 2 O 3 .xH 2 O, nhưng không phải Fe 2 O 3

trong dung dịch kiềm:

10KOH + 2Fe(OH) 3 + 3Br 2 2K 2 FeO 4 + 6KBr + 8H 2 O

10KOH + 2Fe(OH) 3 + 3Cl 2 2K 2 FeO 4 + 6KCl + 8H 2 O

4KOH + 2Fe(OH) 3 + KBrO 3 2K 2 FeO 4 + KBr + 5H 2 O

Hướng 3: Oxi hóa anot sắt trong bình điện phân.

Fe + 2KOH + 2H 2 O → K 2 FeO 4 + 3H 2

Bài 6:

Trong giờ học, thầy giáo dạy Hóa cho các học sinh xem một số chiếc cốc chứa các dung

dịch và một cái lọ chứa chất bột màu nâu đen (gần như là màu đen).

Phát hành PDF bởi Ths Nguyễn Thanh Tú

Đăng ký Word doc qua Zalo 0905779594 Email thanhtuqn88@gmail.com

35


“Tất cả chúng đều chứa các hợp chất của cùng một kim loại M và hôm nay chúng ta sẽ

nghiên cứu về tính chất hóa học của chúng. Tất cả các phản ứng được tôi miêu tả dưới

dạng sơ đồ

A

+ S

M

D xanh hoặc hồng

t 0 t 0

t 0

B xanh lá cây

C tím

G

* Hãy bắt đầu với bột oxit màu nâu đen A, trong đó hàm lượng kim loại là 63,2 %.

Trong tự nhiên, A tồn tại ở dạng khoáng chất X, là nguồn chính để sản xuất kim loại M

và các hợp chất của nó. Khi tôi đun nóng A với KOH và KNO 3 thì xảy ra phản ứng 1, tạo

thành chất B màu xanh lục. Đây không phải là một chất quá bền để có thể lưu trữ, do nó

có thể tham gia vào một phản ứng thú vị, trong đó số oxi hóa của kim loại đồng thời tăng

và giảm. Phản ứng này xảy ra đặc biệt nhanh nếu dung dịch được sục khí CO 2 (phản ứng

2) hoặc chỉ đơn giản là thêm một dung dịch axit vào (phản ứng 3).”

* Dứt lời, thầy giáo liền đổ một ít dung dịch B vào ống nghiệm rồi thêm vào vài giọt axit

H 2 SO 4 loãng - dung dịch đổi sang màu tím, giống như dung dịch chất C.

“Trong chất C, số oxi hóa của kim loại M đạt cực đại, do đó trong công nghiệp thường

điều chế chất này bằng cách sục Cl 2 vào dung dịch của hợp chất B (phản ứng 4). Cũng

như B, chất C có tính oxi hóa mạnh (đặc biệt là trong môi trường axit) và các nhà hóa

học gọi nó là “tắc kè hoa”.”

* Thầy giáo lại tiếp tục thí nghiệm bằng cách thêm vài giọt dung dịch axit H 2 SO 4 và một

lượng nhỏ K 2 S vào dung dịch B, kết quả là dung dịch mất màu (phản ứng 5).

“Chúng ta thu được dung dịch của chất G. Nếu K 2 S dư thì thu được kết tủa màu hồng

nhạt (phản ứng 6). Điều thú vị là khi cho kim loại M phản ứng với lưu huỳnh thì cũng thu

được M, nhưng là với màu xanh lá cây (phản ứng 7).

Phát hành PDF bởi Ths Nguyễn Thanh Tú

Đăng ký Word doc qua Zalo 0905779594 Email thanhtuqn88@gmail.com

36


1) Xác định kim loại M và các chất A, B, C, D, G.

2) Viết các phương trình phản ứng 1 - 7.

3) Xác định tên gọi của khoáng chất M, có thành phần chính là oxit A.

4) Các phản ứng 2 và 3 thuộc loại phản ứng oxi hóa-khử gì?

5) Thầy giáo đã quên nói về phản ứng A → M. Hãy đưa ra phương trình phản ứng.

Phương pháp này dùng để thu được kim loại tinh khiết, được gọi tên là gì?

6) Tại sao C lại được gọi là “tắc kè hoa”? Viết phương trình phản ứng của C với K 2 S

trong các môi trường axit (H 2 SO 4 ), trung tính và kiềm (KOH) (3 phương trình) và mô tả

các biến đổi quan sát được.

7) Thầy giáo cho biết B là một chất oxi hóa mạnh. Viết phương trình phản ứng của B với

K 2 S trong môi trường axit H 2 SO 4

Hướng dẫn

1) Trước tiên, xác định kim loại dựa trên hàm lượng trong oxit, có công thức M2Ox.

16x

( O) 1 0,632 0,368

2A

16x

Trong đó Ar là nguyên tử khối của M.

1 16x

Ar

16x

13,74x

2

0,368

r

A r Công thức oxit Kim loại M

13,74 M 2 O -

27,48 M 2 O 2 MO Al ?

41,22 M 2 O 3 -

54,96 M 2 O 4 MO 2 Mn

68,7 M 2 O 5 -

82,43 M 2 O 6 MO 3 -

Nhôm không phù hợp về mặt hóa trị và cũng không tạo được nhiều hợp chất màu như

mangan

M = Mn, A = MnO 2 , B = K 2 MnO 4 , C = KMnO 4 , G = MnSO 4 , D= MnS

2) Phương trình phản ứng:

(1) MnO 2 + KNO 3 + 2KOH → K 2 MnO 4 + KNO 2 + H 2 O

(hoặc 5MnO 2 + 2KNO 3 + 2KOH → 5K 2 MnO 4 + N 2 + 4H 2 O)

(2) 3K 2 MnO 4 + 4CO 2 + 2H 2 O → 2KMnO 4 + MnO 2 + 4KHCO 3

(hoặc 3K 2 MnO 4 + 2CO 2 + 2H 2 O → 2KMnO 4 + MnO 2 + 2K 2 CO 3 )

Phát hành PDF bởi Ths Nguyễn Thanh Tú

Đăng ký Word doc qua Zalo 0905779594 Email thanhtuqn88@gmail.com

37


(3) 3K 2 MnO 4 + 2H 2 SO 4 (р-р) → 2KMnO 4 + MnO 2 + 2K 2 SO 4 + 2H 2 O

(4) 2K 2 MnO 4 + Cl 2 → 2KMnO 4 + 2KCl

(5) 8KMnO 4 + 5K 2 S + 12H 2 SO 4 → 9K 2 SO 4 + 8MnSO 4 + 12H 2 O

(6) MnSO 4 + K 2 S → MnS↓ + K 2 SO 4

(7) Mn + S → MnS

3) Khoáng chất pyrolusite.

4) Phản ứng dị phân (tự oxi hóa - khử).

5) Khử với nhôm (nhiệt nhôm): 3MnO 2 + Al → 3Mn + Al 2 O 3

6) Gọi là "tắc kè hoa" bởi KMnO 4 biến đổi màu sắc trong các môi trường khác nhau.

2KMnO 4 + 5K 2 SO 3 + 3H 2 SO 4 → 2MnSO 4 + 6K 2 SO 4 + 3H 2 O (dung dịch không màu)

2KMnO 4 + 3K 2 SO 3 + H 2 O → 2MnO 2 + 3K 2 SO 4 + 2KOH (kết tủa nâu)

2KMnO 4 + K 2 SO 3 + 2KOH → 2K 2 MnO 4 + K 2 SO 4 + H 2 O (dung dịch xanh lục)

7) K 2 MnO 4 + 2K 2 SO 3 + 2H 2 SO 4 → MnSO 4 + 3K 2 SO 4 + 2H 2 O

III.3 Tinh thể của kim loại nhóm VIIB, VIIIB

Bài 1: Từ nhiệt độ phòng đến 1185K sắt tồn tại ở dạng Fe với cấu trúc lập phương tâm

khối, từ 1185K đến 1667K ở dạng Fe với cấu trúc lập phương tâm diện. Ở 293K sắt có

khối lượng riêng d = 7,874g/cm 3 .

a) Hãy tính bán kính của nguyên tử Fe.

b) Tính khối lượng riêng của sắt ở 1250K (bỏ qua ảnh hưởng không đáng kể do sự dãn nở

nhiệt).

c) Thép là hợp kim của sắt và cacbon, trong đó một số khoảng trống giữa các nguyên tử

sắt bị chiếm bởi nguyên tử cacbon. Trong lò luyện thép (lò thổi) sắt dễ nóng chảy khi

chứa 4,3% cacbon về khối lượng. Nếu được làm lạnh nhanh thì các nguyên tử cacbon vẫn

được phân tán trong mạng lưới lập phương nội tâm, hợp kim được gọi là martensite cứng

và dòn. Kích thước của tế bào sơ đẳng của Fekhông đổi. Hãy tính số nguyên tử trung

bình của C trong mỗi tế bào sơ đẳng của Fe với hàm lượng của C là 4,3%.

d) Hãy tính khối lượng riêng của martensite.

(cho Fe = 55,847; C = 12,011; số N = 6,022. 10 23 )

Hướng dẫn:

a) Số nguyên tử Fe trong một mạng cơ sở lập phương tâm khối là: 2

Phát hành PDF bởi Ths Nguyễn Thanh Tú

Đăng ký Word doc qua Zalo 0905779594 Email thanhtuqn88@gmail.com

38


m 2.55,847 2.55,847

0

8

d 3

Fe

a 2,87.10 cm 2,87 A

23 3 23

V 6,022.10 . a 6,022.10 .7,874

a 3

0

a 3 4r r 1, 24 A

4

b) ở nhiệt độ 1250 sắt tồn tại dạng Fe với cấu trúc mạng lập phương tâm diện.

Ta có: a 2 2. r 2 2.1, 24 3,51A

; d

0

Fe

4.55,847g

6,022.10 .(3,51.10 cm)

23 8 3

c) Số nguyên tử trung bình của C trong mỗi tế bào sơ đẳng của Fe là:

mC

% C. mFe

4,3.2.55,847

0,418

12,011 % Fe.12,011 95,7.12,011

(2.55,847 0,418.12,011) g

d) Khối lượng riêng của martensite:

23 8 3

6,022.10 .(2,87.10 cm)

Bài 2:

8,58 g / cm

8, 20 g / cm

1. Ở nhiệt độ phòng sắt có cấu tạo tinh thể dạng lập phương tâm khối (sắt- ). Cho biết

khối lượng riêng của sắt kim loại nguyên chất là 7,874g.cm -3 ở 293K. Tính bán kính

nguyên tử của sắt (cm).

2. Thép là hợp kim của sắt và cacbon, trong đó một số khoảng trống giữa nguyên tử sắt

(các hốc) trong mạng tinh thể bị chiếm bởi các nguyên tử nhỏ là cacbon. Hàm lượng

cacbon trong hợp kim này thường trong khoảng 0,1% đến 4%. Trong lò cao, sự nóng

chảy của sắt càng dễ dàng khi thép chứa 4,3% theo khối lượng. Nếu hỗn hợp này được

làm lạnh quá nhanh (đột ngột) thì các nguyên tử cacbon được phân tán trong mạng sắt- .

Chất rắn mới này được gọi là martensite - rất cứng và giòn. Dù hơi bị biến dạng, cấu tạo

tinh thể của chất rắn này là giống như cấu tạo tinh thể của sắt- . Giả thiết rằng các

nguyên tử cacbon được phân bố đều trong cấu trúc của sắt.

a. Ước tính hàm lượng nguyên tử cacbon trong một tế bào đơn vị (ô mạng cơ sở) của

sắt- trong martensite chứa 4,3%C theo khối lượng.

b. Ước tính khối lượng riêng (g.cm -3 ) của vật liệu này.

Khối lượng mol nguyên tử: M Fe = 55,847g.mol -1 ; M C = 12,011g.mol -1 .

Hướng dẫn:

1.

3

3

Phát hành PDF bởi Ths Nguyễn Thanh Tú

Đăng ký Word doc qua Zalo 0905779594 Email thanhtuqn88@gmail.com

39


Số nguyên tử sắt trong 1 ô cơ sở: n = 8.1/8 + 1 = 2.

Theo đề ta có:

a

n.M

N

A .D

Bán kính nguyên tử Fe:

2.55,847

= = 2,356.10 cm

23

6,0221.10 .7,847

-23 3

(1)

a

a

3

2

= 4r

4.r = a 3 a = 4r/ 3

Thể tích 1 ô đơn vị: V 1ô = a 3 2

= 16r / 3 3

(2)

Từ (1) và (2) ta có: r = 1,241.10 -8 cm.

2. a) Trong 100,0 gam martensite có 4,3 gam C n C = 0,36mol và n Fe = 1,71mol.

Số nguyên tử cacbon có trong mỗi ô mạng tinh thể là: 2.(0,36/1,71) = 0,42 nguyên tử.

b. Khối lượng của một ô mạng tinh thể là:

m ô

2.55,847+0,42.12,011

6,022.10

-22

= = 1,939.10 g

23

Vậy khối lượng riêng của martensite có 4,3%C là:

D (martensite có 4,3%C) = 1,939.10 -22 /(2,356.10 -23 ) = 8,228 g.cm -3 .

Bài 3:

Hiện nay, bột màu CoAl 2 O 4 với kích thước hạt siêu mịn dùng nhiều trong lĩnh vực tạo

màu cho sơn, nhựa, gạch, gốm sứ…Trong đó, CoAl 2 O 4 kết tinh ở kiểu mạng spinel có

cấu trúc như hình dưới. Trong đó các ion Co 2+ chiếm các hốc tứ diện và Al 3+ chiếm hốc

bát diện. Ô màu đen biểu thị hốc tứ diện, và ô màu trắng biểu thị hốc bát diện. Các ion

O 2- nằm ở các đỉnh và mặt.

Ở một nhiệt độ T nhất định thì độ dài đường biên giới ô mạng cơ sở (gồm chiều dài và

rộng) của CoAl 2 O 4 là 912 pm. Lúc này các ion oxit có thể tiếp xúc với nhau trực tiếp

được.

a) Tính khối lượng riêng (g/cm 3 ) của CoAl 2 O 4 ở nhiệt độ T.

b) Xác định bán kính cực đại để các ion M 2+ và M 3+ có thể nằm khít vào các hốc

tương ứng trong ô mạng spinel.

Phát hành PDF bởi Ths Nguyễn Thanh Tú

Đăng ký Word doc qua Zalo 0905779594 Email thanhtuqn88@gmail.com

40


Cho biết M của Co = 58,93; Al=26,98; O =16,00; số avogadro N A =6,023.10 23

Hướng dẫn

a. Độ dài các đường biên của một ô mạng cơ sở (gồm chiều dài và rộng) = 912pm nên độ

dài của một ô mạng cơ sở là a = 456 pm

Thể tích của ô mạng cơ sở : V = a 3 = (456.10 -10 ) 3 (cm 3 )

Khối lượng riêng của CoAl 2 O 4 ở nhiệt độ T là

M

CoAl2O

(58,93 2.26,98 4.16,00)

4

D = = 3,097 (g/cm 3 )

3 23 10 3

N .a 6,023.10 .(456.10 )

A

b.

- Các ion O 2- tiếp xúc nhau qua đường chéo của hình lập phương nên bán kính ion O 2-

a 2 456 2

được xác định : 4. 4. r 2

a. 2 r 2

161, 22 pm

O

O

4 4

- Ion M 2+ nằm ở hốc tứ diện, với cạnh của hình tứ diện này là b; thì b được xác định:

b 2r 2.161, 22 322, 44 pm

O

2

- Khoảng cách từ tâm hình tứ diện ra đến đỉnh là c; thì c được xác định bởi công thức :

b 6 322,44. 6

c 197, 45pm

4 4

Vậy để M 2+ nằm khít vào trong hốc tứ diện thì bán kính cực đại của ion là:

r c r 2 2 197, 45 161,22 36,23 pm

M

O

Còn ion M 3+ nằm ở hốc bát diện nên bán kính cực đại của ion này là:

r

M

3

a 2r

2

O

456 2.161, 22

66,78pm

2 2

Bài 4 Hình dưới là ô mạng tinh thể của LaNi 5 . Biết a = b = 511 pm và c = 397 pm, các

góc α = β = 120 o , = 90 và chỉ có một phân tử hợp kim trong ô mạng cơ sở

1. Hãy xác định khối lượng riêng hợp kim.

2. Hợp kim LaNi 5 có khả năng hấp phụ hydro. Bình thường hydro chiếm toàn bộ 6 hốc tứ

diện trong ô mạng và lúc này tinh thể có công thức LaNi 5 H 6 . Cho rằng sự hấp phụ hydro

không làm thay đổi thể tích ô mạng. Hãy xác định tỉ trọng hydro trong ô mạng (số gam

hydro / 1 dm 3 hay 1L).

Phát hành PDF bởi Ths Nguyễn Thanh Tú

Đăng ký Word doc qua Zalo 0905779594 Email thanhtuqn88@gmail.com

41


Ô mạng cơ sở của hợp kim LaNi 5

3. So sánh tỉ trọng hydro trong ô mạng với hydro ở điều kiện tiêu chuẩn và trong hydro

lỏng = 70g/dm 3 . Cho biết hợp kim LaNi 5 hấp phụ hydro tốt hay không.

Hướng dẫn

1. V = a 2 csinα = 8,98.10 -23 cm 3

M

. 8.00

N

A

V

g/cm 3

6M

H

112

2.

N

A.

V

g/dm 3

3. So với không khí

m p.

M H 2

81

V

R.

T

g/cm 3 = 0,081 g/dm 3

Như vậy rõ ràng LaNi 5 là một vật liệu có khả năng hấp phụ hydro rất tốt.

Bài 5

Tinh thể ReO 3 thuộc hệ lập phương, trong đó ion Re 6+ chiếm các vị trí đỉnh của hình lập

phương, ion O 2- chiếm vị trí trung điểm tất cả các cạnh của ô mạng.

Biết bán kính: O 2- = 0,126nm, Re 6+ = 0,061 nm.

Nguyên tử khối: O =16; Re = 186

1. Hãy vẽ một ô mạng cơ sở của tinh thể ReO 3 , tính số nguyên tử Re và O trong một ô

mạng và khối lượng riêng của tinh thể ReO 3 (gam/cm 3 )

2. Cation Li + có thể xâm nhập vào mạng lưới tinh thể ReO 3 ở ngay nhiệt độ phòng.

Cation Li + có kích thước lớn nhất bằng bao nhiêu để khi xâm nhập vào mạng lưới tinh thể

ReO 3 không làm thay đổi kích thước của ô mạng tinh thể ?

Hướng dẫn:

1. Vẽ ô mạng ReO 3

Phát hành PDF bởi Ths Nguyễn Thanh Tú

Đăng ký Word doc qua Zalo 0905779594 Email thanhtuqn88@gmail.com

42


- Trong một ô mạng có

Số ion Re 6+ = 8.1/8 = 1

Số ion O 2- = 12.1/4 = 3

- Độ dài cạnh a= 2(r Re + r O ) = 0,374nm

- Khối lượng riêng của tinh thể:

2. Tính bán kính ion lạ

M (186 16.3) / 6,02.10

D

V

23

ô mang

3

7,43 gam / cm

7 3

ô mang

(0,374.10 )

Ion Li + xâm nhập sẽ chiếm vị trí trung tâm của ô mạng cơ sở. Do vậy khoảng cách từ tâm

đến trung điểm mỗi cạnh tương ứng:

r 2 r a / 2 0, 265 nm

O

r

Bài 6 :

Li

Li

0,139nm

1. Ô mạng cơ sở (tế bào cơ bản) của tinh thể NiSO 4 có 3 cạnh vuông góc với nhau, cạnh

a = 6,338 Å; b = 7,842 Å; c = 5,155 Å. Khối lượng riêng gần đúng của NiSO 4 là 3,9

g/cm 3 . Tìm số phân tử NiSO 4 trong một ô mạng cơ sở và tính khối lượng riêng chính xác

của NiSO 4 .

2. Niken(II) oxit có cấu trúc mạng tinh thể giống mạng tinh thể của natri clorua. Các ion O 2–

tạo thành mạng lập phương tâm mặt, các hốc bát diện có các ion Ni 2+ . Khối lượng riêng của

niken(II) oxit là 6,67 g/cm 3 . Nếu cho niken(II) oxit tác dụng với liti oxit và oxi thì được các

tinh thể trắng có thành phần Li x Ni 1–x O:

x

2 Li 2O + (1 – x)NiO + x 4 O 2 Li x Ni 1–x O

Cấu trúc mạng tinh thể của Li x Ni 1–x O giống cấu trúc mạng tinh thể của NiO,

nhưng một số ion Ni 2+ được thế bằng các ion liti và một số ion Ni 2+ bị oxi hóa để bảo đảm

tính trung hòa điện của phân tử. Khối lượng riêng của tinh thể Li x Ni 1–x O là 6,21 g/cm 3 .

a. Vẽ một ô mạng cơ sở của niken(II) oxit.

b. Tính x (chấp nhận thể tích của ô mạng cơ sở không thay đổi khi chuyển từ NiO thành

Li x Ni 1–x O).

Phát hành PDF bởi Ths Nguyễn Thanh Tú

Đăng ký Word doc qua Zalo 0905779594 Email thanhtuqn88@gmail.com

43


c. Tính phần trăm số ion Ni 2+ đã chuyển thành ion Ni 3+ và viết công thức thực nghiệm

đơn giản nhất của hợp chất Li x Ni 1–x O bằng cách dùng Ni(II), Ni(III) và các chỉ số

nguyên.

Cho Li = 6,84; Ni = 58,69; O = 16; S = 32

Hướng dẫn

1. a = 6,338.10 –8 cm; b = 7,842.10 –8 cm; c = 5,155.10 –8 cm

m m n.M

NiSO4

Từ DNiSO

(1)

4

V a.b.c N .a.b.c

n

D .N .a.b.c

(2)

NiSO 4

A

MNiSO4

A

23 –8 –8 –8

3,9.6,02.10 .6,338.10 .7,842.10 .5,155.10

n = 3,888

154,76

Số phân tử NiSO 4 trong một ô mạng cơ sở phải là số nguyên n = 4

4.154,76

D

NiSO 4

(chính xác) = = 4,012 (g/cm 3 )

23 –8 –8 –8

6,02.10 .6,338.10 .7,842.10 .5,155.10

2a)

Ion oxi (O 2- )

Ion niken (Ni 2+ )

b). Tính x:

Tính cạnh a của ô mạng cơ sở của NiO:

n.MNiO

D = a 3 n.M

NiO

= N .a N .D

NiO 3

A

A

NiO

n = 4 (vì mạng là lập phương tâm diện)

4.74,69

23

3

a = 6,02.10 .6,67

a = 4,206.10 –8 cm

Theo đầu bài, ô mạng cơ sở của NiO và ô mạng cơ sở của Li x Ni 1–x O giống nhau, do đó:

Phát hành PDF bởi Ths Nguyễn Thanh Tú

Đăng ký Word doc qua Zalo 0905779594 Email thanhtuqn88@gmail.com

44


D

x = 0,10

n.M

4. x.6,94 (1 x).58,69 16

6,21 =

23 8 3

6,02.10 .(4,206.10 )

LixNi1

xO

Lix

Ni1

xO 3

N

A.a

c) Thay x vào công thức Li x Ni 1–x O, ta có Li 0,1 Ni 0,9 O hay công thức là LiNi 9 O 10 . Vì phân

tử trung hòa điện nên trong LiNi 9 O 10 có 8 ion Ni 2+ và 1 ion Ni 3+ . Vậy cứ 9 ion Ni 2+ thì có

1 ion chuyển thành Ni 3+ .

Phần trăm số ion Ni 2+ đã chuyển thành ion Ni 3+ là 1 100

9 % = 11,1%

Công thức thực nghiệm đơn giản nhất: LiNi(III)(Ni(II)) 8 O 10 .

Bài 7:

Trong các tinh thể sắt (cấu trúc lập phương tâm khối) các nguyên tử cacbon có thể

chiếm các mặt của ô mạng cơ sở.

a) Bán kính kim loại của sắt là 1,24 o A . Tính độ dài cạnh a của ô mạng cơ sở.

b) Bán kính cộng hóa trị của cacbon là 0,77 A o . Hỏi độ dài cạnh a sẽ tăng lên bao nhiêu

khi sắt có chứa cacbon so với cạnh a khi sắt nguyên chất.

c) Tính độ dài cạnh ô mạng cơ sở cho sắt (cấu trúc lập phương tâm diện) và tính độ tăng

chiều dài cạnh ô mạng biết rằng các nguyên tử C có thể chiếm tâm của các ô mạng cơ sở

và bán kính kim loại sắt là 1,26 A o . Có thể kết luận gì về khả năng xâm nhập của cacbon

vào 2 loại tinh thể sắt trên.

Hướng dẫn

a. sắt : lập phương tâm khối

R = 1,24 a = 2,864 o A

b. Độ dài đường chéo mặt bên a 2 > 2R + 2r (tức là 4,05 > 2.1,24 + 2. 0,77)

Vậy khi C chiếm tâm của mặt bên thì không tiếp xúc với các quả cầu ở các đỉnh

Khoảng cách từ tâm của ô mạng đến tâm của mặt bên:

a

< R + r (tức là 1,432 < 1,24 + 0,77)

2

Phát hành PDF bởi Ths Nguyễn Thanh Tú

Đăng ký Word doc qua Zalo 0905779594 Email thanhtuqn88@gmail.com

45


Vậy C ở tâm mặt bên thì tiếp xúc với quả cầu ở tâm

Vì vậy làm cạnh a tăng lên: [(1,24 + 0,77) – 1,432) * 2 = 1,156 A

o

(nói cách khác: 2R + 2r – a = 1,156)

c. sắt : lập phương tâm diện

Độ dài cạnh a’ 2 = 4R’ a’ = 3,564 A

o

Khi C chiếm tâm của ô mạng làm cho cạnh tăng thêm: 1,26. 2 + 0,77.2 – 3,564 = 0,496A

o

Kết luận: C xâm nhập vào sắt dễ hơn sắt

Bài 8

Tinh thể sắt (II) oxit có cấu tương tự tinh thể NaCl, trong tinh thể FeO có hốc bát diện

được tạo thành bởi các nguyên tử oxi và nguyên tử Fe nằm ở tâm hốc. Trong điều kiện

thường, không tồn tại sắt (II) oxit tinh khiết mà thường có lẫn một lượng sắt (III) trong

FeO. Nên công thức của oxit sắt là Fe x O. Cho hằng số mạng của tinh thể Fe x O a = 4,29 Å

và khối lượng riêng của Fe x O là 5,71 g/cm 3 .

a. Vẽ một ô mạng cơ sở của mạng tinh thể FeO.

b. Tìm x biết nguyên tử khối của Fe là 55,85.

c. Xác định % ion Fe 2+ và Fe 3+ có mặt trong oxit, từ đó, xác định u, v trong công thức oxit

dạng Fe(II) u Fe(III) v O.

Hướng dẫn

a)

nxMFe nMO

4.55,85x 4.16

b) Ta có: d 5,71 x 0,929

23 10 3

N .V 6,022.10 .(429.10 )

A

u v 0,929 u 0,787

c) Ta có: ; u + v = 0,929

2u 3v 2 v 0,142

CT : Fe(II) 0,787 Fe(III) 0,142 O.

Bài 9 Kết quả khảo sát giản đồ nhiễu xạ tia X cho biết tinh thể FeO có cấu trúc kiểu NaCl

với hằng số mạng a = 4,3 Ǻ.

a. Vẽ mạng tinh thể FeO. Xác định khối lượng riêng lí thuyết của FeO (g/cm 3 ).

Phát hành PDF bởi Ths Nguyễn Thanh Tú

Đăng ký Word doc qua Zalo 0905779594 Email thanhtuqn88@gmail.com

46


b. Theo thực nghiệm, mẫu FeO tổng hợp được có khối lượng riêng bằng 5,57 g/cm 3 . Giải

thích sự sai lệch đó. Viết công thức thực tế của oxit đó.

Hướng dẫn

a) FeO có cấu trúc kiểu NaCl → FeO có cấu trúc mạng lập phương tâm diện kép

→ Có 4 phân tử FeO trong một ô đơn vị

Khối lượng riêng của FeO theo lý thuyết:

.

×

ý ế = =

= . ô ạ , . .( , . ) 6,017(g/cm3 )

b) Nhận xét d lý thuyết > d thực tế do thực tế mạng tinh thể bị khuyết một số vị trí

+ TH1: Mạng bị khuyết một số cation

Đặt công thức của oxit sắt là Fe X O

ý

ế

,

Ta có : = = → x = 0,9 → Công thức oxit sắt là : Fe 0,9O

ự ế

,

+ TH2: Mạng bị khuyết một số anion

Đặt công thức sắt là FeO y

Ta có:

ý

ế

ự ế

= =

,

,

→ y = 0,7 → Công thức oxit sắt là FeO 0,7

Bài 10: Tinh thể Pd

Khí H 2 ở nhiệt độ phòng dễ “hòa tan” vào một số kim loại cụ thể, ví dụ như Pd. Ô mạng

cơ sở của Pd có dạng lập phương, tâm của cả các nguyên tử Pd đặt ở các đỉnh và các tâm

mặt của hình lập phương (các nguyên tử được xem như những quả cầu cứng). Khối lượng

riêng của Pd là 12,02 g/cm 3 , khối lượng mol là 106,4 g/mol.

1) Tính số nguyên tử Pd trong 1 ô mạng cơ sở, độ dài cạnh (a) và bán kính của các

nguyên tử Pd theo pm (1 pm = 10 -12 m).

2) Giả sử rằng các nguyên tử là những quả cầu cứng, tính bán kính nguyên tử cực đại có

thể vừa khít với các khoảng trống (hốc) giữa các nguyên tử Pd trong mạng tinh thể.

3) Khoảng cách liên nhân trong các phân tử H 2 là 74 pm và bán kính của nguyên tử H là

54 pm. Hidro trong mạng tinh thể Pd tồn tại ở dạng nào - nguyên tử, phân tử hay cả hai?

4) Tính số nguyên tử Hidro cực đại có thể lấp vào ô mạng cơ sở Pd

5) Về mặt lí thuyết, mỗi thể tích Pd rắn có thể hòa tan tối đa bao nhiêu thể tích khí H 2 (ở

1atm và 25 0 C).

6) Giả sử rằng cân bằng được thiết lập giữa hidro khí và hidro tan trong Pd, hãy dẫn ra

phương trình sự phụ thuộc của nồng độ nguyên tử hidro trong Pd vào áp suất khí hidro

trên Pd rắn ở nhiệt độ không đổi.

Phát hành PDF bởi Ths Nguyễn Thanh Tú

Đăng ký Word doc qua Zalo 0905779594 Email thanhtuqn88@gmail.com

47


Hướng dẫn

M 106, 4

3

1) Thể tích 1 mol palladium là VPd

8,852 cm / mol

12,02

Từ hình vẽ trên, có thể thấy tổng số nguyên tử palladium trong 1 ô mạng cơ sở bằng:

1 1

6. 6. 4

2 8

VPd

8,852

Thể tích ô mạng cơ sở là: V 4 4 5,88.10

23

N 6,02.10

Chiều dài ô mạng cơ sở là

A

3

8

a V 3,89.10 cm 389 nm

cm

23 3

Từ hình vẽ trên, có thể thấy đường chéo của mặt hình lập phương là b = 4r. Áp dụng định

lí Pytago, ta có:

2 2 2 2 2

b a c 16r 2a r 138nm

2)

r

m

a 2r

389 2138

57nm

2 2

a

2 2

3) Hidro trong tinh thể tồn tại ở dạng nguyên tử do phân tử hidro không thể vừa với các

hốc tinh thể.

4) Tổng số hốc bát diện: 1 (ở tâm hình lập phương) + 12 . 1/4 (ở giữa các cạnh) = 4

Đây cũng chính là số nguyên tử hidro cực đại có thể lấp đầy ô mạng cơ sở.

nRT

3

5) VH

12, 23 l 12230 cm

2

P

VH

12230

2

1380

V 8,852

Pd

2

CH

6) H

2

2H

K P

H2

Trong đó: C H - nồng độ hydro nguyên tử trong kim loại, P H2 - áp suất khí hydro

2

C K. P C K.

P hay const P

H H2 H H2 H2

III.4 Bài toán liên quan đến phản ứng hạt nhân

Bài 1:

Hai đồng vị 101 Tc và 104 Tc kém bền, đều phân rã β - , có chu kì bán hủy lần lượt là 14,3

phút và 18,3 phút, sản phẩm của các phân rã trên đều là các nguyên tử bền. Cho khối

lượng các hạt trong bảng sau:

Hạt

101 Ru p n e

101 Tc

Phát hành PDF bởi Ths Nguyễn Thanh Tú

Đăng ký Word doc qua Zalo 0905779594 Email thanhtuqn88@gmail.com

48


Khối lượng (u) 100,9073 100,9056 1,0073 1,0087 0,00055

Xét phản ứng phân rã 101 101 101 0

Tc: Tc Ru e (*)

1. Tính năng lượng tỏa ra của phản ứng (*) theo đơn vị kJ/mol.

2. Tính năng lượng tỏa ra trong quá trình hình thành hạt nhân 101 Ru từ các hạt cơ bản

(kJ/mol).

3. Một lượng 101 Tc có hoạt độ phóng xạ 2016 Ci. Tính khối lượng Tc ban đầu và khối

lượng Tc bị phân rã trong phút đầu tiên.

4. Hỗn hợp gồm hai đồng vị 101 Tc và 104 Tc có hoạt độ phóng xạ tổng cộng là 308 Ci, nếu

để sau 14,3 phút thì hoạt độ phóng xạ chỉ còn 160,462 Ci. Hỏi sau bao lâu (tính từ thời

điểm ban đầu) thì hoạt độ phóng xạ của đồng vị này gấp hai lần hoạt độ phóng xạ đồng vị

kia?

Hướng dẫn

1. ∆m 1 = m e + (m Ru – 44m e ) – (m Tc – 43m e ) = m Ru – m Tc = -1,7.10 -3 (u)

E = -1,7.10 -3 .931,5 = -1,58355 (MeV)

= -1,58355.10 6 .1,602.10 -19 .10 -3 .6,022.10 23 = -152,77.10 6 (kJ/mol)

2. ∆m 2 = m Ru – 44(m e + m p ) – m n .57 = -0,9357 (u)

E = -0,9357.931,5 = -871,60455 (MeV)

= -871,60455.10 6 .1,602.10 -19 . 10 -3 .6,022.10 23 = -8,4086.10 10 (kJ/mol)

2016.3,7.10

16

3. N 101

9,233.10 (nguyên tử)

Tc

ln 2 / (14,3.60)

(g)

= m Tc bị phân rã

4. A A 308

(1)

(1), (2)

16

10

43 44 1

7,48641.10

m

Tc bd

.100,9073 1,5471.10

23

6,022.10

14,3 m

ln 2 m x

Tc bd

7

1 ln x 7,3202.10 (g)

Tc bd

0 0

101 104

Tc Tc

A A 160,462

101 104

Tc Tc

0 (ln 2/14,3).14,3 0 (ln 2/18,3).14,3

101

Tc

104

Tc

A .e A .e 160,462

0 0

101

104

Tc

Tc

A 228,9962(Ci);A 79(Ci)

5

(2)

Phát hành PDF bởi Ths Nguyễn Thanh Tú

Đăng ký Word doc qua Zalo 0905779594 Email thanhtuqn88@gmail.com

49


A 101

Tc

* TH1: 2

A

Tc

104

228,9962.e

79.e

ln 2 .t

18,3

ln 2 .t

14,3

2 t 35,02715(phút)

A

18,3

104

Tc

79.e

* TH2: 2

A 101

228,9962.e

Bài 2

a. Đồng vị phóng xạ 58 Co của Coban có chu kì bán rã 71,3 ngày đêm. Tính độ phóng xạ

của 1 g chất đó theo đơn vị beccơren và curi.

b. Một miligam hỗn hợp của 58 Co với đồng vị phóng xạ 59 Co có độ phóng xạ 2,2.10 10 Bq.

Tính khối lượng của mỗi đồng vị.

Cho biết: số Avogadro N A = 6,022.10 26 nguyên tử/k.mol

Hướng dẫn

m

a) Số N nguyên tử chứa trong 1 g Coban 58 là: N = N A .

M

6,022.10 .1.10

58

26 9

5

ln 2 0,693 693.10

Hằng số rã (hay hằng số phóng xạ) của 58 Co là:

T 71,3.86400 71,3.864

Vậy, độ phóng xạ của 1 g = 10 -9

693.10 6,022.10

kg Coban 58 là: H = . N .

71,3.864 58

H 1,17.10 9 Bq

1,17.10

Theo đơn vị curi, ta có: H =

3,7.10

b) Khối lượng của 58 Co có chứa trong hỗn hợp là:

m 1 =

10

2, 2.10

1,167.10

9

18,851... g hay m 1 18,85 g

Khối lượng của 59 Co trong hỗn hợp là:

9

10

0,0316 hay H 0,032 Ci

m 2 = 1000 – m 1 = 1000 – 18,85 hay m 2 = 981,15 g

Bài 3

Tc

ln 2

.t

ln 2

14,3

.t

2 t 165,872(phút)

5 17

Đồng vị phóng xạ Co-60 được sử dụng để nghiên cứu hiệu quả hấp thụ vitamin-B12, vì

Co là nguyên tử trung tâm của vitamin-B12. Co-60 được tổng hợp quá ba bước, quá trình

tổng có thể hiểu như là phản ứng của Fe-58 với 2 hạt nơtron sinh ra Co-60 và một vi hạt

khác.

1. (a) Vi hạt này là gì?

Phát hành PDF bởi Ths Nguyễn Thanh Tú

Đăng ký Word doc qua Zalo 0905779594 Email thanhtuqn88@gmail.com

50


(b) Bước sóng deBroglie của hạt này bằng bao nhiêu, nếu vận tốc chuyển động của hạt

này bằng 0,90c (c là vận tốc ánh sáng)?

(c) Năng lượng liên kết hạt nhân riêng của Co-60 bằng bao nhiêu? Biết 60 Co =

59,9338u và 1 H = 1,00782u.

2. (a) Cho biết cấu hình electron của nguyên tố chuyển tiếp Bori (Bh, Z = 107).

(b) Quá trình tổng hợp Bh liên quan đến phản ứng bắn phá Bk-247 bằng Ne-22 sinh ra

Bh-267. Viết phương trình phản ứng hạt nhân.

(c) Chu kỳ bán hủy của Bh-267 là 15,0 giây. Nếu tổng hợp được 199 nguyên tử Bh-267

thì sau bao lâu chỉ còn lại 11 nguyên tử?

Hướng dẫn

1. (a) 58

26 Fe + 2 0 1 60

Ne 27 Co +

0 1 e

(b)

34

h 6,626.10 Js

12

2,7.10 m

mv

31 8

9,109.10 kg

0,90

2,998.10 m / s

(c) m (59,9338 27m e) [27(1,00782 m e) 331,00866] 0,5631

E

r

12

2

27 8

0,5931u 1,6605.10 kg / u 2,998.10 m / s

60nucleon

1,401.10 J / nucleon

2. (a) Cấu hình electron: [Rn]5f 14 6d 5 7s 2

(b) Phản ứng: 247 22 267

97 Bk + 10Ne 107 Bh + 4 0 1 n

(c) Từ

Bài 4

199 ln 2

ln t t 62,7 (giây)

11 15

Đồng vị 60 Co (t 27 1/2 = 5,33 năm) được dùng trong y tế, phân rã trước tiên thành 60

28

Ni . Giả

sử 60

58

28

Ni tiếp tục phân rã thành đồng vị bền Ni . 28

a. Viết các phương trình phản ứng hạt nhân xảy ra.

b. Tính khối lượng 60 Co để có hoạt độ phóng xạ 10 Ci. (Biết 1 Ci (Curi)= 27 3,7.1010 phân

rã/ giây)

c. Sau khoảng thời gian t, mẫu phóng xạ có tỉ lệ khối lượng 58

60

28

Ni so với

27

Co là 0,9 (coi

trong mẫu không có sản phẩm trung gian). Tính t theo năm.

Hướng dẫn

Phát hành PDF bởi Ths Nguyễn Thanh Tú

Đăng ký Word doc qua Zalo 0905779594 Email thanhtuqn88@gmail.com

51


b) Chọn t = 0 là thời điểm mẫu phóng xạ là 10 Ci, ta có hoạt độ phóng xạ ban đầu:

Ao = No =

Ta có

Với số nguyên tử ban đầu:

c) Tính t. Số nguyên tử Ni tạo thành sau thời gian t bằng số nguyên tử Co bị phân rã (∆N)

Khối lượng niken sinh ra trong thời gian t:

Mặt khác, khối lượng coban còn lại sau thời gian t phân rã là: m Co =m o .e -λt (2)

Từ (1) và (2) ta có:

Suy ra:

III.5 Cân bằng trong dung dịch điện li

Bài 1: a) Tính pH của các dung dịch FeCl 2 và FeCl 3 0,10M

Cho biết K a (Fe 2+ ) = 3,0 10 -10 ; K a (Fe 3+ ) = 4,0 10 -3

b) Giải thích sự khác nhau về độ pH đó?

Hướng dẫn:

a) - Tính pH của dung dịch FeCl 2 0,1M

FeCl 2 Fe 2+ + 2Cl -

Trong nước, ion Fe 2+ 2+

ngậm 6 phân tử nước Fe(H 2 O) 6 nhưng để đơn giản ta viết Fe 2+

Fe 2+ + 2H 2 O [Fe(OH)] + + H 3 O +

K a = 3.10 -10 2

x

=

CB: (0,1-x)M xM xM

0,1 x

Giải ra x = 5,48.10 -6 = [H 3 O + ] pH = 5,26

- Tính pH của dung dịch FeCl 3 0,1M

3+

Tương tự như trên, thay vì Fe(H 2 O) 6 , ta viết Fe 3+

Phát hành PDF bởi Ths Nguyễn Thanh Tú

Đăng ký Word doc qua Zalo 0905779594 Email thanhtuqn88@gmail.com

52


Fe 3+ + 2H 2 O [Fe(OH)] 2+ + H 3 O +

K a = 4.10 -3 2

x

=

CB: (0,1-x)M xM xM

0,1 x

Giải ra x = 6,32.10 -2 = [H 3 O + ] pH = 1,2

b) Qua kết quả tính toán trên ta thấy pH của dung dịch FeCl 3 thấp hơn pH của dung dịch

FeCl 2 .

Sở dĩ như vậy là do trong phức hidrat [Fe(H 2 O) 6 ] n+ , ion trung tâm Fe 3+ có điện tích 3+ ,

có bán kính r=0,64 A 0 ; ion trung tâm Fe 2+ có điện tích 2+ , có bán kính r=0,76 A 0 ; ion Fe 3+

có điện tích lớn hơn, bán kính nhỏ hơn ion Fe 2+ nên đẩy ion H + ra khỏi phân tử nước

mạnh hơn.

Bài 2: Dựa vào thế điện cực hãy chứng tỏ rằng trong môi trường axit, trạng thái oxi hóa

+2 của sắt bền hơn trạng thái oxi hóa +3 và ngược lại, trong môi trường bazơ, trạng thái

oxi hóa +3 bền hơn.

Cho biết:

Trong môi trường axit:

Fe 3+ (aq) + e Fe 2+ (aq)

O 2 (k) + 2H + (aq) + 2e H 2 O 2 (l)

E 0 = + 0,77V

E 0 = + 0,68V

Trong môi trường bazơ

2Fe(OH) 3 (r) + 2e 2Fe(OH) 2 (r) +2OH -

E 0 = - 0,56V

½ O 2 (k) + H 2 O (l) + 2e 2OH - (aq)

E 0 = + 0,40V

Hướng dẫn:

* Trong môi trường bazơ

2Fe(OH) 2 (r) +2OH - 2Fe(OH) 3 (r) + 2e

E 0 = + 0,56V

½ O 2 (k) + H 2 O (l) + 2e 2OH - (aq)

E 0 = + 0,40V

0

2Fe(OH) 2 + ½ O 2 (k) + H 2 O 2Fe(OH) 3 (r) E

Phản ứng có khả năng tự diễn biến vì

G

nFE mà

0 0

pu

pu

pu

0

pu

0,96V

E 0 nên

0

Trên thực tế, Fe(OH) 2 màu xanh nhạt mới điều chế, khi tiếp xúc với không khí bị oxi oxi

hóa thành Fe(OH) 3 màu nâu.

* Trong môi trường axit

2Fe 2+ (aq) 2Fe 3+ (aq) + 2e

E 0 = - 0,77V

O 2 (k) + 2H + (aq) + 2e H 2 O 2 (aq)

E 0 = + 0,68V

2Fe 2+ (aq) + O 2 (k) + 2H + 2Fe 3+ 0

(aq) + H 2 O 2 E

pu

0,09V

0

G pu

Phát hành PDF bởi Ths Nguyễn Thanh Tú

Đăng ký Word doc qua Zalo 0905779594 Email thanhtuqn88@gmail.com

53


Phản ứng không có khả năng tự diễn biến vì

G

nFE mà

0 0

pu

pu

E 0 nên

0

pu

0

Ở điều kiện chuẩn, trong môi trường axit, Fe (II) không bị oxi trong không khí hay oxi

hòa tan trong nước oxi hóa.

* Kết luận: Trong môi trường axit Fe (II) bền hơn Fe (III) và ngược lại, trong môi trường

bazơ, Fe (III) bền hơn Fe (II)

Bài 3

a. Hãy tính pH của dung dịch FeCl 3 0,010 M. Biết cation Fe(H 2 O) 3+ 6 là một đơn axit có

K a =6,3.10 –3

b. Hãy tính giá trị pH cần thiết lập để dung dịch trên bắt đầu có kết tủa Fe(OH) 3 xuất

hiện. Biết tích số tan của Fe(OH) 3 K sp = 6,3 .10 –38 .

c. Hãy tính giá trị pH tại đó sắt được kết tủa hoàn toàn từ 100,0 ml dung dịch FeCl 3 0,010

M dưới dạng Fe(OH) 3 . Biết rằng quá trình kết tủa được coi là hoàn toàn khi khối lượng

sắt còn lại trong dung dịch nhỏ hơn 0,2 mg. Cho Fe = 55,85.

Hướng dẫn

a) Fe(H 2 O) 3+ 6 + H 2 O [Fe(H 2 O) 5 (OH)] 2+ + H 3 O +

0

G pu

Có:

=> [H 3 O + ] = 5,39.10 -3 M (pH = 2,27)

Kiểm tra: Q = [Fe 3+ ][OH - ] 3 = (0,01 - 5,39.10 -3 ).(1,855.10 -12 ) 3 = 2,94.10 -38 < K sp ,

không có kết tủa xuất hiện.

b) Fe 3+ + H 2 O Fe(OH) 2+ + H + (1)

Fe 3+ + 3 OH - Fe(OH) 3 ↓ (2)

Có: [Fe 3+ ] + [Fe(OH) 2+ ] = 0,01 (3)

K w =[H + ][OH - ] = 10 -14 (4)

(2), (4) =>

=> (5)

(1), (3), (5) =>

=> 6,3.10 4 [H + ] 3 + 396,9 [H + ] 2 - 0,01 = 0

Phát hành PDF bởi Ths Nguyễn Thanh Tú

Đăng ký Word doc qua Zalo 0905779594 Email thanhtuqn88@gmail.com

54


=> [H + ] = 3,938 .10 -3 M (pH = 2,40)

c) Fe 3+ + H 2 O Fe(OH) 2+ + H + (1)

Fe 3+ + 3 OH - Fe(OH) 3 ↓ (2)

Có: (3)

K w =[H + ][OH - ] = 10 -14 (4)

(2), (4) =>

=> (5)

(1), (3), (5) =>

=> 6,3.10 4 [H + ] 3 + 396,9 [H + ] 2 - 3,58.10 -5 = 0

=> [H + ] = 2,936 .10 -4 M (pH = 3,53)

Bài 4 Dung dịch bão hòa H 2 S có nồng độ 0,100 M.

a) Một dung dịch A chứa các cation Mn 2+ , Co 2+ và Ag + với nồng độ ban đầu của mỗi ion

đều bằng 0,010 M. Hòa tan H 2 S vào dung dịch A đến bão hòa và điều chỉnh pH = 2,0 sẽ

thu được kết tủa nào?

b) Khi điều chỉnh pH của dung dịch sau bằng bao nhiêu thì bắt đầu xuất hiện kết tủa

MnS?

Cho: Hằng số axit của H 2 S: K 1 = 1,0⋅10 -7 và K 2 = 1,3 ⋅10 -13 .

T MnS = 2,5⋅10 -10 ; T CoS = 4,0⋅10 -21 ; T Ag S = 6,3 ⋅ 10 .

Hướng dẫn

a) Các cân bằng trong dung dịch:

H 2 S ⇌ H + + ,

⇌ H + + ,

→ H 2 S ⇌ 2 H + + , K = ⋅

H 2 O ⇌ H +

→ [ ]= [ ]⋅ ⋅

[ ]

+ OH , K w

Phát hành PDF bởi Ths Nguyễn Thanh Tú

Đăng ký Word doc qua Zalo 0905779594 Email thanhtuqn88@gmail.com

55


Tại pH = 2,0 → [H + ] = 1⋅10 -2 ; coi [H 2 S] = [ ] = 1,3⋅10 -17 M.

Ta có:

o [Ag + ] 2 [ ] = 0,01 2 ⋅1,3⋅10 -17 = 1,3⋅10 -21 > 6,3 ⋅ 10 → có tủa Ag 2 S.

o [Co 2+ ][

] = 0,01⋅1,3⋅10 -17 = 1,3⋅10 -19 > 4⋅10 -21 → có tủa CoS.

[Mn 2+ ][

] = 0,01⋅1,3⋅10 -17 = 1,3⋅10 -19 < 2,5⋅10 -10 → không có tủa MnS.

b)

Để xuất hiện kết tủa MnS: [Mn 2+ ][ ] = T MnS = 2,5⋅10 -10

→ [ ] = , ⋅

[ ]

= 2,5 ⋅ 10 M

khi [

⇔ [ ]⋅ ⋅

[ ]

] ≥ 2,5⋅10 -8 thì có tủa MnS

≥ 2,5 ⋅ 10

→ [ ]≤ 2,28 ⋅ 10 → ≥ 6,64.

Vậy, khi điều chỉnh pH ≥ 6,64 thì bắt đầu xuất hiện tủa MnS.

III.6. Phản ứng oxi hóa khử và bài tập phần Điện hóa

Bài 1: Cho thế khử chuẩn của một số cặp oxi hóa – khử của Coban như sau:

Co 3+ + e Co 2+

E 0 = 1,808 V

Co(OH) 3 (r) + e Co(OH) 2 (r) + OH - (aq) E 0 = 0,17 V

[Co(NH 3 ) 6 ] 3+ + e [Co(NH 3 ) 6 ] 2+

E 0 = 0,108 V

[Co(CN) 6 ] 3- + e [Co(CN) 5 ] 3- + CN - E 0 = -0,83 V

O 2 (k) + 4H + (10 -7 M) + 4e 2H 2 O (l)

2H 2 O (l) + 2e H 2 (k) + 2OH -

E 0 = 0,815 V

E 0 = -0,828V

Trong số các chất nêu trên, chất nào oxi hóa được nước?

Chất nào khử được nước? Giải thích vì sao?

Hướng dẫn:

a) Ion Co 3+ oxi hóa được nước:

4 Co 3+ + e Co 2+ E 0 = 1,808 V

2H 2 O (l) O 2 (k) + 4H + + 4e E 0 = - 0,815 V

4Co 3+ + 2H 2 O 4Co 2+ + O 2 (k) + 4H + E

0 1,013V

pu

Phát hành PDF bởi Ths Nguyễn Thanh Tú

Đăng ký Word doc qua Zalo 0905779594 Email thanhtuqn88@gmail.com

56


Phản ứng có thể xảy ra vì

G

nFE mà

0 0

pu

pu

E 0 nên

0

pu

0 : phản ứng tự diễn

biến: Co 3+ là một chất oxi hóa rất mạnh.

b) Phức pentaxianocobanat (II) có khả năng khử được nước

2 [Co(CN) 5 ] 3- + CN - [Co(CN) 6 ] 3- + e E 0 = 0,83 V

2H 2 O (l) + 2e H 2 (k) + 2OH -

E 0 = -0,828V

2[Co(CN) 5 ] 3- + 2CN - + 2H 2 O 2[Co(CN) 6 ] 3- + H 2 + 2OH - E

0 0,002V

Phản ứng có thể xảy ra vì

G

nFE mà

0 0

pu

pu

E 0 nên

0

pu

0

G pu

0

G pu

pu

0 : phản ứng tự diễn

biến: phức pentaxianocobanat (II) là một chất khử mạnh.

Bài 2

Tính nồng độ ban đầu của HSO - 4 (K a = 10 -2 ), biết giá trị sức điện động của pin:

PtI - 0,1 M; I - 3 0,02 MMnO - 4 0,05 M, Mn 2+ 0,01 M, HSO - 4 C MPt

ở 25 o C đo được bằng 0,824 V. Cho E o 1,51V và E o 0,5355V.

2

MnO 4 / Mn

Hướng dẫn

Ở điện cực phải: MnO - 4 + 8H + + 5e ⇌ Mn 2+ + 4H 2 O

Ở điện cực trái: 3I - ⇌ I - 3 + 2e

I 3 / I

E

o

p E 2

MnO 4 / Mn

0,059 [MnO 4 ][H

lg

2

5 [Mn ]

]

8

0,059 0,05[H ]

1,51 lg

5 0,01

8

E

t

E

0,059

ΔE = E phải - E trái 0,824 = 1,51 + lg(5[H + ] 8 ) – 0,574

5

Suy ra h = [H + ] = 0,053 M

Mặt khác từ cân bằng:

H 2 SO - 4 ⇌ H + 2-

+ SO 4

[ ] C – h h h

Suy ra

o

I3

/ 3I

2

0,059

2

h

K

C h

a

[I3

]

lg

3

[I ]

h

K

0,059 0,02

0,5355 lg

3

2 (0,1)

K a = 10 -2

Thay giá trị h = 0,053 và K a = 1,0.10 -2 , tính được

2

a

h C

0,574V

Bài 3

1. Cho giản đồ thế khử chuẩn của Mn trong môi trường axit:

C

HSO

4

0,334M

Phát hành PDF bởi Ths Nguyễn Thanh Tú

Đăng ký Word doc qua Zalo 0905779594 Email thanhtuqn88@gmail.com

57


+1,7V

a. Tính thế khử chuẩn của cặp:

MnO /MnO

+1,23V 3

2

và Mn /Mn

- 2-

4 4

b. Hãy cho biết các tiểu phân nào không bền với sự dị phân. Hãy tính hằng số cân bằng

của các phản ứng dị phân đó.

2.a. Hãy biểu diễn sơ đồ pin, tính sức điện động của pin và viết phương trình phản ứng

3+ 2+

xảy ra trong pin (khi pin hoạt động) được tạo thành từ các cặp điện cực Fe /Fe và

2+

Cu /Cu ở điều kiện chuẩn.

b. Tính nồng độ các ion còn lại trong dung dịch khi pin ngừng hoạt động. Giả sử nồng độ

ban đầu của ion có trong dung dịch làm điện cực pin đều bằng 0,010M (Bỏ qua quá trình

thuỷ phân của các ion).

E =0,771V , E 0

= -0,440V , E 0

=0,337V , E 0

=0,521V , RT

2+

2+

+

=0,059

Fe /Fe Fe /Fe Cu /Cu Cu /Cu

F

25 0 C

0

Cho 3+ 2+

-37

K

s,Fe(OH)

=10

3

s,Fe(OH) 2

Hướng dẫn:

1a

2. a.

Thế khử chuẩn của cặp:

2- +

4 2 2

K =10

- 2-

4 4

MnO +4H +2e MnO +2H O (1)

MnO +4H +3e MnO +2H O (2)

- +

4 2 2

Lấy (2) trừ (1) ta có:

-15,1

3

2

MnO /MnO và Mn /Mn

MnO +e MnO (3)

- 2

4 4

ΔG =ΔG -ΔG

0 0 0

3 2 1

0 0 0

3 2 1

0

3

-FE =-3FE -(-2FE )

E =+0,56V

MnO +e+4H Mn +2H O (4)

+ 3+

2 2

MnO +2e+4H Mn +2H O (5)

+ 2+

2 2

Lấy (5) trừ (4) ta có:

3+ 2+

Mn +e Mn (6)

tại

Phát hành PDF bởi Ths Nguyễn Thanh Tú

Đăng ký Word doc qua Zalo 0905779594 Email thanhtuqn88@gmail.com

58


ΔG =ΔG -ΔG

0

6

0 0 0

6 5 4

-FE =-2FE -(-FE )

0 0 0

6 5 4

E =+1,5V

b. Trả lời đúng một tiểu phân không bền được 0,5 điểm. Tính đúng một giá trị K được

0,25 điểm.

2

MnO 3+

4

và Mn không bền với sự dị phân.

MnO +4H +2e MnO +2H O

2- +

4 2 2

2MnO

2MnO +2e

2 -

4 4

2 +

4 4 2 2

0

E

1

=+2,27V

0

-E

3

=-0,56V

3MnO +4H 2MnO +MnO +2H O (7)

0 0 0 0

ΔG

7

=ΔG1 -ΔG

3

=-2FΔE

7

=-3,42F<0 nên phản ứng (7) tự diễn biến.

0

2ΔE7

lgK

7

= 57,966

0,059 K 7 = 9,25.10 57

Mn +e Mn

3+ 2+

0

E

6

=+1,51V

Mn +2H O MnO +e+4H

3+ +

2 2

3+ 2 +

2 2

0

-E

4

=+0,95V

2Mn +2H O MnO +Mn +4H (8)

0 0 0 0

ΔG

8

=ΔG6-ΔG 4=-FΔE 8

=-0,56F<0 nên phản ứng (8) tự diễn biến.

0

ΔE8

lgK

8

= 9, 492

0,059 K 8 = 3,1.10 9

2. b. Sơ đồ pin: (-) Cu Cu 2+ (1M) Fe 3+ (1M), Fe 2+ (1M) Pt (+)

o 0 0

Sức điện động chuẩn của pin: E = E 3+ 2+ - E 2+ =0,434V

Pin Fe /Fe Cu /Cu

Phản ứng xảy ra khi pin hoạt động:

2Fe 3+ + Cu ↔ 2 Fe 2+ + Cu 2+ K =10 14,66

Phát hành PDF bởi Ths Nguyễn Thanh Tú

Đăng ký Word doc qua Zalo 0905779594 Email thanhtuqn88@gmail.com

59


b. K rất lớn nên xem như phản ứng xảy ra hoàn toàn.

2Fe 3+ + Cu → 2 Fe 2+ + Cu 2+

C 0 0,01 0,01 0,01

C 0 0,02 0,015

Xét cân bằng:

2 Fe 2+ + Cu 2+ ↔ 2Fe 3+ + Cu K -1 =10 -14,66

C 0 0,02 0,015 0

CB 0,02 -2x 0,015 - x 2x

K CB = K -1 =10 -14,66 =

(2 x) (2 x)

x x

2 2

2 2

(0,02 2 ) .(0,015 ) 0,02 .0,015

(do K CB bé nên giả thiết x<<0,02)

X = 5,714.10 -11 <<0,02

[Fe 3+ ] = 2x = 1,189.10 -10 [Fe 2+ ] = 0,02-2x = 0,02 [Cu 2+ ] = 0,015-2x =

0,015

Bài 4:

1. Bổ túc và cân bằng các phản ứng oxi hóa khử sau:

a. CuFeS 2 + Fe 2 (SO 4 ) 3 + O 2 + H 2 O CuSO 4 + FeSO 4 + H 2 SO 4

( theo phương pháp thăng bằng electron)

b. Cr 2 S 3 + Mn(NO 3 ) 2 + Na 2 CO 3 Na 2 CrO 4 + Na 2 SO 4 + Na 2 MnO 4 + NO + CO 2

( theo phương pháp thăng bằng electron)

c. Fe x O y + H + + SO 2- 4 SO 2 + …

( theo phương pháp thăng bằng ion - electron)

d. Cu 2 S + HNO 3 …+ NO + SO 2- 4 + …

( theo phương pháp thăng bằng ion - electron)

2. Pin Ni-Cd (“Nicad”) được sử dụng rộng rãi trong các loại thiết bị bỏ túi như điện thoại

di động, máy quay phim xách tay, laptop, v.v… Pin Ni-Cd có giá vừa phải, chu trình

sống cao đồng thời có thể hoạt động được trong một khoảng nhiệt độ rất rộng. Nó không

cần phải được bảo dưỡng và có thể được nạp điện 2000 lần. Một tế bào của pin Ni-Cd

thực hiện hai nửa phản ứng sau:

Cd(OH) 2(r) + 2e → Cd (r) + 2OH -

E o 1 = -0,809 (V)

2NiO(OH) + 2H 2 O + 2e → 2Ni(OH) 2(r) + 2OH - E o 2 = 0,490 (V)

E o 1; E o 2 là thế khử chuẩn ở 25 o C.

Phát hành PDF bởi Ths Nguyễn Thanh Tú

Đăng ký Word doc qua Zalo 0905779594 Email thanhtuqn88@gmail.com

60


a) Phản ứng nào xảy ra ở catot ? ở anot ? Viết phản ứng chung cho pin điện và

phương trình Nernst ở mỗi điện cực. Tính E o pin ?

b) Tính khối lượng Cd chứa trong 1 chiếc điện thoại di động có sử dụng pin Ni-

Cd. Biết công suất thông thường của pin là 700 mAh. ( Cho M Cd = 112 g/mol)

Hướng dẫn

1a. 2CuFeS 2 + 2Fe 2 (SO 4 ) 3 + 7O 2 +2 H 2 O 2CuSO 4 + 6FeSO 4 + 2H 2 SO 4

CuFeS 2 + Fe +3 2 Cu 2+ + 3Fe 2+ + 2S +6 + 14e 2

O 2 + 4e 2O 2- 7

b. Cr 2 S 3 + 15 Mn(NO 3 ) 2 + 20Na 2 CO 3 2Na 2 CrO 4 + 3Na 2 SO 4 + 15Na 2 MnO 4 + 30NO

+ 20CO 2

Cr 2 S 3 2Cr +6 + 3S +6 + 30e 1

Mn 2+ + 2N +5 + 2e Mn +6 + 2N +2 15

c.2 Fe x O y + (12x-4y)H + 2-

+(3x-2y) SO 4 (3x-2y) SO 2 + 2xFe 3+ + (6x-2y) H 2 O

Fe x O y + 2yH + xFe 3+ + yH 2 O +(3x-2y)e 2

4H + + SO 2- 4 + 2e SO 2 + 2H 2 O

(3x-2y)

d. 3Cu 2 S + 16H + +10 NO - 3 6Cu 2+ + 10NO + 3SO 2- 4 +8H 2 O

Cu 2 S + 4H 2 O 2Cu 2+ + SO 2- 4 + 8H + + 10e 3

4H + + NO - 3 + 3e NO + 2 H 2 O 10

2.

a. Cực anod: Cd (r) + 2OH - → Cd(OH) 2(r) + 2e E o a = - 0,809 V

Cực catod : 2NiO(OH) + 2H 2 O + 2e → 2Ni(OH) 2(r) + 2OH - E o c = 0,490 V

Phương trình phản ứng chung :

2NiO(OH) + 2H 2 O + Cd (r) → 2Ni(OH) 2(r) + Cd(OH) 2(r )

Phương trình Nerst :

E

a

o 0,059 lg[OH

]

2

0,059 1

E

a

; Ec

E o

c

lg

2

2

2 [OH ]

E o pin = E o c - E o a = 0,490 – (- 0,809) = 1,299 V

b. 700mAh=0,7.3600=2520C

2520

nCd

0,013 mol nên m Cd = 0,013.112 =1,456 gam

2.96500

Bài 5

Phát hành PDF bởi Ths Nguyễn Thanh Tú

Đăng ký Word doc qua Zalo 0905779594 Email thanhtuqn88@gmail.com

61


Đem điện phân 100ml dung dịch X gồm NiCl 2 0,20M và MCl 2 0,25 M với điện cực trơ,

có cường độ dòng điện một chiều không đổi là 9,65 M. Sau thời gian 10 phút thấy catot

tăng lên 1,734 gam và dung dịch sau điện phân chỉ có một chất tan. Nhỏ 100ml dung dịch

gồm K 2 Cr 2 O 7 0,50M và H 2 SO 4 2M vào 100ml dung dịch MCl 2 0,60M, sau phản ứng

hoàn toàn thu được dung dịch Y.

a) Xác định muối MCl 2 .

b) Thiết lập một pin điện tạo bởi điện cực Pt nhúng trong dung dịch Y với điện cực Ag

nhúng trong dung dịch [ Ag(NH 3 ) 2 ]NO 3 0,50M, KCN 2,10M. Viết các bán phản ứng ở

mỗi điện cực, phản ứng khi pin phóng điện và suất điện động của pin mới được thiết lập.

Cho: E 0 (Cr 2 O 2- 7 /Cr 3+ )= 1,33 V; E 0 (Fe 3+ /Fe 2+ )= 0,77 V; E 0 (Ag + /Ag)= 0,80V;

β[Ag(CN) 3- 4 ] = 10 -20,67 , β[Ag(NH 3 ) + 2 ] = 10 -7,23

Hướng dẫn

1 a) Các bán phản ứng trên điện cực:

Ở anot: 2 Cl - → Cl 2 + 2e

Ở Catot: M 2+ + 2e → M hoặc 2 Ni 2+ + 2e → Ni

Theo bài cho có tổng số mol e thu vào là: n e = I.t/F = 9,65.10.60/9650= 0,06 mol

Ta xét hai trường hợp:

TH 1: Nếu ion M 2+ bị điện phân hết, gọi số mol ion Ni 2+ có thể bị điện phân là x:

Ta có n e = 0,025.2 + x. 2 = 0,06 mol → x= 0,005

→ m = M.0,025 + 58,7.0,005 = 1,734→ M= 57,6 ( loại)

TH 2: Nếu ion Ni 2+ bị điện phân hết, gọi số mol ion M 2+ có thể bị điện phân là y:

Ta có n e = 0,02.2 + y. 2 = 0,06 mol → y= 0,005

→ m = M.0,01 + 58,7.0,02 = 1,734 → M= 56 ( Fe)

b) Với dung dịch loãng chỉ xét phản ứng của ion Fe 2+ với Cr 2 O 2- 7 /H +

6 Fe 2+ + Cr 2 O 2- 7 + 14H + → 6 Fe 3+ + Cr 3+ + 7H 2 O (1)

n 0 0,06 0,05 0,4

n s 0,0 0,04 0,26 0,06 0,02

Thế điện cực Platin

Dung dịch Y gồm các ion : Fe 3+ , H + (0,26/0,2=1,3M); Cr 3+ 2-

(0,02/0,2=0,1M); Cr 2 O 7

(0,04/0,2=0,2M); K + ; Cl - ; và SO 2- 4 .

Khi nhúng thanh Pt vào dung dich Y: cặp oxi hóa khử là Cr 2 O 2- 7 /Cr 3+

Ta có: E 0 (Cr 2 O 2- 7 /Cr 3+ )= 1,33 + (0,0592/6)lg(0,2.1,3 14 /0,1 2 )= 1,36 (V).

Thế điện cực Ag.

Phát hành PDF bởi Ths Nguyễn Thanh Tú

Đăng ký Word doc qua Zalo 0905779594 Email thanhtuqn88@gmail.com

62


Xét cân bằng:

- Cân bằng tạo phức bền:

Ag(NH 3 ) 2 ] + + 4CN - ↔ [Ag(CN) 4 ] 3- + 2NH 3 K=(10 7,23 ) -1 .10 20,67 =10 13,44 >>

C 0 0,5 2,1

C - 0,1 0,5 1

- Cân bằng tạo phân li phức :

[Ag(CN) 4 ] 3- Ag + + 4CN - β -1 = 10 -20,67 .

C 0 0,5 1

[] (0,5-x) x 0,1+4x

β -1 = x.(0,1+4x) 4 /(0,5-x) = 10 -20,67 ; Giả sử x<< 0,1 => x = 10 -16,97 (t/m)

Vậy E [Ag(CN) 4 ] 3- /Ag + = 0,80 + (0,0592/1)lg10 -16,97 = -0,20V

Do E(Cr 2 O 2- 7 /2Cr 3+ ) = 1,35V > E Ag(CN) 3- 4 /Ag = -0,20V

Sơ đồ pin là:

(-) Ag│Ag(CN) 4 ] 3- 0,5M; CN - 2-

0,1M; NH 3 1M ││Cr 2 O 7 0,2M; Cr 3+ 0,1M; H + 1,3M

│Pt(+)

Các bán phản ứng:

Ở cực âm(-): Ag + 4CN - → [Ag(CN) 4 ] 3- + 1e

2-

Ở cực âm(+): Cr 2 O 7 + 6e + 14H + → 2Cr 3+ + 7H 2 O

Phản ứng khi phóng điện:

6Ag + 10CN - 2-

+ Cr 2 O 7 + 14HCN → 6 [Ag(CN) 4 ] 3- + 2Cr 3+ + 7H 2 O

Suất điện động của pin là:

E pin = E (+) - E (-) = 1,36 – (-0,20) = 1,56 (V)

Bài 6. Một pin được cấu tạo bằng cách nối nửa pin A gồm Ni nhúng trong 100 cm 3 Ni 2+

chưa biết nồng độ và nửa pin B gồm Cu nhúng trong 100 cm 3 dung dịch Cu 2+ 0,100 M

qua một cầu muối. Điện thế đo được của hệ là 0,01 mV. Nhiệt độ làm việc là 25 o C. Thêm

một ít CuCl 2 vào dung dịch Cu 2+ làm cho thế của hệ tăng thêm 9,00 mV (bỏ qua thể tích

tăng thêm khi cho thêm CuCl 2 ). Hãy xác định khối lượng CuCl 2 thêm vào (M CuCl2 =

134,45 g.mol -1 . E o (Ni 2+ /Ni) = -0,257V, E o (Cu 2+ /Cu) = 0,342V).

Hướng dẫn:

Phát hành PDF bởi Ths Nguyễn Thanh Tú

Đăng ký Word doc qua Zalo 0905779594 Email thanhtuqn88@gmail.com

63


Pin có thể được biểu diễn như sau:

Ni | Ni 2

( c ) || ( c ) Cu 2

| Cu

2

2

Ni Cu

Thế chuẩn của pin là E

o 0.

599 V .

với || ký hiệu cho cầu muối.

Thế của pin trước khi thêm CuCl 2 được biểu diễn như sau:

R T c

o

2

Ni

E E ln

2 F c

Sau khi thêm

CuCl 2

E'

thì thế mới của pin được biểu diễn như sau:

R T c

' o

2

Ni

E E ln

'

2 F c

'

Với

c

là nồng độ mới của Cu 2+ Cu

2

. Thế tăng một đại lượng,

E E' E

ứng với lượng

CuCl 2 thêm vào được biểu diễn dưới dạng:

E E'

E

R T 2 F

Như vậy không cần phải biết nồng độ của dung dịch Ni 2+ để xác định sự thay đổi thế của

pin sau khi thêm CuCl 2 .

2

Cu

2

Cu

'

c

ln

c

2

Cu

2

Cu

Biến đổi biểu thức thức trên thu được biểu thức cho phép xác định nồng độ mới của Cu 2+ :

'

c Cu

2

c Cu

2

exp 2 E F

0.020 M

R T

Từ đây tính được biến thiên nồng độ sau khi thêm CuCl 2 là

CuCl 2 thêm vào dung dịch là 1.10 -3 mol.

m

Vậy khối lượng CuCl 2 thêm vào: CuCl 2

n CuCl 2

M CuCl 2

1.344 g

.

Bài 7. Một pin điện tạo bởi : Một điện cực gồm tấm Cu nhúng trong dung dịch CuSO4

0,5 M, điện cực thứ hai là một dây Pt nhúng trong dung dịch Fe 2+ , Fe 3+ với lượng

[Fe 3+ ] =2[Fe 2+ ] và một dây dẫn nối Cu với Pt.

a) Viết sơ đồ pin, phản ứng điện cực và tính sức điện động ban đầu của pin.

c

0 010

Cu 2

.

M

và số mol

Phát hành PDF bởi Ths Nguyễn Thanh Tú

Đăng ký Word doc qua Zalo 0905779594 Email thanhtuqn88@gmail.com

64


3

b) Cho rằng thể tích dung dịch CuSO4 khá lớn, xác định tỷ số

Fe

khi pin ngừng hoạt

2

Fe

động.

c) Trộn ba dung dịch: 25 ml Fe(NO 3 ) 2 0,1 M ; 25 ml Fe(NO 3 ) 3 1,0 M ; 50 ml AgNO 3 0,6

M và thêm một số mảnh Ag vụn. Xác định chiều phản ứng và tính giá trị tối thiểu của tỷ

3

số

Fe

để phản ứng đổi chiều?

2

Fe

Hướng dẫn

a) Cho : E 0 (Cu 2+ /Cu) = 0,34 V ; E 0 (Fe 3+ /Fe 2+ ) = 0,77 V ; E 0 (Ag + /Ag) = 0,8 V.

- Theo phương trình Nernst: E(Cu 2+ 0,059

/Cu) = 0,34 + lg [Cu 2+ ] = 0,331 V

2

3

E(Fe 3+ /Fe 2+ 0,059

) = 0,77 + lg

Fe

= 0,788 V

2

2

Fe

- So sánh thấy E(Fe 3+ /Fe 2+ ) > E(Cu 2+ /Cu) Cực Pt là cực dương, cực Cu là cực âm.

- Sơ đồ pin : () Cu Cu 2+ (0,5 M) Fe 2+ ; Fe 3+ Pt (+)

- Phản ứng điện cực : - ở cực Cu xảy ra sự oxihóa : Cu Cu 2+ + 2e

- ở cực Pt xảy ra sự khử : Fe 3+ + e Fe 2+ .

- Phản ứng chung : Cu + 2Fe 3+ Cu 2+ + 2Fe 2+ .

- Sức điện động của pin = 0,788 0,331 = 0,457 V

b) Khi pin ngừng hoạt động thì sức điện động E = E(Fe 3+ /Fe 2+ ) - E(Cu 2+ /Cu) = 0

Do thể tích dung dịch CuSO4 khá lớn nên coi nồng độ Cu 2+ không đổi và = 0,5M.

3

Khi đó 0,77 + 0,059lg = E(Cu 2+ 3

Fe

Fe

/Cu) = 0,331V =4,8.10 8

2

2

Fe

Fe

c) Tổng thể tích = 100 mL

[Fe 2+ ] = 0,025 M ; [Fe 3+ ] = 0,25M; [Ag + ] = 0,3 M

E(Fe 3+ /Fe 2+ ) = 0,77 + 0,059 lg

0,25

0,025

= 0,829 V

E(Ag + /Ag) = 0,8 + 0,059 lg 0,3 = 0,769 V. So sánh thấy E(Fe 3+ /Fe 2+ ) > E(Ag + /Ag) .

Phát hành PDF bởi Ths Nguyễn Thanh Tú

Đăng ký Word doc qua Zalo 0905779594 Email thanhtuqn88@gmail.com

65


nên phản ứng xảy ra theo chiều Fe 3+ + Ag Fe 2+ + Ag + .

Để đổi chiều phản ứng phải có E(Fe 3+ /Fe 2+ ) < E(Ag + /Ag)

Bài 8

3

3

0,77 + 0,059 lg

Fe

< 0,769

Fe

> 0,9617

2

2

Fe

Fe

Có dung dịch X gồm Fe 2 (SO 4 ) 3 0,100M; FeSO 4 0,010M và NaCl 2,000M.

1. Cần đặt điện thế tối thiểu là bao nhiêu để có quá trình oxi hóa và quá trình khử xảy ra đầu

tiên ở mỗi điện cực khi điện phân dung dịch X ở pH = 0.

2. Điện phân 100 ml dung dịch X với cường độ dòng điện một chiều không đổi có

I = 9,650A và trong thời gian 100 giây, thu được dung dịch Y.

a. Tính khối lượng dung dịch giảm trong quá trình điện phân.

b. Tính pH của dung dịch Y.

c. Lắp một pin điện gồm một điện cực hiđro tiêu chuẩn với một điện Pt nhúng vào

dung dịch Y. Viết sơ đồ pin và tính sức điện động của pin khi pin bắt đầu phóng điện.

(Giả thiết rằng H 2 O bay hơi không đáng kể và thể tích của dung dịch không thay đổi trong

quá trình điện phân)

Cho: E o (Fe 3+ /Fe 2+ ) = 0,771 V; E o (2H + /H 2 ) = 0,00 V; * β[Fe(OH)] 2+ = 10 -2,17 ;

* β[Fe(OH)] + = 10 -5,92 ; E o (Cl 2 /2Cl - ) = 1,36 V.

Hướng dẫn:

1. Bán phản ứng đầu xảy ra ở mỗi điện cực là

+ Điện cực (+): 2Cl - ⇌ Cl 2 + 2e

+ Điện cực (-): Fe 3+ + 1e ⇌ Fe 2+

Trong dung dịch X có C(Fe 3+ ) = 0,2M; C(Fe 2+ ) = 0,01M; C(H + ) = 1M; C(Cl - ) = 2M;

Na + ; SO 2- 4 .

E a = E(Cl 2 /2Cl - ) = E 0 (Cl 2 /2Cl - ) + 0,0592

2

= 1,36 + 0,0592

2

1

lg

2

2

p Cl

2

2

lg

[ Cl]

= 1,342 (V)

Ở pH = 0 không có quá trình proton hóa của ion kim loại, do đó:

E c = E(Fe 3+ /Fe 2+ ) = 0,771 + 0,0592lg 0,2 = 0,848 (V)

0,01

Phát hành PDF bởi Ths Nguyễn Thanh Tú

Đăng ký Word doc qua Zalo 0905779594 Email thanhtuqn88@gmail.com

66


Vậy thế cần đặt vào để có quá trình oxi hóa ion Cl - và quá trình khử ion Fe 3+ là:

U = 1,342 – 0,848 = 0,494 (V)

2 a. Số mol e phóng ra hay thu vào trong quá trình điện phân là

ne = It/F = 9,65.100/96500 = 0,01 (mol)

Có các bán phản ứng:

Ở cực (+): 2Cl - Cl 2 + 2e (1)

n o 0,2

Ở cực (-): Fe 3+ + 1e Fe 2+ (2)

n o 0,02 0,001

Theo (1), (2) và giả thiết cho thấy ion Cl - và Fe 3+ đều dư.

Vậy khối lượng dung dịch giảm là: m = mCl 2

= 71.0,01/2 = 0,355 (gam)

2.b. Theo phần (a), cho thấy trong dung dịch Y có:

C(Fe 3+ ) = (0,02-0,01)/0,1=0,1 (M)

C(Fe 2+ ) = (0,001+0,01)/0,1=0,11 (M)

C(Cl - ) = (0,2-0,01)/0,1=1,95 (M);

Na + ; SO 2- 4 .

Có các cân bằng:

Fe 3+ + H 2 O ⇌ Fe(OH) 2+ + H + * β[Fe(OH)] 2+ = 10 -2,17 (3)

Fe 2+ + H 2 O ⇌ Fe(OH) + + H + * β[Fe(OH)] + = 10 -5,92 (4)

H 2 O ⇌ H + + OH - K w = 10 -14 (5)

Do [Fe(OH)] 2+ .[H + ] ≃ 0,1.10 -2,17 >> [Fe(OH)] + .[H + ] ≃ 0,11.10 -5,92 >> K w

Vì vậy pH là do cân bằng (3) quyết định.

Xét cân bằng:

Fe 3+ + H 2 O ⇌ Fe(OH) 2+ + H + * β[Fe(OH)] 2+ = 10 -2,17

C o 0,1

[ ] (0,1-x) x x

=> * β[Fe(OH)] 2+ = x 2 /(0,1-x) = 10 -2,17

Với 0<x<0,1 => x = 0,023

Vậy pH = - lg0,023 = 1,638

2c) (-) Pt, H 2 (1atm)│H + (1M) ║ Fe 2+ (0,11M); Fe 3+ (0,077M) │ Pt (+)

Phát hành PDF bởi Ths Nguyễn Thanh Tú

Đăng ký Word doc qua Zalo 0905779594 Email thanhtuqn88@gmail.com

67


Theo kết quả tính ở phần (b) và cho thấy ion Fe 2+ tạo phức hiđroxo không đáng kể, nên ta

có:E(Fe 3+ /Fe 2+ ) = 0,771 + 0,0592lg(0,1-0,023)/0,11 = 0,762 (V)

Bài 9 :

Vậy E(pin) = E (cao) – E (thấp) = 0,762-0,00 = 0,762 (V)

a. Viết phương trình phản ứng giữa MnO

4(aq)

trung tính biết rằng trong môi trường axit

bazơ tới

MnO

2

4(aq)

SO

, môi trường trung tính tới MnO 2(r)

2

3(aq)

MnO

4(aq)

bị khử tới

trong ba môi trường axit, bazơ và

Mn

2

(aq)

, trong môi trường

b. Tính E 2 ở ba môi trường axit, bazơ, trung tính. Từ đó, rút ra kết luận về khả

MnO 4 /Mn

năng oxi hóa của

MnO

4(aq)

ở mỗi môi trường.

c. Tính hằng số cân bằng của phản ứng tự oxi hóa khử của

sơ đồ thế khử chuẩn sau:

MnO

2

4(aq)

trong nước. Cho biết

Hướng dẫn

a)

2MnO 5SO 6H 2Mn 5SO 3H O

2 2 2

4(aq) 3(aq) (aq) (aq) 4(aq) 2 (l)

2MnO SO 2OH 2MnO SO H O

2 2 2

4(aq) 3(aq) (aq) 4(aq) 4(aq) 2 (l)

2MnO 3SO H O 2MnO 3SO 2OH

2 2

4(aq) 3(aq) 2 (l) 2(r) 4(aq) (aq)

b) Tính E 2 ở ba môi trường

MnO 4 /Mn

MnO 8H 5e Mn 4H O

2

4(aq) (aq) 2 (l)

0 0,059 [MnO

4

][H ]

E E log

2

n [Mn ]

8

Nếu coi [MnO 2

4

] = [Mn

] =1M thì :

0 0,059 8 0 8.0,059

0

E E log[H ] E pH E 0,0944pH

5 5

Môi trường axit, pH càng nhỏ, E càng lớn, khả năng oxi hóa càng mạnh.

Môi trường bazo, pH>7 khả năng oxi hóa kém hơn.

Môi trường trung tính, pH=7 khả năng oxi hóa kém hơn môi trường axit.

Phát hành PDF bởi Ths Nguyễn Thanh Tú

Đăng ký Word doc qua Zalo 0905779594 Email thanhtuqn88@gmail.com

68


c) Tính hằng số cân bằng của phản ứng tự phân hủy

3MnO 2MnO MnO

2

4(aq) 4(aq) 2(r)

2

0

4(aq) 2(r) 2

2

MnO

4(aq)

MnO 4H 2e MnO 2H O E 2, 26V

2

0

2x MnO4(aq)

1e MnO4(aq)

E 0,56V

3MnO 4H 2MnO MnO 2H O E 1,70V

2

0

4(aq) 4(aq) 2(r) 2

0

nE 2.1,70

0,059 0,059

K 10 10

Bài 10

Ngày 1 tháng 7 năm 2000, đường hầm và cầu nối giữa Đan Mạch và Thụy Điển đã

chính thức được mở ra. Nó bao gồm một đường hầm từ Copenhaghen tới một hòn đảo nhân

tạo, và một cầu nối từ đảo Malmo ở Thụy Điển. Vật liệu xây dựng chủ yếu sử dụng là bê

tông và thép.

Sự ăn mòn kim loại có liên quan đến các phản ứng điện hóa. Điều này cũng đúng cho

sự hình thành gỉ trên bề mặt sắt, nơi mà các phản ứng điện hóa ban đầu thường là:

(1) Fe (s) → Fe 2+ (aq) + 2e

(2) O 2 (g) + 2H 2 O(l) + 4e → 4OH - (aq)

Một tế bào điện hóa được hình thành từ các phản ứng trên ở 25 0 C. Sơ đồ pin điện

được mô tả như sau: Fe (s)│Fe 2+ (aq)║OH - (aq), O 2 (g)│Pt (s). Cho giá trị thế điện cực

chuẩn ở 25 0 0 0

C: E 2

0, 44V; E 0,40V ; RTln10/F = 0,0592.

Fe /Fe O ,H O/OH

2 2

a.Tính sức điện động chuẩn của pin điện hóa ở 25 0 C.

b.Viết các bán phương trình xảy ra ở mỗi điện cực và phản ứng tổng quát xảy ra khi

pin hoạt động.

c.Tính hằng số cân bằng của phản ứng điện hóa.

d.Tính sức điện động của pin ở 25 0 C khi [Fe 2+ ]=0,015M; pH= 9; p(O 2 ) = 0,7atm

Hướng dẫn

a. E 0 pin = E 0 + - E 0 - = 0,40 – (-0,44) = 0,84V

b.- Tại anot: 2Fe → 2Fe 2+ + 4e

- Tại catot: O 2 + 2H 2 O + 4e → 2Fe 2+ + 4OH -

Phản ứng tổng quát: 2Fe + O 2 + 2H 2 O → 2Fe 2+ + 4OH -

0

n.E 4.0,84

0,0592 0,0592

56

c. K 10 10 6, 2.10

d. Ta có: pH = 9 → [H + ] = 10 -9 M → [OH - ] = 10 -5 M

Phát hành PDF bởi Ths Nguyễn Thanh Tú

Đăng ký Word doc qua Zalo 0905779594 Email thanhtuqn88@gmail.com

69


E pin = E

0

pin

0,0592 [Fe ] .[OH ]

lg

4 p(O )

2

2 4

2 5 4

0,0592 0,015 .(10 )

= 0,84 lg 1,19V

4 0,7

III.7 Bài tập về Phức của các kim loại nhóm VIIB, VIIIB

Bài 1: Viết công thức và gọi tên phức chất

2

a) Viết công thức của ion phức chất được tạo nên bởi một ion Co 3+ , năm phân tử amoniac

và một ion cacbonat.

b) Gọi tên phức chất K 2 [CoCl 4 ]

c) Gọi tên phức chất [Co(NH 3 ) 4 (H 2 O)Cl]Cl 2

d) Viết công thức của bốn phức chất của coban (III) ứng với dữ kiện thực nghiệm:

Công thức Tổng số ion Ion Cl- tự do Điện tích của ion phức

CoCl 3 .6NH 3 4 3 3+

CoCl 3 .5NH 3 3 2 2+

CoCl 3 .4NH 3 2 1 1+

CoCl 3 .3NH 3 0 0 0

Hướng dẫn:

a) Co 3+ 2

+ 5NH 3 + CO 3

[Co(NH 3 ) 5 CO 3 ] +

b) Trong phức chất có 2 ion K + , vậy ion phức có điện tích 2-. Vì có bốn ion Cl - trong ion

phức nên kim loại trung tâm là Co 2+ . Vì ion phức là anion nên chữ cuối của ion kim loại

coban được thêm “at”. Như vậy tên phức chất trên là: kali tetraclorocobanat

c) [Co(NH 3 ) 4 (H 2 O)Cl]Cl 2 : tetraminaquaclorocoban (III) clorua

d)

[Co(NH 3 ) 6 ]Cl 3 [Co(NH 3 ) 5 Cl]Cl 2 [Co(NH 3 ) 4 Cl 2 ]Cl [Co(NH 3 ) 3 Cl 3 ]

Bài 2. Bằng thực nghiệm xác định được phức bát diện [Co(NH 3 ) 6 ] 3+ nghịch từ, trong khi

đó, phức bát diện [CoF 6 ] 3- thuận từ. Hãy giải thích từ tính của các phức chất trên theo

quan điểm của thuyết liên kết hóa trị (VB).

Hướng dẫn

Cấu hình electron của Co 3+ là [Ar]3d 6 .

- Trong phức chất [CoF 6 ] 3- , Co 3+ có lai hóa ngoài sp 3 d 2 , mỗi AO lai hóa nhận 1 cặp

electron từ F - tạo 1 liên kết cho nhận giữa Co 3+ và F - do vậy phức chất dạng bát diện và

thuận từ.

Phát hành PDF bởi Ths Nguyễn Thanh Tú

Đăng ký Word doc qua Zalo 0905779594 Email thanhtuqn88@gmail.com

70


sp 3 d 2

[CoF 6 ] 3- F - F - F - F - F - F -

- Trong phức chất [Co(NH 3 ) 6 ] 3+ , có sự dồn để ghép đôi electron trong 3d 6 tạo điều kiện

để Co 3+ lai hóa trong d 2 sp 3 , do vậy phức chất có dạng bát diện và nghịch từ.

d 2 sp 3

[Co(NH 3 ) 6 ] 3+ NH 3 NH 3 NH 3 NH 3 NH 3 NH 3

Lưu ý: NH 3 là phối tử trường mạnh trong khi đó F - là phối tử trường yếu.

Bài 3

Coban tạo ra được các ion phức: CoCl 2 (NH 3 ) 4 + (A), Co(CN) 6 3- (B), CoCl 3 (CN) 3 3-

(C),

a. Viết tên của (A), (B), (C).

b. Theo thuyết liên kết hoá trị, các nguyên tử trong B ở trạng thái lai hoá nào?

c. Các ion phức trên có thể có bao nhiêu đồng phân lập thể? Vẽ cấu trúc của chúng.

Hướng dẫn

a. Tên của các ion phức:

(A) Điclorotetraammincoban(III);

(B) Hexaxianocobantat(III);

(C) Triclorotrixianocobantat(III).

b. Co(CN) 6 3- .

Co : d 2 sp 3

C : sp

N : không ở vào trạng thái lai hoá hoặc ở trạng thái lai hoá sp.

c. Ion phức (A) có 2 đồng phân:

H 3 N

H 3 N

Cl

Co

NH 3

NH 3

H 3 N

H 3 N

Cl

Co

Cl

NH 3

Cl

NH 3

Ion phức (B) không có đồng phân:

Phát hành PDF bởi Ths Nguyễn Thanh Tú

Đăng ký Word doc qua Zalo 0905779594 Email thanhtuqn88@gmail.com

71


NC

NC

CN

Co

CN

CN

CN

Ion phức (C) có 2 đồng phân:

Cl

CN

NC

Cl

Co

CN

Cl

NC

Cl

Co

Cl

Cl

CN

CN

Bài 4. Dung dịch chứa ion Fe(SCN) 2+ có màu đỏ bắt đầu từ nồng độ 10 -5 M. Hằng số bền

của ion Fe(SCN) 2+ là β = 2.10 2 .

a. Trong 500 cm 3 dung dịch chứa 10 -3 mol FeCl 3 và 5.10 -3 mol KSCN. Tính nồng độ ion

Fe(SCN) 2+ ở trạng thái cân bằng. Hỏi dung dịch có màu đỏ không?

b. Hòa tan tinh thể NaF vào dung dịch trên (V=const) tạo thành ion FeF 2+ với hằng số

bền là β =1,6.10 5 . Hỏi bắt đầu từ lượng nào của NaF thì màu đỏ biến mất?

Hướng dẫn

a) Fe 3+ + SCN - Fe(SCN) 2+

Cân bằng: 2.10 -3 -x 10 -2 - x x

Ta có: β b = 2.10 2 =

=> x=1,27.10 -3 M > 10 -5 M => Dung dịch có màu đỏ.

b) Màu đỏ biến mất khi [Fe(SCN) 2+ ] < 10 -5 M

Theo ĐLBT nồng độ: [Fe(SCN)] 2+ + [SCN - ]=10 -2

Khi [Fe(SCN) 2+ ] < 10 -5 M thì [SCN - ]=10 -2 M

Ta có: =>

Nồng độ Fe 3+ được bảo toàn nên:

[Fe] 3+ + [Fe(SCN)] 2+ + [FeF 2+ ]=2.10 -3 M

=> [FeF 2+ ]= 2.10 -3 - 5.10 -6 - 10 -5 = 1,985.10 -3 M

Đối với FeF 2+ ta có:

=> => [F - ]=2,481.10 -3 M

=> [F - ] 0 = [FeF 2+ ] + [F - ]= (1,985+ 2,481).10 -3 = 4,466.10 -3 M

Phát hành PDF bởi Ths Nguyễn Thanh Tú

Đăng ký Word doc qua Zalo 0905779594 Email thanhtuqn88@gmail.com

72


Trong 500 cm 3 cần hòa tan 4,466/2. 10 -3 mol NaF, ứng với khối lượng NaF là:

2,233.10 -3 .42=0,0938 gam

Bài 5

1. Momen từ của [Mn(CN) 6 ] 3- là 2,8 MB, của [MnBr 4 ] 2- là 5,9 MB. Hãy xác định số

electron độc thân của ion trung tâm và dự đoán cấu trúc hình học của những ion phức

này?

2. Viết phương trình phản ứng khi cho [PtCl 4 ] 2- tác dụng với dd NH 3 (1:2) và [Pt(NH 3

) 4 ] 2+ tác dụng cới Cl - (1:2) ?

Hướng dẫn

1. Theo thuyết trường tinh thể n( n 2)

Nên phức [Mn(CN) 6 ] 3- có số electron độc thân là 2; phức [MnBr 4 ] 2- có số electron là 5

2.

Phức chất Cấu hình e của

NTTT

Trạng thái lai hóa

NTTT

Số e độc

thân

hình

dạng

3d 4 d 2 sp 3 2 bát diện

3d 5 sp 3 5 tứ diện

Theo quy luật ảnh hưởng trans thì I - > Cl - > NH 3

-Khi cho [PtCl 4 ] 2- tác dụng với dd NH 3 (1:2) ở giai đoạn 2 vì Cl - có ảnh hưởng trans lớn

hơn NH 3 nên Cl - ở vị trí đối diện với Cl - dễ bị thế hơn tạo thành đồng phân cis có màu

vàng da cam.

-Khi cho [Pt(NH 3 ) 4 ] 2+ tác dụng cới Cl - (1:2) ở giai đoạn 2 vì Cl - có ảnh hưởng trans lớn

hơn NH 3 nên NH 3 ở vị trí đối diện với Cl - dễ bị thế hơn tạo thành đồng phân trans có

màu vàng nhạt.

Phát hành PDF bởi Ths Nguyễn Thanh Tú

Đăng ký Word doc qua Zalo 0905779594 Email thanhtuqn88@gmail.com

73


Bài 6 Hợp chất Pt có hóa trị II với công thức chung [PtX 2 (amin) 2 ] (ở đây X là Cl hoặc

X 2 là SO 2- 4 , manolat..) đã có nhiều ứng dụng trong khoa học và cuộc sống vì hoạt tính

sinh học mà đặc biệt là trong việc chữa trị các khối u.

Hợp chất được biết nhất là [PtCl 2 (NH 3 ) 2 ], hợp chất này có cấu trúc vuông phẳng và có

hai đồng phân hình học.Trong đó có một đồng phân có hoạt tính chữa bệnh ung thư.

1. Vẽ phác họa cấu trúc không gian của 2 đồng phân

2. Có bao nhiêu đồng phân [PtClBr(NH 3 ) 2 ]. Vẽ phác họa cấu trúc các đồng phân này

3. Nếu thay 2NH 3 (đơn răng) bằng phối tử hai răng như 1,2-điaminetan( kí hiệu là en) ta

thu được một đồng phân duy nhất có công thức [PtClBr(en)].Vẽ cấu trúc không gian của

phức này

4. Biết dmen và pn là những phối tử hai răng có công thức cấu tạo như sau:

CH 3

NH 2

- CH 2

- CH 2

- N(CH 3

) 2 NH 2

- CH - CH 2

- NH 2

dmen

pn

Viết tất cả các đồng phân có thể có của các chất:

[PtCl 2 (dmen)], [PtCl 2 (pn)], [PtClBr(dmen)], [PtClBr(pn)].

Hướng dẫn

1.

Cl NH 3 Cl NH 3

Cl NH 3 NH 3 Cl

Pt

Pt

2.

Cl NH 3 Cl NH 3

Br NH 3 NH 3 Br

Pt

Pt

3.

Cl NH 2

Pt

Br NH 2

4. Với [PtCl 2 (dmen)]:

Phát hành PDF bởi Ths Nguyễn Thanh Tú

Đăng ký Word doc qua Zalo 0905779594 Email thanhtuqn88@gmail.com

74


Cl NH 2

Pt

CH 3

Br

CH 3

N

Với [PtCl 2 (pn)]:

Cl

Cl

Pt

Với [PtClBr(dmen)]:

Br

NH 2 NH 2

CH CH 3 3

Pt

NH 2

Cl NH 2

Pt

* *

H

H

Br

Pt

N

Cl

CH 3 CH 3

NH 2

NH 2

N

CH 3 CH 3

Cl

Cl

Với [PtClBr(pn)]:

NH 2 Cl

NH 2

CH 3 Pt

H

Pt

NH

H * 2

* NH

Br CH3

2

Cl

Br

NH

NH 2

2 Br

Br

CH

H

Pt

3 Pt

NH

H * CH * NH 2

2

3

Cl

Cl

Bài 7

1. Hai đồng phân A và B của phức [PtBrCl(PR 3 ) 3 ] (với PR 3 là triankylphotphin) có phổ

NMR của photpho khác nhau. Đồng phân A có 1 nhóm cộng hưởng 31 P gồm những vạch

đơn. Đồng phân B có 2 nhóm cộng hưởng, mỗi nhóm cộng hưởng giống cộng hưởng đơn

của A. Hãy cho biết công thức cấu tạo của phức A, B.

2. Viết tất cả các đồng phân có thể có của:

a) [PtCl 2 (NO 3 ) 2 (NH 3 ) 2 ]

b) [CoCl 2 (en)(NH 3 ) 2 ] +

Hướng dẫn

1. Phức [PtBrCl(PR 3 ) 3 ] có cấu tạo vuông phẳng.

Phát hành PDF bởi Ths Nguyễn Thanh Tú

Đăng ký Word doc qua Zalo 0905779594 Email thanhtuqn88@gmail.com

75


Đồng phân A có 1 nhóm cộng hưởng 31 P gồm những vạch đơn nên hai nhóm nguyên tử

PR 3 có vị trí giống nhau trong phân tử đồng phân A là đồng phân trans.(vẽ hình)

Đồng phân B có 2 nhóm cộng hưởng, mỗi nhóm cộng hưởng giống cộng hưởng đơn của

A nên hai nhóm PR 3 có vị trí không giống nhau trong phân tử đồng phân B là đồng

phân cis.(vẽ hình)

(Nếu không vẽ hình của hai đồng phân, trừ 0,25 điểm)

2. a/ Các đồng phân của [PtCl 2 (NO 3 ) 2 (NH 3 ) 2 ]: 3 đồng phân( trans – trans – trans, cis – cis

– cis, trans – cis – cis)

b/ Các đồng phân của [CoCl 2 (en)(NH 3 ) 2 ] + : 3 cặp đồng phân quang học(trans đối với Cl,

cis đối với NH 3 ; trans đối với NH 3 , cis đối với Cl và cis đối với cả Cl, NH 3 )

Bài 8

1. Vẽ tất cả các đồng phân của các tiểu phân sau:

a. [Co III (en) 2 (NO 2 )Cl] + b. [Pt II (NH 3 ) 2 Cl 2 ]

Biết: en: etylenđiamin, H 2 N – CH 2 – CH 2 – NH 2

2. Sử dụng thuyết trường tinh thể giải thích:

[Fe III (CN) 6 ] 3- có 1 electron không ghép đôi trong khi đó [Fe III (H 2 O) 6 ] 3+ có 5 electron

không ghép đôi.

Hướng dẫn

1. a)

b)

2. Ta có: cấu hình electron Fe: [Ar]3d 6 4s 2 ; Fe 3+ : [Ar]3d 5

Ta có CN - là phối tử trường mạnh, Δo lớn nên các electron d được ghép đôi do đó chỉ còn

1 electron chưa ghép đôi.

H 2 O là phối tử trường yếu, Δo nên các electron d được phân bố đều trên các obitan d, do

đó có 5 electron không ghép đôi

Phát hành PDF bởi Ths Nguyễn Thanh Tú

Đăng ký Word doc qua Zalo 0905779594 Email thanhtuqn88@gmail.com

76


[Fe III (CN) 6 ] 3- [Fe III (H 2 O) 6 ] 3+

Bài 9

Phối tử (2-aminoetyl)photphin là phối tử hai càng. Viết các đồng phân hình học và đồng

phân quang học của phức chất đicloro bis(2-aminoetyl)photphin niken(II).

Hướng dẫn

Các đồng phân hình học và đồng phân quang học của phức chất đicloro bis (2-aminoetyl)

photphin) niken(II)

Phối tử (2-aminoetyl)photphin là phối tử 2 càng:

CH 2 CH 2

Đồng phân trans: 2 đồng phân

H 2 N PH 2

Đồng phân cis: có 3 đồng phân, mỗi đồng phân lại có thêm đồng phân quang học

Bài 10

Phức chất [PtCl 2 (NH 3 ) 2 ] được xác định là đồng phân trans-. Nó phản ứng chậm với

Ag 2 O cho phức chất [PtCl 2 (NH 3 ) 2 (OH 2 ) 2 ] 2+ (kí hiệu là X). Phức chất X không phản ứng

được với etylenđiamin (en) khi tỉ lệ mol phức chất X : en = 1 : 1. Hãy giải thích các sự

kiện trên và vẽ (viết) cấu tạo của phức chất X.

Hướng dẫn

Phát hành PDF bởi Ths Nguyễn Thanh Tú

Đăng ký Word doc qua Zalo 0905779594 Email thanhtuqn88@gmail.com

77


[PtCl 2 (NH 3 ) 2 ] (1) là đồng phân trans- đòi hỏi phức chất phải có cấu tạo vuông phẳng:

Cl

H 3 N—Pt—NH 3 (1)

Cl

- Phản ứng của (1) với Ag 2 O:

Trans-[PtCl 2 (NH 3 ) 2 ] + Ag 2 O + H 2 O →

Trans-[PtCl 2 (NH 3 ) 2 (H 2 O) 2 ] 2+ + 2OH -

- Etylenđiamin là phối tử hai càng mạch ngắn. Khi phối trí với các ion kim loại nó chỉ

chiếm 2 vị trí phối trí cạnh nhau (vị trí cis). Hiện tượng en không thể phản ứng với

[PtCl 2 (NH 3 ) 2 (H 2 O) 2 ] 2+ theo phản ứng:

[PtCl 2 (NH 3 ) 2 (H 2 O) 2 ] 2+ + en → [PtCl 2 (NH 3 ) 2 (H 2 O) 2 en] 2+ + 2H 2 O

chứng tỏ rằng 2 phân tử H 2 O nằm ở 2 vị trí trans đối với nhau. Như vậy công thức cấu tạo

của phức chất phải là:

H2O

NH3

Pt

Cl

NH3

Cl

H2O

Bài 11

Kết quả phân tích một phức chất A của Platin (II) cho biết có: 64,78 % khối lượng là Pt,

23,59 % là Cl, 5,65 % là NH3 và 5,98 % còn lại là H2O

1. Tìm công thức phân tử của phức chất biết rằng A là phức chất 1 nhân và Pt có số phối

trí là 4. Viết công thức cấu tạo 2 đồng phân cis và trans của nó

2. Entanpi tự do chuẩn tạo thành ở 25 o C của các đồng phân cis, trans lần lượt là: -396 và -

402 kJ.mol -1 . Tính hằng số cân bằng K của phản ứng sau: cis(A) trans(A)

3. Tính nồng độ mol/lit mỗi đồng phân trong dung dịch, biết rằng lúc đầu chỉ có đồng

phân cis nồng độ 0,01M.

Cho Pt = 195 ; Cl = 35,5 ; N = 14 ; O = 16 ; H = 1

Hướng dẫn

1. Đặt CTPT của A là: Pt x Cl y (NH 3 ) z (H 2 O) t .

Phát hành PDF bởi Ths Nguyễn Thanh Tú

Đăng ký Word doc qua Zalo 0905779594 Email thanhtuqn88@gmail.com

78


- Vì phức chất A là phức 1 nhân nên phân tử khối của A:

M

A

M

Pt

100% 195100

301 g / mol

% Pt 64,78

- Từ % của các thành phần có trong A x = 1, y = 2, z = 1, t = 1

CTPT là: PtCl 2 (NH 3 )(H 2 O)

CTCT 2 đồng phân cis, trans:

2. Xét phản ứng chuyển hóa: Cis Trans

G 298K = -402 + 396 = -6kJ = -6000J = -RTlnK

3. Xét phản ứng chuyển hóa: Cis Trans K = 11,27

Cân bằng: 10 -2 – x x

G

6000

RT

8,314 298

K e e 11, 27

x

K 11, 27

2 10 x

x= [trans] = 9,2.10 -3 ; [cis]=10 -2 - 9,2.10 -3 = 8.10 -4

Bài 12

Năm 1912, Alfred Werner đã tổng hợp một số hợp chất phức đồng phân của cobalt, được

sử dụng để làm bằng chứng cho sự phù hợp về lí thuyết cấu trúc của các hợp chất phức

do chính ông đề xuất.

Một dung dịch chứa 10 gam CoCl 2 ∙6H 2 O trong 150 gam ethyelenediamine (H 2 N-CH 2 -

CH 2 -NH 2 , en) 10 % được để trong không khí trong nhiều giờ. Acid hóa dung dịch màu

nâu được bởi HCl rồi làm bay hơi cho đến khi bắt đầu xảy ra sự kết tinh, sau đó thêm

NH 4 NO 3 vào dịch cái, thì có một lượng nhỏ tinh thể dạng phiến màu xanh lục (chất I)

được tạo thành. Lọc kết tủa, rồi thêm NaBr vào dịch lọc, thì một lượng lớn các tinh thể

hình kim màu vàng cam (chất III) được tạo thành. Dung dịch nước chứa 100 gam chất III

được xử lí với một lượng bạc tartrate (tartric acid: HOOC-CH (OH)-CH(OH)-COOH)

vừa đủ (68.3 gam bạc tartrate), thu được 2 đương lượng kết tủa của các ion halogenua.

AgCl kết tủa được lọc tách rồi làm bay hơi dung dịch. d-tartrate kết tinh (chất IV) sau khi

làm bay hơi được lọc tách, và dịch lọc được làm nguội rồi chuyển thành một khối gelatin

Phát hành PDF bởi Ths Nguyễn Thanh Tú

Đăng ký Word doc qua Zalo 0905779594 Email thanhtuqn88@gmail.com

79


chứa l-tartrate (chất V). Các tinh thể IV và V được nghiền riêng trong các cối giã với HBr

đặc, hơi ấm. Lọc tách kết tủa trong cả hai trường hợp, rồi tái kết tinh trong nước ấm, lần

lượt thu được các chất VI và VII. Tính chất của các chất được cho trong bảng sau:

Hàm lượng, %

độ dẫn

điện []D

STT

màu

[M]D

Co N Br(Cl)

phân (1 %)

sắc

tử

xanh

I 18.89 22.44 (22.72)

105 – –

lục

II 18.89 22.44 (22.72) tím 107 – –

III 11.06 15.77 44,.97 vàng 415 – –

IV 10.58 15.08 14.34 vàng 265 +98° +555°

V 10.58 15.08 14.34 vàng 273 – –

VI 11.44 16.32 46.54 vàng 420 +117° +602°

VII 11.44 16,.32 46.54 vàng 418 -115° -592°

1) Xác định công thức các chất I - VII.

2) Viết phương trình điều chế I, III - VII.

3) Cho biết I và II, VI và VII thuộc loại đồng phân nào?

4) Với các hợp chất này, xác định:

a) số oxid hóa của nguyên tử trung tâm;

b) cấu hình electron của cobalt ion;

c) số phối trí của nguyên tử trung tâm;

d) đa diện phối trí (hình đa diện tạo thành bởi các nguyên tử phối trí). Vẽ sự sắp xếp (cấu

trúc) các đồng phân với các nguyên tử phối trí nằm ở đỉnh đa diện.

Hướng dẫn

1) - Theo dữ kiện trong bảng, thành phần chất I là:

Co : N : Cl = (18.89: 58.933): (22.44:14.007): (22.72: 35.453) = 0.3205: 1.6021: 0.6408

= 1.0: 4.99: 1.99 = 1: 5: 2

Khối lượng mol M(I) = M(Co) : ω(NO) = 312 gam/mol.

Công thức chất I gồm 1 Co 3+ , 2 phân tử ethylenediamine (tương ứng với 4 nguyên

tử nitrogen), 1 NO - 3 , 2 Cl - .

Phát hành PDF bởi Ths Nguyễn Thanh Tú

Đăng ký Word doc qua Zalo 0905779594 Email thanhtuqn88@gmail.com

80


Dựa vào dữ liệu về độ dẫn điện, xác định được công thức chất I là [Co(en) 2 Cl 2 ](NO 3 )

(xanh lục).

- Công thức chất II cũng tương tự: [Co(en) 2 Cl 2 ](NO 3 ) (tím).

- Công thức chất III: [Co(en) 3 ]Br 3 .3H2O

- Công thức chất III phù hợp với dữ kiện về độ dẫn điện phân tử. Độ dẫn điện tăng

(nhưng

không tuyến tính) theo số ion tạo thành trong quá trình phân li phức chất. Đây từng là

phương pháp chính để xác định công thức cầu phối trí ngoài cùng của Werner. Từ 1

mol

I có 2 mol ion được tạo thành, còn từ 1 mol chất III thì có 4.

- Thành phần các chất IV và V là

Co: N: Br = (10.58: 58.933): (15.08: 14.007): (15.08:79.904) =1.0: 6.0: 1.0.

Khối lượng mol M(IV) và M(V) = 58.933: 0.1058 = 557 gam /mol.

Thành phần của IV và V gồm 1 Co 3+ , 3 ethylenediamine, 1 Br - - tương đương với

hợp phần Co(en) 3 Br = 319 - vậy phần còn lại phải là tartrate ion (C 4 H 4 O 6 2- ) và 5 phân

tử H 2 O. Độ dẫn diện cao hơn I nhưng thấp hơn III, do đó, trong quá trình

phân li có 3

vi hạt được tạo thành.

Công thức của IV và V là [Co(en) 3 ]Br(C 4 H 4 O 6 ).5H 2 O

Tương tự tìm được công thức VI và VII là [Co(en) 3 ]Br 3 .2H 2 O

2) - Điều chế chất III:

4CoCl 2 + 12en + O 2

HCl

4[Co(en) 3 ]Cl 3 + 2H 2 O

Phát hành PDF bởi Ths Nguyễn Thanh Tú

Đăng ký Word doc qua Zalo 0905779594 Email thanhtuqn88@gmail.com

81


[Co(en) 3 ]Cl 3 + 3NaBr + 3H 2 O → [Co(en) 3 ]Br 3 .3H 2 O + 3NaCl (III)

Có một lượng nhỏ tạp chất tạo thành, khi thêm ammonium nitrate thì tách ra ở dạng

muối nitrate màu xanh lục:

HCl

4CoCl 2 + 12en + O 2 4[Co(en) 2 Cl 2 ]Cl + 2H 2 O

[t-Co(en) 2 Cl 2 ]Cl + NH 4 NO 3 → [t-Co(en) 2 Cl 2 ]NO 3 + NH 4 NO 3 (I)

- Điều chế chất IV - VII:

[Co(en) 3 ]Br 3 + Ag 2 C 4 H 4 O 6 → [Co(en)3]Br(C 4 H 4 O 6 ) + 2AgBr

[d-Co(en) 3 ]Br(d-C 4 H 4 O 6 ) + 5H 2 O → [d-Co(en) 3 ]Br(d-C 4 H 4 O 6 ).5H 2 O (IV)

[l-Co(en) 3 ]Br(l-C 4 H 4 O 6 ) + 5H 2 O → [l-Co(en) 3 ]Br(l-C 4 H 4 O 6 ).5H 2 O (V)

[d-Co(en) 3 ]Br(d-C 4 H 4 O 6 ).5H 2 O + 2HBr → [d-Co(en) 3 ]Br 3 .2H 2 O + 3H 2 O + d-C 4 H 4 O 6

(VI)

[l-Co(en) 3 ]Br(l-С 4 H 4 O 6 ).5H 2 O + 2HBr → [l-Co(en) 3 ]Br 3 .2H 2 O +3H 2 O + l-С 4 H 4 O 6 (VII)

3) I và II là đồng phân hình học (cis/trans).

VI và VII là đồng phân quang học.

4) - Số oxi hóa +3;

- Cấu hình electron: 1s 2 2s 2 2p 6 3s 2 3p 6 3d 6

- Số phối trí 6

cis-[Co(en) 2 Cl 2 ] + trans-[Co(en) 2 Cl 2 ] +

Đồng phân quang

học [Co(en) 3 ] 3+

Đồng phân quang

học [Co(en) 3 ] 3+

III.8. Các bài tập tính toán khác

Bài 1: Hàng năm, thế giới sản xuất được khoảng 75.000 tấn niken. Một trong những

phương pháp sản xuất niken tinh khiết được tiến hành như sau:

Phát hành PDF bởi Ths Nguyễn Thanh Tú

Đăng ký Word doc qua Zalo 0905779594 Email thanhtuqn88@gmail.com

82


Niken oxit được xử lí bằng khí than ướt (H 2 + CO), khi đó hidro khử niken oxit thành

niken chưa tinh khiết. Ở khoảng 50 0 C và ở áp suất 1 atm, niken tác dụng với khí CO tạo

thành hợp chất Ni (0) tetracacbonyl: Ni(CO) 4 là một chất dễ bay hơi. Cho Ni(CO) 4 qua

một lò phản ứng ở nhiệt độ 250 0 C, nó bị phân hủy thành CO và Ni nguyên chất.

a) Viết các phương trình hóa học xảy ra

b) Muốn sản xuất một tấn niken cần bao nhiêu niken oxit, hidro và cacbon monooxit

c) Liên kết trong phân tử Ni(CO) 4

Hướng dẫn:

a) NiO (r) + H 2 (k) Ni (r) + H 2 O (k)

Ni (r) + 4CO (k)

0

50 C

Ni(CO) 4

0

250 C

1000000g

b) 17035 mol Ni

1

58,7 g.

mol

- Muốn sản xuất 1 tấn Ni cần 17035 mol NiO tức là 17035 74,7 = 1272,514 kg NiO

- Cần 17035 mol H 2 tức là 34070 g H 2 hay 34,07 kg H 2

- C2ần 417035 mol CO tức là 417035 28 = 1907,92 kg CO

c) Trong phân tử Ni(CO) 4 Ni có số oxi hóa bằng 0

Ni: 1s 2 2s 2 2p 6 3s 2 3p 6 3d 8 4s 2 (có 10e hóa trị)

Liên kết trong phân tử CO :C:::O:

Trong phân tử Ni(CO) 4 , mỗi nguyên tử C cho nguyên tử trung tâm Ni 2e, 4 nguyên tử C

trong 4 nhóm CO cho nguyên tử trung tâm Ni 8e, tạo thành 4 liên kết phối trí. Số e hóa trị

của Ni trong hợp chất Ni(CO) 4 bây giờ là 10+8=18e, cấu hình của khí hiếm bền.

Bài 2: Người ta hòa tan 15,2 gam sắt (II) sunfat thành một lít dung dịch D. Để trong

không khí một thời gian, một phần ion Fe(II) trong dung dịch bị oxi hóa thành ion Fe(III)

(dung dịch D’)

Người ta lấy 120 cm 3 dung dịch D’, nhỏ thêm vào vài giọt axit sunfuric rồi cho tác dụng

từ từ với một dung dịch thuốc thử KMnO 4 0,1M. Sau khi tác dụng với 20cm 3 dung dịch

thì KMnO 4 không mất màu nữa.

a) Viết phương trình hóa học biến đổi Fe (II) thành Fe (III) bởi KMnO 4

b) Tính số mol ion Fe 2+ có mặt trong dung dịch D’

c) Tính tỉ lệ phần trăm số mol ion Fe 2+ đã bị oxi hóa

Hướng dẫn:

a) 5Fe 2+ (aq) + MnO - 4 (aq) + 8H + (aq) 5Fe 3+ (aq) + Mn 2+ (aq) + 4H 2 O

Phát hành PDF bởi Ths Nguyễn Thanh Tú

Đăng ký Word doc qua Zalo 0905779594 Email thanhtuqn88@gmail.com

83


(dung dịch màu tím)

(mất màu)

b) - Số mol MnO 4

-

trong 20cm 3 dung dịch KMnO 4 :

- Số mol ion Fe 2+ bằng 5 lần số mol MnO 4

-

: 2

n

MnO4

3

0,1 20 10 ( mol)

3 2

n 0,1 20 10 5 10 mol

Fe

trong 120

cm 3 dung dịch D’

- Số mol ion Fe 2+ 2+ 2 1000

2

trong 1 lít dung dịch D’ : [Fe ] 10 8,33.10 ( mol / l)

120

c) Số mol Fe 2+ m

2+ FeSO 15, 2

4

trong một lít dung dịch D: [Fe ] 0,1 mol / l

M 152

và [Fe 3+ ] = 0,1 – 0,083 = 0,017 mol/l

Như vậy có 17% ion Fe (II) đã bị oxi của không khí oxi hóa thành ion Fe (III)

Bài 3: Hòa tan 10 gam muối Fe (II) không nguyên chất trong nước thành 200cm 3 dung

dịch. Lấy 20cm 3 dung dịch đó axit hóa bằng axit sunfuric loãng rồi chuẩn độ bằng dung

dịch KMnO 4 0,03M. Thể tích dung dịch KMnO 4 đã dùng là 25 cm 3 . Tính tỉ lệ phần trăm

khối lượng sắt trong muối sắt (II) không nguyên chất nói trên.

Hướng dẫn:

5Fe 2+ (aq) + MnO - 4 (aq) + 8H + (aq) 5Fe 3+ (aq) + Mn 2+ (aq) + 4H 2 O

- Số mol MnO 4

-

đã dùng để chuẩn độ:

n

MnO4

- Số mol Fe 2+ trong 20cm 3 dung dịch muối sắt (II):

n n 4 3

mol

2

Fe MnO4

5 57,5.10 3,75.10

FeSO4

25.10 0,03 7,5.10

3 4

- Số mol Fe 2+ trong 200cm 3 dung dịch tức là trong 10 gam muối sắt (II) không nguyên

chất là

2

3,75.10 mol 56 2,1

gam

- Tỉ lệ phần trăm Fe 2+ trong muối sắt (II) không nguyên chất là 21%

Bài 4: Một hỗn hợp có khối lượng 4,72 gam gồm Fe, FeO, Fe 2 O 3 được đun nóng trong

một bình kín trong môi trường khí hidro. Sản phẩm thu được là 3,92 gam Fe và 0,9 gam

H 2 O.

Nếu cho cùng một lượng hỗn hợp trên tác dụng với CuSO 4 dư thì sẽ thu được 4,96 gam

một hỗn hợp chất rắn.

a) Tính thể tích dung dịch axit HCl 7,3% (d=1,03 g/cm 3 ) cần thiết để hòa tan 4,72 gam

hỗn hợp ban đầu.

b) Tính thể tích khí giải phóng ra khi hòa tan hỗn hợp trên trong axit HCl ở đktc?

Hướng dẫn:

a) * Phản ứng khử bởi H 2 :

mol

Phát hành PDF bởi Ths Nguyễn Thanh Tú

Đăng ký Word doc qua Zalo 0905779594 Email thanhtuqn88@gmail.com

84


FeO (r) + H 2 (k)

Fe 2 O 3 (r) + 3H 2 (k)

o

t

Fe (r) + H 2 O (k)

o

t

2Fe (r) + 3H 2 O (k)

Khối lượng của sắt sau phản ứng là 3,92 gam

Số mol sắt trong hỗn hợp ban đầu là n Fe

Số mol sắt tạo ra trong phản ứng a là n FeO

(a)

(b)

Số mol sắt tạo ra trong phản ứng b là n Fe2O3

3,92g

3,92

Ta có: nFe nFeO nFe 0, 07

2O

mol (1)

3

M 56

Fe

* Phản ứng với dung dịch CuSO 4 : Fe(r)+ CuSO 4 (aq) Cu(r) + FeSO 4 (aq)

Nếu 1 mol Fe phản ứng, khối lượng chất rắn tăng 64 – 56 = 8 gam

Đề cho khối lượng chất rắn tăng 4,96 - 4,72 = 0,24 gam

Vậy lượng Fe trong hỗn hợp ban đầu: n Fe = 0,24 / 8 = 0,03 mol (2)

* Sự tạo thành nước sau phản ứng khử oxit sắt: n

2

Theo phản ứng a và b: nH 3 0,05

2O nFeO

nFe2O

mol (3)

3

Giải (1), (2), (3) ta được n FeO = 0,02 mol; n Fe2O3 = 0,01 mol

* Phản ứng với axit:

Fe (r) + 2HCl (aq) FeCl 2 (aq) + H 2 (k)

FeO (r) + 2HCl (aq) FeCl 2 (aq) + H 2 O

Fe 2 O 3 (r) + 6HCl (aq) 2FeCl 3 (aq) + 3H 2 O

n 2n 2n 6n 0,06 0,04 0,06 0,16 mol

HCl Fe FeO Fe2O3

H O

0,9

0,05 mol

18

Cần lưu ý một phần Fe đã phản ứng với FeCl 3 : Fe + 2FeCl 3 3FeCl 2

1

Lượng Fe tác dụng: nFe

nFeCl

nFe

3 2O3

0,01mol

2

Điều đó có nghĩa là lượng axit tác dụng giảm đi 0,02 mol, như vậy lượng axit đã tác dụng

là 0,14 mol.

0,1436,5

Thể tích axit HCl 7,3% tác dụng với hỗn hợp trên là: V

68 cm

0,0731,03

b) Thể tích H 2 tạo ra ở đktc: Fe (r) + 2HCl (aq) FeCl 2 (aq) + H 2 (k)

Lượng sắt trong hỗn hợp là 0,03 mol nhưng đã tác dụng với FeCl 3 0,01 mol

Vậy lượng sắt tác dụng với axit HCl là 0,02 mol và tạo ra 0,02 mol khí H 2

Vậy V

2

0,02 22,4 0,448l

H

3

Phát hành PDF bởi Ths Nguyễn Thanh Tú

Đăng ký Word doc qua Zalo 0905779594 Email thanhtuqn88@gmail.com

85


Bài 5

Chọn 7 chất rắn khác nhau mà khi cho mỗi chất đó tác dụng với dung dịch H 2 SO 4 đặc,

nóng, dư đều cho sản phẩm là Fe 2 (SO 4 ) 3 , SO 2 và H 2 O. Viết các phương trình hóa học.

Hướng dẫn

Các chất rắn có thể chọn: Fe;FeO;Fe 3 O 4 ;Fe(OH) 2 ;FeS;FeS 2 ;FeSO 4

Các pthh :

2Fe + 6H 2 SO 4 (đặc)

0

t

Fe 2 (SO 4 ) 3 + 3SO 2 + 6H 2 O

2FeO + 4H 2 SO 4 (đặc)

0

t

Fe 2 (SO 4 ) 3 +SO 2 + 4H 2 O

2Fe 3 O 4 + 10H 2 SO 4 (đặc)

2Fe(OH) 2 + 4H 2 SO 4 (đặc)

0

t

3 Fe 2 (SO 4 ) 3 + SO 2 + 10H 2 O

0

t

Fe 2 (SO 4 ) 3 + SO 2 + 6H 2 O

2FeS + 10H 2 SO 4 (đặc)

2FeS 2 + 14H 2 SO 4 (đặc)

0

t

Fe 2 (SO 4 ) 3 + 9SO 2 + 10H 2 O

0

t

Fe 2 (SO 4 ) 3 + 15SO 2 + 14H 2 O

0

t

2FeSO 4 + 2H 2 SO 4 (đặc) Fe 2 (SO 4 ) 3 + SO 2 + 2H 2 O

Bài 6

Cho mẫu chất chứa Fe 3 O 4 , Fe 2 O 3 và các tạp chất trơ. Hòa tan mẫu vào lượng dư

dung dịch KI trong môi trường axit (khử tất cả Fe 3+ thành Fe 2+ ) tạo thành dung dịch A.

Pha loãng A đến thể tích 50 ml. Lượng I 2 có trong 10 ml dung dịch A phản ứng vừa đủ

với 5,5 ml dung dịch Na 2 S 2 O 3 1M (sinh ra S 4 O 2

6

). Lấy 25 ml mẫu dung dịch A khác,

chiết tách I 2 , lượng Fe 2+ trong dung dịch còn lại phản ứng vừa đủ với 3,2 ml dung dịch

MnO 4

1M trong H 2 SO 4 .

a. Viết phương trình phản ứng xảy ra (dưới dạng phương trình ion thu gọn).

b. Tính phần trăm khối lượng Fe 3 O 4 và Fe 2 O 3 trong mẫu ban đầu.

Hướng dẫn

a) Phương trình phản ứng:

Fe 3 O 4 + 2I - + 8H + → 3Fe 3+ + I 2 + 4H 2 O (1)

Fe 2 O 3 + 2I - + 6H + → 2Fe 3+ + I 2 + 3H 2 O (2)

2S 2 O 2

3

+ I 2 → S 4 O 2

6

+ 2I - (3)

5Fe 2+ + MnO 4

+ 8H + → 5Fe 3+ + Mn 2+ + 4H 2 O (4)

Phát hành PDF bởi Ths Nguyễn Thanh Tú

Đăng ký Word doc qua Zalo 0905779594 Email thanhtuqn88@gmail.com

86


b) (3) => n 1 1

I

n .0,0055.1 0,00275 mol

2 (3)

2

2

S 2 O 3 2

(4) => n 5n 5.0,0032.1 0,016 mol

2

Fe (4)

MnO4

Đặt số mol Fe 3 O 4 và Fe 2 O 3 lần lượt là x và y, ta có:

3x + 2y = 0,016.2 = 0,032

x + y = 0,00275.5 = 0,01375

x = 0,0045 và y = 0,00925

%m Fe3O4 = 17,4% và %m Fe2O3 = 24,7%

Bài 7 : Hoà tan hoàn toàn hỗn hợp X gồm a mol Cu x FeS y và b mol FeS y (a:b=1:3; x, y

nguyên dương) trong dung dịch HNO 3 đặc nóng thu được dung dịch Y chỉ gồm hai muối

sunfat. Xác định x, y.

Hướng dẫn:

Áp dụng định luật bảo toàn nguyên tố Cu, Fe, S ta có:

6 + x = 4y

Do x, y nguyên dương nên chỉ có thể x = 2 và y = 2

Bài 8. Sự gỉ sắt diễn ra ở 25 0 C, 1atm theo phương trình phản ứng:

4Fe + 3O 2 2Fe 2 O 3 .

a. Tính S 0 của phản ứng. Biết S 0 của Fe, O 2 và Fe 2 O 3 tương ứng bằng 27,3; 205 và

87,4 J/K.mol.

b. Bằng cách tính biến thiên entropi của hệ cô lập hãy cho biết quá trình trên là tự diễn

biến hay không? Biết nhiệt sinh của Fe 2 O 3 ở điều kiện đã cho là -824,2 kJ/mol.

Hướng dẫn

a. S 0 pứ = 2 S 0 (Fe 2 O 3 ) – 4 S 0 (Fe) – 3 S 0 (O 2 )

= 2.87,4 – 4.27,3 – 3.205 = -549,4 J/K.mol

b. Sự gỉ của sắt đã tỏa năng lượng dưới dạng nhiệt ra môi trường xung quanh một lượng

bằng: -824,2.2 = -1648,4 kJ/mol, do đó làm tăng entropi của môi trường một lượng bằng:

S 0 mt = 1648400/298 = 5531,5 J/K.mol

S 0 hệ = -549,4 J/K.mol

=> S 0 cô lập = S 0 mt + S 0 hệ = 5531,5 + (-549,4) = 4982,1.mol

S 0 cô lập > 0 chứng tỏ sự gỉ là quá trình tự phá hủy của kim loại ở điều kiện thường về

nhiệt độ và áp suất.

Bài 9

Phát hành PDF bởi Ths Nguyễn Thanh Tú

Đăng ký Word doc qua Zalo 0905779594 Email thanhtuqn88@gmail.com

87


Trộn 1 ml dung dịch A gồm HCl 0,1M, FeCl 3 0,02M và NiCl 2 0,001M vào 1 ml dung

dịch NaOH 0,162M tạo thành hỗn hợp X.

(a) Tính pH X .

(b) Tính nồng độ cân bằng của ion Fe 3+ . So sánh với nồng độ cân bằng của Fe 3+ trong

dung dịch Fe(OH) 3 bão hòa.

Cho: pK S (Fe(OH) 3 ) = 37; pK S (Ni(OH) 2 ) = 14,7; lg*β (FeOH 2+ ) = -2,17; lg*β

(NiOH 2+ ) = -8,94.

Hướng dẫn

(a) H + + OH - → H 2 O

[] 0 0,031

Fe 3+ + 3OH - → Fe(OH) 3

[] 0 0,001

Ni 2+ +2OH - → Ni(OH) 2

[] 0 0

TPGH: Fe(OH) 3 ; Ni(OH) 2 ; H 2 O

Xét dung dịch X:

2 14,7

2

s1

Ni(OH) Ni 2OH K 10 (1)

2 8,94

s1

Ni NiOH H * 10 (2)

H OH H O K 10 (3)

2 w

9,64

2

1

Ni(OH) NiOH OH K 10 (4)

3 37

3

s2

Fe(OH) Fe 3OH K 10 (5)

2 25,17

3

2

Fe(OH) FeOH 2OH K 10 (6)

14

2

w

H O H OH K 10 (7)

So sánh: K 4 K 7 K 1 K 6 K 5 bỏ qua K 6 ; K 5 .

Đánh giá K 4 , K 1 , K 7 :

7 K

3 s1

5

[OH ] 7 Kw 10 ;[OH ] 1 2 1,59.10 ;

4

5

[OH ] K 1,51.10 [OH ]

4 1 1

bỏ qua (7) tính theo (1) và (4)

14

Phát hành PDF bởi Ths Nguyễn Thanh Tú

Đăng ký Word doc qua Zalo 0905779594 Email thanhtuqn88@gmail.com

88


OH 2 Ni 2 NiOH

2 Ks1 K

1

=2,057.10 -5

[OH ] 2 [OH ]

pH = 9,31. [Fe 3+ ] X = 1,15.10 -23

Có thể không tính gần đúng mà dùng điều kiện proton vẫn cho ra đúng kết quả.

(b) Xét dung dịch bão hòa Fe(OH) 3 chỉ có các cân bằng (5), (6), (7)

K 7 K 6 K 5

37

7 3 10 16

w 7.3

h K 10 [Fe ] 10

10

Vậy [Fe 3+ ] X nhỏ hơn nhiều so với [Fe 3+ ] bão hòa trong dung dịch Fe(OH) 3 .

Bài 10

Có thể điều chế tinh thể FeCl 3 .6H 2 O theo cách sau: Hoà tan sắt kim loại vào trong dung

dịch axit clohydric 25%. Dung dịch tạo thành được oxy hóa bằng cách sục khí clo qua

cho đến khi cho kết quả âm tính với K 3 [Fe(CN) 6 ].

Dung dịch được cô bay hơi ở 95 o C cho đến khi tỉ trọng của nó đạt chính xác 1,695 g/cm 3

và sau đó làm lạnh đến 4 o C. Tách kết tủa thu được bằng cách hút chân không rồi cho vào

một dụng cụ chứa được niêm kín.

1. Viết các phản ứng dẫn đến sự kết tủa FeCl 3 .6H 2 O

2. Có bao nhiêu gam sắt và bao nhiêu mL dung dịch axit clohydric 36% (d=1,18g/cm 3 )

cần để điều chế 1,00kg tinh thể này. Biết rằng hiệu suất quá trình chỉ đạt 65%

3. Đun nóng 2,752g FeCl 3 .6H 2 O trong không khí đến 350 o C thu được 0,8977g bã rắn.

Xác định thành phần định tính và định lượng của bã rắn.

Hướng dẫn

1. Các phương trình phản ứng:

Fe + 2HCl FeCl 2 + H 2

2FeCl 2 + Cl 2 2FeCl 3

3FeCl 2 + 2K 3 [Fe(CN) 6 ] Fe 3 [Fe(CN) 6 ] 2 + 6KCl

FeCl 3 + 6H 2 O FeCl 3 .6H 2 O

2.

1000

3,7 mol FeCl3.6H2O

270,3

3,7 236,5

Như vậy cần

978 mL dung dịch HCl 36%

0,361,180,65

3. - Khi đun nóng thì FeCl 3 .6H 2 O phân huỷ theo phương trình sau:

Phát hành PDF bởi Ths Nguyễn Thanh Tú

Đăng ký Word doc qua Zalo 0905779594 Email thanhtuqn88@gmail.com

89


FeCl 3 .6H 2 O FeOCl + 5H 2 O + 6HCl

- Khi nhiệt độ tăng thì FeOCl sẽ tiếp tục phân huỷ:

3FeOCl FeCl 3 + Fe 2 O 3 (Hơi FeCl 3 bay ra)

- Lượng FeCl 3 .6H 2 O trong mẫu là 2,752 10,18mmol

270,3

- Điều này ứng với khối lượng FeCl 3 là 107,3. 0,01018 = 1,092g

- Do khối lượng thu được của bã rắn bé hơn nên ta biết được FeOCl sẽ bị phân hủy một

phần thành Fe 2 O 3 .

Khối lượng FeCl 3 mất mát do bay hơi là: 1,902 0,8977 1, 2mmol

162, 2

=> Bã rắn cuối cùng chứa (0,01018 – 3.0,00120) = 6,58 mmol FeOCl và 1,20 mmol

Fe 2 O 3 .

Bài 11

Hoà tan hoàn toàn 0,8120 gam một mẫu quặng sắt gồm FeO, Fe 2 O 3 và 35% tạp chất trơ

trong dung dịch HCl (dư), thu được dung dịch X. Sục khí SO 2 vào dung dịch X, thu được

dung dịch Y. Dung dịch Y phản ứng vừa đủ với 22,21 ml dung dịch KMnO 4 0,10 M. Mặt

khác, hoà tan hết 1,2180 gam mẫu quặng trên trong dung dịch HCl (dư) rồi thêm ngay

dung dịch KMnO 4 0,10 M vào dung dịch thu được cho đến khi phản ứng xảy ra hoàn toàn,

thì hết 15,26 ml dung dịch KMnO 4 0,10 M.

a) Viết phương trình hóa học của các phản ứng xảy ra.

b) Tính thể tích SO 2 (ở điều kiện tiêu chuẩn) đã dùng và thành phần phần trăm theo khối

lượng của FeO, Fe 2 O 3 có trong mẫu quặng.

Hướng dẫn:

a) FeO + 2 HCl FeCl 2 + H 2 O (1)

Fe 2 O 3 + 6 HCl 2 FeCl 3 + 3 H 2 O (2)

2 FeCl 3 + 2 H 2 O + SO 2 2 FeCl 2 + H 2 SO 4 + 2 HCl (3)

5 FeCl 2 + KMnO 4 + 8 HCl 5 FeCl 3 + MnCl 2 + KCl + 4 H 2 O (4)

5 SO 2 + 2 KMnO 4 + 2 H 2 O 2 H 2 SO 4 + 2 MnSO 4 + K 2 SO 4 (5)

b) Từ (1) và (4) ta có:

n FeO (trong 1,2180 gam mẫu) = n 2

Fe = 5. n = 5 . 0,10 . 15,26.10 -3 = 7,63.10 -3 (mol)

MnO 4

-3

7,63.10 . 0,8120

n FeO (trong 0,8120 gam mẫu) =

= 5,087.10 -3 (mol)

1,2180

m FeO (trong 0,8120 gam mẫu) = 72 . 5,087.10 -3 = 0,3663 (g)

Phát hành PDF bởi Ths Nguyễn Thanh Tú

Đăng ký Word doc qua Zalo 0905779594 Email thanhtuqn88@gmail.com

90


và m

Fe2O3

(trong 0,8120 gam mẫu) = 0,8120 . 0,65 – 0,3663 = 0,1615 (g)

n

Fe2O

(trong 0,8120 gam mẫu) = 0,1615

3

160

1,01.10 -3 (mol)

Tương tự, từ (3) và (5) ta có: n n n

2 2 2

1

Trong đó: n

SO 2 (3)

= . nFeCl3

2

SO SO (3) SO (5)

(trong 0,8120 gam mẫu) = n

Fe2O

(trong 0,8120 gam mẫu) = 1,01.10 -3 (mol)

3

5

n SO -

2 (5)

n

MnO 4 (5)

2

= 5 1

( n

MnO n )

-

4

Fe

2 5

2

với: n 2

Fe = n FeO (trong 0,8120 gam mẫu) + 2.n

Fe2O

(trong 0,8120 gam mẫu)

3

n

SO 2 (5)

=

5 ( n 1 (n

2 5

MnO -

FeO (trong 0,8120 gam mẫu) Fe

4

2O3

3 3 3

SO 2 (5)

+ 2.n (trong 0,8120 gam mẫu))

5

- 1

- -

n = 0,10 . 22,21.10 - (5,087.10 + 2 . 1,01.10 )

2

5

2.10-3 (mol).

Vậy: n SO

3,01.10 -3 (mol) V

2

SO 2

= 22,4 . 3,01.10 -3 = 0,0674 (lit)

% FeO = 0,3663 .100

0,8120

= 45,11 %

% Fe 2 O 3 = 65 % – 45,11 % = 19,89 %

KẾT LUẬN VÀ KIẾN NGHỊ

KẾT LUẬN

Sau một quá trình nghiên cứu, đề tài đã thu được những kết quả sau:

Tiến hành xây dựng (sưu tầm, lựa chọn, biên soạn, phân loại) được 90 bài tập trắc

nghiệm ôn tập kiến thức và 61 bài tập dãy chuyển hóa, tinh thể, tính toán, các phản ứng

tạo phức, phản ứng trong cân bằng dung dịch,... của kim loại nhóm VIIB, VIIIB. Tất cả

các bài tập đều có đáp án, hướng dẫn giải chi tiết, đầy đủ. Đây là nguồn bài tập giáo viên

có thể đễ dàng sử dụng trong quá trình giảng dạy, ôn luyện học sinh giỏi, ra đề kiểm tra,

Phát hành PDF bởi Ths Nguyễn Thanh Tú

Đăng ký Word doc qua Zalo 0905779594 Email thanhtuqn88@gmail.com

91


đề thi; làm tài liệu học tập cho học sinh đặc biệt cho học sinh chuyên về phần kim loại

chuyển tiếp.

- Hệ thống bài tập trắc nghiệm: 90 bài

- Hệ thống bài tập tự luận: 61 bài được chia thành các dạng

+ Cấu tạo nguyên tử, phân tử, từ tính: 4 bài

+ Sơ đồ phản ứng, dãy chuyển hóa kim loại nhóm VIIB, VIIIB: 6 bài

+ Tinh thể của kim loại nhóm VIIB, VIIIB: 10 bài

+ Bài toán liên quan đến phản ứng hạt nhân: 4 bài

+ Cân bằng trong dung dịch điện li: 4 bài

+ Phản ứng oxh-k và bài tập phần điện hóa: 10 bài

+ Bài tập về phức của các kim loại nhóm VIIB, VIIIB: 12 bài

+ Các bài tập tính toán khác: 11 bài

KIẾN NGHỊ

Tiếp tục xây dựng các chuyên đề khác về hóa vô cơ

Khi xây dựng và sử dụng hệ thống bài tập hóa vô cơ, cần lưu ý:

- Xây dựng hệ thống bài tập phải đảm bảo tính chính xác, khoa học.

- Để xây dựng được một hệ thống bài tập tốt, thiết thực và sử dụng hiệu quả đòi hỏi:

+ GV phải nắm kiến thức lý thuyết vững, GV buộc phải giải qua các đề thi HSG hóa học

các cấp. Có như vậy GV mới có được sự nhìn nhận bao quát về chương trình, dự đoán

hướng ra đề thi HSG, từ đó nâng cao chất lượng bồi dưỡng.

+ Hệ thống bài tập kim loại nhóm VIIB, VIIIB tôi xây dựng trên cơ sở tuyển chọn những

bài tập ở mức độ dễ đến khá khó từ các sách tham khảo, nguồn bài tập trên mạng và đề

thi HSG các cấp. Để phù hợp với mục đích rèn luyện kỹ năng và phát triển nhận thức của

HSG hóa học, tùy tình hình thực tế HS ở mỗi trường mà GV lọc tách bài để luyện tập cho

phù hợp với nội dung và mục đích rèn luyện.

+ Giáo viên cần sáng tạo để được những bài tập tương đương cho học sinh giải. Từ bài

tập đã giải, thay đổi, thêm, bớt các dữ kiện thành bài tập mới. Dần dần khuyến khích, yêu

cầu học sinh tự biến đổi thành bài tập mới. Như vậy, học sinh vừa được làm quen với

Phát hành PDF bởi Ths Nguyễn Thanh Tú

Đăng ký Word doc qua Zalo 0905779594 Email thanhtuqn88@gmail.com

92


phương pháp giải bài tập, vừa biết được phương pháp đó áp dụng trong những tình huống

nào.

+ Bài tập phải gắn liền hoá học với thực tế.

TÀI LIỆU THAM KHẢO

1. Hoàng Nhâm (2017), Hóa học vô cơ cơ bản 3. NXB Giáo dục Việt Nam.

2. Nguyễn Duy Ái, Đào Hữu Vinh (2008), Bài tập đại cương và vô cơ. NXB Giáo dục,

Hà Nội

3. Các đề thi HSG Quốc Gia (chính thức và thi thử)

4. Các đề thi Hóa Quốc tế ICHO

Huế, ngày 4 tháng 8 năm 2019

Người viết chuyên đề:

Giáo viên: NGUYỄN THỊ HỒNG HÀ

Tổ: Hóa học

Đơn vị: Trường THPT Chuyên Quốc Học

Phát hành PDF bởi Ths Nguyễn Thanh Tú

Đăng ký Word doc qua Zalo 0905779594 Email thanhtuqn88@gmail.com

93


CHUYÊN ĐỀ DHBB

XÂY DỰNG HỆ THỐNG CÂU HỎI

VÀ BÀI TẬP NHÓM VII.B VÀ VIII.B

NĂM HỌC 2019-2020

Phát hành PDF bởi Ths Nguyễn Thanh Tú

Đăng ký Word doc qua Zalo 0905779594 Email thanhtuqn88@gmail.com


1.1. Lý do chọn đề tài

1.1.1. Thực trạng

Phần 1. MỞ ĐẦU

Quá trình vận dụng kiến thức thông qua các bài tập có rất nhiều hình thức

phong phú và đa dạng. Thông qua bài tập, giáo viên sẽ đánh giá được khả

năng nhận thức, khả năng vận dụng kiến thức của học sinh, thông qua việc

giải các bài tập mà kiến thức được củng cố khắc sâu, chính xác hoá, mở rộng

và nâng cao, đồng thời còn bồi dưỡng năng lực tư duy sáng tạo cho học sinh.

Trong các tài liệu hiện hành thì chủ yếu là các tài liệu cho học sinh ôn

luyện thi đại học và cao đẳng, các tài liệu dành cho học sinh giỏi, học sinh

chuyên còn ít và đặc biệt tài liệu dành cho thi HSG Quốc gia, Quốc tế thực sự

là quá ít, với môn Hóa học cũng không nằm ngoài bối cảnh khó khăn chung

đó, sự thiếu hệ thống trong mỗi loại chuyên đề bồi dưỡng gây ra một sự rời

rạc và mất tính kế thừa.

Trong các tài liệu hiện hành dành cho học sinh giỏi, học sinh chuyên

không có nhiều câu hỏi và bài tập về các nguyên tố nhóm VII.B và VIII.B.

Đồng thời, phần kiến thức mà các đề thi HSG Quốc Gia, Quốc tế hàng năm

lại chiếm tỉ lệ lớn về các nguyên tố nhóm VII.B và VIII.B.

Từ những khó khăn vừa mang tính chủ quan, vừa mang tính khách quan

nêu trên, tôi thiết nghĩ cần có các bộ chuyên đề bồi dưỡng để tổng hợp lại

những dạng bài tập trong các kỳ thi học sinh giỏi để tạo ra một sự thuận lợi

trong việc giảng dạy của giáo viên và học tập của học sinh.

1.1.2. Ý tưởng

Xuất phát từ vấn đề khó khăn đã nêu ở trên, qua kinh nghiệm thực tế giảng

dạy với mong muốn góp phần vào việc nâng cao chất lượng giảng dạy bộ

môn Hóa học, tôi mạnh dạn trình bày chuyên đề về “Xây dựng hệ thống câu

hỏi và bài tập nhóm VII.B và VIII.B” với các bài tập được lựa chọn từ nhiều

nguồn khác nhau mang tính cập nhật từ các kì thi trong nước và quốc tế với

mong muốn phần nào làm giảm được những khó khăn đó, thông qua một số

ví dụ cụ thể về các dạng bài tập có lời giải và không có lời giải, nhằm giúp

cho học sinh rút ra kinh nghiệm, rèn luyện kĩ năng khi làm bài, đồng thời để

góp phần nâng cao hiệu quả giảng dạy ở trường trung học phổ thông Chuyên.

Phát hành PDF bởi Ths Nguyễn Thanh Tú

Đăng ký Word doc qua Zalo 0905779594 Email thanhtuqn88@gmail.com


1.2. Mục đích của đề tài

1. Nguyên cứu và hệ thống hóa lý thuyết về các nguyên tố nhóm VII.B và

VIII.B.

2. Xây dựng hệ thống câu hỏi và bài tập theo từng dạng, từng đơn vị kiến

thức và đáp án của từng câu hỏi, bài tập.

3. Tạo ra nguồn tài liệu về các nguyên tố nhóm VII.B và VIII.B có chất

lượng để phục vụ công tác bồi dưỡng HSG của bản thân, đồng thời chia sẻ

với đồng nghiệp để hoàn thiện hơn.

1.3. Nhiệm vụ

1. Nghiên cứu chương trình hóa học phổ thông nâng cao và chuyên hóa học,

phân tích các đề thi học sinh giỏi cấp tỉnh, khu vực, cấp quốc gia, quốc tế và

đi sâu về nội dung liên quan đến các dạng bài tập về các nguyên tố nhóm

VII.B và VIII.B.

2. Hệ thống hóa lý thuyết về các nguyên tố nhóm VII.B và VIII.B.

3. Sưu tầm, lựa chọn trong tài liệu giáo khoa, sách bài tập cho học sinh, trong

các tài liệu tham khảo. Các đề thi học sinh giỏi các cấp có nội dung liên quan;

phân loại, xây dựng các bài tập lí thuyết và tính toán.

4. Đề xuất phương pháp xây dựng và sử dụng hệ thống bài tập dùng cho việc

giảng dạy, bồi dưỡng học sinh giỏi các cấp ở trường THPT chuyên.

1.4. Giả thuyết khoa học

Nếu giáo viên giúp học sinh nắm vững vấn đề lí thuyết và xây dựng

được hệ thống bài tập chất lượng, đa dạng, phong phú đồng thời có phương

pháp sử dụng chúng một cách thích hợp thì sẽ nâng cao được hiệu quả của

quá trình bồi dưỡng học sinh giỏi ở trường THPT Chuyên, học sinh phát huy

được tính tích cực, tư duy sáng tạo và tăng niềm đam mê nghiên cứu khoa

học.

1.5. Phương pháp nghiên cứu

1. Nghiên cứu thực tiễn dạy học và bồi dưỡng học sinh giỏi hóa học ở

trường THPT Chuyên.

2. Nghiên cứu các tài liệu về phương pháp dạy học hóa học, các tài liệu về

bồi dưỡng học sinh giỏi, các đề thi học sinh giỏi,…

3. Thu thập tài liệu và truy cập thông tin trên internet có liên quan đến

chuyên đề.

Phát hành PDF bởi Ths Nguyễn Thanh Tú

Đăng ký Word doc qua Zalo 0905779594 Email thanhtuqn88@gmail.com


4. Đọc, nghiên cứu và xử lý các tài liệu.

1.6. Điểm mới của chuyên đề

- Chuyên đề đã xây dựng được hệ thống lí thuyết cơ bản có mở rộng và nâng

cao đầy đủ, hệ thống bài tập lí thuyết và bài tập tính toán có phân loại rõ ràng

các dạng bài để làm tài liệu phục vụ cho học sinh và giáo viên trường chuyên

học tập, giảng dạy, ôn luyện, bồi dưỡng trong các kì thi học sinh giỏi các cấp

và làm tài liệu học tập cho học sinh. Ngoài ra, chuyên đề còn là tài liệu tham

khảo mở rộng và nâng cao cho giáo viên môn hóa học và học sinh yêu thích

môn hóa học nói chung.

- Chuyên đề mở rộng hướng sang cả dạng bài tập về tinh thể, phức chất... và

phần thực hành – chuẩn độ tạo phức để giúp học sinh có tầm nhìn bao quát

hơn và rèn kĩ năng thí nghiệm tốt hơn.

- Đề xuất phương pháp xây dựng và sử dụng có hiệu quả hệ thống bài tập hóa

học.

1.7. Cấu trúc của đề tài

1. Tổng quan kiến thức của các nguyên tố nhóm VII.B và VIII.B

2. Hệ thống câu hỏi dạng lý thuyết của các nguyên tố nhóm VII.B và

VIII.B

3. Hệ thống câu hỏi dạng bài tập của các nguyên tố nhóm VII.B và VIII.B

4. Một số bài thực hành có liên quan đến các nguyên tố nhóm VII.B và

VIII.B

Phát hành PDF bởi Ths Nguyễn Thanh Tú

Đăng ký Word doc qua Zalo 0905779594 Email thanhtuqn88@gmail.com


Phần 2. NỘI DUNG

2.1. Tổng quan kiến thức của các nguyên tố nhóm VII.B và VIII.B

2.1.1. Các nguyên tố nhóm VII.B ( Mangan – Tecnexi – Reni)

2.1.1.1. Nhận xét chung về các nguyên tố nhóm VII.B

(1). Ba nguyên tố Mn (mangan), Tc (tecnexi), Re (reni) thuộc họ d ở các chu

kỳ 4, 5, 6 thuộc bảng tuần hoàn.

a. Mangan được Silơ tìm ra năm 1774, là nguyên tố nhân tạo.

Tecnexi được tổng hợp vào năm 1973 do các nhà khoa học Italia là Perie

và Segro khi chiếu chùm đơteri vào bản molipden.

Reni được hát hiện vào năm 1925 liên quan đến tên tuổi của các nhà bác

học Đức là W.Noddak, I.Noddak, O.Berg.

b. Nguyên tử khối, số thứ tự nguyên tố, sự phân bố electron và trạng thái hóa

trị của ba nguyên tố trên:

Bảng 1. Bảng nguyên tử khối, số thứ tự nguyên tố, sự phân bố electron và

trạng thái hóa trị

Nguyên Kí STT NTK Phân bố electron Hóa trị

tố hiệu

Mangan Mn 25 55,938 2 8 13 2 I, II, III, IV, V,

VI, VII

Tecnexi Tc 43 [99] 2 8 18 13 2 IV , VI, VII

Reni Re 75 186,210 2 8 18 32 13 2 I, II, III, IV, V,

VI, VII

(2) a. Cả ba nguyên tố có số electron đều được phân bố

như sau: (n-1)d 5 ns 2

(n-1)d 5 ns 2 np

Phát hành PDF bởi Ths Nguyễn Thanh Tú

Đăng ký Word doc qua Zalo 0905779594 Email thanhtuqn88@gmail.com


b. Mn, Tc, Re là những nguyên tố đa hóa trị gây ra bởi các electron hóa trị.

Bậc oxi hóa đặc trưng của Mn là +2, +4 và +7, ngoài ra còn tạo ra những hợp

chất ứng với bậc oxi hóa +3, +4 và +6. Bậc oxi hóa đặc trưng của Te, Re là

+7. Khi theo chiều tăng của bậc oxi hóa, khuynh hướng tạo ra anion phức tăng

lên, còn khuynh hướng tạo ra cation phức giảm xuống.

(3). a. Tương tự các halogen – đặc biệt là clo – Mn và Re tạo ra hợp chất

Mn 2 O 7 và Re 2 O 7 có tính chất tương tự Cl 2 O 7 ; các muối pemanganat (MnO 4 - )

renat (ReO 4 - ) đồng hình với muối perclorat (ClO 4 - ); cả hai axit pemanganic

(HMnO 4 ) và axit percloric (HClO 4 ) đều là axit đơn chức và đều có tính oxi

hóa mạnh.

b. Trong số các hợp chất của mangan, có một số giống hợp chất của crom

và của sắt.

Mangan và crom tạo ra oxit có số oxi hóa thấp (tính bazơ) và trioxit (tính

axit). Muối cromat (CrO 4 2- ) đồng hình với muối manganat (MnO 4 - ). Mangan

cũng tạo nên phèn tương tự như phèn sắt K 2 SO 4 .Mn 2 (SO 4 ) 3 . 24H 2 O hoặc dạng

muối kép như (NH 4 ) 2 SO 4 .MnSO 4 .6H 2 O tương tự muối kép của sắt (II). Ngoài

ra mangan còn tạo ra các oxit MnO, Mn 2 O 3 , Mn 3 O 4 (MnO.Mn 2 O 3 ) tương tự

các oxit của sắt.

(4) Bảng 2. Một số đặc điểm của Mn, Tc, Re.

Mn Tc Re

Electron hóa trị 3d 5 4s 2 4d 5 5s 2 5d 5 6s 2

Bán kính nguyên tử (A 0 ) 1,30 1,36 1,37

Bán kính ion M 2+ (A 0 ) 0,91 0,95 -

Bán kính ion M 3+ (A 0 ) 0,70 - -

Bán kính ion M 4+ (A 0 ) 0,52 0,72 0,72

Bán kính ion M 7+ (A 0 ) 0,46 0,57 0,57

Thế ion hóa I 1 (eV) 7,34 7,23 7,87

Bán kính nguyên tử tăng từ Mn đến Re, nhưng không đáng kể, đặc biệt khi

chuyển từ Tc đến Re, do đó Tc và Re có tính chất gần nhau hơn so với Mn.

(5) Sơ đồ thế điện cực của Mn và Re

a. Trong môi trường axit

-

2-

3-

MnO 4 MnO 4 MnO 4 MnO 2 Mn 3+ Mn 2+ Mn

+0,56 +0,27 +4,27 +0,95 +1,5 -1,15

Phát hành PDF bởi Ths Nguyễn Thanh Tú

Đăng ký Word doc qua Zalo 0905779594 Email thanhtuqn88@gmail.com


+2,27 +1,23

+1,7

ReO 4

-

+0,77 +0,63

ReO 3 +1,51 ReO 2 Re 3+ +0,3

Re 0

+0,51 +0,26

+0,12 +0,51

2-

ReCl 6

+0,37

b. Trong môi trường bazơ

-

2-

3-

MnO 4 MnO 4 MnO 4 MnO 2 Mn 2 O 3 Mn(OH) 2

Mn +0,56 +0,27 +0,96 +0,15 -0,25 -1,51

+0,62 -0,05

+0,60

+0,34

-

ReO 4

Re 0

ReO 3 ReO 2 .2H 2 O Re 2 O 3

-0,89 -0,446 -1,25 -0,333

-0,594 -0,56

-0,584

(6) Về mặt cấu tạo, các dạng thù hình của mangan đều kết tinh theo kiểu lập

phương; còn reni kết tinh theo kiểu lục phương.

Ví dụ: dạng thù hình - Mn tồn tại ở 1070 -1130 0 C có dạng cấu trúc mạng lập

phương tâm diện có cạnh = 3,862 A 0 , theo dạng cấu trúc của Cu. Còn thù

Phát hành PDF bởi Ths Nguyễn Thanh Tú

Đăng ký Word doc qua Zalo 0905779594 Email thanhtuqn88@gmail.com


hình d - Mn theo dạng - Fe, nghĩa là kết tinh theo kiểu lập phương tâm khối

có cạnh = 3,081 A 0 .

2.1.1.2. Trạng thái tự nhiên và thành phần các đồng vị

(1). *Khoáng vật chủ yếu của mangan là pirolusit (MnO 2 ); ngoài ra một số

khoáng vật khác có chứa mangan như bronit (Mn 2 O 3 ); manganit Mn 2 O 3 .H 2 O

và các sunfua MnS; MnS 2 .

*Tecnexi là nguyên tố nhân tạo.

*Lượng reni ở trong vỏ quả đất có rất ít. Quặng giàu reni nhất là các

molipdenit cũng chỉ chứa khoảng 2.10 -3 % về khối lượng.

(2). *Trong cơ thể người, mangan có khoảng 4.10 -4 % trong tim, gan và tuyến

thượng thận, ảnh hưởng đến sự trưởng thành của cơ thể và sự tạo máu.

*Trong nước biển có chứa rất ít mangan, nhưng dưới đáy biển có một

lượng rất lớn mangan.

(3). Hàm lượng các kim loại trên ở Mặt Trăng do các tàu vũ trụ Apolo và tàu

Luna được có giá trị sau:

Bảng 3. Hàm lượng trung bình của Mn, Tc, Re trên Mặt Trăng

Nguyên tố Hàm lượng trung bình (số gam/1 g mẫu đá)

Mn

Tc

Apolo – 11 Apolo - 12 Luna – 6

1,9.10 -3 1,9.10 -3 -

- - -

Re 0,01.10 -6 - -

(4). *Mangan có nhiều đồng vị từ 49 Mn đến 57 Mn, trong đó chỉ có 55 Mn là

đồng vị thiên nhiên chiếm 100%.. Đồng vị phóng xạ bền nhất là 53 Mn có chu

kỳ bán hủy là 14 năm và kém bền nhất là 49 Mn có chu kỳ bán hủy là 0,4 giây.

*Tecnexi là nguyên tố nhân tạo, các đồng vị đều có tính phóng xạ, trong

số đó đồng vị 99 Tc bền nhất có chu kỳ bán hủy là 2,12.10 5 .

*Reni có 14 đồng vị. Các đồng vị thiên nhiên là 185 Re (37,07%); 187 Re

(62,93%); còn lại là các đồng vị phóng xạ.

2.1.1.3. Điều chế Mn, Tc, Re

(1). *Mangan được điều chế bằng phương pháp nhiệt nhôm từ các oxit MnO

hoặc Mn 3 O 4 t 0

Phát hành PDF bởi Ths Nguyễn Thanh Tú

Đăng ký Word doc qua Zalo 0905779594 Email thanhtuqn88@gmail.com


3Mn 3 O 4 +8Al 4Al 2 O 3 + 9Mn

*Mangan cũng có thể điều chế bằng phương pháp nhiệt silic

t 0

MnO 2 + Si Mn + SiO 2

*Trong công nghiệp, mangan được điều chế bằng cách dùng cabon để

khử oxit mangan trong lò điện.

*Mangan cũng được điều chế bằng phương pháp điện phân dung dịch

muối sunfat.

*Mangan tinh khiết được điều chế bằng cách điện phân dung dịch MnCl 2

với catot bằng thủy ngân. Mangan hòa tan trong thủy ngân tạo ra hỗn hống

Mn-Hg. Chưng cất hỗn hống trong chân không, tách được Mn và thu hồi lại

Hg.

(2). *Người ta điều chế một lượng nhỏ tecnexi (vài mg) trong lò phản ứng hạt

nhân khi bắn phá oxit molipđen bằng notron

+ +

+ -

*Cũng điều chế trong lò phản ứng hạt nhân khi phân hạch urani.

(3). Người ta điều chế reni bằng cách dùng H 2 để khử amoni renat ở nhiệt độ

cao

t 0

2NH 4 ReO 4 + 4H 2 2Re + N 2 + 8H 2 O

2.1.1.4. Tính chất lý học của Mn, Tc, Re

(1). *Tùy theo phương pháp điều chế, mangan tạo ra ở bốn dạng thù hình.

*Mangan được điều chế bằng phương pháp nhiệt nhôm, tồn tại ở hai dạng

- Mn và - Mn.

Dạng - Mn tồn tại ở nhiệt độ thường có khối lượng riêng là 7,21 g/cm 3 .

Dạng - Mn tồn tại ở nhiệt độ cao 742 – 1070 0 C có khối lượng riêng là 7,29

g/cm 3 .

*Nếu mangan được kết tủa bằng phương pháp điện phân, mangan tồn tại

ở dạng - Mn, bền trong khoảng 1070 – 1130 0 C, có khối lượng riêng là 7,21

g/cm 3 . Dạng thù hình tồn tại ở nhiệt độ cao hơn 1130 0 C là dạng d - Mn.

- Mn - Mn - Mn d - Mn

Phát hành PDF bởi Ths Nguyễn Thanh Tú

Đăng ký Word doc qua Zalo 0905779594 Email thanhtuqn88@gmail.com


Tồn tại:

Nhiệt độ thường 742 -1070 0 C 1070-1130 0 C >1130 0 C

*Các dạng - Mn và - Mn đều cứng và giòn; dạng - Mn thì mềm và

dẻo.

*Mn, Re có màu trắng bạc, còn Tc có màu xám.

*Một số hằng số vật lý của Mn, Tc, Re

Bảng 4. Một số hằng số vật lý quan trọng của Mn, Tc, Re

Tính chất Mn Tc Re

Khối lượng riêng (g/cm 3 )

Nhiệt độ nóng chảy (t nc 0 C)

Nhiệt độ sôi (t s 0 C)

Độ âm điện

7,4 11,5 21

1244 2700 3180

2120 4900 5670

1,5 1,9 1,9

(2) *Một lượng khá lớn mangan được dùng để điều chế hợp kim fero –

mangan (60% - 90% Mn; 40% - 10% Fe) khi khử hỗn hợp sắt và quặng

mangan. Loại hợp kim này rất bền và cứng, thường dùng làm ổ bi, các bộ

phận máy nghiền và làm đường ray xe lửa.

Người ta cũng dùng hợp kim của mangan để điều chế trực tiếp gang trắng

trong lò cao.

Hợp kim bronzơ – mangan (95%Cu và 5%Mn) có độ bền cơ học cao.

*Reni được dùng trong công nghiệp điện vì có độ dẫn điện cao ( ớn hơn

Hg 4,5 lần và thấp hơn W) là nguyên liệu rất tốt để làm dây tóc bóng đèn

điện, bền hơn W. Hợp kim Re và Pb dùng làm pin nhiệt điện.

Ngoài ra reni làm chất xúc tác cho nhiều phản ứng hóa học hữu cơ.

2.1.1.5. Tính chất hóa học của Mn, Tc, Re

Hoạt tính hóa học của kim loại trong nhóm giảm dần từ Mn đến Re do đó khả

năng phản ứng của các chất giảm dần.

(1) *Mangan và reni không phản ứng trực tiếp với hidro, nhưng khí H 2 tan

được trong mangan nóng chảy (70 – 100 cm 3 H 2 trong 100 g Mn ở nhiệt độ

1244 0 C)

*Reni không hòa tan được khí H 2 .

(2).*Trong không khí, mangan ở dạng khối rắn, không bị oxi hóa, ngay cả khi

đun nóng vì được bao bọc bởi một lớp oxit mỏng bảo vệ cho kim loại; nếu ở

Phát hành PDF bởi Ths Nguyễn Thanh Tú

Đăng ký Word doc qua Zalo 0905779594 Email thanhtuqn88@gmail.com


trạng thái vụn lại dễ bị oxi hóa hơn; nhưng mangan khó phản ứng với oxi, tạo

Mn 3 O 4 ở 940 0 C.

3Mn + 2O 2 Mn 3 O 4

Khi đun nóng bột reni trong khí quyển oxi ở 300 0 C tạo ra oxit Re 2 O 7 .

*Mangan và reni đều phản ứng trực tiếp với lưu huỳnh, selen, telu tạo ra

các hợp chất như MnSe, MnSe 2 , ReSe 2 , MnTe, MnTe 2 ,...

(3). *Mangan hóa hợp trực tiếp với nitơ tạo ra Mn 3 N 2 ở nhiệt độ khoảng 600

– 1000 0 C khi cho bột mangan tác dụng với nitơ:

N 2 + 3Mn Mn 3 N 2

*Mangan và reni phản ứng trực tiếp với photpho khi nung trong ampun

hàn tạo ra các chất Mn 3 P 2 , MnP, ReP,...

(4). Mangan hóa hợp trực tiếp với cacbon và silic tạo ra các hợp chất Mn 3 C,

Mn 7 C 3 , Mn 3 Si, MnSi,...

(5). Cả mangan và reni đều phản ứng mạnh với các halogen tạo thành muối

dạng MnX 2 , ví dụ:

Mn + Cl 2 MnCl 2

(6). *Mangan có thế điện cực đứng trước Zn, nên khi đun nóng phân hủy

được H 2 O, đặc biệt khi có tạp chất như cacbon, mangan dễ bị nước và không

khí ẩm ăn mòn:

Mn + 2H 2 O Mn(OH) 2 + H 2

Tecnexi và reni không có khả năng này.

*Mangan tan trong các axit loãng HCl, H 2 tạo ra H 2

Mn + H 2 SO 4 MnSO 4 + H 2

Tecnexi và reni không có khả năng này.

*Mangan tan trong H 2 SO 4 đặc, tạo ra SO 2 , nếu H 2 SO 4 đặc nguội phản ứng

xảy ra rất chậm, nhưng khi đun nóng phản ứng xảy ra rất nhanh.

Với HNO 3 tạo ra khí NO

t 0 C

Mn + 2H 2 SO 4(đặc, nóng) MnSO 4 + SO 2 + 2H 2 O

3Mn + 8HNO 3 3Mn(NO 3 ) 2 + 2NO + 4H 2 O

Mangan không thể hiện tính thụ động khi ngâm trong HNO 3 đặc nguội.

Phát hành PDF bởi Ths Nguyễn Thanh Tú

Đăng ký Word doc qua Zalo 0905779594 Email thanhtuqn88@gmail.com


Tecnexi và reni phản ứng mạnh với HNO 3 và H 2 SO 4 đặc tạo ra hợp chất ứng

với hóa trị bền của Tc(VII) và Re(VII) là

3Tc + 7HNO 3 3HTcO 4 + 7NO + 2H 2 O

2Re + 7H 2 SO 4 2HReO 4 + 7SO 2 + 6H 2 O

(7). Mangan không phản ứng với kiềm.

2.1.1.6. Các hợp chất với số oxi hóa +2

(1). Số phối trí đặc trưng của Mn(II) là bằng 6 ứng với dạng lai hóa sp 3 d 2 với

sự phân bố liên kết tám mặt.

Các hợp chất Mn(II) đều có tính thuận từ và đều có 5 electron độc thân. Cấu

tạo của phức chất tám mặt spin cao của Mn(II) tương ứng với cấu hình

electron

d d d d d d d d d d ∗ d ∗

Đa số các muối đều dễ hòa tan trong nước và đều là muối bền nhất của

Mn(II). Khi tan trong nước, các muối Mn(II) phân ly tạo ra phức – aquo

[Mn(OH) 6 ] 2+ và dung dịch có màu hồng.

Một số hợp chất không tan trong nước như MnO, MnS, MnF 2 , Mn(OH) 2 ,

MnCO 3 , Mn 3 (PO 4 ) 2 .

(2). Mangan (II) oxit (MnO) được điều chế bằng cách nhiệt phân Mn(II)

oxalat hoặc cacbonat khi không có không khí (nung trong luồng khí H 2 hoặc

N 2 )

t 0

COO

Mn MnO + CO + CO 2

COO

t 0

MnCO 3 MnO + H 2 O

Cũng dùng H 2 hoặc CO để khử oxit bậc oxi hóa cao như MnO 2 , Mn 3 O 4

MnO 2 + H 2 MnO + H 2 O

MnO là chất tinh thể, màu xanh xám, t nc

0

= 1780 0 C. Không tan trong nước,

không phản ứng với nước. Ở trạng thái tinh thể, hoàn toàn bền trong không

khí, nhưng ở dạng bột dễ bị oxi hóa tạo thành các oxit cao

6MnO + O 2 2Mn 3 O 4 (MnO 2 ; Mn 2 O 3 )

MnO bị H 2 khử thành kim loại nhưng phải ở nhiệt độ rất cao:

Phát hành PDF bởi Ths Nguyễn Thanh Tú

Đăng ký Word doc qua Zalo 0905779594 Email thanhtuqn88@gmail.com


MnO + H 2 Mn

+ H 2 O

MnO tan trong các axit tạo thành muối Mn(II)

MnO + 2HCl MnCl 2 + H 2 O

(3). Mangan (II) hidroxit (Mn(OH) 2 ) tương tự như Fe(OH) 2 , khi cho kiềm tác

dụng với dung dịch muối Mn(II) tạo ra kết tủa màu trắng:

MnSO 4 + 2NaOH Mn(OH) 2 + Na 2 SO 4

Một số phức amoniacat của Mn(II) như [Mn(NH 3 ) 6 ]Cl 2 , [Mn(NH 3 ) 6 ](ClO 4 ) 2

cũng dễ dàng bị thủy phân tạo thành Mn(OH) 2 :

[Mn(NH 3 ) 6 ]Cl 2 + 2H 2 O ⇌ Mn(OH) 2 + 2NH 4 Cl + 4NH 3

Vì vậy các loại hức trên chỉ tồn tại ở trạng thái rắn hoặc trong dung dịch

nhưng hải có lượng dư NH 3 rất lớn.

Mn(OH) 2 để trong không khí dần dần bị hóa nâu vì bị oxi hóa theo phương

trình:

2Mn(OH) 2 + O 2 + 2H 2 O 2Mn(OH) 4

Hay 2Mn(OH) 2 + O 2 2[MnO 2 .H 2 O] (rắn)

Mn(OH) 2 không tan trong nước (T t = 4,5.10 -13 ), là một bazơ, dễ tan trong axit

tạo thành các muối Mn(II).

Mn(OH) 2 + H 2 SO 4 MnSO 4 + 2H 2 O

Trong môi trường kiềm, Mn(OH) 2 sẽ bị oxi hóa, ngay cả O 2 của không khí:

6Mn(OH) 2 + O 2 2Mn 2 MnO 4 + H 2 O

(4). Mangan (II) clorua (MnCl 2 .4H 2 O) được điều chế bằng cách cho kim loại,

oxit, hidroxit, cacbonat tác dụng với axit HCl và kết tinh từ dung dich nước:

MnO 2 + 4HCl MnCl 2 + Cl 2 + 2H 2 O

Trong tinh thể hidrat hóa, một số phân tử H 2 O đã đóng vai trò phối tử. Ở trạng

thái khan tạo ra tinh thể hình phiến màu hồng. Nóng chảy ở 650 0 C và trong

luồng khí H 2 , MnCl 2 bay hơi ở 1190 0 C; H 2 không khử được MnCl 2 nhưng khi

đun nóng trong luồng khí O 2 hoặc trong hơi nước chuyển thành Mn 2 O 3

4MnCl 2 + O 2 + 4H 2 O 2Mn 2 O 3 + 8HCl

(5). Mangan (II) sunfat (MnSO 4 )

t 0

Phát hành PDF bởi Ths Nguyễn Thanh Tú

Đăng ký Word doc qua Zalo 0905779594 Email thanhtuqn88@gmail.com


Phương pháp điều chế MnSO 4 tương tụ như MnCl 2 là cho kim loại, oxit,

hidroxit, muối cacbonat tan trong dung dịch H 2 SO 4 :

MnO + H 2 SO 4 MnSO 4 + H 2 O

Trong công nghiệp, MnSO 4 được điều chế bằng cách cho pirolusit (MnO 2 ) tan

trong H 2 SO 4 đặc nóng hoặc nung hỗn hợp MnO 2 với FeSO 4 :

2MnO 2 + 2H 2 SO 4 2MnSO 4 + 2H 2 O + O 2

4MnO 2 + 4FeSO 4 4MnSO 4 + 2Fe 2 O 3 + O 2

Ở trạng thái khan hầu như là màu trắng. Khi kết tinh từ dung dịch tạo ra tinh

thể có màu hồng khác nhau phụ thuộc vào hàm lượng kết tinh.

MnSO 4 .H 2 O; MnSO 4 .4H 2 O; MnSO 4 .5H 2 O; MnSO 4 .7H 2 O

Trong tinh thể các sunfat đó, các nhóm SO 4 đóng vai trò cầu nối giữa các

nguyên tử Mn(II)

O

S

O

H 2 O O O OH 2

H 2 O OH 2

Mn Mn (MnSO 4 .4H 2 O)

H 2 O OH 2

H 2 O O O OH 2

S

O

Từ dung dịch MnSO 4 và muối sunfat của kim loại kiềm có thể tách ra những

tinh thể muối kép dạng M 2 SO 4 .MnSO 4 .nH 2 O (n = 2; 4; 6)

Khi tác dụng với chất oxi hóa, MnSO 4 thể hiện tính khử phụ thuộc vào môi

trường:

Trong môi trường kiềm mạnh Mn 2+ MnO 4 2- . Ví dụ:

3MnSO 4 + 2KClO 3 + 12KOH (nóng chảy) 3K 2 MnO 4 + 2KCl + 3K 2 SO 4 + 6H 2 O

Trong môi trường axit, Mn 2+

ví dụ PbO 2 :

O

MnO 4 - , khi tác dụng với chất oxi hóa mạnh,

2MnSO 4 + 5PbO 2 + 6HNO 3 2HMnO 4 + 3Pb(NO 3 ) 2 + 2 PbSO 4 + 2H 2 O

Phát hành PDF bởi Ths Nguyễn Thanh Tú

Đăng ký Word doc qua Zalo 0905779594 Email thanhtuqn88@gmail.com


(6). Mangan (II) nitrat (Mn(NO 3 ) 2 ) được điều chế bằng cách cho kim loại,

oxit, hidroxit, cacbonat tác dụng với HNO 3 loãng tạo ra dung dịch màu hồng

nhạt:

Mn(OH) 2 + 2HNO 3 Mn(NO 3 ) 2 + 2H 2 O

Ở nhiệt độ thường, tinh thể tách ra ở dạng Mn(NO 3 ) 2 .6H 2 O; nếu cao hơn ở

25 0 C ở dạng Mn(NO 3 ) 2 .3H 2 O.

Dễ tan trong nước, khi đun nóng tách ra NO 2 và MnO 2 :

Mn(NO 3 ) 2 MnO 2 + 2NO 2

Đấy là phương pháp điều chế MnO 2 nguyên chất.

(7). Mangan (II) cacbonat (MnCO 3 ) được điều chế bằng cách cho muối

Mn(II) tác dụng với dung dịch cacbonat.

bazơ:

Nếu tác dụng với dung dịch muối cacbonat sẽ thu được muối cacbonat

2MnSO 4 + 2Na 2 CO 3 + H 2 O 2Na 2 SO 4 + [MnCO 3 .Mn(OH) 2 ] + CO 2

Nếu tác dụng với dung dịch muối hidro cacbonat đã bão hòa khí CO 2

tạo ra kết tủa màu trắng:

Mn(NO 3 ) 2 + 2NaHCO 3 MnCO 3 + 2NaNO 3 + CO 2 + H 2 O

MnCO 3 là chất bột màu trắng, mịn như lông tơ, không tan trong nước (T t =

1.10 -10 ở 25 0 C). Khi đun nóng đến 100 0 C, bị phân hủy:

100 0 C

MnCO 3 MnO + CO 2

Khi để trong không khí ẩm dễ bị oxi hóa chuyển thành màu thẫm lại do tạo ra

Mn 2 O 3 .

(8). Mangan (II) sunfat (MnS). Khi cho muối mangan (II) tác dụng với muối

sunfat tan, tạo ra kết tủa màu hồng thẫm. Màu của kết tủa thay đổi tùy theo

điều kiện tạo kết tủa.

Khi để lâu (không có không khí) tạo ra dạng khan MnS màu xanh.

Để trong không khí, bị oxi hóa chuyển dần sang màu nâu:

MnS + O 2 + 2H 2 O S + MnO 2 .2H 2 O

Không tan trong nước (T t = 2,5.10 -10 )

2.1.1.7. Các hợp chất với số oxi hóa +4

Phát hành PDF bởi Ths Nguyễn Thanh Tú

Đăng ký Word doc qua Zalo 0905779594 Email thanhtuqn88@gmail.com


(1). Các muối đơn giản của Mn(IV) đều không bền. Đa số các muối đều được

biết ở dạng muối phức axido như M 2 1 [MnF 6 ], M 2 1 [MnCl 6 ], còn dạng MnF 4 ,

MnCl 4 đều không bền, dễ bị phân hủy.

Với mangan (IV) thì bền nhất là MnO 2 và Mn(OH) 2 hay MnO 2 .2H 2 O.

(2). Mangan(IV) oxit (MnO 2 ) là thành phần chính của khoáng chất piroluzit.

MnO 2 tinh khiết được điều chế bằng cách đun nóng Mn(NO 3 ) 2 đến 200 0 C:

Mn(NO 3 ) 2 MnO 2 + 2NO 2

Các dung dịch muối mangan(II) cũng đều bị oxi hóa bởi bột tẩy, hipoclorit,

KMnO 4 và những tác nhân oxi hoá khác đều tạo ra MnO 2 trong môi trường

kiềm:

MnSO 4 + CaOCl 2 + 2NaOH MnO 2 + Na 2 SO 4 + CaCl 2 + H 2 O

2KMnO 4 + 3MnSO 4 + 4KOH 5MnO 2 + 3K 2 SO 4 + 2H 2 O

MnO 2 là chất bột màu nâu đen, không tan trong nước.

Khi đun nóng bị phân hủy:

3MnO 2 Mn 3 O 4 + O 2

Và khi đun nóng với H 2 SO 4 đặc nóng tạo ra O 2

2MnO 2 + 2H 2 SO 4(đặc nóng) 2MnSO 4 + O 2 + 2H 2 O

MnO 2 và Mn(OH) 2 đều có tính lưỡng tính. Tác dụng với H 2 SO 4 đặc tạo ra

muối Mn(SO 4 ) 2 không bền, còn khi nung nóng chảy với kiềm tạo ra các hợp

chất M 2 1 MnO 3 và M 4 1 MnO 4 :

2KOH + MnO 2 K 2 MnO 3 + H 2 O

4KOH + MnO 2 K 4 MnO 4 + 2H 2 O

MnO 2 có tính oxi hóa mạnh và có cả tính khử.

Oxi hóa HCl đặc tạo ra khí clo:

MnO 2 + 4HCl MnCl 2 + Cl 2 + 2H 2 O

Trong môi trường axit, MnO 2 oxi hóa muối Fe(II) thành muối Fe(III), oxi

hóa axit oxalic thành CO 2 :

MnO 2 + 2FeSO 4 + H 2 SO 4 MnSO 4 + Fe 2 (SO 4 ) 3 + H 2 O

MnO 2 + (COOH) 2 + H 2 SO 4 MnSO 4 + 2CO 2 + 2H 2 O

Oxi hóa axit sunfurơ thành Mn(II) đithionat; oxi hóa H 2 O 2 thành O 2 :

Phát hành PDF bởi Ths Nguyễn Thanh Tú

Đăng ký Word doc qua Zalo 0905779594 Email thanhtuqn88@gmail.com


MnO 2 + 2H 2 SO 3 MnS 2 O 6 + 2H 2 O

MnO 2 +

H 2 O 2 + H 2 SO 4 MnSO 4 + O 2 + 2H 2 O

Khi tác dụng với chất oxi hóa mạnh hơn, MnO 2 thể hiện tính khử.

Khi cho MnO 2 trộn với dung dịch KOH 5% đun nóng ở 250 o C rồi cho không

khí đi qua sẽ tạo ra K 2 MnO 4 màu lục:

2MnO 2 + 4KOH + O 2 2K 2 MnO 4 + 2H 2 O

Hoặc 3MnO 2 + KClO 3 + 6KOH 3K 2 MnO 4 + KCl + 3H 2 O

Trong môi trường axit, tạo ra dung dịch màu tím:

2MnO 2 + 3PbO 2 + 6HNO 3 2HMnO 4 + KCl + 3H 2 O

(3) Người ta cũng biết được các hợp chất hóa trị (IV) của tecnexi và reni như

TcO 2 , ReO 2 , các halogenua như ReCl 4 , M 2 1 ReCl 6 ...

(4) MnO 2 là hợp chất có nhiều ứng dụng trong thực tế. Dùng trong công

nghiệp thủy tinh để oxi hóa các hợp chất sunfua và các hợp chất của sắt; dùng

trong các mặt nạ phòng độc tránh cacbon oxit. Dùng làm chất oxi hóa trong

pin để khử hiđro; dùng trong công nghệ chế tạo diêm.

Ngoài ra, MnO 2 được dùng làm chất oxi hóa trong môi trường axit, làm chất

xúc tác cho quá trình nhiệt phân KClO 3 ...

2.1.1.8. Các hợp chất với số oxi hóa +6

(1) Các hợp chất ứng với số oxi hóa +6 đều không bền. Với mangan, được

biết rõ nhất là muối manganat MnO 4 2- ; trong dung dịch nước tồn tại cân bằng

sau:

3MnO 4

2-

+ 2H 2 O ⇌ 2MnO 4

-

+ MnO 2 + 4OH -

vì vậy các muối manganat bền trong môi trường kiềm, còn muối pemanganat

bền trong môi trường axit.

Các muối TcO 4 2- , ReO 4 2- trong dung dịch nước cũng tồn tại cân bằng như

trên.

(2) Kali manganat (K 2 MnO 4 ). Như trên đã nêu, nếu nung nóng hỗn hợp gồm

MnO 2 với NaOH hoặc KOH khi có mặt của không khí hoặc một tác nhân oxi

hóa khác như clorat, nitrat tạo ra manganat:

MnO 2 + KNO 2 + 2KOH K 2 MnO 4 + KNO 2 + H 2 O

Phát hành PDF bởi Ths Nguyễn Thanh Tú

Đăng ký Word doc qua Zalo 0905779594 Email thanhtuqn88@gmail.com


Hòa tan sản phẩm vào nước tạo ra dung dịch màu xanh lá cây, và khi làm bay

hơi trong chân không thu được tinh thể màu xanh lá cây là K 2 MnO 4 hoặc

Na 2 MnO 4 .10H 2 O, đồng hình với các muối sunfat.

Khi axit hóa dung dịch K 2 MnO 4 tạo ra axit H 2 MnO 4 nhưng lại không bền,

phân hủy ngay theo phương trình:

3H 2 MnO 4 2HMnO 4 + MnO 2 + 2H 2 O

Khi đun nóng dung dịch K 2 MnO 4 tạo ra dung dịch màu tím và kết tủa màu

nâu đen:

3K 2 MnO 4 + 2H 2 O 2KMnO 4 + MnO 2 + 4KOH

Các muối K 2 MnO 4 , Na 2 MnO 4 đều dễ tan trong nước; BaMnO 4 ít tan (Tt =

2,5.10 -10 )

Nói chung, tất cả các dẫn xuất của mangan (VI) đều có tính oxi hóa mạnh.

Trong môi trường kiềm sẽ bị khử đến MnO 2 , còn trong môi trường axit tạo ra

muối Mn(II):

K 2 MnO 4 + 2H 2 S + 2H 2 SO 4 2S

+ MnSO 4 + K 2 SO 4 + 4H 2 O

K 2 MnO 4 + 2Fe(OH) 2 + 2H 2 O MnO 2 + 2Fe(OH) 3 + 2KOH

Vì Mn(VI) ứng với số oxi hóa trung gian, nên khi tác dụng với chất oxi hóa

mạnh, các manganat có thể bị oxi hóa tạo thành pemanganat MnO 4 - . Ví dụ:

2KMnO 4 + Cl 2 2KMnO 4 + 2KCl

4K 2 MnO 4 + O 2 + 2H 2 O 4KMnO 4 + 4KOH

(3) Người ta đã biết được một số hợp chất của Re(VI) như ReF 6 có màu vàng

nhạt; ReCl 6 có màu xanh; ReOCl 4 , ReOF 4 ; ReO 3 có màu đỏ. Các hợp chất đó

đều có tính axit. Trừ ReO 3 , còn lại dễ dàng bị nước phân hủy:

3ReF 6 + 12H 2 O 2HReO 4 + Re(OH) 4 + 18HF

3ReOF 4 + 9H 2 O 2HReO 4 + Re(OH) 4 + 12HF

Reni trioxit không tác dụng với H 2 O và với axit, còn trong kiềm xảy ra quá

trình tự oxi hóa khử:

3ReO 3 + 4KOH 2KReO 4 + K 2 ReO 3 + 2H 2 O

Các dẫn xuất của Re(VI) thể hiện tính khử, dễ dàng bị oxi hóa, ngay cả

oxi của không khí:

4K 2 ReO 4 + O 2 + 2H 2 O 4KReO 4 + 4KOH

Phát hành PDF bởi Ths Nguyễn Thanh Tú

Đăng ký Word doc qua Zalo 0905779594 Email thanhtuqn88@gmail.com


2.1.1.9. Các hợp chất với số oxi hóa +7

(1) Theo dãy các hợp chất Mn(VII), Tc(VII), Re(VII) thì độ bền tăng dần.

Chẳng hạn, với Mn(VII) chỉ được biết Mn 2 O 7 và oxoflorua MnO 3 F, còn với

Re

(VII) đã điều chế một dãy hợp chất ReF 7 , ReOF 5 , ReO 2 F 3 , ReO 3 F, Re 2 O 7 ;

Ví dụ:

Mn 2 O 7 có màu xanh thẫm T nc = 5,9 o C.

Re 2 O 7 màu vàng

T nc = 296 o C

Trong các dẫn xuất của Mn(VII) thì quan trọng nhất trong thực tế là KMnO 4

(2) Axit pemanganic (HMnO 4 ) và anhiđrit penanganic (Mn 2 O 7 ). Khi đun nóng

một dung dịch gồm muối Mn(II) với PbO 2 và HNO 3 tạo ra axit pemanganic:

2Mn(NO 3 ) 2 + 5PbO 2 + 6HNO 3 2HMnO 4 + 5Pb(NO 3 ) 2 + 2H 2 O

Khi làm lạnh một dung dịch đặc KMnO 4 và AgNO 3 thu được kết tủa màu đỏ

AgMnO 4 (T t = 1,6.10 -3 ).

Nếu cho BaCl 2 vào dung dịch có chứa AgMnO 4 thì AgCl sẽ kết tủa (T t =

1,78.10 -10 ), còn lại Ba(MnO 4 ) 2 trong dung dịch:

2AgMnO 4 + BaCl 2 Ba(MnO 4 ) 2 + 2AgCl

Từ dung dịch bari pemanganat, cho thêm một lượng H 2 SO 4 cần thiết, sau khi

tách kết tủa BaSO 4 , dung dịch còn lại có chứa HMnO 4 :

Ba(MnO 4 ) 2 + H 2 SO 4 2HMnO 4 + BaSO 4

Axit pemanganic là một axit mạnh tương đương HCl và HNO 3 ; dung dịch chỉ

có thể cô đặc đến 20%, sau đó bắt đầu bị phân hủy:

4HMnO 4 4MnO 2 + 3O 2 + 2H 2 O

Anhiđrit tương ứng với HMnO 4 là Mn 2 O 7 . Mn 2 O 7 là chất lỏng, màu xanh

thẫm thu được khi cho H 2 SO 4 đặc 90% tác dụng với KMnO 4 :

2KMnO 4 + H 2 SO 4 K 2 SO 4 + Mn 2 O 7 + H 2 O

Khi bay hơi, tạo ra hơi màu tím cùng với hơi nước. Khi đun nóng bị phân

hủy:

2Mn 2 O 7 4MnO 2 + 3O 2

Vì vậy Mn 2 O 7 là chất oxi hóa rất mạnh, ete, rượu và nhiều chất hữu cơ khác

bốc cháy khi tiếp xúc với Mn 2 O 7 .

Phát hành PDF bởi Ths Nguyễn Thanh Tú

Đăng ký Word doc qua Zalo 0905779594 Email thanhtuqn88@gmail.com


Tác dụng với nước tạo ra HMnO 4 :

Mn 2 O 7 + H 2 O 2HMnO 4

(3) Kali pemanganat ( KMnO 4 ) là một muối quan trọng nhất của axit

pemanganic. Để điều chế KMnO 4 có thểbằng cách cho K 2 MnO 4 tác dụng với

H 2 SO 4 :

3K 2 MnO 4 + 2H 2 SO 4 2K 2 SO 4 + 2KMnO 4 + MnO 2 + 2H 2 O

Lọc tách MnO 2 , sau đó cô dung dịch nước lọc. Kali pemanganat có độ hòa tan

bé hơn kali sunfat nên kết tinh trước.

• Cũng có thể điều chế bằng cách cho clo tác dụng với K 2 MnO 4 :

K 2 MnO 4 + Cl 2 2KMnO 4 + 2KCl

hoặc cho khí CO 2 tác dụng với dung dịch K 2 MnO 4 :

3K 2 MnO 4 + 4CO 2 + 2H 2 O 2KMnO 4 + MnO 2 + 4KHCO 3

3K 2 MnO 4 + 2CO 2 2KMnO 4 + MnO 2 + 2K 2 CO 3

• KMnO 4 ở trạng thái rắn là những tinh thể hình thoi, dễ kết tinh màu tím đỏ

gần như đen, có ánh kim. Tan trong nước có màu tím đậm; dung dịch loãng

có màu đỏ.

• Ở trạng thái rắn kết tinh đồng hình với KClO 4 . Khi nung nóng bị phân hủy ở

200 o C:

2KMnO 4 K 2 MnO 4 + MnO 2 + O 2

và khi đun sôi với dung dịch kiềm tạo ra K 2 MnO 4 và O 2 :

4KMnO 4 + 4KOH 4K 2 MnO 4 + 2H 2 O + O 2

• Kali pemanganat là chất oxi hóa mạnh có nhiều ứng dụng thực tế trong

phòng thí nghiệm. Tính oxi hóa của KMnO 4 phụ thuộc vào môi trường của

dung dịch, mạnh nhất là môi trường axit. Ví dụ:

Trong môi trường axit:

2KMnO 4 + 5K 2 SO 3 + 3H 2 SO 4 2MnSO 4 + 6K 2 SO 4 + 3H 2 O

Trong môi trường trung tính:

2KMnO 4 + K 2 SO 3 + H 2 O 2MnO 2 _+ 3K 2 SO 4 + 2KOH

Trong môi trường kiềm:

2KMnO 4 + K 2 SO 3 + 2KOH 2K 2 MnO 4 + K 2 SO 4 + H 2 O

Phát hành PDF bởi Ths Nguyễn Thanh Tú

Đăng ký Word doc qua Zalo 0905779594 Email thanhtuqn88@gmail.com


Phản ứng tạo ra K 2 MnO 4 phải có nồng độ kiềm cao và chất khử ít, hơn nữa

các phản ứng trên thường xảy ra trong dung dịch loãng, do đó sản phẩm khử

trong môi trường kiềm hay trung tính thường là MnO 2 .

• Trong dung dịch, KMnO 4 oxi hóa được các muối Fe(II) thành muối Fe(III);

axit H 2 SO 4 thành H 2 SO 4 ; sunfit thành sunfat; axit oxalic tạo ra CO 2 và H 2 O

(60 o C); iotua thành iot; axit halogenhiđric thành halogen tương ứng; nitrit

thành nitrat; hiđro mới sinh thành H 2 O; hiđro sunfua thành lưu huỳnh;

thiosunfat thành sunfat; amoniac và muối amoni thành nitơ; muối stanit thành

muối stanat... Ví dụ:

2KMnO 4 + 10H + 3H 2 SO 4 2MnSO 4 + K 2 SO 4 + 8H 2 O

6KMnO 4 + 10NH 3 + 9H 2 SO 4 5N 2 + 6MnSO 4 + 3K 2 SO 4 + 24H 2 O

• Trong môi trường axit, oxi hóa được cả H 2 O 2 :

2KMnO 4 + H 2 O 2 + 4H 2 SO 4 2MnSO 4 + 2KHSO 4 + 8H 2 O + 5O 2

• KMnO 4 oxi hóa được muối Mn(II) thành MnO 2 :

3MnSO 4 + 2KMnO 4 + 2H 2 O 5MnO 2 + K 2 SO 4 + 2H 2 SO 4

(4) Người ta cũng đã biết các muối kim loại kiềm khác như LiMnO 4 .3H 2 O dễ

tan; NaMnO 4 .3H 2 O dễ tan; RbMnO 4 ít tan (T t = 2,9.10 -3 ), CsMnO 4 khó tan

(T t = 9,1.10 -5 ); muối pemanganat của kim loại kiềm thổ, kim loại nặng như

Ca(MnO 4 ) 2 .5H 2 O dễ tan; Ba(MnO 4 ) 2 tan; AgMnO 4 ít tan (T t = 1,6.10 -3 );

(5) Các hợp chất Tc(II), Re(VII) tương tự các hợp chất của Mn(VII).

• Các oxit Tc 2 O 7 và Re 2 O 7 cũng được điều chế bằng cách đun nóng các

axit HTcO 4 và HReO 4 :

2HReO 4 Re 2 O 7 + H 2 O

2HTcO 4 Tc 2 O 7 + H 2 O

Cũng có thể thu được khi đun nóng bột Tc hay Re trong luồng khí oxi:

4Re + 7O 2 2Re 2 O 7

4Tc + 7O 2 2Tc 2 O 7

Cả hai đều là chất rắn màu vàng và đều dễ tan trong nước tạo ra axit tương

ứng:

Re 2 O 7 + H 2 O 2HReO 4

Tc 2 O 7 + H 2 O 2HTcO 4

Phát hành PDF bởi Ths Nguyễn Thanh Tú

Đăng ký Word doc qua Zalo 0905779594 Email thanhtuqn88@gmail.com


• Các axit HTcO 4 và HReO 4 (axit petecnexic và axit perenic) đều là những

axit mạnh. Phương pháp điều chế chủ yếu là cho kim loại tương ứng tác dụng

với HNO 3 30%:

3Re + 7HNO 3 3HReO 4 + 7NO + 2H 2 O

Các dung dịch loãng HTcO 4 và HReO 4 đều không có màu và khi đun nóng bị

phân hủy thành anhiđrit tương ứng.

• Các muối petecnat và perenat đều không có màu (nếu cation không màu).

Đa số đều dễ tan trong nước, một số muối perenat khó tan KReO 4 (T t = 1,9.10 -

3 ); RbReO 4 (T t = 9,6.10 -4 ); CsReO 4 (T t = 4.10 -4 ); AgReO 4 (T t = 7,9.10 -5 );

Ba(ReO 4 ) 2 (T t = 5,3.10 -2 ).

(6) So sánh với các dẫn xuất của Mn(VII) thì các dẫn xuất của Tc(VII) và

Re(VII) bền hơn.

Chẳng hạn: Mn 2 O 7 ởtrạng thái lỏng; ΔH 298 = - 743 kJ/mol.

Tc 2 O 7 ởtrạng thái rắn; ΔH 298 : - 1117 kJ/mol.

Re 2 O 7 ởtrạng thái rắn; ΔH 298 = - 1243 kJ/mol.

• HMnO 4 chỉ cô đặc đến 20%, còn HTcO 4 và HReO 4 có thể cô dung dịch đến

khô.

Các axit đó đều có tính oxi hóa mạnh và tính axit mạnh, nhưng tính oxi hóa

và tính axit giảm dần từ HMnO 4 đến HReO 4 .

• Muối perenat rất bền khi nung nóng, ví dụ KReO 4 nóng chảy ở 555 o C và

đến 1370 o C vẫn chưa bị phân hủy; trong khi đó KMnO 4 bị phân hủy ở 200 o C.

• Người ta chưa biết được hợp chất không chứa oxi của Mn(VII) nhưng đã

điều chế được các hợp chất của Re(VII) như Re 2 S 7 , HReS 4 ...

(7) Với mangan, tạo ra các oxit và các hiđroxit ứng với bậc oxi hóa từ thấp

đến cao và sự biến đổi bậc oxi hóa ảnh hưởng đến tính chất của chúng:

MnO Mn 2 O 3 MnO 2 (MnO 3 ) Mn 2 O 7

Tính axit tăng dần

Mn(OH) 2 Mn(OH) 3 Mn(OH) 4 H 2 MnO 4 HMnO 4

Tính axit tăng dần

• Tính axit tăng theo dãy trên cũng có thể giải thích trong khuôn khổ sự tương

quan điện tích và bán kính của các ion. Khi chuyển từ Mn(II) - Mn(VII), bán

Phát hành PDF bởi Ths Nguyễn Thanh Tú

Đăng ký Word doc qua Zalo 0905779594 Email thanhtuqn88@gmail.com


kính của ion giảm dần, điện tích của ion tăng dần, nên khả năng tách ion H +

tăng dần theo chiều từ Mn(OH) 2 đến HMnO 4 .

• Độ bền của ion MnO 4

-

lớn hơn độ bền của ion MnO 4 2- cũng được giải thích

trong phạm vi của thuyết obitan phân tử như sau:

Trong ion phức hình 4 mặt, chẳng hạn như MnO 4 - , CrO 4 - , MnO 4 2- ... thì các

obitan của chất tạo phức tham gia hình thành liên kết gồm 5 obitan (n-1)d;

một obitan ns và ba obitan np, với các obitan p của bốn phối tử.

• Trong ion MnO 4 - có 24 electron hóa trị gồm 7 electron của nguyên tử Mn

(3d 5 4s 2 ), 16 electron của 4 nguyên tử oxi O (2p 4 ) và 1 electron do tích điện

của ion. Trong ion MnO 4 2- có 25 electron hóa trị đã tham gia hình thành liên

kết.

Sự phân bố electron trong các obitan phân tử (MO) trong ion MnO 4 - và ion

MnO 4

2-

có cấu hình như sau (hình 65 ).

MnO 4 - : (d ) (p ) (p)

MnO 4 2- : (d ) (p ) (p) (p )

Như vậy, trong ion MnO 4 - có một electron hóa trị chiếm MO phản liên kết, do

đó ion MnO 4 - kém bền hơn ion MnO 4 - .

Tương tự như vậy, trong các ion MnO 3- 4 ; MnO 4- 4 có 2 và 3 electron chiếm

MO phản liên kết nên càng kém bền, và vì vậy chúng ta hiểu được tính không

bền của số phối trí 4 trong hợp chất Mn(IV) và cũng hiểu được vì sao các ion

3-

MnO 4 MnO 4- 4 không tồn tại.

Phát hành PDF bởi Ths Nguyễn Thanh Tú

Đăng ký Word doc qua Zalo 0905779594 Email thanhtuqn88@gmail.com


2.1.2. Các nguyên tố nhóm VII.B ( Mangan – Tecnexi – Reni)

2.1.2.1. Nhận xét chung về các nguyên tố nhóm VIIIB

(1) Nhóm VIIIB gồm 9 nguyên tố thuộc họ d ở các chu kỳ 4; 5; 6 thuộc bảng

tuần hoàn.

• Sắt (Ferrum) được biết từ thời kỳ cổ đại. Coban (Cobaltum) được một nhà

khoa học Thụy Điển là Bran(G.Bandt) tìm ra năm 1775. Niken (Niccolum)

được tìm ra năm 1751 bởi nhà hóa học kiêm khoáng vật học người Thụy Điển

là Cronstet (A. Cronstedt) và đến đầu thế kỷ thứ 19 mới được nhà hóa học

người Đức là Richte (J. Richter) nghiên cứu kỹ về tính chất của niken kim

loại.

• Ruteni (Ruthenium) do nhà hóa học người Nga là Claoxơ (K.Klaus) tìm

ra năm 1843; hai nguyên tố rođi (Rhodium) và palađi ( Palladium) đều tìm ra

năm 1803 do phát minh của nhà hóa học người Anh là Volastơn (W.

Wollaston); osmi (Osmium) và iriđi (Iriđium) do nhà hóa học người Anh là

Thenơn (S. Tennant ) tìm ra năm 1804, còn platin (Platinium) đã biết từ thời

cổ đại.

Phát hành PDF bởi Ths Nguyễn Thanh Tú

Đăng ký Word doc qua Zalo 0905779594 Email thanhtuqn88@gmail.com


(2) Khi so sánh tính chất lý học và hóa học cơ bản của các nguyên tố nhóm

VIIB, người ta thấy các nguyên tố sắt, coban và niken có tính chất tương tự

nhau, nên được xếp chung thành họ sắt; các nguyên tố còn lại có tính chất

giống nhau theo chiều thẳng đứng, nên được xếp chung thành họ platin. Nói

cách khác, khi khảo sát các tính chất của các nguyên tố nhóm VIIIB, người ta

không xem nhóm này là một nhóm thống nhất như các nhóm khác, mà xem là

những bộ ba chuyển tiếp giữa các nguyên tố nhóm VIIB (Mn, Tc, Re) và

nhóm IB (Cu, Ag, Au). Tóm tắt như sau:

HỌ SẮT

25 26 27 28

29

Mn Sắt Coban Niken

Cu

HỌ PLATIN

43 44 45 46

47

Tc Ruteni Rodi Paladi

Ag

75 76 77 78

79

Re Osmi Iridi Platin

Au

(3) Nguyên tử khối, sự phân bố electron và trạng thái hóa trị của các nguyên

tố đó như sau:

Nguyên tố Kí

hiệu

Nguyên

khối

tử

Phân bố electron

Hóa trị

Sắt Fe 55,847 2 8 14 2 I, II, III, IV, VI

Coban Co 58,933 2 8 15 2 I, II, III, IV

Phát hành PDF bởi Ths Nguyễn Thanh Tú

Đăng ký Word doc qua Zalo 0905779594 Email thanhtuqn88@gmail.com


Niken Ni 58,710 2 8 16 2 I, II, III, IV

Ruteni Ru 101,070 2 8 18 1

5

Rođi Rh 102,905 2 8 18 1

6

Palađi Pd 106,400 2 8 18 1

8

Osmi Os 190,200 2 8 18 3

2

Iriđi Ir 192,220 2 8 18 3

2

Platin Pt 195,090 2 8 18 3

2

1 II, III, IV, V, VI, VII,

VIII

1 I, II, III, IV, VI

0 II, III, IV

14 2 II, III, IV, VI, VIII

15 2 I, II, III, IV, VI

17 1 I, II, III, IV, VI

(4) Về tính chất , các nguyên tố thuộc nhóm VIIIB có những nét chung như

sau:

• Đều có tính chất của kim loại, màu sắc từ xám đến xám trắng; rất khó

nóng chảy và rất khó bay hơi; thể tích nguyên tử thấp.

• Tất cả đều có khả năng hấp thụ hiđro trên bề mặt ít hoặc nhiều và gây ra

hoạt tính cao của hiđro (hiđro hoạt động).

• Tất cả đều có tác dụng xúc tác cho phản ứng hóa học vô cơ hoặc hữu cơ.

• Đều có khuynh hướng tạo phức, đặc trưng nhất là phản ứng tạo phức với

NH 3 , với CO và cả với NO.

• Có khả năng tạo ra nhiều hợp chất có hóa trị khác nhau và có thể dễ

chuyển hóa từ trạng thái hóa trị này đến trạng thái hóa trị khác.

• Đều tạo ra hợp chất có màu ngay cả ở trạng thái tự do (dạng hiđrat hóa ).

• Hiđroxit của chúng đều có tính bazơ yếu, hoặc axit yếu, hoặc có tính

lưỡng tính.

Phát hành PDF bởi Ths Nguyễn Thanh Tú

Đăng ký Word doc qua Zalo 0905779594 Email thanhtuqn88@gmail.com


• Có ái lực yếu đối với oxi và giảm dần từ trái sang phải; nhưng lại có ái

lực mạnh với lưu huỳnh và tăng dần từ trái sang phải. Về mặt này, các nguyên

tố nhóm VIIIB tương tự nhóm IB.

(5) Về cấu hình elcctron, nguyên tố nhóm VIIIB đều thuộc họ d mà nguyên tử

lắp đầy dần các obitan d ở lớp (n - l) - (n là số thứ tự của chu kỳ).

(6) Tinh thể các kim loại đó có cấu trúc theo mạng như sau:

Fe

Mạng lập phương tâm

khối

a = 2,8664 o A

Ru

Mạng lục phương

a = 2,7085 o A ;

c = 4,2816 o A

Os

Mạng lục phương

a = 2,7353 o A ;

c = 4,3191 o A

Co

Mạng lục phương

a = 1,5063 o A ;

c = 4,0795 o A

Rh

Mạng lập phương tâm

diện

a = 3,8044 o A

Ir

Mạng lập phương tâm

diện

a = 3,8938 o A

Ni

Mạng lập phương tâm

diện

a = 3,5238 o A

Pd

Mạng lập phương tâm

diện

a = 3,8907 o A

Pt

Mạng lập phương tâm

diện

a = 3,9239 o A

CÁC KIM LOẠI HỌ SẮT

2.1.2.2. Nhận xét chung về kim loại họ sắt

(l) Cả ba kim loại sắt, coban và niken ở trạng thái kim loại cũng như ở trạng

thái hợp chất đều có những đặc điểm giống nhau về tính chất:

- Cả ba kim loại đều tạo ra các muối có số oxi hóa +2; ngoài ra sắt tạo ra

các muối có số oxi hóa +3; coban(II) dễ dàng chuyển thành Co(III) nhưng

thường gặp trong các hợp chất phức; với niken thì chủ yếu tạo ra hợp chất

Ni(II).

- Khuynh hướng tạo ra hợp chất hóa trị I tăng từ sắt đến niken; niken dễ

dàng tạo ra hợp chất hóa trị I tương tự đồng.

Phát hành PDF bởi Ths Nguyễn Thanh Tú

Đăng ký Word doc qua Zalo 0905779594 Email thanhtuqn88@gmail.com


Hóa trị cực đại của sắt là VI ( trong ferat FeO 4 2- ) tương tự mangan; trong

khi đó hóa trị của coban và niken là (IV) trong các hợp chất của đioxit dễ

dàng bị phân bảy tách ra oxi.

(2) Một số đặc điểm nguyên tử của ba nguyên tố trên.

Fe Co Ni

Electron hóa trị 3d 6 4s 2 3d 7 4s 2 3d 8 4s 2

Bán kính nguyên tử ( o A )

Bán kính ion M 2+ ( o A )

Bán kính ion M 3+ ( o A )

1,26 1,25 1,24

0,80 0,78 0,74

0,67 0,64 -

Thế ion hóa I 1 (eV) 7,90 7,86 7,63

(3) Thế điện cực : tóm tắt theo sơ đồ sau :

• Trong môi trường axit:

6 4 3 2 0

3 Fe

0,77 2 Fe

0,44

Fe

0,04

[Fe(CN) ] [Fe(CN) ]

3

4

6 6

0,361 1,16

1,416 3 1,92 2 0,28

CoO2

Co Co Co

NiO NiO Ni Ni

2 1,8 1,56 2 0,257

4 2

1,6

Trong môi trường bazơ:

6 4 3 2 0

FeO FeO FeO(OH) Fe

2 0,55 0,68 0,8

4 2

NiO NiO Ni Ni

2 0,4 0,49 2 0,72

2 2

(4) Về cấu tạo tinh thể :

• Sắt có bốn dạng thù hình, có cấu tạo như hình 66.

Phát hành PDF bởi Ths Nguyễn Thanh Tú

Đăng ký Word doc qua Zalo 0905779594 Email thanhtuqn88@gmail.com


Hình 1. Mạng tinh thể các dạng thù hình của Fe

- Ở điều kiện thường đến 770 o C tồn tại dạng Fe có mạng lập phương

tâm khối;

- Ở 770 o C dạng Fe chuyển thành dạng Fe, mạng tinh thể không thay

đổi nhưng độ dài giữa hai nguyên tử tăng lên.

- Ở 910 o C chuyển thành dạng Fe, mạng tinh thể thay đổi thành mạng

lập phương tâm diện.

- Đến 1390 o C lại chuyển thành mạng lập phương tâm khối là dạng d Fe.

o

770 C

o o o

770 C 910 C 1390 C

Fe Fe Fe d Fe

• Coban có hai dạng thù hình: Ở điều kiện thường đến 417 o C tồn tại dạng

Co có mạng lục phương (a=2,5063 o A ; c=4,0795 o A ); đến ~480 o C tồn tại dạng

Co có mạng lập phương tâm diện (a=3,5441 o A ).

• Niken tồn tại hai dạng thù hình: <250 o C ở dạng Ni có mạng lục phương

(a = 2,65 o A , c = 4,32 o A ); cao hơn 250 o C chuyển thành dạng Ni với mạng

lập phương tâm diện (a = 3,5238 o A ).

2.1.2.3. Trạng thái thiên nhiên và thành phần các đồng vị

(1) • Sắt thuộc các nguyên tố phổ biến nhất cấu tạo nên vỏ quả đất. Khoáng

vật chủ yếu của sắt là manhetit (Fe 3 O 4 ), hematit đỏ(Fe 2 O 3 ), hematit nâu

[Fe 2 O 3 .2Fe(OH) 3 ]. Ngoài ra, một lượng sắt khá lớn ở dạng khoáng chất

xiđerit (FeCO 3 ) và cả trong quặng với lưu huỳnh, asen như pirít FeS 2 ... tuy

nhiên pirit không phải là chất dùng để điều chế sắt mà dùng điều chế H 2 SO 4 .

Hàm lượng của coban trong vỏ quả đất ít hơn nhiều so với sắt. Khoáng vật

Phát hành PDF bởi Ths Nguyễn Thanh Tú

Đăng ký Word doc qua Zalo 0905779594 Email thanhtuqn88@gmail.com


quan trọng của coban là cobantin (CoAsS). Coban ở lẫn với các kim loại khác

như Cu, Ni, Ag, Fe, Mn trong các quặng đa kim.

• Hàm lượng của niken trong vỏ quả đất nhiều hơn coban. Khoáng vật chủ

yếu của niken là penlađit là quặng sunfua của niken, đồng và sắt. Ngoài ra

còn có các loại quặng khác như gacnierit (NiSiO 3 .MgSiO 3 ), quặng smantit là

quặng arsenua của Ni, Co, Fe.

(2) • Trong các thiên thạch đá có khoảng 5,15%Fe; 0,01%Co; 0,14%Ni (%

nguyên tử), nhưng trong các thiên thạch sắt có đến 90%Fe; 8,5Ni; 0,5%Co

cùng với một ít tạp chất khác. Hàng năm có đến trên 1000 thiên thạch rơi

xuống trái đất chủ yếu là thiên thạch sắt. Thiên thạch sắt lớn nhất nặng 60 tấn

phát hiện năm 1920 ở Tây Nam châu Phi.

• Trong cơ thể động vật, sắt có trong hemoglobin, chức năng chính của

phức chất này là liên kết oxi phân tử và chuyển oxi đó vào các mô. Ngoài ra,

còn có trong mioglobin là chất protein dự trữ oxi trong cơ bắp; có trong phức

chất feritin cũng là protein có chức năng tạo nên những hợp chất khác chứa

sắt cần cho cơ thể của sinh vật. Gan và lá lách là bộ phận giàu sắt nhất trong

cơ thể.

• Coban thuộc số nguyên tố quan trọng đối với con người, vitamin B 12 hay

cobanamin là phức chất của coban có chứa 4,5% Co về khối lượng, dùng để

chữa bệnh thiếu máu.

(3) Trong nước đại dương, các nguyên tố họ sắt, coban, niken ở dạng

Fe(OH) 3 ; Co 2+ , CoSO 4 ; Ni 2+ , NiSO 4 [theo D. Golldberg (1963)] ứng với thành

phần hóa học trung bình như sau (theo A.P. Vinogradov -1967):

Nguyên

tố

Hàm lượng

% mg/l

Fe 1.10 -6 0,01

Co 5.10 -8 0,0005

Ni 2.10 -7 0,002

(4) Hàm lượng ba nguyên tố trong các mẫu đá ở Mặt Trăng có các hàm lượng

như sau:

Phát hành PDF bởi Ths Nguyễn Thanh Tú

Đăng ký Word doc qua Zalo 0905779594 Email thanhtuqn88@gmail.com


Nguyên

tố

Hàm lượng trung bình

Apollo-

11

Apollo-12 Luna-6

Fe 0,134 0,132 -

Co 2,5.10 -5 4,2.10 -5 4,1.10 -6

Ni - 2.10 -4 1,171.10 -

(5) • Sắt có bốn đồng vị bền là 54 Fe (5,84%); 56 Fe (91,68%); 57 Fe (2,17%);

58 Fe (0,31%); trong số các đồng vị phóng xạ thì đồng vị 55 Fe là bền hơn cả, có

chu kỳ bán hủy là 2,9 năm. Coban có tám đồng vị từ 54 Co đến 61 Co nhưng chỉ

có 59 Co là đồngvị thiên nhiên (100%) , số đồng vị còn lại đều là đồng vị

phóng xạ trong đó bền nhất là 60 Co có chu kỳ bán hủy là 5,2 năm, kém bền

nhất là 54 Co với Chu kỳ bán hủy là 0,18 giây.

• Niken có 11 đồng vị từ 56 Ni đến 66 Ni , trong đó có năm đồng vị thiên

nhiên là 58 Ni (67,76%);

60 Ni (26,16%); 61 Ni (l,25%); 62 Ni (3,66%); 64 Ni

(l,16%). Trong các đồng vị phóng xạ thì đồng vị 59 Ni có chu bán hủy là 7,5.

104 năm bền nhất, và đồng vị 65 Ni kém bền nhất có chu kỳ bán hủy là 0,108

ngày đêm.

2.1.2.4. Điều chế Fe, Co, Ni

(1) • Sắt tinh khiết được điều chế bằng cách dùng H 2 tinh khiết để khử oxit sắt

tinh khiết :

Fe 2 O 3 + 3H 2

o

t

2Fe + 3H 2 O

Quá trình khử xảy ra ở khoảng 175-270 o C; ở nhiệt độ đó, sản phẩm thu

được là loại sắt tự cháy khi tiếp xúc với không khí loại sắt này sẽ bị oxi hóa

nhanh và nóng đỏ lên. Để thu được sản phẩm có độ bền cao đối với không

khí, quá trình khử phải thực hiện ở nhiệt độ cao hơn khoảng 550-650 o C; ở

điều kiện đó, sắt thu được ở dạng bột mịn không còn có khả năng tự cháy.

Quá trình khử xảy ra theo từng giai đoạn, ban đầu tạo ra Fe 3 O 4 , sau đó mới

tạo thành Fe:

3Fe 2 O 3 + H 2

o

t

2Fe 3 O 4 + H 2 O

Nếu nung ở 573 o C thì tạo ra oxit FeO:

4

Phát hành PDF bởi Ths Nguyễn Thanh Tú

Đăng ký Word doc qua Zalo 0905779594 Email thanhtuqn88@gmail.com


Fe 2 O 3 + H 2

o

t

2FeO + H 2 O

Nếu quá trình khử xảy ra khi tăng nhiệt độ đến khoảng 700 – 1000 o C

không thu được dạng bột mịn mà ở dạng sắt rèn tức là ở dạng khối chảy.

• Có thể điều chế bằng phương pháp nhiệt phân Fe(CO) 5 sắt pentacacbonyl:

Fe(CO) 5

o

t

Fe + 5CO

• Sắt rất tinh khiết có thể điều chế bằng phương pháp điện phân dung dịch

muối Fe(II) chẳng hạn như FeSO 4 .FeCl 2 , với dương cực là tấm Fe - Cr, còn

âm cực là sắt tinh khiết. Quá trình điện phân phụ thuộc vào các yếu tố như pH

của dung dịch, nồng độ của chất điện phân, mật độ dòng của anot và catot.

(2) • Coban tinh khiết được điều chế bằng cách dùng H 2 để khử coban(II) oxit

CoO hoặc các oxit khác:

CoO + H 2

o

t

Co + H 2 O

Nếu nung đến khoảng 400 o C, sản phẩm thu được ở dạng bột hoàn toàn bền

đối với không khí khô.

Sản phẩm thu được phụ thuộc vào nhiệt độ, chẳng hạn ở 450 o C thu được dạng

thù hình - Co (mạng lập phương), ở nhiệt độ thấp hơn 450 o C sẽ thu được

dạng - Co (mạng lục phương).

• Coban tinh khiết có thể điều chế bằng phương pháp điện phân dung dịch

CoSO 4 trong nước với dương cực bằng thép không gỉ đã được xử lý bề mặt và

âm cực bằng tấm chì tinh khiết hóa học. Sản phẩm thu được có chứa 99,1% -

99,2% coban, tạp chất còn lại là niken.

(3) • Niken tinh khiết cũng được điều chế bằng các phương pháp như trên.

Khi dùng H 2 để khử oxit NiO:

NiO + H 2

o

t

Ni + H 2 O

Nếu nung ở 270 – 280 o C thu được niken tự (hay), còn ở 350 - 400 o C hoặc cao

hơn thu được niken bột, hoàn toàn bền đối với không khí.

• Niken bột cũng được điều chế bằng phương pháp điện phân dung dịch

NiSO 4 trong nước với dương cực bằng các lá (Fe - Ni ), còn âm cực bằng Ni.

• Niken tinh khiết cũng có thể

tetracacbonyl Ni(CO) 4 :

Ni(CO) 4

điều chế bằng cách nhiệt phân niken

t

o

Ni + 4CO

Phát hành PDF bởi Ths Nguyễn Thanh Tú

Đăng ký Word doc qua Zalo 0905779594 Email thanhtuqn88@gmail.com


(4) Ngoài các phương pháp đã nêu, các kim loại trên đều có thể điều chế bằng

phương pháp nhiệt nhôm thu được kim loại khối nóng chảy:

3Fe 3 O 4 + 8Al

o

2700 C

9Fe + 4Al 2 O 3

3Co 3 O 4 + 8Al

o

3300 C

9Co + 4Al 2 O 3

3NiO + 2Al

o

2450 C

3Ni + Al 2 O 3

Các phản ứng tỏa ra một lượng nhiệt khá lớn đủ để cho chất phản ứng nóng

chảy. Sản phẩm thu được không tinh khiết.

(5) Như trên đã nêu , các kim loại Fe, Co, Ni ở trạng thái bột rất nhỏ tách ra từ

các hợp chất có khả năng tự cháy, nghĩa là tự bốc cháy trong không khí ở

ngay nhiệt độ thường. Bột tự cháy của sắt cũng có thể điều chế bằng cách

nung cẩn thận Fe(II) oxalat:

Fe(COO) 2

o

t

Fe + 2CO 2

Các hạt sắt điều chế bằng cách đó có đường kính khoảng 5.10 -8 cm do đó bề

mặt tiếp xúc rất lớn nên đã làm tăng tốc độ oxi hóa, tuy nhiên nguyên nhân

quan trọng là do cấu trúc mạng tinh thể của các hạt so với kiến trúc bền của

kim loại đó.

2.1.2.5. Tính chất lý học của Fe, Co, Ni và ứng dụng

(1) • Cả ba nguyên tố là những kim loại màu trắng có ánh kim ; Fe và Co có

màu xám, còn Ni có màu trắng bạc.

• Một số hằng số lý học của Fe, Co, Ni nêu ra ở bảng 43.

Bảng 5. Một số hằng số lý học quan trọng của Fe, Co, Ni

Tính chất Fe Co Ni

Khối lượng riêng (g/cm 3 ) 7,9 8,9 8,9

Nhiệt độ nóng chảy (T nc , o C) 153

9

Nhiệt độ sôi (T s , o C) 274

0

1495 1455

2900 2730

Độ dẫn điện (Hg = 1) 10 10 14

Độ dẫn nhiệt (Hg = 1) 10 8 7

Phát hành PDF bởi Ths Nguyễn Thanh Tú

Đăng ký Word doc qua Zalo 0905779594 Email thanhtuqn88@gmail.com


Độ âm điện (theo Pauling) 1,8 1,7 1,8

• Sắt và niken dễ rèn, dễ dát mỏng; coban cứng và giòn.

(2) • Ở điều kiện thường, khác với tất cả các kim loại, ba nguyên tố Fe, Co, Ni

đều là chất sắt từ tức là chất bị nam châm hút. Tuy nhiên, trong bốn dạng thù

hình của Fe, chỉ có - Fe mới có tính sắt từ, nghĩa là tính sắt từ của sắt chỉ ở

nhiệt độ đến 770 o C, trên nhiệt độ đó tính sắt từ của sắt biến mất, mặc dù mạng

tinh thể không đổi, sắt chỉ còn lại tính thuận từ.

• Coban có tính sắt từ ở nhiệt độ l075 o C, còn niken - ở 362 o C; trên nhiệt độ

đó tính sắt từ của coban và niken sẽ mất.

• Một số hợp chất của các nguyên tố trên cũng có tính sắt từ, do đó người ta

đã làm giàu quặng sắt bằng phương pháp tử học.

• Ngoài tính chất bị nam châm hút, dưới tác dụng của dòng điện các chất

sắt từ sẽ trở thành nam châm. Tuy nhiên, sắt nguyên chất chỉ tác dụng như

một nam châm khi chịu tác dụng của dòng điện, còn thanh thép khi đã nam

châm hóa thì sau đó vẫn tác dụng như một nam châm vĩnh cửu.

(3) • Sắt là kim loại quan trọng nhất đối với các ngành kỹ thuật và công

nghiệp hiện đại. Cả ba kim loại được dùng chủ yếu để tạo ra các hợp kim đặc

biệt là các loại thép.

• Nhiều hợp chất của sắt có ý nghĩa quan trọng trong thực tế như FeCl3

dùng làm chất đông tụ khi làm sạch nước, làm chất cầm máu, làm chất xúc tác

trong hóa học hữu cơ. Các muối ferit của kim loại hóa trị hai dùng trong kỹ

thuật máy tính.

Các oxit của sắt không những là nguồn điều chế nhiều hợp chất của sắt mà

còn là nguồn nguyên liệu quan trọng của ngành luyện kim đen.

(4) • Hơn 3/4 lượng coban được sản xuất dùng để chế tạo thép và hợp kim đặc

biệt. Coban truyền cho thép tính cứng và tính chịu mòn nên loại thép có chứa

coban dùng để chế tạo các dụng cụ cắt gọt khác nhau.

• Một số hợp kim của coban dùng trong kỹ thuật quốc phòng và kỹ thuật

tên lửa.

Coban là vật liệu sắt từ, nhiều hợp kim của co ban cũng có tính sắt từ. Hợp

chất giữa các kim loại của coban và samari (SmCo 5 ) và một số kim loại đất

hiếm khác được dùng làm vật liệu để chế tạo nam châm vĩnh cửu mạnh.

Phát hành PDF bởi Ths Nguyễn Thanh Tú

Đăng ký Word doc qua Zalo 0905779594 Email thanhtuqn88@gmail.com


• Đồng vị 60

59 60

27Co được điều chế theo phản ứng:

27Co (n, )

27

rãi trong y khoa để chữa bệnh ung thư.

Co

được dùng rộng

(5) • Niken có nhiều ứng dụng hơn coban, có nhiều ứng dụng trong nhiều

ngành kinh tế quốc dân, hơn 80% lượng niken được sản xuất, dùng trong

ngành luyện kim.

• Thép có chứa niken có độ bền cao về mặt hóa học và cơ học; niken được

dùng để mạ lên bề mặt các sản phẩm làm bằng vật liệu dễ bị ăn mòn.

• Hợp kim Monen (A. Monel là giám đốc một công ty ở Canada đã chế tạo

ra loại hợp kim này) có độ bền cao với môi trường ăn mòn, được sử dụng

nhiều trong ngành chế tạo máy cho công nghệ hóa học, đóng tàu... Một số hợp

kim khác của niken được dùng để sản xuất các loại tiền thay cho bạc.

• Niken và hợp kim của nó được dùng trong kỹ thuật hàng không và vũ trụ,

trong công nghiệp hóa chất dùng làm chất xúc tác.

(6) Một điều cần chú ý là khi nói đến tính chất lý học của sắt cần để ý đến

thành phần tạp chất có trong sắt với hai loại :

- Sắt có độ tinh khiết cao có chứa dưới 0,001% tạp chất, có từ tính tốt và dễ

hàn, nhưng tính chất cơ học không cao, độ bền kém hơn nhiều so với gang và

các loại thép; không thể dùng trong vật liệu chế tạo.

- Sắt tinh khiết kỹ thuật còn gọi là thép điện kỹ thuật có chứa 0,02 - 0,04%

cacbon, ngoài ra còn có các tạp chất khác như oxi , nhơ, lưu huỳnh, photpho

cũng có từ tính tết và dễ hàn, có tính cơ học cao, dùng làm vật liệu chế tạo.

2.1.2.6. Tính chất hóa học của Fe, Co, Ni

Fe, Co, Ni là những kim loại hoạt động trung bình. Tác dụng được với nhiều

đơn chất và hợp chất. Hoạt tính hóa học giảm dần từ Fe đến Ni. Chẳng hạn,

Fe dễ dàng bị oxi hóa trong không khí ẩm; Co bền ở điều kiện thường, đun

nóng đến 300 o C thì bị oxi oxi hóa; còn niken bắt đầu bị oxi hóa ở 500 o C.

(1) • Cả ba kim loại đều không phản ứng trực tiếp với hiđro, nhưng ở trạng

thái bột nhỏ và ở nhiệt độ cao đều hấp thụ hiđro với lượng khá lớn. Các hợp

chất với hiđro của chúng đều không có thành phần xác định và điều chế bằng

phương pháp gián tiếp. Các hiđrua đều bền như : FeH, FeH 2 , FeH 3 ; CoH,

CoH 2 ; NiH, NiH 2

• Độ hòa tan của hiđro trong sắt nóng chảy phụ thuộc vào dạng thù hình

của sắt. Dạng - Fe là dung môi tốt hơn cả so với các dạng thù hình khác, ở

Phát hành PDF bởi Ths Nguyễn Thanh Tú

Đăng ký Word doc qua Zalo 0905779594 Email thanhtuqn88@gmail.com


1540 o C có thể hòa tan được 18gH 2 trong 100g Fe; người ta cũng đã nghiên

cứu thấy rằng sự giải hấp thụ bắt đầu ở 150 - 300 o C.

• Coban hấp thụ hiđro kém hơn nhiều. Ở 1200 o C, 100g coban hấp thụ được

5,46 cm 3 hiđro. Coban điện phân hấp thụ kém, còn coban tấm hầu như không

hấp thụ hiđro.

• Đặc biệt là niken hấp thụ mạnh hiđro, khả năng đó cho phép dùng niken

làm chất xúc tác trong quá trình hiđro hóa các chất hữu cơ.

Độ hòa tan của hiđro trong niken có khác nhau phụ thuộc vào trạng thái rắn

hay lỏng. Ở 1600 o C, 100g niken hòa tan được 43cm 3 hiđro.

(2) Ở điều kiện thường sắt, coban và niken đều bền với không khí và nước.

Khi nung bột Fe trong không khí hoặc trong oxi tạo ra Fe 3 O 4 :

3Fe + 2O 2

o

t

Fe 3 O 4

Ở nhiệt độ nóng trắng, coban bị oxi hóa tạo ra Co 3 O 4 :

3Co + 2O 2

o

t

Co 3 O 4

Còn niken bị oxi hóa chậm hơn, bắt đầu ở 500 o C tạo ra NiO:

2Ni + O 2

o

t

2NiO

(3) Trong không khí ẩm, sắt có lẫn các tạp chất sẽ bị gỉ, tức là bị ăn mòn trên

bề mặt. Lớp gỉ tạo nên có thành phần chủ yếu là Fe 2 O 3 .xH 2 O, lớp gỉ này xốp

và giòn nên không bảo vệ được cho sắt. Phản ứng tạo gỉ chủ yếu theo

phương trình:

4Fe + 3O 2 + 2xH 2 O 2[Fe 2 O 3 . xH 2 O]

Tốc độ tạo ra gỉ sắt lớn nhất ở các đầu mút, ở chỗ bẻ cong đặc biệt ở các vết

cắt; còn ở phần phẳng của tấm sắt bị ăn mòn ở mức độ nhỏ.

(4) • Cả ba kim loại đều phản ứng mạnh với các halogen. Khi đun nóng sắt

kim loại với các halogen thu được Fe(III) halogenua khan FeX 3 ; ví dụ:

o

t

2Fe + 3Cl 2 2FeCl 3

Tuy nhiên khi nghiền bột tốt với bột sắt tạo ra sản phẩm có thành phần là

Fe 3 I 8 ( hay 2FeI 3 .FeI 2 ).

3Fe + 4I 2 Fe 3 I 8

nghĩa là hỗn hợp có cả iotua Fe(III) và Fe(II). Nói cách khác, FeI 3 chỉ bền

trong cân bằng với lượng dư khá lớn FeI 2 :

Phát hành PDF bởi Ths Nguyễn Thanh Tú

Đăng ký Word doc qua Zalo 0905779594 Email thanhtuqn88@gmail.com


2FeI 3 2FeI 2 + I 2

• Coban và niken tác dụng trực tiếp với các halogenua tạo ra muối ứng với số

oxi hóa +2 của kim loại. Ví dụ: khi đốt cháy coban hay niken trong luồng khí

chỉ tạo ra CoCl 2 và NiCl 2 ; tương tự như vậy với brom.

M + Cl 2

o

t

MCl 2 (M là Co, Ni)

nhưng với flo, coban tạo ra hỗn hợp COF 2 + COF 3 .

(5) Khi nung hỗn hợp gồm S với các kim loại họ sắt tạo ra các sunfua:

Fe + S

o

t

FeS

Co và Ni cũng có phản ứng tương tự tạo ra CoS và NiS.

Selen và telu phản ứng trực tiếp với Fe, Co, Ni:

- Với Fe tạo ra : Fese, FeSe 2 ; FeTe, FeTe ; trong đó FeSe và FeSe 2 là chất

bán dẫn.

- Với Co tạo ra : CoSe, CoSe 2 ; CoTe, CoTe 2 .

- Với Ni tạo ra : NiSe, NiSe 2 ; NiTe, NiTe 2 .

(6) Cả ba kim loại đều không phản ứng trực tiếp với nitơ. Các hợp chất như

Fe 2 N, Fe 4 N; Co 2 N, Co 3 N, Co 3 N 2 ; Ni 3 N, Ni 3 N 2 đều được điều chế bằng

phương pháp gián tiếp và đều ít bền. Tác dụng với photpho trong các ampun

hàn tạo ra dung dịch rắn ứng với các hợp chất có thành phần Fe 3 P, Fe 2 P, FeP,

FeP 2 ; Co 2 P, CoP, CoP 3 ; Ni 3 P, Ni 2 P, NiP 3 .

(7) • Fe, Co, Ni tạo ra hợp chất với cacbon ứng với thành phần M 3 C và M 2 C,

trong đó quan trọng nhất là Fe 3 C:

3Fe + C

o

t

Fe 3 C

Fe 2 C được điều chế bằng cách nung Fe với C ở 250 o C.

Các hợp chất ứng với thành phần như trên của Co và Ni được điều chế bằng

phương pháp gián tiếp, chẳng hạn khi nung coban bột với CO:

3Co + 2CO

o

t

Co 3 C + CO 2

• Fe tạo ra các silixua quan trọng như FeSi 2 , FeSi, Fe 3 Si 2 và Fe 2 Si có trong

thành phần của ferosilic.

• Coban tạo ra các hợp chất Co 3 Si, Co 2 Si , CoSi, CoSi 2 trực tiếp từ các

nguyên tố, nóng chảy ở nhiệt độ trong khoảng từ 1200 – 1400 o C ; ví dụ :

Phát hành PDF bởi Ths Nguyễn Thanh Tú

Đăng ký Word doc qua Zalo 0905779594 Email thanhtuqn88@gmail.com


3Co + Si

o

t

Co 3 Si

• Niken cũng phản ứng trực tiếp với silic tạo ra các silixua Ni 3 Si, Ni 2 Si, NiSi,

NiSi; chẳng hạn:

2Ni + Si

o

t

Ni 2 Si

Nóng chảy trong khoảng từ 1000 – 1200 o C.

(8) Fe, Co, Ni đều phản ứng trực tiếp với B:

Nung nóng bột Fe với B ở 1300 – 1400 o C trong bầu khí trơ tạo ra FeB và

FeB 2 :

2Fe + 3B

o

t

FeB + FeB 2

Khi nung Co với bột B vó định hình trong ống thạch anh hàn kín tạo ra

các hợp chất Co 3 B, Co 2 B, CoB, CoB 2 .

Cũng tương tự cách điều chế như trên, niken tạo ra các hợp chất Ni 3 B,

Ni 2 B, NiB, Ni 3 B 2 , Ni 4 B 3 .

(9) • Nước không chứa không khí hòa tan hầu như không ăn mòn sắt ở nhiệt

độ thường, vì trên bề mặt dễ tạo ra lớp mỏng Fe(OH) 2 bảo vệ cho kim loại.

Nhưng khi cho hơi nước qua Fe nung đỏ tạo ra Fe 3 O 4 :

3Fe + 4H 2 O

o

t

Fe 3 O 4 + 4H 2

• Co và Ni không bị H 2 O ăn mòn ở nhiệt độ thường, nhưng khi cho H 2 O qua

Ni nung đỏ tạo ra NiO:

Ni + H 2 O

o

t

NiO + H 2

(10) Với dung dịch kiềm, ở điều kiện thường thực tế không tác dụng với cả ba

kim loại. Tuy nhiên, khi đun sôi bột sắt khử với dung dịch NaOH 50%, trong

điều kiện không có không khí, thu được dung dịch natri hipoferit:

Fe + 2NaOH + 2H 2 O Na 2 [Fe(OH) 4 ] + H 2

(11) • Trong dãy thế điện cực, Co và Ni đứng giữa Fe và Sn nhưng gần Sn

hơn nên cả hai kim loại tan trong axit loãng chậm hơn Fe.

Sắt tác dụng với axit không có tính oxi hóa tạo ra muối Fe(II) và H 2 :

Fe + 2H 3 O + Fe 2+ + H 2 + 2H 2 O

sắt khử được các ion kim loại đứng sau sắt trong dãy thế điện cực:

Fe + Cu 2+ Fe 2+ + Cu

Phát hành PDF bởi Ths Nguyễn Thanh Tú

Đăng ký Word doc qua Zalo 0905779594 Email thanhtuqn88@gmail.com


Fe + 2Fe 3+ 3Fe 2+

H 2 SO 4 đặc nguội gây ra tính thụ động cho sắt, nhưng với H 2 SO 4 đặc nóng sẽ

oxi hóa sắt tạo ra muối Fe(III) và đồng thời bị khử đến SO 2 :

2Fe + 6H 2 SO 4 đặc

o

t

Fe 2 (SO 4 ) 3 + 3SO 2 + 6H 2 O

Hình 2. Tốc độ hòa tan của Fe trong H 2 SO 4

Hình 2 cho thấy sắt hòa tan mạnh nhất trong dung dịch H2SO4 khoảng 50%.

• Khi nhúng thanh sắt vào HNO 3 bốc khói một thời gian, sau đó nhúng vào

dung dịch chứa ion Cu 2+ hoặc dung dịch axit loãng như HCl và H 2 SO 4 , sắt sẽ

mất khả năng phản ứng, nghĩa là sắt đã bị thụ động hóa bởi HNO 3 bốc khói,

HNO 3 loãng, lạnh hoà tan sắt tạo ra sắt (II) nitrat:

8Fe + 20HNO 3 loãng 8Fe(NO 3 ) 2 + 2NH 4 NO 3 + 6H 2 O

(12) • Trong dung dịch axit loãng như HCl, H 2 SO 4 , coban khó tan hơn so với

sắt, tạo ra H 2 :

Co + H 2 SO 4 CoSO 4 + H 2

• Coban phản ứng với H 2 SO 4 đặc nóng tạo ra SO 2 :

Co + 2H 2 SO 4 đặc

o

t

CoSO 4 + SO 2 + 2H 2 O

• Coban tan trong HNO 3 loãng, và cũng tương tự như sắt, coban cũng bị thụ

động hóa bởi HNO 3 đặc.

(13) • Các axit vô cơ loãng tác dụng với niken chậm hơn sắt tạo ra H 2 :

nhưng dễ tan trong HNO 3 loãng:

Ni + 2HCl NiCl 2 + H 2

3Ni + 8HNO 3 loãng 3Ni(NO 3 ) 2 + 2NO + 4H 2 O

Phát hành PDF bởi Ths Nguyễn Thanh Tú

Đăng ký Word doc qua Zalo 0905779594 Email thanhtuqn88@gmail.com


• Tương tự Fe và Co, Ni cũng bị thụ động hóa bởi HNO 3 bốc khói.

2.1.2.7. Hợp chất cacbonyl của Fe, Co, Ni

Cũng tương tự như các nguyên tố Mn, Cr, V, Ti, các nguyên tố Fe, Co, Ni

cũng có khả năng phản ứng với cạcbon oxit tạo ra các hợp chất cacbonyl mà

trong đó các nguyên tố trung tâm này mang số oxi hóa "0".

(1) • Khi đun nóng bột sắt trong ống chứa CO ở 150-200 o C ở áp suất khoảng

100atm tạo ra sắt pentacbonyl: Fe + 5CO Fe(CO) 5

• Fe(CO) 5 là chất lỏng màu vàng nhạt (T nc = -20 o C ; T s = l03 o C) có cấu tạo

hình chóp kép với nguyên tử Fe ở trung tâm [d(Fe - C) = 1,84 o A ; d(C - O) =

1,15 o A ], ứng với obitan lai hóa dsp 3 của nguyên tử sắt (hình 68 ).

3d 4s 4p

• Khi đốt cháy hơi Fe(CO) 5 trong không khí tạo nên Fe 2 O 3 :

o

t

4Fe(CO) 5 + 13O 2 2Fe 2 O 3 + 20CO 2

• Bị nhiệt phân hủy ở 140 o C tạo thành sắt kim loại:

o

t

Fe(CO) 5 Fe + 5CO

Người ta dùng phản ứng này để điều chế sắt tinh khiết hóa học.

Fe(CO) 5 không tan trong nước, tan trong ete và benzen; trong dung dịch ete,

Fe(CO) 5 phản ứng với H 2 SO 4 theo phương trình:

Fe(CO) 5 + H 2 SO 4 FeSO 4 + 5CO + H 2

điều đó chứng tỏ sắt ở trong hợp chất đó có tác dụng như là sắt kim loại.

• Fe(CO) 5 có tính chất oxi hóa và khử. Chẳng hạn trong amoniac lỏng, tác

dụng với natri kim loại thể hiện tính oxi hóa:

dsp

Fe(CO) 5 + 2Na Na 2 [Fe(CO) 4 ] + CO

và tác dụng với dung dịch kiềm trong rượu thể hiện tính khử:

Fe(CO) 5 + 4KOH K 2 [Fe(CO) 4 ] + K 2 CO 3 + 2H 2 O

(2) • Khi đun nóng bột coban trong khí quyển cacbon oxit ở 150 – 200 o C và

250 atm tạo ra coban octa cacbonyl:

2Co + 8CO Co 2 (CO) 8

3

Phát hành PDF bởi Ths Nguyễn Thanh Tú

Đăng ký Word doc qua Zalo 0905779594 Email thanhtuqn88@gmail.com


• Co 2 (CO) 8 là hợp chất hai nhân, trong đó nguyên tử coban tạo ra sáu liên kết

- d . Bốn liên kết d tạo ra bởi bốn cặp electron của bốn phân tử CO đặt vào

bốn obitan tự do của coban; liên kết d thứ năm tạo ra từ một cặp electron d

đặt vào obitan p của phân tử CO. Liên kết giữa Co - Co tạo ra do sự ghép đôi

bởi hai electron độc thân của hai nguyên tử coban:

lienket p

lienket

Co 2 (CO) 8 là tinh thể có màu da cam, bắt đầu bị phân hủy ở 60 o C:

Co 2 (CO) 8

o

t

2Co + 8CO

Khi đun nóng dưới áp suất của khí hiđro tạo ra hợp chất hidrua cacbonyl:

Co 2 (CO) 8 + H 2

O

OC C CO

OC Co Co CO

OC C CO

O

3d 4s 4p

o

t

2H[Co(CO) 4 ]

Chất này có màu vàng, T nc = - 33 o C, tan trong dung dịch kiềm tạo thành muối

như K[Co(CO) 4 ] , NH 4 [Co(CO) 4 ].

(3) • Đã biết được nikel tetracabonyl Ni(CO) 4 là hợp chất của Ni(0) có cấu

hình bốn mặt ứng với dạng lai hóa sp 3 (hình 69).

3d 4s 4p

3

sp

Phát hành PDF bởi Ths Nguyễn Thanh Tú

Đăng ký Word doc qua Zalo 0905779594 Email thanhtuqn88@gmail.com


Hình 3. Sơ đồ cấu tạo phân tử Fe(CO) 6

Hình 4. Sơ đồ cấu tạo phân tử

Ni(CO) 4

• Ni(CO) 4 là chất lỏng không màu ( T nc = 19,3 o C ; T s = 43 o C ) dễ dàng điều

chế bằng cách cho CO tác dụng với bột niken ở 60 – 80 o C:

o

t

Ni + 4CO Ni(CO) 4

Ni(CO) 4 không tan trong nước, không phản ứng với axit loãng, kiềm loãng,

dễ tan trong các dung môi hữu cơ.

Khi đun nóng khoảng 180 – 200 o C, Ni(CO) 4 bị phân hủy tạo ra Ni nguyên

chất:

o

t

Ni(CO) 4 Ni + 4CO

Phản ứng trên dùng để điều chế niken tinh khiết.

CÁC HỢP CHẤT VỚI SỐ OXI HÓA +2

Số phối trí của Fe(II), Co(II) và Ni(II) là bằng 6 và 4 ứng với cấu hình tám

mặt và bốn mặt hoặc mặt phẳng vuông.

(1) Số phối trí 6 của Fe(II) trong các hợp chất như FeO (hình 70); FeF 2 (hình

71); FeCl 2 và đa số các ion phức như [Fe(OH) 2 ] 6 2+ ; [Fe(OH) 6 ] 4 - ; [Fe(CN) 6 ] 4

-

.... Một số phức bốn mặt ứng với số phối trí 4 của Fe(II) như [FeCl 4 ] 2- ,

[Fe(NCS) 4 ] 2- ....

Phát hành PDF bởi Ths Nguyễn Thanh Tú

Đăng ký Word doc qua Zalo 0905779594 Email thanhtuqn88@gmail.com


Hình 5. Cấu trúc tinh thể (dạng NaCl)

của FeO

Hình 6. Cấu trúc tinh thể (dạng

rutin) của FeF 2

(2) Số phối trí 6 của coban trong các phức spin cao như [Co(OH) 2 ] 6 2+ ;

[Co(OH 2 ) 6 ] 2+ ; [CoF 6 ] 4

-

hoặc trong các phức spin thấp [Co(CN) 6 ] 4 - , [Co(NO 2 ) 6 ] 4 - .... Một số phức bốn

mặt ứng với số phối trí 4 như [CoCl 4 ] 2- , [CoBr 4 ] 2- , [Co(NCS) 4 ] 2- .

Đa số các hợp chất của Co(II) đều có tính thuận từ.

(3) Các hợp chất lưỡng tố ứng với số phối trí 6 của Ni chẳng hạn như NiO có

cấu trúc tinh thể dạng NaCl (hình 70); NiF 2 - có cấu trúc dạng rutin (hình 71);

NiCl 2 , NiBr 2 , NiI 2 có cấu trúc lớp.

2.1.2.8. Các oxit FeO, CoO, NiO

Như trên đã nêu, các oxit trên đều kết tinh theo mạng tinh thể như NaCl với

a(FeO) = 4,29 o A ; a(CoO) = 4,24 o A ; a(NiO) = 4,17 o A (hình 70).

(l) Sắt (II) oxit (FeO)

• Sắt (II) oxit là chất bột màu đen, được điều chế bằng cách dùng H 2 để khử

Fe 2 O 3 ở 300 o C.

Fe 2 O 3 + H 2

o

t

2FeO + H 2 O

Hoặc bằng cách nhiệt phân Fe(II) oxalat (chất rắn màu vàng) trong điều kiện

không có không khí:

COO

COO

Fe

t o FeO + CO + CO 2

• Hàm lượng của sắt trong FeO thường bé hơn so với hàm lượng của nguyên

tố đó ứng với công thức phân tử, vì các nguyên tử Fe chiếm không hoàn toàn

các mắt của mạng lưới tinh thể, ứng với công thức Fe 0,95 O.

Phát hành PDF bởi Ths Nguyễn Thanh Tú

Đăng ký Word doc qua Zalo 0905779594 Email thanhtuqn88@gmail.com


• FeO không tan trong nước, nhưng có phản ứng với H 2 O đặc biệt khi đun

nóng. Sau khi nung nóng mạnh, FeO trở nên trơ, nghĩa là mất hoạt tính hóa

học cao.

• FeO dễ tan trong axit , không tan trong dung dịch kiềm, là oxit bazơ. Khi tan

trong axit loãng tạo ra ion [Fe(OH 2 ) 6 ] 2+ :

(2) Coban(II) oxit (CoO)

FeO + 2H 3 O + + 3H 2 O [Fe(OH 2 ) 6 ] 2+

• Coban(II) oxit là oxit bazơ. CoO là chất rắn màu xanh, tạo ra khi nung

Co(OH) 2 , CoCO 3 hay Co(NO 3 ) 2 trong bầu khí trơ hay trong chân không:

Co(OH) 2

CoCO 3

2Co(NO 3 ) 2

o

t

CoO + H 2 O

o

t

CoO + CO 2

o

t

2CoO + 4NO 2 + O 2

nếu nung trong không khí sẽ chuyển thành Co 3 O 4 :

3Co(OH) 2 + 1/2O 2

3Co(NO 3 ) 2

o

t

Co 3 O 4 + 3H 2 O

o

t

Co 3 O 4 + 6NO 2 + O 2

• Khi nung rất mạnh, tất cả các oxit của coban đều chuyển thành CoO, và khi

nung trong luồng khí H 2 tất cả các oxit đó đều bị khử thành kim loại:

2Co 2 O 3

CoO + H 2

o

t

4CoO + O 2

o

t

Co + H 2 O

• Màu sắc của CoO khan cũng khác nhau, phụ thuộc vào cách sắp xếp nguyên

tử trong tinh thể, ngoài màu xanh còn có các màu vàng, xám, hung, đỏ hoặc

đen, tuy nhiên cho đến nay chỉ mới biết được dạng lập phương, chưa biết

được dạng thù hình khác.

(3) Niken(II) oxit (NiO)

• Niken(II) oxit là chất rắn màu xanh, được tạo ra khi nhiệt phân Ni(OH) 2 ,

NiCO 3 , Ni(NO 3 ) 2 .

• Thực tế không tan trong nước, nhưng dễ tan trong axit.

• Hiđro dễ dàng khử NiO thành kim loại:

NiO + H 2

o

t

Ni + H 2 O

Phát hành PDF bởi Ths Nguyễn Thanh Tú

Đăng ký Word doc qua Zalo 0905779594 Email thanhtuqn88@gmail.com


• Khi nung mạnh, NiO chuyển thành dạng hình tám mặt đúng, màu đen xám,

có vẻ sáng kim loại, lúc đó NiO mất khả năng hoà tan trong axit.

• NiO được dùng làm chất tạo màu xám cho thủy tinh, làm chất xúc tác cho

các phản ứng hữu cơ.

2.1.2.9. Các hiđroxit Fe(OH) 2 , Co(OH) 2 , Ni(OH) 2

Phương pháp chung để điều chế các hiđroxit trên là cho muối tác dụng với

dung dịch kiềm mạnh :

M 2+ + 2OH - M(OH) 2

(1) • Fe(OH) 2 kết tủa màu trắng, hấp thụ mạnh O 2 , màu sắc xanh thẫm dần và

cuối cùng thành màu nâu của Fe(OH) 3 :

4Fe(OH) 2 + O 2 + 2H 2 O 4Fe(OH) 3

Cần chú ý rằng amoniac không thể kết tủa hoàn toàn Fe(OH) 2 , vì khi có mặt

lượng lớn muối amoni, kết tủa Fe(OH) 2 hầu như không xảy ra do sự tạo thành

phức chất amoniacat của Fe(II):

FeCl 2 + 6NH 3 ⇌ [Fe(NH 3 ) 6 ]Cl 2

nồng độ ion OH - không thể tăng rất lớn vượt quá giới hạn, vì khi tăng nồng độ

OH - đồng thời lại giảm nồng độ Fe 2+ nên nồng độ các ion không đủ để đạt đến

tích số tan T = [Fe 2+ ][OH - ] 2 . Phức chất amoniacat Fe 2+ chỉ bền ở trạng thái

rắn, trong dung dịch nước dễ dàng bị thủy phân:

[Fe(NH 3 ) 6 ]Cl 2 + 2H 2 O ⇌ Fe(OH) 2 + 2NH 4 Cl + 4NH 3

Fe(OH) 2 tan trong dung dịch axit loãng không có không khí; tan trong dung

dịch kiềm đặc tạo ra hipoferit :

Fe(OH) 2 + 2NaOH đặc

o

t

Na 2 [Fe(OH)) 4 ]

khi làm lạnh dung dịch, tinh thể hipoferit tách ra ở dạng màu xanh - nâu. Khi

tan trong nước bị phân hủy.

Khi cho Sr(OH) 2 hay Ba(OH) 2 tác dụng lên Fe(OH) 2 tạo ra các hipoferit

tương tự Sr 2 [Fe(OH) 6 ] và Ba 2 [Fe(OH) 6 ].

(2) • Co(OH) 2 tồn tại ở hai dạng thù hình : xanh và hồng. Khi tác dụng dung

dịch kiềm với muối Co(II), lạnh, tạo ra dạng thù hình màu xanh; khi đun

nóng, kết tủa Co(OH) 2 chuyển thành dạng màu hồng:

o

2 OH

t

Co

Co(OH)

(lanh)

2

Co(OH)

2

xanh

hong

Phát hành PDF bởi Ths Nguyễn Thanh Tú

Đăng ký Word doc qua Zalo 0905779594 Email thanhtuqn88@gmail.com


• Trong không khí, Co(OH) 2 bị oxi hóa chậm chuyển thành Co(III) hiđroxit

màu hung.

4Co(OH) 2 + O 2 + 2H 2 O 4Co(OH) 3

• Co(OH) 2 có tính lưỡng tính, nhưng nặng về tính bazơ. Dễ tan trong axit, tan

trong dung dịch kiềm đặc nóng tạo thành cobantit màu tím – xanh.

Co(OH) 2 + 2NaOH đặc

o

t

Na 2 [Co(OH)) 4 ]

Cũng tương tự, các hợp chất Sr 2 [Co(OH) 6 ] và Ba 2 [Co(OH) 6 ] cũng được hình

thành ở dạng tinh thể màu tím - đỏ.

• Khi cho dung dịch amoniac dư tác dụng với muối Co(II) tạo ra phức chất

amoniacat:

CoCl 2 + 6NH 3 ⇌ [Co(NH 3 ) 6 ]Cl 2

Phức chất này bền hơn amoniacat Fe(II), nhưng cũng bị H 2 O phân hủy:

[Co(NH 3 ) 6 ]Cl 2 + 2H 2 O ⇌ [Co(OH 2 ) 6 ]Cl 2 + 6NH 3

• Ni(OH) 2 kết tủa màu xanh quả táo, không bị oxi hóa trong không khí. Có

tính bazơ, tan trong axit tạo thành muối tương ứng.

- Tan trong dung dịch amoniac dư và trong dung dịch muối amoni do tạo ra

phức chất amoniacat :

Ni(OH) 2 + 6NH 3 ⇌ [Ni(NH 3 ) 6 ](OH) 2

• Cũng tương tự như các amoniacat của Fe 2+ và Co 2+ , amoniacat của Ni(II)

cũng bị nước phân huỷ một phần :

[Ni(NH 3 ) 6 ]Cl 2 + 2H 2 O ⇌ Ni(OH) 2 + 2NH 4 Cl + 4NH 3

Khi dư amoniac và muối amoni, cân bằng trên chuyển sang trái dễ hơn trường

hợp của Fe 2+ và Co 2+ . Như vậy, dung dịch amoniac kết tủa không hoàn toàn

các hiđroxit trên.

2.1.2.10. Các halogenua của Fe(II), Co(II), Ni(II)

(1) • Sắt (II) florua và clorua không thể điều chế trực tiếp từ các đơn chất.

Cả hai muối trên ở dạng khan được điều chế bằng cách cho hiđro halogenua

tác dụng với sắt nung nóng:

Fe + 2HF khí

o

t

FeF 2 + H 2

Phát hành PDF bởi Ths Nguyễn Thanh Tú

Đăng ký Word doc qua Zalo 0905779594 Email thanhtuqn88@gmail.com


FeCl 2 cũng có thể điều chế bằng cách nung muối kép FeCl 2 .2NH 4 Cl hay

(NH 4 ) 2 FeCl 4 trong điều kiện không có không khí:

FeCl 2 .2NH 4 Cl

o

t

FeCl 2 + 2HCl + 2NH 3

Cũng có thể điều chế bằng cách dùng H 2 để khử FeCl 3 nung nóng:

2FeCl 3 + H 2

o

t

2FeCl 2 + 2HCl

• FeCl 2 là chất kết tinh màu trắng. Ở 1500 o C ứng với công thức FeCl 2 , nhiệt

độ thấp hơn ứng với công thức Fe 2 Cl 4 .

• Tinh thể hiđrat hóa FeCl 2 .4H 2 O tạo ra khi làm bay hơi dung dịch sau khi cho

sắt tác dụng với axit HCl. Tinh thể màu lam. Để ngoài không khí có màu lục

vì bị oxi hóa một phần.

Khi đun nóng nhẹ hoặc để trên H 2 SO 4 đặc chuyển thành FeCl 2 .2H 2 O.

FeCl 2 bị oxi hóa thành Fe 2 O 3 và FeCl 3 khi đun nóng trong không khí:

o

t

6FeCl 2 + 3/2O 2 Fe 2 O 3 + 4FeCl 3

• Sắt(II) bromua và iotua được điều chế trực tiếp từ sắt và các halogen.

Fe + Br 2

Fe + I 2

o

t

FeBr 2

o

t

FeI 2

nếu dư I 2 tạo ra Fe 3 I 8 (hay FeI 2 . 2FeI 3 ).

FeBr 2 là những vảy có màu vàng lục, hút ẩm. Khi đun nóng đến 300 o C

chuyển thành Fe 2 O 3 .

(2) • Coban (II) clorua khan được điều chế bằng cách nung coban trong

luồng khí clo:

o

t

Co + Cl 2 CoCl 2

• Dạng khan là chất bột màu xanh lơ, hấp thụ mạnh hơi H 2 O tạo ra

CoCl 2 .6H 2 O. T nc = 722 o C, dễ tan trong nước.

• Muối CoCl 2 .6H 2 O bị mất một phần nước kết tinh kèm theo sự thay đổi màu

sắc khá rõ rệt:

p p p p

49 C 58 C 90 C 140 C

2 2

2 2

2 2

2 2

oCl2

CoCl .6H O CoCl .4H O CoCl .2H O CoCl .H O C

hong do tim hong tim xanh xanh lo

Phát hành PDF bởi Ths Nguyễn Thanh Tú

Đăng ký Word doc qua Zalo 0905779594 Email thanhtuqn88@gmail.com


• Khi cho H 2 O tác dụng lên CoCl 2 khan quá trình lại xảy ra ngược lại. Sự thay

đổi màu sắc đó được dùng trong các ẩm kế để xác định độ ẩm của không khí.

• Khi đun nóng dung dịch CoCl 2 hay muối coban khác cũng có sự thay đổi

màu sắc. Màu hồng của dung dịch CoCl 2 loãng sẽ chuyển thành màu xanh khi

đun nóng hoặc khi tăng nồng độ, hoặc khi cho thêm các chất như HCl đặc,

CaCl 2 , làm phá vỡ sự hiđrat hóa của Co 2+ , thay thế các phân tử H 2 O trong cầu

nội bằng ion Cl - :

[Co(OH 2 ) 6 ] 2+ + 4Cl - ⇌ [CoCl 4 ] 2- + 6H 2 O

hồng

xanh

• Ngược lại, một dung dịch CoCl 2 trong rượu có màu xanh, khi cho thêm

ZnCl 2 hay HgCl 2 lại chuyển thành màu hồng do phản ứng:

[CoCl 4 ] 2- + 2HgCl 2 + 6H 2 O ⇌ [Co(OH 2 ) 6 ] 2+ +

2[HgCl 4 ] 2-

xanh không màu hồng

không màu

• Coban(II) florua và bromua được điều chế bằng cách cho coban(II)

cacbonat tác dụng với axit tương ứng:

CoCO 3 + 2HF CoF 2 + CO 2 + H 2 O

còn coban(II) iotua lại có thể điều chế trực tiếp từ các đơn chất :

Co + I 2 CoI 2

(3) • Niken (II) clorua có thể điều chế bằng cách cho niken tác dụng với

nước cường thủy hoặc cho tác dụng với niken nung nóng:

hay Ni + Cl 2

3Ni + 2HNO 3 + 6HCl 3NiCl 2 + 2NO + 4H 2 O

o

t

NiCl 2

• Niken (II) bromua và iotua được điều chế bằng phản ứng trực tiếp từ các

đơn chất:

Ni + Br 2 NiBr 2

còn niken florua được điều chế bằng cách nhiệt phân florua kép niken và

amoni.

Phát hành PDF bởi Ths Nguyễn Thanh Tú

Đăng ký Word doc qua Zalo 0905779594 Email thanhtuqn88@gmail.com


• Nói chung các muối halogenua của Fe 2+ , Co 2+ , Ni 2+ , ở điều kiện thường

phần lớn đều kết tinh với sáu phân tử H 2 O ( trừ florua ), đều dễ tan trong nước

và trong rượu.

Ở trạng thái khan, đều có màu khác nhau do chịu ảnh hưởng của anion:

F- Cl - Br - I -

Fe 2+ không màu không

màu

vàng lục

hung đỏ

Co 2+ đỏ nhạt xanh nhạt nhạt lục đen

Ni 2+ vàng nhạt hung vàng hung sẫm đen

2.1.2.11. Các sunfua của Fe(II), Co(II), Ni(II)

(1) Các sunfua FeS, CoS, NiS được điều chế bằng cách nung các kim loại

tương ứng với lưu huỳnh, hoặc bằng cách cho (NH 4 ) 2 S tác dụng với dung dịch

muối của các kim loại đó, tạo ra kết tủa màu đen, thực tế không tan trong

nước. ( T(FeS) = 5. 10 -18 ; T( -CoS) = 2,0. 10 -25 ; T( -NiS) =1.10 -24 ). Ngay

sau khi kết tủa, cả ba sunfua đó đều dễ tan trong axit loãng kể cả axit yếu như

axit axetic. Để lâu trong dung dịch, CoS và NiS chuyển sang dạng khó tan

trong axit. Về cấu trúc tinh thể, cả ba sunfua đều kết tinh theo kiểu NiAs (hình

7; 8).

Với FeS thì a = 3,43 A o ; c = 5,86 A o ; dFeS

2, 45A . Kiểu kiến trúc đó giống

như kiểu NaCl, đều có số phối trí 6, nhưng khoảng cách từ một tiểu phân đến

các tiểu phân khác bao quanh nó là hoàn toàn khác nhau (bốn tiểu phân sắp

xếp gần hơn hai tiểu phân kia).

o

Phát hành PDF bởi Ths Nguyễn Thanh Tú

Đăng ký Word doc qua Zalo 0905779594 Email thanhtuqn88@gmail.com


Hình 7. Tế bào tinh thể của NiAs

Hình 8. Mạng tinh thể của NiAs

Do kiến trúc theo kiểu NiAs nên có sự giảm khoảng cách trong mạng lưới so

với tổng bán kính hiệu dụng của các ion. Ví dụ với FeS có d = 2,45 o A trong

khi đó bán kính hiệu dụng của ion Fe 2+ = 0,83 o A và của S 2- = 1,74 o A , điều đó

đã được giải thích bằng sự biến dạng tương hỗ của hai ion.

Một điều cần chú ý là trong tinh thể, ví dụ FeS, một phần nguyên tử sắt có thể

thay thế bởi cùng một số nguyên tử lưu huỳnh mà cấu trúc và độ bền của tinh

thể không thay đổi. Trong ba sunfua trên thì FeS có nhiều ứng dụng thực tế,

được dùng làm nguyên liệu để điều chế H 2 S.

FeS thực tế không tan trong nước, nhưng tan được trong axit loãng tạo ra H 2 S.

Trong không khí ẩm, kết tủa FeS dần dần bị oxi hóa thành sunfat và cuối cùng

tạo thành Fe 2 O 3 :

FeS + 2O 2

4FeS + 7O 2

o

t

FeSO 4

o

t

2Fe 2 O 3 + 4SO 2

(2) • Sắt(II) đisunfua FeS 2 thường gặp trong thiên nhiên ở dạng quặng pirit sắt

và quặng marcasit.

• FeS 2 có thể được điều chế bằng cách nung nóng FeS với S:

o

t

FeS + S FeS 2

Trong không khí ẩm, FeS 2 bị oxi hóa chậm tạo ra FeSO 4 ; ở nhiệt độ cao tách

ra lưu huỳnh và thuận nghịch rõ rệt ở khoảng 550 – 700 o C; khi nung trong

không khí tạo ra Fe 2 O 3 và SO 2 :

2FeS 2 + 7O 2 + 2H 2 O 2FeSO 4 + 2H 2 SO 4

FeS 2

o

t

FeS + S

Phát hành PDF bởi Ths Nguyễn Thanh Tú

Đăng ký Word doc qua Zalo 0905779594 Email thanhtuqn88@gmail.com


4FeS 2 + 11O 2

o

t

2Fe 2 O 3 + 8SO 2

Cả hai dạng pirit và marcasit đều có màu vàng đồng thau, đều có vẻ sáng kim

loại, và có cấu tạo mạng tinh thể khác nhau.

2.1.2.12. Các xianua của Fe(II), CO(II), NI(II)

(1) • Các hợp chất xianua trên đều tạo ra ở dạng kết tủa vô định hình màu

vàng (Fe), màu đỏ (Co) và màu xanh (Ni) khi cho KCN tác dụng với các

muối.

• Xianua của sắt (II) ở dạng tinh khiết chưa được biết. Khi cho ion Fe 2+ tác

dụng với ion CN - , kết tủa lắng xuống ban đầu sẽ nhanh chóng tạo thành anion

phức [Fe(CN) 6 ] 4- :

FeSO 4 + 2KCN Fe(CN) 2 + K 2 SO 4

Fe(CN) 2 + 4KCN K 4 [Fe(CN) 6 ]

• Đã biết được nhiều muối dạng M 4 [Fe(CN) 6 ] và thường kết tinh ở dạng hiđrat

hóa, trong đó quan trọng nhất là kali feroxianua K 4 [Fe(CN) 6 ] tách ra từ dung

dịch ở dạng hiđrat tinh thể K 4 [Fe(CN) 6 ].3H 2 O có màu vàng, mất nước kết tinh

ở 87 o C, thường được gọi là muối vàng - máu (vì trước kia người ta điều chế

chất này từ máu bò).

• Các feroxianua của kim loại kiềm và kiềm thổ đều dễ tan trong nước, trừ

bari feroxianua Ba 2 [Fe(CN) 6 ] là khó tan.

Trong dung dịch nước, không có phản ứng của con Fe 2+ và CN - ban đầu vì ion

[Fe(CN) 6 ] 4 - rất bền (K kb ≈ 10 -37 ):

K 4 [Fe(CN) 6 ] 4K + + [Fe(CN) 6 ] 4-

• Nói chung, các hợp chất đó đều bền với không khí, với axit, với kiềm, tuy

nhiên khi nung nóng bị phân hủy và khi tác dụng với H 2 SO 4 đặc nóng tạo ra

CO:

FeSO 4

K 4 [Fe(CN) 6 ] + 6H 2 SO 4 + 6H 2 O 2K 2 SO 4 + 3(NH 4 ) 2 SO 4 + 6CO +

• Khi cho HCl đặc tác dụng với dung dịch đậm K 4 [Fe(CN) 6 ] tạo ra kết tủa

trắng H 4 [Fe(CN) 6 ]:

K 4 [Fe(CN) 6 ] + 4HCl 4KCl + H 4 [Fe(CN) 6 ]

Axit feroxianhiđric

Phát hành PDF bởi Ths Nguyễn Thanh Tú

Đăng ký Word doc qua Zalo 0905779594 Email thanhtuqn88@gmail.com


• Là axit mạnh ở hai chức đầu, còn sự phân ly tiếp theo thì yếu hơn (K 3 = 10 -3 ;

K 4 = 5,0.10 -5 ). Dễ tan trong nước và trong rượu.

Khi đun nóng với axit loãng tạo ra HCN ( T s = 26 o C ) tự do:

K 4 [Fe(CN) 6 ] + 3H 2 SO 4 2K 2 SO 4 + FeSO 4 + 6HCN

còn khi tác dụng với CuSO 4 tạo ra kết tủa màu nâu socola Cu 2 [Fe(CN) 6 ]:

CuSO 4 + K 4 [Fe(CN) 6 ] Cu 2 [Fe(CN) 6 ] + K 2 SO 4

Nâu

(2) • Khi cho KCN tác dụng với dung dịch muối coban (II) hình thành kết tủa

màu đỏ nâu coban (II) xianua Co(CN) 2 :

CoCl 2 + 2KCN Co(CN) 2 + 2KCl

• Co(CN) 2 tan trong KCN dư tạo ra dung dịch màu vàng , sau đó đỏ dần và

cuối cùng tạo ra tinh thể màu tím có thành phần K 4 [Co(CN) 6 ] ( kali

cobantoxianua):

Co(CN) 2 + 4KCN K 4 [Co(CN) 6 ]

• Hợp chất này bền trong điều kiện không có không khí, dễ tan trong nước

cho dung dịch màu đỏ, nhưng khi đun nóng dung dịch sẽ bị H 2 O oxi hóa (

không có không khí):

2K 4 [Co(CN) 6 ] + 2H 2 O 2K 3 [Co(CN) 6 ] + 2KOH + H 2

(3) • Khi cho KCN tác dụng với muối niken(II) tạo ra Ni(CN) 2 kết tủa màu

xanh sáng.

• Khi cho dư KCN, kết tủa ban đầu sẽ tan tạo thành phức chất màu vàng

K 2 Ni(CN) 4 ] , kết tinh dạng monohiđrat K 2 Ni(CN) 4 ].H 2 O màu da cam.

NiSO 4 + 2KCN Ni(CN) 2 + K 2 SO 4

Ni(CN) 2 + 2KCN K 2 [Ni(CN) 4 ]

• Hợp chất này dễ mất nước và tan nhiều trong nước, bị axit mạnh phân hủy

giải phóng Ni(CN) 2

• Khi thêm nhiều KCN vào dung dịch chứa ton [Ni(CN) 4 ] 2- màu của dung

dịch từ vàng chuyển thành màu đỏ, do sự tạo thành con [Ni(CN) 4 ] 4- :

K 2 [Ni(CN) 4 ] + 2KCN K 4 [Ni(CN) 6 ]

2.1.2.13. Các sunfat của Fe(II), Co(II), Ni(II)

Phát hành PDF bởi Ths Nguyễn Thanh Tú

Đăng ký Word doc qua Zalo 0905779594 Email thanhtuqn88@gmail.com


(1) • Sắt(II) sunfat là muối quan trọng nhất trong thực tế, ở dạng lỏng tinh thể

có thành phần FeSO 4.7 H 2 O.

Trong công nghiệp được điều chế bằng cách dùng O 2 của không khí và H 2 O

oxi; hóa pirit Fe (II) ở nhiệt độ thường:

2FeS 2 + 7O 2 + 2H 2 O 2FeSO 4 + 2H 2 SO 4

Trong phòng thí nghiệm điều chế bằng cách hòa tan sắt tinh khiết trong

H 2 SO 4 loãng:

Fe + H 2 SO 4 FeSO 4 + H 2

• Kết tinh từ dung dịch nước tạo ra tinh thể màu xanh lam

[Fe(OH 2 ) 6 ]SO 4 .H 2 O.

• Khi nung nóng tạo ra muối khan màu trắng và khi nung nóng mạnh tạo ra

Fe 2 O 3 :

4FeSO 4

o

t

2Fe 2 O 3 + 4SO 2 + O 2

Phản ứng trên được dùng để điều chế Fe 2 O 3 .

• Nhiều kim loại khác cũng kết tinh đồng hình với FeSO 4 với công thức tổng

quát: MSO 4 .7H 2 O hoặc [M(OH 2 ) 6 ]SO 4 .H 2 O, trong đó M là Fe, Zn, Cd, Mn,

Cr, Co, Ni.

• FeSO 4 tạo ra dung dịch màu nâu tối khi tác dụng với khí NO do tạo ra phức

chất nitrozo Fe(II) sunfat:

FeSO 4 + NO [FeNO]SO 4

nâu

• Cũng như các muối Fe(II) khác, FeSO 4 bị oxi hóa chậm khi để trong không

khí. Trong môi trường axit tạo ra muối Fe(II); còn khi không có mặt axit tạo

ra muối bazơ:

4FeSO 4 + O 2 + 2H 2 O 4Fe(OH)SO 4

4FeSO 4 + O 2 + 2H 2 SO 4 2Fe 2 (SO 4 ) 3 + 2H 2 O

FeSO 4 là chất khử tốt, khử được muối Ag + và muối Au 3+ thành kim loại:

Ag + + Fe 2+ Fe 3+ + Ag

Au 3+ + 3Fe 2+ 3Fe 3+ + Au

Khử được HgCl 2 (hợp chất cộng hóa trị) thành Hg 2 Cl 2 :

Phát hành PDF bởi Ths Nguyễn Thanh Tú

Đăng ký Word doc qua Zalo 0905779594 Email thanhtuqn88@gmail.com


2HgCl 2 + 2Fe 2+ Hg 2 Cl 2 + 2Fe 3+ + 2Cl -

• FeSO 4 hình thành muối kép với kim loại kiềm hoặc amoni:

R 2 SO 4 .FeSO 4 .6H 2 O (R là kim loại kiềm)

(NH 4 ) 2 SO 4 .FeSO 4 .6H 2 O (muối Mohr)

(2) • Coban(II) sunfat được điều chế bằng cách cho kim loại, oxit, hiđroxit

tan trong H 2 SO 4 . Ví dụ:

CoO + H 2 SO 4 CoSO 4 + H 2 O

CoSO 4 kết tinh từ dung dịch ở dạng CoSO 4 .7H 2 O hay [Co(OH 2 ) 6 ]SO 4 .H 2 O

màu đỏ son.

• Để trong không khí không hề bị biến đổi, nhưng khi nung mạnh thì ban đầu

mất nước kết tinh trở thành dạng khan CoSO 4 màu đỏ và sau đó bị phân hủy

thành Co 3 O 4 :

3CoSO 4

o

t

Co 3 O 4 + SO 2 + 2SO 3

• CoSO 4 dễ tan trong nước nhưng không tan trong rượu. Trong dung dịch

nước ở 40 – 50 o C kết tinh ở dạng CoSO 4 .6H 2 O.

CoSO 4 cũng tạo thành muối kép với kim loại kiềm và amoni, ví dụ amoni

coban(II) sunfat màu đỏ (NH 4 ) 2 SO 4 .CoSO 4 .6H 2 Ođồng hình với muối amoni

magie sunfat.

• Coban(II) sunfat tạo ra dạng muối bazơ: dạng màu xanh có thành phần

CoSO 4 .3Co(OH) 2 và dạng màu tím có thành phần 2CoSO 4 .3Co(OH) 2 .5H 2 O.

(3)

• Niken (II) sunfat được điều chế bằng cách hòa tan niken, oxit hoặc hiđroxit

trong H 2 SO 4 :

Ni(OH) 2 + H 2 SO 4 NiSO 4 2H 2 O

Kết tinh từ dung dịch nước ở nhiệt độ thấp hơn 32 o C tạo ra tinh thể

NiSO 4 .7H 2 O hay [Ni(OH 2 ) 6 ]SO 4 .H 2 O màu xanh. Trên 32 o C khoảng 31,5 -

53,3 o C kết tinh dạng NiSO 4 .6H 2 O màu xanh, còn khi kết tinh ở trên 53,3 o C

tạo ra dạng NiSO 4 .6H 2 O màu lục.

• Tác dụng với khí NH 3 tạo thành amoniacat:

NiSO 4 + 6NH 3 [Ni(NH 3 ) 6 ]SO 4

Phát hành PDF bởi Ths Nguyễn Thanh Tú

Đăng ký Word doc qua Zalo 0905779594 Email thanhtuqn88@gmail.com


nhưng với (NH 4 ) 2 SO 4 thì tạo muối kép màu lục xanh: (NH 4 ) 2 SO 4 .NiSO 4 .6H 2 O

được dùng làm chất mạ niken.

NiSO 4 cũng tạo ra muối kép với các kim loại kiềm, ví dụ như

K 2 SO 4 .NiSO 4 .6H 2 O.

2.1.2.14. Các nitrat của Fe(II), Co(III), Ni(II)

(1) • Sắt(II) nitrat tạo ra khi hòa tan sắt trong HNO 3 loãng, lạnh:

8Fe + 20HNO 3 8Fe(NO 3 ) 2 + 2NH 4 NO 3 + 6H 2 O

Tốt nhất là bằng phản ứng trao đổi giữa FeSO 4 và Pb(NO 3 ) 2 :

FeSO 4 + Pb(NO 3 ) 2 Fe(NO 3 ) 2 + PbSO 4

• Kết tinh ở dạng hexahiđrat Fe(NO 3 ) 2 .6H 2 O màu xanh sáng ở nhiệt độ

thường, còn ở - 10 o C kết tinh ở dạng nonahiđrat Fe(NO 3 ) 2 .9H 2 O

Khi đun nóng dung dịch sắt(ll) nitrat bị phân hủy tạo thành muối bazơ sắt

(III).

(2) • Coban(II) nitrat được điều chế bằng cách hòa tan coban, oxit Co(II),

Co(OH) 2 .

CoCO 3 .6H 2 O trong axit HNO 3 loãng :

Co(OH) 2 + 2HNO 3 Co(NO 3 ) 2 + 2H 2 O

• Kết tinh ở trạng tinh thể hexahiđrat Co(NO 3 ) 2 .6H 2 O màu đỏ thẫm, chảy rữa

ngoài không khí ẩm, khi giữ lâu trên H 2 SO 4 sẽ mất một phân tử H 2 O. Khi đun

nóng sẽ nóng chảy ở gần 55 o C rồi mất nước kết tinh, sau đó bị phân hủy:

3Co(NO 3 ) 2

o

t

Co 3 O 4 + 6NO 2 + O 2

(3) Niken (II) nitrat được điều chế bằng cách hòa tan niken kim loại, NiCO 3

trong HNO 3 :

NiCO 3 + 2HNO 3 Ni(NO 3 ) 2 + CO 2 + H 2 O

Kết tinh ở dạng hexahiđrat Ni(NO 3 ) 2 .H 2 O màu lam ngọc bích, đồng hình với

muối coban tương ứng.

Niken(II) nitrat dễ lên hoa trong không khí khô và chảy rữa nhanh trong

không khí ẩm.

Nóng chảy trong nước kết tinh ở 57 o C, tiếp tục đun nóng cao hơn 85,4 o C mất

dần nước kết tinh và đến 300 o C phân hủy còn lại Ni 2 O 3 :

2Ni(NO 3 ) 2

o

t

Ni 2 O 3 + 4NO 2 + 1/2O 2

Phát hành PDF bởi Ths Nguyễn Thanh Tú

Đăng ký Word doc qua Zalo 0905779594 Email thanhtuqn88@gmail.com


Ni 2 O 3 là chất bột màu xám hoặc đen, do đó người ta dùng Ni(NO 3 ) 2 để tạo

màu xám trong công nghiệp đồ gốm.

2.1.2.15. Muối cacbonat của Fe(II), CO(II), Ni(II)

(1) • Sắt(II) cacbonat gặp trong thiên nhiên thành những mỏ lớn ở dạng

khoáng chất xiđerit, được điều chế bằng cách cho dung dịch muối Fe 2+ tác

dụng với dung dịch muối cacbonat trung tính chẳng hạn Na 2 CO 3 :

FeSO 4 + Na 2 CO 3 FeCO 3 + Na 2 SO 4

• FeCO 3 là chất rắn màu trắng không tan trong nước, nhưng tương tự như

CaCO 3 tan rõ rệt trong nước có chứa CO 2 vì tạo nên muối hiđrocacbonat :

FeCO 3 + CO 2 + H 2 O Fe(HCO 3 ) 2

• Fe(HCO 3 ) 2 cũng dễ bị phân hủy khi đun nóng. Trong nước thiên nhiên

thường có sắt ở dạng sắt(II) hiđrocacbonat cùng với Ca(HCO 3 ) 2 và

Mg(HCO 3 ) 2 .

• Ở trạng thái ẩm, nhanh chóng trở thành màu lục và sau đó chuyển thành nâu

khi để trong không khí, do quá trình thủy phân dần dần và quá trình oxi hóa

tạo thành Fe(OH) 3 :

Khi đun nóng, FeCO 3 bị phân hủy:

4FeCO 3 + O 2 + 6H 2 O 4Fe(OH) 3 + 4CO 2

FeCO 3

o

t

FeO + CO 2

áp suất hơi khi phân hủy là 1 atm ở 282 o C.

Fe(HCO 3 ) 2 cũng bị oxi hóa thành Fe 2 O 3 :

4Fe(HCO 3 ) 2 + O 2 2Fe 2 O 3 + 8CO 2 + 4H 2 O

(2) • Coban(II) cacbonat; niken(II) cacbonat kết tủa ở dạng hiđrat tinh thể khi

cho các hiđrocacbonat của kim loại kiềm tác dụng với muối của CO(II) và

NI(II):

CoCl 2 + 2NaHCO 3 CoCO 3 + 2NaCl + CO 2 + H 2 O

NiCl 2 + 2NaHCO 3 NiCO 3 + 2NaCl + CO 2 + H 2 O

Khác với cách điều chế FeCO 3 nếu cho các muối CO(II), Ni(II) tác dụng với

Na 2 CO 3 thì sản phẩm thu được lại là muối cacbonat bazơ kết tủa

CoCO 3 .nCo(OH) 2 (màu xanh); NiCO 3 .mNi(OH) 2 màu lục sáng.

Phát hành PDF bởi Ths Nguyễn Thanh Tú

Đăng ký Word doc qua Zalo 0905779594 Email thanhtuqn88@gmail.com


Muối khan CoCO 3 và NiCO 3 có thể điều chế bằng cách đun nóng dạng

hiđrat tinh thể trong ống hàn kín ở 140 o C.

CoCO 3 thực tế không tan trong nước (T = 8.10 -13 ở 25 o C ), còn NiCO 3 có tích

số tan T = 6,6.10 -9 nhưng NiCO 3 .mNi(OH) 2 lại có tích số tan T = 1,7. 10 -7 .

CÁC HỢP CHẤT VỚI OXI HÓA +3

Như đã nêu ở mục 2 hợp chất ứng với số oxi hóa +3 của niken là không đặc

trưng, nên trong phần này chủ yếu trình bày các hợp chất của Fe(III) và

Co(III).

Số phối trí của Fe(III) là bằng 6 và 4 ứng với sự phân bố tám mặt và bốn mặt.

Chẳng hạn FeCl 3 ở trạng thái hơi ứng với công thức Fe 2 Cl 6 (dạng Al 2 Cl 6 ) còn

khi hòa vào nước tạo ra phức chất tám mặt [Fe(OH 2 ) 6 ].

Số phối trí đặc trưng của Co(III) là bằng 6, chẳng hạn các ion Co(OH 2 ) 6 ] 3+ ,

[Co(NH 3 ) 6 ] 3+ .

Màu sắc của các hợp chất sắt (III) phụ thuộc vào bản chất của anion. Ví dụ:

FeCl 3 - màu nâu đen; Fe(CNS) 3 – màu đỏ máu; Fe 2 S 3 – màu đen;

Fe(NO 3 ) 3 .6H 2 O – màu tím sáng.

Các hợp chất của Fe(III) đều có tính oxi hóa; trong dung dịch các muối Fe(III)

bị khử đến muối Fe(II).

2.1.2.16. Các oxit Fe 2 O 3 , Co 2 O 3 , Fe 3 O 4 , Co 3 O 4

(1) • Sắt (III) oxit tạo ra ba dạng thù hình là - Fe 2 O 3 dạng thuận từ, dạng -

Fe 2 O 3 là dạng sắt từ và dạng d - Fe 2 O 3 có cấu trúc kiểu corinđon.

• Khi nung sắt (III) hiđroxit, nói đúng hơn là oxit bị hiđrat hóa Fe 2 O 3 .nH 2 O, ở

nhiệt độ thấp hơn 650 o C tạo ra chất rắn ở dạng bột màu đỏ nâu, nhưng nếu

nung ở nhiệt độ cao hơn tạo thành tinh thể màu xám đen không còn khả năng

tan trong axit, tương tự như Cr 2 O 3 , Al 2 O 3 dạng tinh thể:

Fe 2 O 3 .nH 2 O

o

t

Fe 2 O 3 + nH 2 O

• Fe 2 O 3 Cũng có thể điều chế bằng cách nung FeSO 4 .7H 2 O; FeO hoặc một

muối sắt (II) của axit dễ bay hơi khác:

FeSO 4 .7H 2 O

2FeSO 4

o

t

FeSO 4 + 7H 2 O

o

t

Fe 2 O 3 + SO 2 + SO 3

Phát hành PDF bởi Ths Nguyễn Thanh Tú

Đăng ký Word doc qua Zalo 0905779594 Email thanhtuqn88@gmail.com


Ngoài ra trong công nghiệp được điều chế bằng cách nung quặng pirit mà

thành phần chính là FeS 2 .

4FeS 2 + 11O 2

o

t

2Fe 2 O 3 + 8SO 2

• Fe 2 O 3 nóng chảy ở 1565 o C và thăng hoa ở 2000 o C.

• Fe 2 O 3 tan trong axit tạo thành ion phức [Fe(OH 2 ) 6 ] 3+ không màu ; màu nâu

của dung dịch muối sắt (III) là do màu của sản phẩm phản ứng thủy phân, tức

là màu của các con phức hiđroxo - aqua:

[Fe(OH 2 ) 6 ] 3+ + H 2 O [Fe(OH 2 ) 5 OH] 2+ + H 3 O +

vàng nâu

• Bên cạnh tính chất chủ yếu là tính bazơ, Fe 2 O 3 còn có tính axit tạo thành

muối ferit màu vàng hoặc đỏ, khi nung hỗn hợp Na 2 CO 3 + Fe 2 O 3 :

Na 2 CO 3 + Fe 2 O 3

o

t

2NaFeO 2 + CO 2

Khi nung với C, hoặc nung trong luồng khí CO, H 2 hoặc khí than đá, Fe 2 O 3 sẽ

bị khử thành Fe:

Fe 2 O 3 + 3C

Fe 2 O 3 + 3CO

Fe 2 O 3 + 3H 2

o

t

2Fe + 3CO

o

t

2Fe + 3CO 2

o

t

2Fe + 3H 2 O

(2) • Coban(III) oxit được điều chế bằng cách nhiệt phân Co(NO 3 ) 2 :

4Co(NO 3 ) 2

o

t

2Co 2 O 3 + 8NO 2 + O 2

Cũng được điều chế dạng tinh khiết bằng cách nung Co(OH) 2 trong lò điện ở

350 – 370oC trong chén sứ:

4Co(OH) 2 + O 2

o

t

2CO 2 O 3 + 4H 2 O

• Để có sản phẩm thật tinh khiết người ta đã dùng phản ứng sau:

4CoCl 2 + 4NH 4 Cl + 16NH 3 + O 2 4[Co(NH 3 ) 5 Cl]Cl 2 + 2H 2 O

2[Co(NH 3 ) 5 Cl]Cl 2 + 3H 2 O ⇌ Co 2 O 3 + 4NH 3 + 6NH 4 Cl

• Co 2 O 3 là chất bột màu nâu sẫm, nung đến gần 600oC tạo thành CoO 4 là chất

bột màu

6Co 2 O 3

o

t

4Co 3 O 4 + O 2

• Nung ở 1300 o C phân hủy tiếp tạo ra 4CoO.Co 2 O 3 và cuối cùng là CoO :

Phát hành PDF bởi Ths Nguyễn Thanh Tú

Đăng ký Word doc qua Zalo 0905779594 Email thanhtuqn88@gmail.com


2Co 2 O 3

o

t

4CoO + O 2

• Co 2 O 3 cũng bị hiđro khử đến kim loại :

t

Co 2 O 3 + 3H o

2 2Co + 3H 2 O

• Co 2 O 3 tan trong axit HCl tạo ra Cl 2 và với H 2 SO 4 tạo ra O 2 :

Co 2 O 3 + 6HCl 2CoCl 2 + Cl 2 + 3H 2 O

Co 2 O 3 + 4H 2 SO 4 4CoSO 4 + O 2 + 4H 2 O

(3) • Niken(III) oxit tạo ra khi nung trong không khí muối cacbonat hoặc

nitrat niken(II) ở 300 o C:

4NiCO 3 + O 2

4Ni(NO 3 ) 2

o

t

2Ni 2 O 3 + 4CO 2

o

t

2NiO 3 + 8NO 2 + O 2

tuy nhiên thành phần của sản phẩm thu được từ các phản ứng đó có hàm

lượng oxi thấp hơn so với công thức Ni 2 O 3 .

Là chất bột màu đen hoặc xám; cũng như Co 2 O 3 , Ni 2 O 3 là chất oxi hóa mạnh,

chẳng hạn tan trong HCl tạo ra khí Cl 2 :

Ni 2 O 3 + 6HCl 2NiCl 2 + Cl 2 + 3H 2 O

(4) • Sắt(II, III) oxit (Fe 3 O 4 ) có thành phần ứng với khoảng chất oxít sắt từ

được điều chế bằng cách nung nóng Fe 2 O 3 ở nhiệt độ trên 1450 o C :

6Fe 2 O 3

o

t

4Fe 3 O 4 + O 2

Khi nung sắt trong không khí tạo ra Fe 3 O 4 (cùng với các sản phẩm khác):

3Fe + 2O 2

o

t

Fe 3 O 4

• Fe 3 O 4 cũng tạo ra khi cho luồng hơi H 2 O hoặc CO 2 qua sắt nung đỏ :

3Fe + 4H 2 O Fe 3 O 4 + 4H 2

3Fe + 4CO 2

o

t

Fe 3 O 4 + 4CO

Cũng có thể điều chế bằng cách cho hỗn hợp hiđro và hơi nước đi qua Fe 2 O 3

nung ở 400 o C:

3Fe 2 O 3 + H 2

o

t

2Fe 3 O 4 + H 2 O

• Ngoài ra để điều chế dạng tinh khiết người ta đã cho dung dịch FeSO 4 và

Fe 2 (SO 4 ) 3 tác dụng với dung dịch KOH, đun sôi thu được kết tủa Fe 3 O 4 :

FeSO 4 + Fe 2 (SO 4 ) 3 + 8KOH Fe 3 O 4 + 4K 2 SO 4 + 4H 2 O

Phát hành PDF bởi Ths Nguyễn Thanh Tú

Đăng ký Word doc qua Zalo 0905779594 Email thanhtuqn88@gmail.com


làm khô kết tủa trong khí quyển H 2 thu được sản phẩm với thành phần Fe 3 O 4

.2H 2 O.

• Fe 3 O 4 là chất bột màu đen, nóng chảy ở 1540 o C; nung ở nhiệt độ cao hơn,

mất một phần oxi tạo ra FeO:

2Fe 3 O 4

o

t

6FeO + O 2

Ở trạng thái ẩm, dễ bị oxi hóa ngoài không khí tạo ra Fe 2 O 3 :

4Fe 3 O 4 + O 2 6Fe 2 O 3

• Khi tác dụng với ít axit tạo ra Fe 2 O 3 Và dung dịch muối sắt (II), sau đó thêm

tiếp axit đến dư, sẽ tan hoàn toàn tạo nên muối sắt(II) và sắt(III):

Fe 3 O 4 + 2HCl Fe 2 O 3 + FeCl 2 + H 2 O

Fe 3 O 4 + 8HCl 2FeCl 3 + FeCl 2 + 4H 2 O

• Fe 3 O 4 bị (Al, H 2 ), CO khử thành kim loại:

3Fe 3 O 4 + 8Al

o

t

9Fe + 4Al 2 O 3

• Oxit sắt từ thiên nhiên (manhetit) không tan trong kiềm và axit, lại có độ dẫn

điện cao nên đôi khi được dùng làm điện cực trong một số quá trình điện hóa,

như điện phân muối clorua kim loại kiềm....

Người ta cũng xem Fe 3 O 4 như là một muối của axit ferơ HFeO 2 và Fe(OH) 2 :

II III

Fe

FeO2

2

(5) • Coban(II, III) oxit (Co 3 O 4 ) được tạo ra khi nung coban(II) nitrat:

o

t

3Co(NO 3 ) 2 Co 3 O 4 + 6NO 2 + O 2

Co 3 O 4 cũng được tạo ra khi nung CoO trong không khí:

o

t

6CoO + O 2 2Co 3 O 4

• Co 3 O 4 là chất bột màu đen, bị H 2 khử thành kim loại khi nung nóng:

o

t

Co 3 O 4 + 4H 2 3Co + 4H 2 O

tác dụng với axit HCl tạo ra muối Co(II) và Cl 2 :

Co 3 O 4 + 8HCl 3CoCl 2 + Cl 2 + 4H 2 O

Phát hành PDF bởi Ths Nguyễn Thanh Tú

Đăng ký Word doc qua Zalo 0905779594 Email thanhtuqn88@gmail.com


Khác với Fe 3 O 4 , trong mạng tinh thể có ion kim loại hóa trị II và hóa trị III,

trong Co 3 O 4 có ion kim loại hóa trị II và hóa trị IV, và được xem là một muối

của Co 2+ tương tự Mn 3 O 4 : Co

II

Co IV

O

.

2 4

2.1.2.17. Các hiđroxit Fe(OH) 3 , Co(OH) 3 , Ni(OH) 3

(1) • Sắt (III) hiđroxit là chất kết tủa màu đỏ nâu được tạo ra khi cho một tác

nhân kết tủa như kiềm, amoniac, dung dịch cacbonat tác dụng với muối

Fe(III):

3CO 2

FeCl 3 + 3NH 3 + 3H 2 O Fe(OH) 3 + 3NH 4 Cl

2FeCl 3 + 3Na 2 CO 3 + 3H 2 O 2Fe(OH) 3 + 6NaCl +

• Thực ra, dạng kết tủa keo màu nâu đỏ vô định hình đó là dạng Fe 2 O 3 .nH 2 O,

với hàm lượng nước khác nhau. Trong công thức thường viết Fe(OH) 3 chính

là Fe 2 O 3 .3H 2 O; trong thiên nhiên, dạng hematit nâu Fe 2 O 3 .H 2 O hay là

FeO(OH).

• Fe(OH) 3 khô là những cục xốp, khối lượng riêng thay đổi trong khoảng từ

3,4g/cm 3 đến 3,9g/cm 3 , hầu như không tan trong nước (T = 4.10 -38 ).

• Khả năng hòa tan trong axit phụ thuộc vào "tuổi" của kết tủa. Kết tủa vừa

mới điều chế dê tan trong axit vô cơ và hữu cơ, nhưng để lâu một thời gian thì

lại khó tan.

• Khi đun nóng đến 500 – 700 o C sẽ mất nước hoàn toàn biến thành Fe 2 O 3 :

2Fe(OH) 3

o

t

Fe 2 O 3 + 3H 2 O

• Bên cạnh tính chất chủ yếu là tính bazơ, Fe(OH) 3 còn thể hiện tính axit yếu

(axit ferơ HFeO 2 ) nên khi cho Fe(OH) 3 tác dụng với kiềm đặc nóng hoặc bằng

cách nấu chảy với Na 2 CO 3 hay K 2 CO 3 tạo thành các muối ferit NaFeO 2 hay

KFeO 2 :

Fe(OH) 3 + NaOH đặc

o

t

NaFeO 2 + 2H 2 O

o

t

2Fe(OH) 3 + K 2 CO 3 2KFeO 2 + CO 2 + 3H 2 O

• NaFeO 2 hay các ferit khác đều bị thủy phân đến kiềm và Fe 2 O 3 :

2NaFeO 2 + H 2 O Fe 2 O 3 + 2NaOH

Người ta cũng lợi dụng phản ứng trên để điều chế NaOH trong công nghiệp

bằng cách nung hỗn hợp Na 2 CO 3 và Fe 2 O 3 ở 1100 o C:

Phát hành PDF bởi Ths Nguyễn Thanh Tú

Đăng ký Word doc qua Zalo 0905779594 Email thanhtuqn88@gmail.com


Na 2 CO 3 + Fe 2 O 3

sau đó cho NaFeO 2 thủy phân như trên.

o

t

2NaFeO 2 + CO 2

• Ngoài ferit của kim loại kiềm, người ta còn biết một số ferit khác như

Mg(FeO 2 ) 2 Zn(FeO 2 ) 2 ....

(2) • Coban(III) hiđroxit là chất bột màu nâu đen, tạo ra khi oxi hóa Co(OH))

bằng oxi của không khí tương tự Fe(OH) 2 hoặc oxi hóa nhanh các muối

Co(II) khi cho tác dụng với các chất như NaOCl, Cl 2 , Br 2 , H 2 O:

2CoCl 2 + NaOCl + 4NaOH + H 2 O 2Co(OH) 3 + 5NaCl

2CoCl 2 + H 2 O 2 + 4NaOH 2Co(OH) 3 + 4NaCl

• Co(OH) 3 là chất không tan trong nước ( T = 4.10 -45 ), tan trong axit tạo thành

muối CO(III), trong kiềm đặc dư tạo ra muối hiđroxo, ví dụ: K 3 [Co(OH) 6 ].

(3) • Niken(III) hiđroxit là chất bột màu đen, tạo ra khi oxi hóa Ni(OH) 2 bằng

oxi không khí hoặc bằng brom trong môi trường kiềm tạo thành oxit bị hiđrat

hóa NiO(OH), thường viết ở dạng Ni(OH) 3 .

Quá trình oxi hóa đó cũng được thực hiện bằng cách oxi hóa điện phân

trong môi trường kiềm. Người ta đã lợi dụng tính oxi hóa của Ni(OH) 3 để chế

tạo ăcquy kiềm (Acquy Edisen). Khác với ăcquy chì, ăcquy kiềm gồm một

điện cực bằng Fe, và một điện cực được chế tạo bằng bột oxit Ni(III) ngậm

nước với chất điện ly là KOH. Sự vận hành của loại ắcquy trên theo sơ đồ:

Fe + 2Ni(OH) 3 Fe(OH) 2 + 2Ni(OH) 2

(1,3V)

Nhược điểm của loại ăcquy này là hệ số hiệu dụng thấp.

CÁC MUỐI SẮT(III), COBAN(III)

• Các muối Fe(III) được điều chế bằng cách oxi hóa các muối Fe(II) hoặc

Fe(OH) 2 bằng axit tương ứng. Ví dụ hòa tan Fe(OH) 2 trong HNO 3 :

3Fe(OH) 2 + 10HNO 3 3Fe(NO 3 ) 3 + NO + 8H 2 O

• Dung dịch của các muối Fe(III) không có axit dư, có màu thay đổi từ vàng -

nâu đến nâu thẫm, màu sắc đó không phải là màu của ion Fe 3+ mà do dạng

keo của muối bazơ gây ra bởi phản ứng thủy phân.

Bản thân của ion Fe 3+ không màu, trong dung dịch ở dạng hiđrat hóa

[Fe(OH 2 ) 6 ] 3+ màu vàng nhạt, màu vàng đậm dần do quá trình:

Phát hành PDF bởi Ths Nguyễn Thanh Tú

Đăng ký Word doc qua Zalo 0905779594 Email thanhtuqn88@gmail.com


2H +

[Fe(OH 2 ) 6 ] 2+ [Fe(OH 2 ) 5 OH] 2+ + H + [Fe(OH 2 ) 4 (OH) 2 ] + +

Trong môi trường axit dư, màu của dung dịch lại phụ thuộc vào bản chất của

anion và axit thêm vào. Ví dụ dung dịch FeF 3 khi thêm axit HCl có màu hồng

, còn dung dịch FeCl 3 có màu vàng.

• Các muối Fe(III) đều dễ bị khử đến muối Fe(II) bởi các chất như hiđro mới

sinh , SO 2 , SnCl 2 , H 2 S, KI và một số kim loại âm điện hơn như Zn... Ví dụ:

Fe 3+ + H Fe 2+ + H +

2Fe 3+ + SO 2 + 2H 2 O 2Fe 2+ + SO 4 2-+ 4H +

2Fe 3+ + Sn 2+ 2Fe 2+ + Sn 4

+

2Fe 3+ + H 2 S 2Fe 2+ + S + 2H +

2Fe 3+ + Zn 2Fe 2+ + Zn 2+

2Fe 3+ + 2I - 2Fe 2+ + I 2

• Trừ oxit Co(III) và các phức chất, nói chung các muối Co(III) đều kém bền

và đều có tính oxi hóa.

2.1.2.18. Các halogenua của Fe(III), Co(III)

(1) • Sắt (III) nghĩa được điều chế khi hòa tan Fe(OH) 3 trong axit HF, sau khi

làm bay hơi trên nồi cách thủy thu được tinh thể FeF 3 . 3H 2 O.

Tinh thể hiđrat hóa có màu hồng nhạt; dạng khan có màu hơi xanh.

(2) • Sắt (III) clorua thu được khi cho khí clo khô qua bột sắt nung nóng đỏ

tạo ra FeCl 3 khan :

o

t

2Fe + 3Cl 2 2FeCl 3

Khi hòa tan sắt trong axit HCl, sau đó cho khí clo qua dung dịch, từ dung dịch

này tinh thể hexahiđrat FeCl 3 .6H 2 O sẽ tách ra:

Fe + 2HCl FeCl 2 + H 2

2FeCl 2 + Cl 2 2FeCl 3

• FeCl 3 khan là những tinh thể màu hung đen dạng vẩy hoặc phiến, có màu đỏ

lựu trong ánh sáng truyền qua và có màu lục trong ánh sáng phản chiếu. Nóng

chảy ở 300 o C thành chất lỏng linh động màu đỏ. Sôi và phân hủy một phần ở

317 o C.

Phát hành PDF bởi Ths Nguyễn Thanh Tú

Đăng ký Word doc qua Zalo 0905779594 Email thanhtuqn88@gmail.com


Ở 440 o C, tỉ khối hơi ứng với công thức kép Fe 2 Cl 6 (hình 74 ); ở 750 o C ứng

với công thức lớn FeCl 3 ; trên 750 o C ứng với sự phân hủy FeCl 3 thành FeCl 2

và Cl 2 :

2FeCl 3

o

750 C

2FeCl 2 + Cl 2

Trong không khí, FeCl 3 dễ bị chảy rữa, nhưng khi nung FeCl 3 trong không

khí hoặc cho luồng hơi nước đi qua sẽ chuyển thành Fe 2 O 3 :

o

t

4FeCl 3 + 3O 2 2Fe 2 O 3 + 6Cl 2

o

t

2FeCl 3 + 3H 2 O Fe 2 O 3 + 6HCl

Hình 9. Sơ đồ cấu tạo phân tử Fe 2 Cl 6

(3) • Fe(III) bromua màu đỏ hung, cũng được điều chế tương tự FeCl 3 :

2Fe + 3Br 2 2FeBr 3

Dạng khan có màu nâu đỏ; dạng hiđrat hóa FeBr 3.6 H 2 O là những tinh thể có

màu lục thẫm. Cả hai dạng đều dễ tan trong nước. Khi nung ở nhiệt độ cao

FeBr 3 bị phân hủy dễ hơn so với FeCl 3 :

2FeBr 3 2FeBr 2 + Br 2

(4) Sắt (III) iotua không bền ngay ở nhiệt độ thường, chỉ bền trong cân bằng

với lượng dư lớn FeI 2

2FeI 3 2FeI 2 + Br 2

do đó không thể tách ra ở dạng nguyên chất. Khi nghiền I 2 với bột sắt tạo nên

sản phẩm có thành phần Fe 3 I 8 (tức 2FeI 3 .FeI 2 ). Khi nung sản phẩm này với

K 2 CO 3 tạo ra KI:

Fe 3 I 8 + 4K 2 CO 3 8KI + Fe 3 O 4 + 4CO 2

(4) • Coban(III) florua bền nhất trong halogenua của Co(III) . CoF 3 là chất

bột màu hung lục có thể điều chế bằng cách cho flo tác dụng với CoF 2 ở

300 o C:

2CoF 2 + F 2

o

300 C

2CoF 3

Phát hành PDF bởi Ths Nguyễn Thanh Tú

Đăng ký Word doc qua Zalo 0905779594 Email thanhtuqn88@gmail.com


Cũng được điều chế ở dạng hiđrat CoF 3 .3,5H 2 O khi điện phân với điện cực

platin một dung dịch bão hòa CoF 2 trong dung dịch HF 40%, thu được tinh

thể màu lục tươi ở anot. Nước tinh khiết phân hủy COF 3 thành Co(OH) 3 :

CoF 3 + 3H 2 O Co(OH) 3 + 3HF

Khi đun nóng đến 300 o C trong luồng CO 2 , CoF 3 bị phân hủy thành CoF 2 và

F 2 .

(5) • Coban(III) clorua rất không bền. Khi cho Co(OH) 3 tác dụng với HCl

không thu được CoCl 3 do phản ứng:

2Co(OH) 3 + 6HCl 2CoCl 3 + 6H 2 O

nhưng có thể điều chế bằng cách cho Co 2 O 3 tác dụng với hiđro clorua trong

bóng tối ở - 5oC dưới lớp ete khô.

2.1.2.19. Các sunfat của Fe(III), Co(III)

(1) • Sắt (III) sunfat được điều chế bằng các cách sau đây :

Hòa tan hiđroxit sắt(III) trong H 2 SO 4 :

Đun nóng FeSO 4 với H 2 SO 4 đặc:

2Fe(OH) 3 + 3H 2 SO 4 Fe 2 (SO 4 ) 3 + 6H 2 O

2FeSO 4 + 2H 2 SO 4 Fe 2 (SO 4 ) 3 + SO 2 + 2H 2 O

Đun nóng FeSO 4 với HNO 3 và H 2 SO 4 đặc theo phương trình:

2FeSO 4 + 2HNO 3 đặc + H 2 SO 4 đặc Fe 2 (SO 4 ) 3 + 2NO 2 + 2H 2 O

Chế hóa Fe 2 O 3 với axit sunfuric đặc :

Fe 2 O 3 + 3H 2 SO 4 đặc Fe 2 (SO 4 ) 3 + 3H 2 O

• Ở trạng thái khan là chất bột màu trắng hoặc vàng nhạt, hút ẩm mạnh, để

ngoài không khí sẽ chảy rữa thành chất lỏng màu hung.

• Tạo nên với nước một số dạng hiđrat hóa như Fe 2 (SO 4 ) 3 . 9H 2 O và

Fe 2 (SO 4 ) 3 .l 0 H 2 O. Cả hai dạng này cũng gặp trong thiên nhiên. Khi tan trong

nước có khả năng tạo thành dung dịch rất đậm đặc, nhưng quá trình tan khá

chậm. Tan trong rượu , nhưng không tan trong H 2 SO 4 . Dung dịch nước có

màu nâu đỏ do bị thủy phân mạnh, nhưng nếu cho thêm H 2 SO 4 , phản ứng

thủy phân bị hạn chế và dung dịch hầu như không màu. Khi đun sôi dung dịch

loãng, muối bazơ sẽ kết tủa:

Fe 2 (SO 4 ) 3 + 2H 2 O ⇌ 2Fe(OH)SO 4 + H 2 SO 4

Phát hành PDF bởi Ths Nguyễn Thanh Tú

Đăng ký Word doc qua Zalo 0905779594 Email thanhtuqn88@gmail.com


Khi đun nóng, sẽ mất dần nước kết tinh tạo ra Fe 2 (SO 4 ) 3 khan và sau đó bị

phân hủy:

Fe 2 (SO 4 ) 3

o

t

Fe 2 O 3 + 3SO 3

• Sắt(III) sunfat có khả năng tạo ra muối kép dạng M.Fe(SO 4 ) 2 .12H 2 O được

gọi là phèn – sắt. Quan trọng hơn cả là phèn sắt – amoni NH 4 . Fe(SO 4 ) 2 .

12H 2 O và phèn sắt – kali K.Fe(SO 4 ) 2 .12H 2 O, được dùng làm chất cầm màu

vải. Ở trạng thái tinh khiết các loại phèn sắt này đều không màu, nhưng

thường có màu tím nhạt là do có dấu vết của mangan(III) sunfat.

Phèn được điều chế bằng cách oxi hóa dung dịch FeSO 4 .7H 2 O bằng HNO 3 , cô

dung dịch đến trạng thái bão hòa thì thêm một lượng đồng phân tử amoni

sunfat hoặc kali sunfat:

6FeSO 4 .7H 2 O + 3H 2 SO 4 + 2HNO 3 3Fe 2 (SO 4 ) 3 + 2NO + 46H 2 O

(NH 4 ) 2 SO 4 + Fe 2 (SO 4 ) 3 + 24H 2 O (NH 4 ) 2 SO 4 .Fe 2 (SO 4 ) 3 .24H 2 O

Ở 33 o C phèn – sắt amoni biến thành màu nâu; nóng chảy ở khoảng 40 - 41 o C,

đến 150 o C mất 23 phân tử H 2 O và ở 750 o C tạo thành phèn khan.

(2) • Coban(III) sunfat được điều chế bằng cách điện phân dung dịch đặc

CoSO 4 trong H 2 SO 4 , thu được tinh thể mỏng hình kim màu xanh nhạt, ở anot

đã làm lạnh đến 0 o C, có dạng hiđrat hóa Co 2 (SO 4 ) 3 .18H 2 O; bị H 2 O phân hủy,

nhưng trong dung dịch H 2 SO 4 loãng lại không bị phân hủy.

H 2 O phân hủy Co 2 (SO 4 ) 3 tạo ra oxit:

2Co 2 (SO 4 ) 3 + 2H 2 O 4CoSO 4 + O 2 + 2H 2 SO 4

• Với sunfat kim loại kiềm và amoni, Co 2 (SO 4 ) 3 tạo ra phèn - coban, ví dụ

phèn coban - kali được điều chế khi làm lạnh dung dịch đồng phân tử K 2 SO 4

và Co 2 (SO 4 ) 3 . Phèn K 2 SO 4 .Co 2 (SO 4 ) 3 .24H 2 O là những tinh thể tám mặt màu

xanh thẫm, bị H 2 O phân hủy ở nhiệt độ phòng tạo ra O 2 .

Phèn coban - rubiđi và phèn coban - xezi cũng có màu xanh thẫm nhưng khó

tan hơn nhiều so với phèn coban - kali.

Phèn coban - amoni (NH 4 ) 2 SO 4 .Co 2 (SO 4 ) 3 .24H 2 O cũng thu được khi điện

phân dung dịch gồm (NH 4 ) 2 SO 4 và CoSO 4 .

2.1.2.20. Sắt (III) nitrat

• Được tạo ra khi hòa tan vỏ bào sắt trong HNO 3 loãng gần 25% tạo ra Fe(III)

nitrat và hỗn hợp các oxit nào khác nhau. Ở nhiệt độ thường, phụ thuộc vào

nồng độ, muối đó có thể kết tinh ở dạng không màu hình lập phương có thành

Phát hành PDF bởi Ths Nguyễn Thanh Tú

Đăng ký Word doc qua Zalo 0905779594 Email thanhtuqn88@gmail.com


phần Fe(NO 3 ) 3 .6H 2 O hoặc dạng đơn tà màu tím có thành phần

Fe(NO 3 ) 3 .9H 2 O.

2.1.2.21. Các sunfua của Fe(III) và Co(III)

(1) • Fe(III) sunfua ( Fe 2 S 3 ) được tạo ra ở dạng kết tủa màu đen khi cho H 2 S

hoặc (NH 4 ) 2 S tác dụng với muối Fe 3+ trong môi trường trung tính hoặc kiềm

yếu (amoniac).

Ở trạng thái ẩm, Fe 2 S 3 bị phân hủy nhanh chóng trong không khí tạo thành

Fe(OH) 3 và lưu huỳnh tự do:

2Fe 2 S 3 + 3O 2 + 6H 2 O 4Fe(OH) 3 + 6S

• Fe 2 S 3 thực tế không tan trong nước; tan được trong dung dịch HCl loãng:

Fe 2 S 3 + 4HCl 2FeCl 2 + 2H 2 S + S

Tuy nhiên, người ta nghi ngờ kết tủa đó có phải là hoàn toàn là Fe 2 S 3 không,

hay là một hỗn hợp gồm FeS và S tạo ra do phản ứng:

2Fe 3+ + 3S 2- 2FeS + S

Khi nung sắt với S không tạo ra sắt(III) sunfua. Trong thiên nhiên gặp ở dạng

muối kép Cu 2 S. Fe 2 S 3 và 3Cu 2 S. Fe 2 S 3 .

(2) • Coban(III) sunfua (Co 2 S 3 ) chỉ tạo ra khi nung nóng chảy muối Co 2+ với

S và Na 2 CO 3 ; sản phẩm thu được là những tinh thể màu xám, không tan trong

nước và rất khó tan trong axit đặc.

2.1.2.22. Phức chất xianua của Fe(III) và Co(III)

(1) • Kali ferixianua (K 3 [Fe(CN) 6 ]) được điều chế bằng cách oxi hóa kém

feroxianua K 4 [Fe(CN) 6 ] bởi các chất như Cl 2 , KMnO 4 , HNO 3 hoặc H 2 O 2 :

2K 4 [Fe(CN) 6 ] + Cl 2 2K 3 [Fe(CN) 6 ] + 2KCl

Hợp chất này không thể điều chế bằng phương pháp như khi điều chế

K 4 [Fe(CN) 6 ], nghĩa là không thể cho muối Fe 3+ tác dụng với dung dịch KCN,

vì sẽ tạo ra kết tủa Fe(OH) 3 do sự thủy phân KCN tạo ra môi trường kiềm.

• K 3 [Fe(CN) 6 ] là chất kết tinh màu đỏ - gạch, khi nghiền nhỏ chuyển thành

chất bột màu vàng, tan trong nước cho dung dịch màu vàng.

Trong dung dịch nước, không có phản ứng của ion Fe 3+ và ion CN - vì ion

[Fe(CN) 6 ] 3- rất bền (K kb ≈ 10 -44 ):

K 3 [Fe(CN) 6 ] 3K + + [Fe(CN) 6 ] 3-

Phát hành PDF bởi Ths Nguyễn Thanh Tú

Đăng ký Word doc qua Zalo 0905779594 Email thanhtuqn88@gmail.com


• K 3 [Fe(CN) 6 ] là chất oxi hóa mạnh, đặc biệt hoạt động trong môi trường

kiềm, kali ferixianua có thể oxi hóa PbO thành PbO 2 ; hợp chất Cr 3+ thành

Cro 4 2- ; H 2 S thành S:

2K 3 [Fe(CN) 6 ] + 2KOH + PbO 2K 4 [Fe(CN) 6 ] + H 2 O + PbO 2

2K 3 [Fe(CN) 6 ] + 2KOH + H 2 S 2K 4 [Fe(CN) 6 ] + 2H 2 O + S

Khi cho HCl đặc tác dụng với dung dịch bão hòa K 3 [Fe(CN) 6 ] tạo ra tinh thể

màu đỏ nâu là axit ferixian hiđric H 3 [Fe(CN) 6 ]:

K 3 [Fe(CN) 6 ] + 3HCl H 3 [Fe(CN) 6 ] + 3KCl

Đun nóng K 3 [Fe(CN) 6 ] với dung dịch KOH đặc (với tỉ lệ l: 1) tạo ra

K 4 [Fe(CN) 6 ] và O 2 :

2H 2 O

4K 3 [Fe(CN) 6 ] + 4KOH đặc 4K 4 [Fe(CN) 6 ] + O 2 +

Với muối Fe 2+ , dung dịch K 3 [Fe(CN) 6 ] tạo ra kết tủa màu "xanh Tuabun"

(Tumbull):

[Fe(CN) 6 ] 3- + 3Fe 2+ Fe 3 [Fe(CN) 6 ]

Ngược lại, các muối Fe 3+ lại tác dụng với dung dịch kali feroxianua

K 4 [Fe(CN) 6 ] tạo ra kết tủa màu "xanh phổ" còn gọi là "xanh Beclin" hay

"xanh Prussian):

3[Fe(CN) 6 ] 4- + 4Fe 3+ Fe 4 [Fe(CN) 6 ] 3

• Trong ion [Fe(CN) 6 ] 3- có 17 electron hóa trị (12 electron của 6 nhóm CN - và

lk 12 5

5 electron của ion trung tâm), có cấu hình: [ ] [ p (d)]

nhưng muốn có cấu hình bền [n(d)] 6 cần phải thêm một electron, do đó ion

[Fe(CN) 6 ] 3- là chất oxi hóa mạnh. Tính oxi hóa của ion [Fe(OH 2 ) 6 ] 3+ cũng giải

thích tương tự, tuy nhiên, [Fe(CN) 6 ] 3- có tác dụng oxi hóa mạnh trong môi

trường kiềm, còn ion [Fe(OH 2 ) 6 ] 3+ thì trong môi trường axit.

(2) • Kali cobantixianua (K 3 [Co(CN) 6 ]) là những tinh thể màu vàng, thu được

khi cho muối Co 2+ tác dụng với KCN trong điều kiện có chất oxi hóa:

4K 4 [Co(CN) 6 ] + 4HCl + O 2 4K 3 [Co(CN) 6 ] + 4KCl + 2H 2 O

Nếu muối đó tác dụng với HNO 3 hay H 2 SO 4 Sẽ thu được tinh thể không màu

là axit cobantixian hiđric:

2K 3 [Co(CN) 6 ] + 3H 2 SO 4 2H 3 [Co(CN) 6 ] + 3K 2 SO 4

Phát hành PDF bởi Ths Nguyễn Thanh Tú

Đăng ký Word doc qua Zalo 0905779594 Email thanhtuqn88@gmail.com


Axit đó cũng có thể điều chế bằng cách cho K 3 [Co(CN) 6 ] tác dụng với

CuSO 4 để tạo ra kết tủa màu xanh là Cu 3 [Co(CN) 6 ] 2 . Sau đó cho phân hủy với

H 2 S:

2K 3 [Co(CN) 6 ] + 3CuSO 4 Cu 3 [Co(CN) 6 ] 2 (xanh) + 3K 2 SO 4

Cu 3 [Co(CN) 6 ] 2 + 3H 2 S 3CuS + 2H 3 [Co(CN) 6 ]

• Axit cobantixianhiđric là axit mạnh ba lần axit, kết tinh ở dạng hiđrat hóa

H 3 [Co(CN) 6 ].5H 2 O, trong dung dịch nước chỉ bền ở dưới 50 o C. Khi đun nóng

lâu đến 120 o C tạo ra Co(CN) 3 màu xanh:

H 3 [Co(CN) 6 ]

o

t

Co(CN) 3 + 3HCN

(3) • Sắt(III) thioxianat (Fe(NCS) 3 ) hay sắt(III) sunfoxianua được tạo ra khi

cho muối Fe 3+ tác dụng với KNCS hay (NH 4 )NCS :

FeCl 3 + 3KNCS Fe(NCS) 3 + 3KCl

là chất màu đỏ máu, có thể tách ra từ dung dịch ở dạng hiđrat tinh thể

Fe(NCS) 3 .3H 2 O màu đỏ thẫm, dễ tan trong nước. Màu đỏ máu của dung dịch

chính là màu của ion [FeNCS] 2+ .

HỢP CHẤT CỦA SẮT VỚI SỐ OXI HÓA +6

• Hợp chất gặp ở dạng ion FeO 4 2- tạo ra khi oxi hóa kim loại Fe hoặc hợp chất

của Fe(III) bằng chất oxi hóa mạnh trong môi trường kiềm mạnh.

Khi nấu chảy Fe 2 O 3 với KNO 3 và KOH tạo nên muối K 2 FeO 4 theo phản ứng:

Fe 2 O 3 + 3KNO 3 + 4KOH 2K 2 FeO 4 + 3KNO 2 + 2H 2 O

hoặc khi nấu chảy Fe 2 O 3 với Na 2 O 2 :

Fe 2 O 3 + 3Na 2 O 2 2Na 2 FeO 4 + Na 2 O

hoặc cho clo tác dụng với huyền phù Fe(OH) 3 trong kiềm đặc:

3Cl 2 + 2Fe(OH) 3 + 10NaOH

o

t

2Na 2 FeO 4 + 6NaCl + 8H 2 O

• Các ferat có màu đỏ sẫm, ít bền trong dung dịch nước, khi đun nóng nhẹ giải

phóng ra oxi

4Na 2 FeO 4 + 2H 2 O 4NaFeO 2 + 3O 2 + 4NaOH

Cho tác dụng với BaCl 2 tạo ra kết tủa màu đỏ tím của bari ferat:

2Na 2 FeO 4 + BaCl 2 BaFeO 4 + 2NaCl

Phát hành PDF bởi Ths Nguyễn Thanh Tú

Đăng ký Word doc qua Zalo 0905779594 Email thanhtuqn88@gmail.com


• Các ferat đều là chất oxi hóa mạnh, ví dụ oxi hóa amoniac ở điều kiện

thường dẫn đến nitơ tự do:

2Na 2 FeO 4 + 2NH 3 2NaFeO 2 + N 2 + 2NaOH + 2H 2 O

2.2. Hệ thống các câu hỏi và bài tập về các nguyên tố nhóm VII.B và

VIII.B

2.2.1. Hệ thống câu hỏi lý thuyết

Câu 1. a) Người ta có thể điều chế Mangan bằng phương pháp điện phân

dung dịch MnSO 4 . Hỏi có những quá trình nào đã xảy ra trên bề mặt điện cực

khi điện phân dung dịch đó?

b) Ngoài phương pháp điện phân có thể dùng phương pháp nào để điều chế

Mangan?

c) Từ MnO bằng phương pháp nào có thể thu được Mn(OH) 2 biết rằng MnO

không tan trong nước?

d) Bằng phản ứng nào chứng minh rằng Mn(OH) 2 có tính khử?

Hướng dẫn

a) Điện phân dung dịch MnSO 4 tương tự như quá trình điện phân dung dịch

NiSO 4 hoặc CuSO 4 .

b) Có thể điều chế Mn bằng phương pháp nhiệt kim hoặc nhiệt silic:

3Mn 3 O 4 + 8Al → 9Mn + 4Al 2 O 3

MnO 2 + Si → Mn + SiO 2

c) Chuyển MnO thành MnSO 4 hoặc MnCl 2 , sau đó cho dung dịch muối Mn 2+

tác dụng với kiềm thu được kết tủa Mn(OH) 2 mầu trắng.

d) Có thể dùng phản ứng :

2Mn(OH) 2 + O 2 (không khí) + 2H 2 O → Mn(OH) 4

Câu 2. Viết phương trình của các phản ứng sau:

1) MnSO 4 + KClO 3 + KOH(nóng chảy) →

2) MnSO 4 + PbO 2 + HNO 3 →

3) MnSO 4 + Br 2 + NaOH →

4) MnBr 2 + H 2 O 2 + KOH →

5) MnSO 4 + CaOCl 2 + NaOH →

Phát hành PDF bởi Ths Nguyễn Thanh Tú

Đăng ký Word doc qua Zalo 0905779594 Email thanhtuqn88@gmail.com


Hướng dẫn

1) 3MnSO 4 + 2KClO 3 + 12KOH → 3K 2 MnO 4 + 2KCl + 6H 2 O + 3K 2 SO 4

2) 2MnSO 4 + 5PbO 2 + 6HNO 3 → 2HMnO 4 + 3Pb(NO 3 ) 2 + 2PbSO 4 + 2H 2 O

3) MnSO 4 + 2H 2 O 2 + 4KOH → K 2 MnO 4 + 4H 2 O + K 2 SO 4

4) MnSO 4 + 2Br 2 + 8NaOH → Na 2 MnO 4 + 4H 2 O + 4NaBr + Mn 2 SO 4

5) MnSO 4 + CaOCl 2 + 2NaOH → MnO 2 + Na 2 SO 4 + CaCl 2 + H 2 O

Câu 3. a) Từ MnO 2 bằng phản ứng nào có thể thu được: MnCl 2 , KMnO 4 ,

Mn 2 O 7 ?

Hướng dẫn

b) Từ MnO 2 điều chế Ba(MnO 4 ) 2 . Viết các phương trình phản ứng.

a) Có thể cho MnO 2 tác dụng với HCl đặc thu được MnCl 2 :

MnO 2 + 4HCl → MnCl 2 + Cl 2 + 2H 2 O

Nung hỗn hợp MnO 2 + KClO 3 + KOH rắn phản ứng tạo ra K 2 MnO 4 , hòa tan,

lọc dung dịch nước lọc có K 2 MnO 4 . Axit hóa dung dịch K 2 MnO 4 thu được

KMnO 4 . Đun nóng dung dịch ở 80 0 C, sau đó làm nguội, tinh thể KMnO 4 xuất

hiện.

Muốn thu được Mn 2 O 7 , cho H 2 SO 4 đặc tác dụng với tinh thể KMnO 4 :

2KMnO 4 + H 2 SO 4 → 2HMnO 4 + K 2 SO 4

2HMnO 4 → Mn 2 O 7 + H 2 O

b) Nung hỗn hợp Ba(OH) 2 và MnO 2 trong không khí:

2Ba(OH) 2 + 2MnO 2 + O 2 → 2BaMnO 4 + 2H 2 O

3BaMnO 4 + 2H 2 O → Ba(MnO 4 ) 2 + MnO 2 + 2Ba(OH) 2

Ba(OH) 2 + CO 2 → BaCO 3 + H 2 O

lọc và rửa sản phẩm, Ba(MnO 4 ) 2 còn lại trong dung dịch.

Câu 4. a) Các ion MnO 4 2- và MnO 4 - bền trong môi trường nào? Giải thích

nguyên nhân.

b) Thêm từ từ từng giọt dung dich NaOH cho đến môi trường kiềm

vào một dung dịch KMnO 4 sau đó cho thêm từng giọt H 2 SO 4 loãng cho đến

môi trường axit. Hãy nêu các quá trình xẩy ra trong quá trình trên và giải thích

nguyên nhân.

Phát hành PDF bởi Ths Nguyễn Thanh Tú

Đăng ký Word doc qua Zalo 0905779594 Email thanhtuqn88@gmail.com


c) Viết phương trình phản ứng mô tả tính oxi hóa và tính khử của K 2 MnO 4 .

d) Có thể thu được H 2 MnO 4 bằng phương pháp cho H 2 SO 4 đặc tác dụng với

muối K 2 MnO 4 được không?

Hướng dẫn

a,b). Trong dung dịch có tồn tại cân bằng sau:

3MnO 4

2-

+ 2H 2 O ⇌ 2MnO 4 - + MnO 2 + 4OH -

từ cân bằng đó có thể thấy được ion MnO 2- tồn tại trong môi trường kiềm;

MnO 4 - bền trong môi trường axit.

c) Có thể bằng các phản ứng:

K 2 MnO 4 + 2H 2 S + 2H 2 SO 4 → 2S + MnSO 4 + K 2 SO 4 + 4H 2 O

2K 2 MnO 4 + Cl 2 →2KMnO 4 + 2KCl

4K 2 MnO 4 + O 2 + 2H 2 O → 4KMnO 4 + 4KOH

d) H 2 MnO 4 không bền nhanh chóng bị phân hủy:

K 2 MnO 4 + H 2 SO 4 → H 2 MnO 4 + K 2 SO 4

2H 2 MnO 4 → 2HMnO 4 + MnO 2 + 2H 2 O

Câu 5. a) Viết các phương trình phản ứng sau:

Hướng dẫn

1) KMnO 4 + MnCl 2 →

2) K 2 MnO 4 + Cl 2 →

3) KMnO 4 + KI + H 2 SO 4 →

4) KMnO 4 + KI + H 2 O →

5) KMnO 4 + FeSO 4 + H 2 SO 4 →

b) Viết các phương trình phản ứng sau đây dưới dạng phân tử:

1) Mn 2+ + ClO - + OH - →

2) MnO 4 - + NO 2 - + H + →

3) MnO - 4 + Fe + H + →

4) Mn 2+ -

+ BrO 3 + H 2 O →

5) MnO 4 - + H 2 O 2 + OH - →

a) 1) 2KMnO 4 + 3MnCl 2 + 2H 2 O → 5MnO 2 + 2KCl + 4HCl

Phát hành PDF bởi Ths Nguyễn Thanh Tú

Đăng ký Word doc qua Zalo 0905779594 Email thanhtuqn88@gmail.com


2) 2K 2 MnO 4 + Cl 2 → 2KMnO 4 + 2KCl

3) 2KMnO 4 + 10KI + 3H 2 SO 4 → 2MnSO 4 + 6K 2 SO 4 + 5I 2 + 8H 2 O

4) 2KMnO 4 + 6KI + 4H 2 O → 2MnO 2 + 3I 2 + 8KOH

b) 1) 2MnCl 2 + 4KClO + 8KOH → 2K 2 MnO 4 + 8KCl + 4H 2 O

với phương trình dạng ion:

2Mn 2+ + 4ClO - +8OH - → 2MnO 4

2-

+ 4Cl - + 4H 2 O

theo ví dụ trên, viết phương trình phân tử dựa vào các phương trình ion sau:

2) 2MnO 4 - + 5NO 2 - + 6H + → 2Mn 2+ + 5NO 3 - + 3H 2 O

3) 3MnO 4 - + 5Fe + 24H + → 3Mn 2+ + 5Fe 3+ + 12H 2 O

4) 5Mn 2+ + 2BrO 3 - + 4H 2 O → 5MnO 2 + Br 2 + 8H -

5) 2MnO 4 - + H 2 O 2 + 2OH - → 2MnO 4 2- + O 2 + 2H 2 O

Câu 6. a) Hãy giải thích sự hình thành liên kết trong phân tử hợp chất

Fe(CO) 5 .

Hướng dẫn

b) Phương pháp điều chế và ứng dụng của Fe(CO) 5

Phân tử được hình thành theo cơ chế "cho - nhận" nhờ các obital lai hóa dsp 3

của nguyên tử Fe.

3d 4s 4p

và các cặp electron của 5 phân tử CO

Fe(CO) 5 điều chế bằng cách nung bột sắt trong dòng khí CO ở 150 - 200 0 C

với áp suất khoảng 100at.

Fe + 5CO → Fe(CO) 5

Câu 7. a. Trong hai chất K 4 [Fe(CN) 6 ] và FeSO 4 chất nào có tính khử mạnh

hơn? Tại sao?

b. Viết phương trình phản ứng khi cho K 3 [Fe(CN) 6 ] tác dụng với H 2 O 2

trong môi trường KOH.

c. Viết phương trình phản ứng khi cho K 4 [Fe(CN) 6 ] tác dụng với H 2 O 2

Phát hành PDF bởi Ths Nguyễn Thanh Tú

Đăng ký Word doc qua Zalo 0905779594 Email thanhtuqn88@gmail.com


trong dung dịch HCl.

Hướng dẫn

a) Trong dung dịch nước ion Fe 2+ ở dạng ion phức [Fe(H 2 O) 6 ] 2+ có độ bền

kém hơn ion phức [Fe(CN) 6 ] 4- , nên [Fe(H 2 O) 6 ] 2+ có tính khử mạnh hơn.

b) 2K 3 [Fe(CN) 6 ] + H 2 O 2 +2KOH→2K 4 [Fe(CN) 6 ] + O 2 +2H 2 O

c) 2K 4 [Fe(CN) 6 ] + H 2 O 2 + 2HCl → 2K 3 [Fe(CN) 6 ] + 2KCl + 2H 2 O

Câu 8. Viết phương trình của các phản ứng sau:

Hướng dẫn

1). Fe(SO 4 ) 3 + Na 2 SO 3 + H 2 O →

2). FeSO 4 + HNO 3 + H 2 SO 4 →

3). FeSO 4 + HNO 3 →

4). FeSO 4 + KMnO 4 + H 2 SO 4 →

5). FeCl 3 + Na 2 CO 3 + H 2 O →

6) FeSO 4 + KBrO 3 + H 2 SO 4 →

7) K 4 [Fe(CN) 6 ] + KMnO 4 + H 2 SO 4 →

8) K 4 [Fe(CN) 6 ] + H 2 O 2 + H 2 SO 4 →

1). Fe 2 (SO 4 ) 3 + Na 2 SO 3 + H 2 O → 2FeSO 4 + Na 2 SO 4 + H 2 SO 4

2). 6FeSO 4 + 2HNO 3 + 3H 2 SO 4 → 3Fe 2 (SO 4 ) 3 + 2NO + 4H 2 O

3). 3FeSO 4 + 10HNO 3 → 3Fe(NO 3 ) 3 + NO + 3H 2 SO 3 + 2H 2 O

4). 10FeSO 4 +2KMnO 4 +8H 2 SO 4 → 5Fe 2 (SO 4 ) 2 + K 2 SO 4 + 2MnSO 4 +

8H 2 O

5) 2FeCl 3 + 3Na 2 CO 3 + 3H 2 O → 2Fe(OH) 3 + 6NaCl + 3CO 2

6) 6Fe 2+ + BrO - 3 + 6H + → 6Fe 3+ + Br - + 3H 2 O

7) 5[Fe(CN) 6 ] 4- + MnO 4 - + 8H + → 5[Fe(CN) 6 ] 3- + 4H 2 O + Mn 2+

8) 2[Fe(CN) 6 ] 4- + H 2 O 2 + 2H + → 2[Fe(CN) 6 ] 3- + 2H 2 O

Phát hành PDF bởi Ths Nguyễn Thanh Tú

Đăng ký Word doc qua Zalo 0905779594 Email thanhtuqn88@gmail.com


Câu 9. Cho sơ đồ sau:

Hướng dẫn

Câu 10. Cho dung dịch NaOH từ từ vào dung dịch CoCl 2 (hồng) thu được kết

tủa màu hồng. Tiếp tục cho dung dịch NH 3 đến dư vào thì kết tủa màu hồng ta

Phát hành PDF bởi Ths Nguyễn Thanh Tú

Đăng ký Word doc qua Zalo 0905779594 Email thanhtuqn88@gmail.com


và dung dịch thu được có màu vàng. Cho hỗn hợp axeton và amonithioxianua

và thu được dung dịch có màu xanh.

Mặt khác, cho dung dịch H 2 O 2 vào dung dịch màu vàng thì thu được dung

dịch có màu hồng thẫm. Thêm tiếp dung dịch H 2 SO 4 loãng vào thì thấy có khí

bay ra. Viết phương trình phản ứng xảy ra.

Hướng dẫn

Co 2+ + 2OH - Co(OH) 2 (hồng)

Co(OH) 2 (hồng) + 6NH 3 [Co(NH 3 ) 6 ] 2+ (màu vàng) + 2OH -

[Co(NH 3 ) 6 ] 2+ (màu vàng) + 4SCN - [Co(SCN) 4 ] 2+ + NH 3

Mặt khác, cho dung dịch H 2 O 2 vào dung dịch [Co(NH 3 ) 6 ] 2+ (màu vàng)

H 2 O 2 + 2[Co(NH 3 ) 6 ] 2+ (màu vàng) 2[Co(NH 3 ) 6 ] 3+ (hồng thẫm) + 2OH -

[Co(NH 3 ) 6 ] 3+ (hỗng thẫm) + H + Co 3+ + 6NH 4

+

4Co 3+ + 2H 2 O 4Co 2+ + O 2 + 4H +

2.2.2. Hệ thống các dạng bài tập

2.2.2.1. Xác định công thức của các chất

Câu 1. Khi phân tích nguyên tố các tinh thể ngậm nước của một muối tan A

của kim loại X, người ta thu được các số liệu sau:

Theo dõi sự thay đổi khối lượng của A khi nung nóng dần lên nhiệt độ cao,

người ta thấy rằng, trước khi bị phân hủy hoàn toàn, A đã mất 32% khối

lượng.

Trong dung dịch nước, A phản ứng được với hỗn hợp gồm PbO 2 và HNO 3

(nóng), với dung dịch BaCl 2 tạo thành kết tủa trắng không tan trong HCl.

Hãy xác định kim loại X, muối A và viết các phương trình phản ứng xảy ra.

Biết X không thuộc họ Lantan và không phóng xạ.

Hướng dẫn.

Biện luận được X là Mn; A là MnSO 4 .4H 2 O

Viết phương trình phản ứng

2MnSO 4 . + 5PbO 2 + 6HNO 3 2HMnO 4 + 3Pb(NO 3 ) 2 + 2PbSO 4 +

2H 2 O

MnSO 4 +BaCl 2 BaSO 4 + MnCl 2

Phát hành PDF bởi Ths Nguyễn Thanh Tú

Đăng ký Word doc qua Zalo 0905779594 Email thanhtuqn88@gmail.com


Câu 2. Hoà tan sản phẩm rắn của quá trình nấu chảy hỗn hợp gồm bột của

một khoáng vật màu đen, kali hiđroxit và kali clorat, thu được dung dịch có

màu lục đậm. Khi để trong không khí, màu lục của dung dịch chuyển dần

thành màu tím. Quá trình chuyển đó còn xảy ra nhanh hơn nếu sục khí clo vào

dung dịch hay khi điện phân dung dịch.

Hãy cho biết khoáng vật màu đen là chất gì.

Viết phương trình của tất cả các phản ứng xảy ra trong quá trình thí nghiệm.

Hướng dẫn:

1. Khoáng vật màu đen là MnO 2 .

2. Dung dịch màu lục đậm chuyển dần thành màu tím khi để trong không khí

chỉ có thể là dung dịch MnO 4 2- vậy phản ứng xảy ra khi nấu chảy hỗn hợp là

3MnO 2 + 6KOH + 6KClO 3 3K 2 MnO 4 + 3H 2 O + KCl (1)

3K 2 MnO 4 + 2H 2 O 2KMnO 4 + MnO 2 + 4KOH (2)

2KOH + CO 2 K 2 CO 3 (3)

Phản ứng này làm cân bằng (2) chuyển dịch dần sang phải

2K 2 MnO 4 + Cl 2 2KMnO 4 + 2KCl

â

2K 2 MnO 4 + 2H 2 O đ ⎯⎯⎯⎯⎯ ệ

2KMnO 4 + 2KOH + H 2

Câu 3. Cho 6,000 g mẫu chất chứa Fe 3 O 4 , Fe 2 O 3 và các tạp chất trơ. Hòa tan

mẫu vào lượng dư dung dịch KI trong môi trường axit (khử tất cả sắt thành

Fe 2+ ) tạo ra dung dịch A. Pha loãng dung dịch A đến thể tích 50 mL. Lượng I 2

có trong 10 mL dung dịch A phản ứng vừa đủ với 5,500 mL dung dịch

Na 2 S 2 O 3 1,00M (sinh ra S 4 O 2- 6 ). Lấy 25 mL mẫu dung dịch A khác, chiết tách

I 2 , lượng Fe 2+ trong dung dịch còn lại phản ứng vừa đủ với 3,20 mL dung dịch

MnO - 4 1,000M trong H 2 SO 4 .

1. Viết các phương trình phản ứng xảy ra (dạng phương trình ion thu gọn).

2. Tính phần trăm khối lượng Fe 3 O 4 và Fe 2 O 3 trong mẫu ban đầu.

Hướng dẫn.

1. Phương trình phản ứng:

Fe 3 O 4 + 2I - + 8H + 3Fe 2+ + I 2 + 4H 2 O (1)

Fe 2 O 3 + 2I - + 6H + 2Fe 2+ + I 2 + 3H 2 O (2)

2S 2 O 3 2- + I 2 S 4 O 6 2- + 2I - (3)

Phát hành PDF bởi Ths Nguyễn Thanh Tú

Đăng ký Word doc qua Zalo 0905779594 Email thanhtuqn88@gmail.com


5Fe 2+ + MnO 4 - + 8H + 5Fe 3+ + Mn 2+ + 4H 2 O (4)

2. số mol Fe 3 O 4 = 0,0045; số mol Fe 2 O 3 = 0,00925

% khối lượng của Fe 3 O 4 = 17,4%

% khối lượng của Fe 2 O 3 = 24,7%

Câu 4. Một hỗn hợp của 2 g bột rất mịn A trong 50 ml dung dịch NaOH 28%

được nghiền nhỏ trong một bình Erlenmeyer với 3,5 g bột Na 2 SO 3 .7H 2 O; bình

dược nhúng trong một chậu nước đá. Nghiền khoảng 10 phút cho đến khi thu

được một tinh thể màu xanh. Hỗn hợp này sau đó được vận chuyển trong

chân không vào một kính lọc được làm mát bằng đá, và các sản phẩm được

làm sạch kỹ bằng 28% sodium hydroxide tại 0 0 C. Hỗn hợp sau đó được nhanh

chóng trải rộng thành một lớp mỏng trên đất sét và bảo quản ở 0 0 C trong một

bình hút ẩm chân không (không có chất làm khô) ... Các quá trình chuẩn bị

phải được thiết kế để tránh ô nhiễm bởi silicat hoặc aluminat ... Sản phẩm B

tồn tại dưới dạng thanh tinh thể màu xanh da trời, vẫn ổn định ở mức 0 0 C nếu

giữ tránh khỏi H 2 O và CO 2 ... Một hỗn hợp của B trong KOH 50% chuyển

sang màu xanh cỏ khi đc nung nóng hoặc pha loãng; đồng thời, C được kết

tủa.

Ở dạng tinh khiết, muối D, thành phần chính của B, được chuẩn bị theo các

quá trình sau đây: “NaOH được làm mất nước hoàn toàn bằng cách đun nóng

trong nồi bạc tại 400ºС và trộn với C với tỉ lệ Na: tỷ lệ X mol là 3: 1. Hỗn hợp

được làm nóng tới 800ºС trong một nồi bạc và giữ trong oxy trong 5 giờ. Sản

phẩm D được hình thành và được nhanh chóng làm nguội đến nhiệt độ

phòng”. Muối D là một hợp chất xanh lá cây đậm và trơ với CO 2 .

Một hỗn hợp 30 g KOH trong 50 ml nước được chuẩn bị; 10 g А được thêm

vào và hỗn hợp này được đun sôi trong bình 250 ml Erlenmeyer cho đến khi

thu được một dung dịch xanh nguyên chất. Nước bị mất do bay hơi được thay

thế và bình được đặt trong đá. Các tinh thể màu đen-xanh kết tủa, phát ra ánh

sáng tím, được thu thập trên một kính lọc Pyrex, rửa sạch (hút cao) với KOH

1M, và được làm khô bằng P 2 O 5 . Hợp chất được hình thành E có thể được kết

tinh bằng cách hòa tan trong KOH loãng và bốc hơi trong chân không.

Xác định nguyên tố Х và công thức phân tử của A-Е sử dụng các dữ liệu sau

đây:

- hàm lượng natri trong В là 18,1%;

- hàm lượng của các nguyên tố Х trong А, В, С, D, và Е là 34,8, 13,3, 63,2,

Phát hành PDF bởi Ths Nguyễn Thanh Tú

Đăng ký Word doc qua Zalo 0905779594 Email thanhtuqn88@gmail.com


29,3, và 27,9% tương ứng.

Viết tất cả các phương trình hóa học

Hướng dẫn

1. Muối khan D là thành phần chủ yếu của hợp chất B. Chúng ta có thể cho

rằng B là một hydrat của D. Tỉ lệ mol Na : X trong D là 3: 1. D không phải là

một hợp chất nhị tố Na 3 X vì như vậy М Х = (29,3 · 69 / 70,7) = 28,6. Không

có nguyên tố nào như vậy. Vì vậy, D có chứa một số nguyên tố khác. Oxi có

khả năng cao nhất, ví dụ, D là Na 3 XO n (muối D không thể có công thức

Na 3 Н m XO n vì tất cả các chất dễ bay hơi đều bị loại bỏ dưới điều kiện đã thực

hiện để tổng hợp D (nung nóng tại 800ºС)). Hàm lượng Х có nhiều trong С

cho phép chúng ta giả sử rằng C là một hợp chất nhị tố, tức là, nó là một oxit

của Х. Bây giờ chúng ta có thể xác định Х.

Oxit X 2 O XO X 2 O 3 XO 2 X 2 O 5 XO 3 X 2 O 7 XO 4

M X 13,74 27,48 41,22 54,96 68,70 82,43 96,17 109,91

Do đó, Х là Mn và С là MnO 2 . Từ thành phần của Mn trong D chúng ta suy ra

được công thức của nó, Na 3 MnO 4 . Các trạng thái oxi hóa mangan trong hợp

chất này là +5. Qua nung nóng hoặc làm mát, dung dịch kiềm của D phân

hủy, cho ra MnO 2 rắn và dung dịch màu xanh lá cây. Các dẫn xuất của

Mangan thường tím nhưng không phải màu xanh lá cây. Do đó, dung dịch có

chứa muối mangan (VI). Dung dịch xanh tương tự được hình thành ở bước

cuối cùng. Chúng ta có thể kết luận rằng bước này dẫn đến manganate,

K 2 MnO 4 . Thật vậy, thành phần của Mn trong K 2 MnO 4 (hợp chất E) là 27,9%.

Hợp chất В (một dẫn xuất của Mn (V)) thu được bằng phản ứng của А với

natri sulfit (một chất khử thông dụng). Nung nóng dung dịch kiềm của A thu

được K 2 MnO 4 . Nó chỉ có thể xảy ra khi A là 1 dẫn xuất của Mn (VII). Thật

vậy, hàm lượng Mn trong A tương ứng với công thức KMnO 4 . Các hợp chất

không rõ còn lại là B. Ở trên chúng ta cho rằng B là một hydrat của D. Các

tính toán sử dụng công thức của Na 3 MnO 4 · nH 2 O dẫn đến М В = 413,5. Nó

tương ứng với n = 12,5. Tuy nhiên, М В = 381,2 nếu tính từ hàm lượng Na.

Nói cách khác, tỷ lệ Na: Mn trong В không phải là 3: 1 mà là 3,25: 1.

Natri bổ sung này xuất hiện do sự hiện diện của một số hợp chất Na khác ở

dạng solvat. Để xác định hợp chất này, việc phân tích các quá trình tổng hợp

là cần thiết. Trong quá trình tổng hợp B hòa tan được làm sạch bằng dung

Phát hành PDF bởi Ths Nguyễn Thanh Tú

Đăng ký Word doc qua Zalo 0905779594 Email thanhtuqn88@gmail.com


dịch NaOH. Vì vậy, công thức có thể có của B là Na 3 MnO 4 .0,25NaOH.nH 2 O.

Từ hàm lượng Na và Mn, chúng ta kết luận rằng n = 12. Cuối cùng, В là

[4Na 3 MnO 4 .NaOH.48H 2 O].

2. Các phản ứng xảy ra là:

4KMnO 4 +4Na 2 SO 3 .7H 2 O+13NaOH+ 16H 2 O →

[4Na 3 MnO 4 .NaOH.48H 2 O]↓+ 4Na 2 SO 4 + 4KOH

2Na 3 MnO 4 + 2H 2 O → Na 2 MnO 4 + MnO 2 + 4NaOH

12NaOH + 4MnO 2 + O 2 → 4Na 3 MnO 4 + 6H 2 O

4KMnO 4 + 4KOH → 4K 2 MnO 4 + O 2 + 2H 2 O

Câu 5. Một chất rắn màu trắng X tham gia một loạt các thí nghiệm trong đó X

bị đốt thành tro dưới tác dụng của các luồng khí vào khác nhau. Kết qủa thí

nghiệm được thống kê ở bảng sau:

Thí nghiệm số Khí vào Sự chênh lệch khối lượng mẫu

so với ban đầu

1 N 2 -37,9

2 NH 3 -51,7

3 O 2 -31,0

4 HCl +9,5

5 HCl + Cl 2 -100,0

Trong tất cả các thí nghiệm thì trong hỗn hợp sau phản ứng ngoài khí ban đầu

còn có một khí chưa biết Y. Ở thí nghiệm số 5 xuất hiện một hợp chất màu đỏ

nâu Z ngưng tụ khi tiến hành bước làm lạnh trong thí nghiệm.

a. Sử dụng các giá trị cho ở bảng trên hãy xác định các chất được ký hiệu

bằng chữ cái.

b. Viết các phản ứng xảy ra trong thí nghiệm.

c. Cho biết cấu trúc của Z trong pha khí.

Hướng dẫn

Phát hành PDF bởi Ths Nguyễn Thanh Tú

Đăng ký Word doc qua Zalo 0905779594 Email thanhtuqn88@gmail.com


X là FeCO 3 Y là CO 2

Z là FeCl 3

Các phản ứng sau đây đã xảy ra

FeCO 3 → FeO + CO 2

3FeCO 3 + 2NH 3 → 3Fe + 3CO 2 + 3H 2 O

4FeCO 3 → 2Fe 2 O 3 + 4CO 2

FeCO 3 + 2HCl → FeCl 2 + CO 2 + H 2 O

2FeCO 3 + 4HCl + Cl 2 → 2FeCl 3 + 2CO 2 + 2H 2 O

Ở pha hơi thì sắt (III) clorua tồn tại ở dạng dime (FeCl 3 ) 2

Câu 6. Có thể điều chế tinh thể FeCl 3 .6H 2 O theo cách sau: Hoà tan sắt kim

loại vào trong dung dịch axit clohydric 25%. Dung dịch tạo thành được oxy

hóa bằng cách sục khí clo qua cho đến khi cho kết qủa âm tính với

K 3 [Fe(CN) 6 ]. Dung dịch được cô bay hơi ở 95 o C cho đến khi tỉ trọng của nó

đạt chính xác 1,695 g/cm 3 và sau đó làm lạnh đến 4 o C. Tách kết tủa thu được

bằng cách hút chân không rồi cho vào một dụng cụ chứa được niêm kín.

a. Viết các phản ứng dẫn đến sự kết tủa FeCl 3 .6H 2 O

b. Có bao nhiêu gam sắt và bao nhiêu mL dung dịch axit clohydric 36%

(d=1,18g/cm 3 ) cần để điều chế 1,00kg tinh thể này. Biết rằng hiệu suất quá

trình chỉ đạt 65%

c. Đun nóng 2,752g FeCl 3 .6H 2 O trong không khí đến 350 o C thu được 0,8977g

bã rắn. Xác định thành phần định tính và định lượng của bã rắn.

Hướng dẫn

a. Các phản ứng:

Fe + 2HCl FeCl 2 + H 2

2FeCl 2 + Cl 2 2FeCl 3

Phát hành PDF bởi Ths Nguyễn Thanh Tú

Đăng ký Word doc qua Zalo 0905779594 Email thanhtuqn88@gmail.com


3FeCl 2 + 2K 3 [Fe(CN) 6 ] Fe 3 [Fe(CN) 6 ] 2 + 6KCl

FeCl 3 + 6H 2 O FeCl 3 .6H 2 O

b. Số mol FeCl 3 .6H 2 O = 1000/270,3 = 3,7 mol

Thể tích HCl 3% cần dùng là:

, . . ,

, . . . ,

978ml

Khi đun nóng thì FeCl 3 .6H 2 O phân huỷ theo phương trình sau:

FeCl 3 .6H 2 O FeOCl + 5H 2 O + 6HCl

Khi nhiệt độ tăng thì FeOCl sẽ tiếp tục phân huỷ:

3FeOCl FeCl 3 + Fe 2 O 3 (Hơi FeCl 3 bay ra)

Lượng FeCl 3 .6H 2 O trong mẫu là: 2,752/270,3 =10,18 mmol

Điều này ứng với khối lượng FeCl 3 là 107,3. 0,01018 = 1,092g FeOCl

Do khối lượng thu được của bã rắn bé hơn nên ta biết được FeOCl sẽ bị phân

hủy một phần thành Fe 2 O 3 .

Lượng FeCl 3 mất mát do bay hơi là: , ,

,

= 1,20 mmol

Như vậy bã rắn cuối cùng chứa (0,01018 – 3.0,00120) = 6,58 mmol FeOCl và

1,20 mmol Fe 2 O 3 .

2.2.2.2. Bài tập về phức chất

Câu 1. Coban tạo ra được các ion phức: [CoCl 2 (NH 3 ) 4 ] +

1. Viết tên của (A), (B), (C).

[Co(CN) 6 ] 3-

[CoCl 3 (CN) 3 ] 3-

2. Theo thuyết liên kết hoá trị, các nguyên tử trong B ở trạng thái lai hoá nào?

3. Các ion phức trên có thể có bao nhiêu đồng phân lập thể? Vẽ cấu trúc của

chúng.

Hướng dẫn

1. Tên của các ion phức:

(A) Điclorotetraammincoban (III);

(B) Hexaxianocobantat (III);

(C) Triclorotrixianocobantat (III).

(A)

(B)

(C)

Phát hành PDF bởi Ths Nguyễn Thanh Tú

Đăng ký Word doc qua Zalo 0905779594 Email thanhtuqn88@gmail.com


2. [Co(CN) 6 ] 3- . Co : d 2 sp 3 ; C : sp ;

Câu 2. (Đề thi HSGQG – 2019) Trong một số trường hợp, theo quy tắc 18

electron của Sidgwich, một phức chất sẽ được dự đoán là bền nếu các nguyên

tử hoặc ion kim loại chuyển tiếp trung tâm đạt được cấu hình 18 electron (n-

1)d 10 ns 2 np 6 như của khí hiếm.

Dựa vào quy tắc này:

a. Giải thích tại sao Fe(0) có thể tạo phức chất đơn nhân bền X với CO,

trong khi không quan sát thấy điều tương tự ở Co(0)? Cho X phản ứng với I 2

theo tỉ lệ mol 1:1 thu được phức chất đơn nhân Y và khí Z. Xác định công

thức hóa học của Y và Z.

b. Giải thích tại sao Co(0) lại tạo được phức chất hai nhân bền với CO có

công thức là [Co 2 (CO) 8 ]. Mô tả sự tạo thành liên kết và vẽ cấu trúc hình học

của phức chất này.

Cho biết: Z C = 6; Z Fe = 26; Z Co = 27.

Phát hành PDF bởi Ths Nguyễn Thanh Tú

Đăng ký Word doc qua Zalo 0905779594 Email thanhtuqn88@gmail.com


Hướng dẫn

Nguyên tử Fe có 8 electron hoá trị nằm thuộc các phân lớp 3d 4s, do vậy nó

có thể kết hợp với n phối tử CO để tạo phức chất đơn nhân [Fe(CO) n ]. CO là

phối tử cho 2 electron. Như vậy, tổng số electron hoá trị và electron liên kết

của phức chất là 18 electron → 8+ 2n =18 → n = 5. Vậy X là [Fe(CO) 5 ].

(0,5 điểm)

Nguyên tử Co có số lẻ electron hoá trị (9 electron) thuộc các phân lớp 3d 4s,

do vậy khi kết hợp với n phối tử CO để tạo phức chất đơn nhân thì tổng số

electron hoá trị và electron liên kết của phức chất là số lẻ (9+2.n), không đạt

được cấu hình bền của khí hiếm. Do đó, Co(0) không tạo được phức chất

cacbonyl đơn nhân bền. (0,25 điểm)

Y là phức chất đơn nhân, thỏa mãn quy tắc 18 electron → Y là [Fe(CO) 4 (I) 2 ]

(0,25 điểm)

Z là CO.

(0,25 điểm)

(với phản ứng là [Fe(CO) 5 ] + I 2 → [Fe(CO) 4 (I) 2 ] + CO)

b. (0,75 điểm)

- Trong phức chất [Co 2 (CO) 8 ], mỗi nguyên tử Co có 9 electron hoá trị, 8

electron từ 4 phối tử CO và 1 electron từ liên kết Co-Co. Vậy mỗi nguyên tử

Co có 18 electron nên phức chất [Co 2 (CO) 8 ] bền.

(0,25 điểm)

Mô tả sự tạo thành liên kết và vẽ cấu trúc của phức chất [Co 2 (CO) 8 ]:

- Trong phân tử [Co 2 (CO) 8 ], mỗi nguyên tử Co tạo nên 6 liên kết: 4 liên kết

σ-cho-nhận tạo nên từ cặp electron trên MO σ liên kết của CO, một liên kết σ

cho-nhận tạo nên từ cặp electron d của Co với MO π* trống của CO và một

liên kết σ tạo nên giữa hai nguyên tử Co. Như vậy, hai liên kết σ của mỗi cầu

CO ở đây được coi là hai liên kết cho nhận ngược nhau: một từ CO và một từ

kim loại. Liên kết giữa Co với các phân tử CO còn được làm bền nhờ liên kết

π cho. (0,25 điểm)

- Vẽ cấu trúc của phức chất [Co 2 (CO) 8 ]:

Phát hành PDF bởi Ths Nguyễn Thanh Tú

Đăng ký Word doc qua Zalo 0905779594 Email thanhtuqn88@gmail.com


(Chỉ cần vẽ được một trong 3 dạng trên)

Nếu học sinh giải thích theo lai hóa của Co dạng d 2 sp 3 thì cũng cho đủ điểm.

Phát hành PDF bởi Ths Nguyễn Thanh Tú

Đăng ký Word doc qua Zalo 0905779594 Email thanhtuqn88@gmail.com


2.2.2.3. Bài tập về tinh thể

Câu 1. (Đề thi HSGQG -2019)

Một ô mạng lập phương của oxit sắt M có hằng số mạng là 4,30 Å được mô tả

ở hình bên.

a. Xác định công thức hoá học của M. Tính khối lượng riêng (g.cm –3 ) của M

và bán kính của ion sắt (Å). Cho biết bán kính của ion O 2– là 1,40 Å.

b. Trong quá trình tổng hợp M, thu được oxit sắt N có cùng kiểu ô mạng và

hằng số mạng như của M, nhưng thiếu hụt một số lượng ion sắt. Khối lượng

riêng của N đo được là 5,70 g.cm –3 . Xác định công thức hoá học của N. Đề

xuất quy trình hoá học đơn giản để phân biệt (định tính) M và N.

Hướng dẫn

Phát hành PDF bởi Ths Nguyễn Thanh Tú

Đăng ký Word doc qua Zalo 0905779594 Email thanhtuqn88@gmail.com


+ Dùng dung dịch SCN − để nhận ra ion Fe 3+ được tạo ra từ oxit N bằng hiện

tượng có dung dịch màu đỏ máu xuất hiện.

Fe 3+ + 3SCN − →[Fe(SCN) 3 ]

2.2.3. Một số bài thực hành

(0,25 điểm)

Bài thực hành số 1. Xác định hàm lượng Fe trong muối Fe (III) (hoặc trong dd

Fe 3+ pha sẵn).

(Phương pháp chuẩn độ phức chất – Kỹ thuật chuẩn độ trực tiếp)

1- Nguyên tắc xác định:

Dùng dung dịch EDTA tiêu chuẩn để chuẩn độ trực tiếp xuống dung dịch

mẫu chứa ion Fe 3+ ở môi trương pH = 2 3, dung dịch nóng. Nhận biết điểm

tương đương bằng chỉ thị H 2 SSal. Tại điểm tương đương dung dịch đổi màu

từ đỏ tím sang vàng chanh.

Phương trình: Fe 3+ + H 2 Y 2- = FeY - + 2H +

Kết quả được tính theo công thức:

mDg

Fe

( NV )

EDTA

Vdm

% Fe = 100

G V

xd

55.85

; mĐg

Fe

= 0.02793g

3

210

Vì 1H 2 Y 2- ~ 1Fe 3+ ~ 2H +

Nếu xác định Fe từ dung dịch Fe 3+ pha sẵn ta tính g/l:

mDg

Fe

( NV )

EDTA

g/l

Fe

1000

V

( ml )

Phát hành PDF bởi Ths Nguyễn Thanh Tú

Đăng ký Word doc qua Zalo 0905779594 Email thanhtuqn88@gmail.com


2- Điều kiện xác định

- Nếu trong mẫu chứa Fe 2+ phải oxy hóa hoàn toàn lên Fe 3+ bằng HNO 3 và

đun sôi 1 2 phút.

- Thực hiện môi trường pH = 2 3 để tránh Fe 3+ bị thủy phân dưới dạng

Fe(OH) 3 , ở môi trường này phản ứng thực hiện hoàn toàn và chỉ thị H 2 SSal có

màu đặc trưng đỏ tím sẽ đổi màu rõ rệt nhất.

- Kỹ thuật điều chỉnh môi trường: có 2 cách

Cách 1: Dựa vào sự có mặt của ion Fe 3+ trong dung dịch, dùng dung dịch

NH 3 làm kết tủa Fe(OH) 3 lúc này pH khoảng 3 4 (lúc xuất hiện vẩn nâu

không tan). Rồi nhỏ từ từ dung dịch HCl 1/1 phá tan vẩn nâu (pH = 3) sau đó

cho dư khoảng 1ml HCl nữa ta được pH=2 3.

Cách 2: Dựa vào màu của phức sắt salisilat ở các môi trường khác nhau thì

có màu khác nhau: ở pH = 2 3 có màu đỏ tím.

pH = 5 6 có màu đỏ nâu.

pH = 8 10 có màu vàng chanh

Theo cách này cho 7 8 giọt chỉ thị H 2 SSal vào dung dịch xác định rồi

dùng dung dịch NH 3 1/1 và HCl 1/1 (nhỏ từ từ) để điều chỉnh môi trường đến

khi dung dịch có màu đỏ tím là được pH = 2 3.

3- Quy trình xác định

Xác định Fe từ dung dịch Fe 3+ pha sẵn

EDTA t/c

Chuẩn độ từ màu đỏ tím sang màu vàng chanh (ghi V EDTA tiêu tốn)

Đun nóng dung dịch 60 70 o C

Nhỏ từ từ dung dịch NH 3 1/1 đến màu đỏ tím

+ 7 8 giọt chỉ thị H 2 SSal

10.00ml dung dịch Fe 3+ + nước cất tới ~ 2V

Làm thí nghiệm song song lấy kết quả trung bình, sai lệch giữa hai lần

chuẩn độ không quá 0.1ml dung dịch tiêu chuẩn.

Bài thực hành số 2. Xác định Fe trong muối FeSO 4 .7H 2 O (phương pháp

khối lượng)

Phát hành PDF bởi Ths Nguyễn Thanh Tú

Đăng ký Word doc qua Zalo 0905779594 Email thanhtuqn88@gmail.com


1- Nguyên tắc:

- Hòa tan lượng mẫu muối Fe (II) vào dung dịch bằng nước cất. Đem oxi

hóa hoàn toàn Fe 2+ Fe 3+ bằng HNO 3 , dung dịch nóng.

3Fe 2+ + HNO 3 +3H + = 3Fe 3+ + 2H 2 O + NO

- Kết tủa hoàn toàn Fe 3+ dưới dạng kết tủa vô định hình Fe(OH) 3 bằng

thuốc thử NH 3 dung dịch nóng.

Fe 3+ + NH 3 + H 2 O = Fe(OH) 3 + NH 4

+

- Lọc, rửa sạch kết tủa, sấy và nung ở nhiệt độ 850 o C khoảng 1giờ.

850

2Fe(OH) 3 0 C Fe 2 O 3 + 3H 2 O

1

a

%Fe = f 100 G

2M

Fe

56

2

f = 0. 69942 0.7

M 160

Fe2O3

a: khối lượng dạng cân

G: số gam mẫu cân

f: hệ số chuyển đổi

2- Điều kiện xác định

- Khi hòa tan mẫu cần tẩm trước bằng H 2 SO 4 đặc để tránh sự thủy phân.

- Làm kết tủa: kết tủa Fe(OH) 3 bằng NH 3 đặc 10%, pH=3 4. Đây là kết

tủa vô định hình nên phải tuân thủ đầy đủ các điều kiện sau: dung dịch nóng,

thuốc thử đặc, dung dịch đặc, thuốc thử cho nhanh, khuấy nhẹ, đun nóng già

dung dịch để kết tủa vón chắc rễ lọc rửa.

- lọc, rửa kết tủa: lọc kết tủa trong điều kiện dung dịch nóng (phương pháp

lắng gạn). Rửa kết tủa vài lần đầu bằng NH 4 NO 3 1% là chất điện ly mạnh (để

tránh hiện tượng keo hóa kết tủa, khó lọc rửa)

- Nung kết tủa: nung kết tủa 850 o C được dạng cân Fe 2 O 3 tránh nung ở

nhiệt độ cao hơn sẽ sinh ra Fe 3 O 4 gây sai số

6Fe 2 O 3

o C

3- Quy trình xác định

900

4Fe 3 O 4 + O 2

Phát hành PDF bởi Ths Nguyễn Thanh Tú

Đăng ký Word doc qua Zalo 0905779594 Email thanhtuqn88@gmail.com


Cân chính xác 0.2000g mẫu FeSO 4 .7H 2 O. Tẩm ướt mẫu bằng vài ml

H 2 SO 4 đặc, dằm nhẹ mẫu, dùng nước cất để hòa tan mẫu rồi chuyển vào cốc

thủy tinh chịu nhiệt 250ml, thêm nước cất tới thể tích khoảng 100ml.

- Làm kết tủa Fe(OH) 3 : Khuấy nhẹ, đun nóng già dung dịch kết tủa vón

chắc

+ NH 3 10% dd hóa vàng (pH=3 4)+1 2ml NH 3 nữa

+ 1 giọt MO 0.1%

t

+ 2 3ml HNO 3 1/1

0

sôi 3 phút, để nguội bớt

100ml dung dịch mẫu vừa hòa tan

- Lọc kết tủa qua giấy lọc băng vàng(hoặc băng đỏ, trước khi lọc cần phải

tẩm ướt giấy lọc bằng nước cất sôi).

- Rửa kết tủa vài lần đầu bằng NH 4 NO 3 1% (đã được kiềm hóa bằng NH 3 ).

- Sau rửa bằng nước cất nóng tới sạch ion Cl - (thử bằng AgNO 3 ).

- Dùng giấy lọc ướt để lau sạch đũa và cốc thủy tinh rồi chuyển vào phễu

lọc .

- Chuyển giấy lọc có chứa kết tủa vào trong chén nung đã biết trước khối

lượng.

- Đem hóa tro giấy lọc trước cửa lò nung.

- Nung kết tủa ở 850 o C trong một giờ.

- Lấy ra để nguội trong bình hút ẩm đến nhiệt độ phòng và đem cân.

- Tính kết quả.

Bài thực hành số 3. Quy trình phân tích crom trong mẫu thép không gỉ chứa

Fe, Cr và Mn được tiến hành như sau:

Pha dung dịch chuẩn FeSO 4 :

Hòa tan 11,0252 gam muối Mohr ( FeSO 4 .(NH 4 ) 2 SO 4 .6H 2 O) vào bình định

mức 250ml có H 2 SO 4 và định mức đến vạch bằng nước cất. Tính nồng độ mol

của dung dịch FeSO 4 thu được?

Chuẩn hóa dung dịch KMnO 4 :

Lấy 25,0 ml dung dịch FeSO 4 vừa pha chế ở trên cho vào bình nón, thêm 1 ml

dung dịch H 3 PO 4 đặc( để tạo phức không màu với Fe 3+ ), chuẩn độ dung dịch

Phát hành PDF bởi Ths Nguyễn Thanh Tú

Đăng ký Word doc qua Zalo 0905779594 Email thanhtuqn88@gmail.com


thu được bằng dung dịch KMnO 4 thấy vừa hết 24,64 ml. Tính nồng độ mol

của dung dịch KMnO 4 .

Chuẩn bị mẫu: Hòa tan 0,2800 gam mẫu thép trong dung dịch hỗn hợp H 3 PO 4

và H 2 SO 4 đặc, đun nóng cho đến khi thu được dung dịch trong suốt màu xanh

(khi đó, Fe Fe 3+ ; Cr Cr 3+ ; Mn Mn 2+ ). Làm lạnh dung dịch đến nhiệt

độ phòng. Thêm 5 ml dung dịch AgNO 3 1%, 20 ml dung dịch (NH 4 ) 2 S 2 O 8

20%. Sau vài phút dung dịch có màu hồng( pesunfat oxi hóa Mn 2+ MnO 4 - ;

Cr 3+ Cr 2 O 7

2-

; Ag + đóng vai trò làm xúc tác). Đun sôi dung dịch để phân hủy

hết ion pesunfat còn dư ( sinh ra SO 4 2- và O 2 ). Thêm từ từ từng giọt HCl đặc

đến khi dung dịch chuyển từ màu hồng sang màu vàng( HCl phản ứng chọn

lọc với MnO 4 - ) Sau khi kết thúc phản ứng thấy có kết tủa trắng ở đáy bình.

3. Viết phương trình phản ứng của Cr 3+ , Mn 2+ với S 2 O 8

2-

trong môi trường

axit.

4. Viết phương trình phản ứng loại MnO 4

-

bằng dung dịch HCl đặc.

5. Hãy cho biết kết tủa trắng là chất gì? Được tạo thành như thế nào?

Tiến hành chuẩn độ: Chuyển dung dịch thu được ở trên vào bình định mức

250 ml rồi định mức đến vạch bằng nước cất được dung dịch A. Lấy 50 ml

dung dịch A cho vào bình nón, thêm tiếp 25,0 ml dung dịch FeSO 4 ở trên.

Lượng FeSO 4 dư được chuẩn độ bằng dung dịch KMnO 4 đã được chuẩn hóa ở

trên thấy vừa hết 19,89 ml.

6. Viết phương trình phản ứng của Fe 2+ 2-

với Cr 2 O 7

7. Tính thành phần phần trăm của Cr trong mẫu thép?

Hướng dẫn

1. Số mol của FeSO 4 = 11,0252 / 392.

Nồng độ mol của FeSO 4 là 0,1125M

2. Phương trình phản ứng: 5Fe 2+ + MnO 4 + 8H + 5Fe 3+ + Mn 2+ + 4H 2 O

Nồng độ mol của KMnO 4 là (0,1125.25)/(5.24,64) = 0,02283M

3. Phương trình các phản ứng là:

2Cr 3+ + 3S 2 O 8

2-

2Mn 2+ + 5S 2 O 8

2-

4. 10Cl - + 2MnO 4

-

+ 7H 2 O Cr 2 O 7

2-

+ 6SO 4

2-

+ 14H +

+ 8H 2 O 2MnO 4

-

+ 10SO 4

2-

+ 16H + 2Mn 2+ + 5Cl 2 + 8H 2 O

+ 16H +

5. Kết tủa trắng thu được là AgCl, được tạo thành là do khi khử hết MnO 4 - ,

Phát hành PDF bởi Ths Nguyễn Thanh Tú

Đăng ký Word doc qua Zalo 0905779594 Email thanhtuqn88@gmail.com


lượng HCl dư sẽ phản ứng với Ag + .

6. 6Fe 2+ + Cr 2 O 7

2-

+ 14H + 6Fe 3+ + 2Cr 3+ + 7H 2 O

7. Số mol Cr 2 O 7 2- (A) = 5.(0,025.0,1125 – 5.0,01989. 0,02283)/ 6

Thành phần % khối lượng của Cr trong mẫu thép là 16,78%

Bài thực hành số 4: ( Trích đề chuẩn bị thi Olympic quốc tế lần thứ 30)

Câu 1: Tổng hợp [Co(NH 3 ) 5 Cl]Cl 2 và các đồng phân [Co(NH 3 ) 5 ONO]Cl 2 và

[Co(NH 3 ) 5 NO 2 ]Cl 2 .

a). Điều chế [Co(NH 3 ) 5 Cl]Cl 2

Hòa tan 10.0 g amoni clorua trong 60 mL nước amoniac đặc (CẨN

THẬN!) trong một erlenmeyer 500 mL. Trong khi khuấy liên tục dung dịch

với máy khuấy từ, thêm vào từng lượng nhỏ 20 g bột coban clorua hidrat

(ngậm 6 phân tử nước) đã tán nhuyễn (COI CHỪNG: tránh để bột phơi ra

ngoài. Nghiền cẩn thận trong tủ hút).

Vẫn liên tục khuấy đều chất bột nhão màu nâu, thêm từ từ 16 mL hidro

peoxit 30 % (CẨN THẬN!) từ phễu nhỏ giọt. Khi kết thúc sủi bọt, thêm từ từ

60 mL HCl đậm đặc.

Tiếp tục khuấy trên bếp đun, giữ nhiệt độ khoảng 85oC trong 20 phút;

rồi hạ xuống nhiệt độ phòng và lọc lấy kết tủa [Co(NH 3 ) 5 Cl]Cl 2 . Rửa vài lần

với 40 mL nước đá, rồi với 40 mL HCl 6 M lạnh. Sấy khô sản phẩm trong lò

tại 100oC trong vài giờ. Thu được khoảng 18 g sản phẩm màu tím.

Phức này có thể được kết tinh lại trong nước nóng để có các tinh thể

màu tím xẫm với hiệu suất cao, nhưng không cần thiết cho quá trình tổng hợp

tiếp sau.

b). Điều chế [Co(NH 3 ) 5 ONO]Cl 2 và [Co(NH 3 ) 5 NO 2 ]Cl 2

Hoà tan 10 g [Co(NH 3 ) 5 Cl]Cl 2 trong một dung dịch gồm 15 mL nước

amoniac đặc (CẨN THẬN!) trong 160 mL nước vừa khuấy vừa đun. Lọc bỏ

lượng chút ít coban oxit kết tủa nếu có, và làm lạnh nước lọc xuống khoảng

10oC.

Chuẩn độ dung dịch, làm lạnh liên tục, với HCl 2 M đến khi cho phản

ứng trung tính với quỳ. Dung dịch sẽ đổi thành màu đỏ rượu vang.

Hòa tan 10,0 g natri nitrit trong dung dịch, rồi thêm 10 mL HCl 6 M.

Để dung dịch trong chậu nước đá trong khoảng một, hai giờ rồi lọc lấy kết tủa

Phát hành PDF bởi Ths Nguyễn Thanh Tú

Đăng ký Word doc qua Zalo 0905779594 Email thanhtuqn88@gmail.com


gồm các tinh thể màu hồng của [Co(NH 3 ) 5 ONO]Cl 2 . Rửa với 50 mL nước đá,

tiếp theo với 50 mL etanol, và để khô trong không khí tại nhiệt độ phòng.

Thu được khoảng 9 g sản phẩm.

Để tại chỗ mát và có bóng tối. Sau một thời gian, sự đồng phân hóa từ

đồng phân nitrit thành đồng phân nitro sẽ từ từ xảy ra.

Để điều chế một mẫu tinh khiết đồng phân nitro, hòa tan 4,0 g

[Co(NH 3 ) 5 ONO]Cl 2 trong 40 mL nước nóng chứa vài giọt nước amoniac, rồi

vừ thêm, vừa làm lạnh, 40 mL HCl đậm đặc. Làm lạnh dung dịch trong một

chậu nước đá và lọc lấy [Co(NH 3 ) 5 NO 2 ]Cl 2 màu cam. Rửa sản phẩm với 25

mL etanol và để khô trong không khí tại nhiệt độ phòng. Thu được khoảng

3,5 g sản phẩm màu vàng cam.

Câu 2:

Tách riêng coban và niken bằng sắc kí trao đổi anion và chuẩn độ EDTA các

kim loại.

Coban và niken được tách riêng trên một cột trao đổi anion bazơ mạnh

(dạng clorua) bằng cách rửa với HCl 9 M HCl rồi HCl 3 M. Trong HCl 9 M,

niken không tạo phức clo anion trong khi coban sẽ tạo phức; từ đó, Niken sẽ

được rửa trôi. Trong HCl 3 M, phức clo anion của coban có màu xanh phân li

để tạo thành cation coban hidrat hóa có màu hồng, sẽ bị rứa trôi. Sau khi

tách, các kim loại được chuẩn độ gián tiếp với EDTA chuẩn để định lượng;

lượng dư EDTA được thêm vào và EDTA chưa phản ứng ứng được chuẩn độ

lại với dung dịch kẽm chuẩn trong dung dịch hơi axit, dùng chất chỉ thị

xilenol cam.

Giai đoạn tách:

Chuẩn độ:

Ni 2+ + Cl – NiCI +

Co 2+ + 4 Cl – CoCl 4

2-

Co 2+ + H 2 Y 2- CoY 2- + 2H +

Ni 2- + H 2 Y 2- NiY 2- + 2H +

Điểm dừng:

H 2 Y 2- + Zn 2+

ZnY 2- + 2H + (chuẩn độ lại)

H 4 In + Zn 2+ ZnIn 2- + 4H +

Phát hành PDF bởi Ths Nguyễn Thanh Tú

Đăng ký Word doc qua Zalo 0905779594 Email thanhtuqn88@gmail.com


vàng-lục

Hóa chất cần thiết

đỏ-tím

Hóa chất. NaOH 3M, HCl 9M, HCl 3M, chất chỉ thị màu xilenol cam

0,5% (khối lượng/thể tích) trong etanol 10% (0,5 g hòa tan trong 10 mL

etanol và pha loãng đến 100 mL với nước cất), kẽm dạng hạt, phenolptalein

0,2% trong 90% etanol, hexamin (hexametylentetramin), nhựa trao đổi anion

Dowex 1-X8 (dạng clorua) hoặc tương tự.

Dung dịch EDTA chuẩn 0,01 M. Điều chế từ Na 2 H 2 Y.2H 2 O sấy khô

tại 80oC trong 2 giờ và làm nguội trong tủ sấy bằng cách cân khoảng 1,9 g

(đọc đến số milligam gần nhất), hòa tan trong nước cất đã khử ion, rồi pha

loãng đến 500,0 mL trong ống đong. Tính nồng độ mol phân tử.

Dung dịch kẽm 0,01 M chuẩn. Cân chính xác khoảng1 0,33 g kẽm

tinh khiết dạng hạt (đọc chính xác số đo đến 0,1 mg gần nhất) và cho vào cốc

400-mL. Không dùng bột kẽm. Để có độ chính xác cao, các viên kẽm được

xử lí với HCl 2 M để loại bỏ lớp màng kẽm oxit nếu có. Gạn bỏ axit và rửa

kẽm nhiều lần với nước. Sau đó, rửa vài lần với etanol và sau cùng với ete

trong tủ hút (DỄ CHÁY). Sấy khô các hạt kẽm trước khi cân. Hòa tan với

lượng tối thiểu HCl cần thiết. Đun cách thủy để giúp hòa tan nhanh chóng

hơn. Đậy bằng kính thủy tinh suốt quá trình hòa tan. Sau khi kẽm tan hết,

rửa bằng nước cất các giọt nước đọng dưới kính thủy tinh và cho vào cốc, rửa

sạch phía ngoài cốc rồi rót vào ống đong 500-mL. Pha loãng đến 500 mL

bằng nước cất. Tính nồng độ mol phân tử của dung dịch.

Tiến hành

1. Chuẩn bị cột trao đổi ion. Chuẩn bị một buret 50-mL có bông thủy

tinh ở đáy buret để chặn nhựa. Thêm vào nhựa trao đổi anion Dowex 1-X8

(sệt, dạng clorua) trong HCl 9 M vào buret đến khi cột nhựa có chiều cao

khoảng 15-20 cm. Không để mức chất lỏng xuống dưới mức nhựa. Giữ

khoảng 2 mL chất lỏng phía trên nhựa.

Rửa cột hai lần, mỗi lần với 10-mL HCl 9 M với lưu lượng khoảng 2

hoặc 3 mL mỗi phút. Cột sẽ xẫm màu đôi chút khi xử lí với HCl, nhưng sẽ

trở lại màu bình thường khi rửa bằng nước. Để lại 2–3 cm HCl phía trên mức

nhựa.

1 Cân chính xác khoảng 0,33g: Cân khoảng 0,33g nhưng phải đọc số đo thật chính xác theo giả thiết.

Phát hành PDF bởi Ths Nguyễn Thanh Tú

Đăng ký Word doc qua Zalo 0905779594 Email thanhtuqn88@gmail.com


2. Tách hỗn hợp chưa biết. Nhận mẫu chưa biết trong ống đong 50-mL.

Pha loãng theo thể tích với HCl 9 M HCl. Mẫu chưa biết sẽ chứa khoảng 5

mmol hoặc ít hơn niken(II) và coban(II) trong HCl 9 M.

Dùng pipet để cho 2 mL dung dịch mẫu chưa biết vào cột. Rửa giải

(elute: rửa phân tích) niken với khoảng 75 mL HCl 9 M thêm vào từng

15-mL mỗi lần, với lưu lượng (tốc độ chảy) khoảng 1-2 mL mỗi phút. Thu

vào một erlenmeyer 250-mL. Phức NiCl+ màu vàng lục nhạt sẽ chảy qua cột

và làm cho nhựa trao đổi ion xẫm màu. Dải màu xanh lam của coban (nhưng

sẽ xuất hiên thành màu lục vì nhựa màu vàng) cũng di chuyển một đoạn

xuống phía dưới cột. Trong khi mẫu thử đang được rửa giải, nên tiến hành

chuẩn độ thử như trình bày dưới đây.

Sau khi toàn bộ niken được rửa giải nhưng trước khi dải coban trôi

xuống cuối cột, ngưng cho dung dịch vào cột và thay erlenmeyer thu dung

dịch rửa bằng một erlenmeyer sạch khác. Rửa giải coban với khoảng năm

lần, mỗi lần 10-mL, dung dịch HCl 3 M HCl, với lưu lượng 1 mL mỗi phút.

Khi HCl loang ra trên cột, phức CoCl 4 2- sẽ phân li chuyển thành màu hồng Co

2+ . Sau khi coban được rửa giải hết, ngưng thêm dung dịch vào cột và tiến

hành chuẩn độ các kim loại riêng biệt. (Có thể phải rửa giải thêm một lần cuối

cùng với nước để đảm bảo thu hết toàn bộ coban.)

3. Chuẩn độ Ccoban và niken. Cho bay hơi cẩn thận các mẫu thu được

đến gần khô trên bếp điện (TỦ HÚT). Làm nguội và pha loãng với 50 mL

nước cất đã khử tách ion. Trong khi các mẫu đang bay hơi, trước khi chuẩn độ

các mẫu chưa biết, tiến hành chuẩn độ thử một, hai lần với các dung dịch

coban hoặc niken chuẩn bị sẵn. Chuẩn độ gián tiếp các dung dịch niken và

coban như sau. Trung hòa mỗi dung dịch đến điểm dừng của phenolptalein

với NaOH 3M, tránh dùng dư NaOH. Thêm từng giọt HCl 6M vừa đủ để mất

màu đỏ của chất chỉ thị. Thêm tiếp 25,00 mL EDTA 0,01 M chuẩn vào cốc,

rồi thêm năm giọt HCl 9 M, 1 g hexamin, và bốn giọt chất chỉ thị xilenol màu

cam. Dung dịch được đệm tại pH 5-6 nhờ hexamin. Nếu dung dịch có màu

đỏ-tím, đun nhẹ dung dịch và thêm vào 10,00 mL EDTA nữa. Chuẩn độ lại

với dung dịch kẽm 0,01 M tiêu chuẩn cho đến khi chất chỉ thị đổi màu từ

vàng-lục sang đỏ-tím.

Cách tính

Từ số milimol EDTA đã dùng và số milimol EDTA dư tìm được do sự

chuẩn độ lại, tính và cho biết số milimol niken và coban có trong mẫu chưa

biết.

Phát hành PDF bởi Ths Nguyễn Thanh Tú

Đăng ký Word doc qua Zalo 0905779594 Email thanhtuqn88@gmail.com


Bài thực hành số 2: ( Trích đề thi Olympic quốc tế lần thứ 39)

PHƯƠNG PHÁP CHUẨN ĐỘ XÁC ĐỊNH Fe Ở CÁC TRẠNG THÁI OXI

HÓA KHÁC NHAU

Một số phương pháp xác định Fe ở trạng thái oxy hóa +2 và +3 được

thảo luận ở bài tập 12. Bạn được mời thử thêm một phương pháp nữa để giải

quyết bài tập đó bằng thực nghiệm.

Thuốc thử và dung dịch cần thiết

KIO 3 (R9, R22, R36/37/38, S35)

Axit Ascorbic, rắn

KI (R36/38, R42-43, R61, S26, S36/37/39, S45)

HCl (R34, R37, S26, S36, S45), đặc và 2M

HNO 3 (R8, R35, S1/2, S23, S26, S36, S45), đặc

Axit sunfo salicylic 25 %

Dung dịch NH 3 10% (R10, R23, R34, R50, S1/2, S16, S36/37/39, S45,

S61)

Dung dịch EDTA chuẩn khoảng 0,05M )(R36, S26), nồng độ chính xác sẽ

được cho sau

1. Chuẩn bị dung dịch chuẩn đầu tiên KIO 3 .

1.1 Tính khối lượng KIO 3 chính xác đến 0,0001 g cần thiết để chuẩn bị 200,0

mL dung dịch KIO 3 0,01000 M.

1.2 Dùng cân phân tích lấy chính xác lượng KIO 3 . Lượng KIO 3 này có thể

khác so với tính toán không quá 0,05 g và phải được cân với độ chính xác

0,0001 g.

1.3 Chuyển lượng KIO 3 sang bình định mức 200,0 mL, hòa tan nó trong nước,

pha loãng đến vạch định mức và trộn đều.

1.4 Tính toán chính xác nồng độ mol/L dung dịch vừa mới pha.

2. Chuẩn bị dung dịch chuẩn độ - axit ascorbic.

2.1 Tính khối lượng axit ascorbic với độ chính xác đến 0.01 gam cần thiết để

pha 200 mL dung dịch nồng độ 0,1 M.

2.2 Dùng cân kĩ thuật cân. Lượng cân có thể khác so với lượng tính toán

nhưng không quá 0,05 g.

Phát hành PDF bởi Ths Nguyễn Thanh Tú

Đăng ký Word doc qua Zalo 0905779594 Email thanhtuqn88@gmail.com


2.3 Hòa tan lượng axit ascorbic mới cân vào khoảng 200 mL nước, trộn đều,

chuyển dung dịch sang một lọ và dùng nút đóng chặt .

3. Chuẩn hóa dung dịch axit ascorbic.

3.1 Đổ đầy vào buret dung dịch axit ascorbic.

3.2 Dùng pipet chuyển 10,00 mL dung dịch chuẩn KIO 3 sang bình nón

(Erlen), thêm 20 mL dung dịch KI 5% và 5 mL dung dịch HCl 2M.

3.3 Chuẩn độ hỗn hợp với axit ascorbic đến khi màu của Iot biến mất.

Chú ý: khi chuẩn độ Iot với những dung dịch chất khử, hồ tinh bột thường

được thêm vào như một chất chỉ thị. Ở đây không khuyên làm như vậy bởi vì

tốc độ phản ứng sẽ giảm rất mạnh khi có hồ tinh bột.

3.4 Lặp lại phép chuẩn độ đến khi thu được 3 thể tích dung chuẩn độ khác

nhau không quá 0,10 mL.

3.5 Tính trung bình thể tích dung chuẩn độ.

3.6 Tính nồng độ mol/L của axit ascorbic.

Câu hỏi

1. Viết và cân bằng tất cả những phương trình xảy ra trong quá trình chuẩn

hóa dung dịch axit ascorbic. Biết axit ascorbic C 6 H 8 O 6 bị oxi hóa thành axit

dehydro ascorbic C 6 H 6 O 6 .

2. KIO 3 trong KI dư cũng có thể được dùng như chất chuẩn để chuẩn hóa

HCl. Phương pháp tương tự như phương pháp đã mô tả ở trên nhưng trong

trường hợp này không thêm HCl vào dung dịch chuẩn độ. Những chất nào có

thể dùng làm chỉ thị cho phép chuẩn độ này?

a. Hồ tinh bột

b. Axit sunfosalicylic

c. Metyl da cam

d. Metyl da cam + Na 2 S 2 O 3 dư

3. Xác định Fe(III) bằng phép chuẩn độ ascorbic.

Lấy dung dịch mẫu chứa Fe(II) và Fe(III) (trong bình định mức 100,0 mL) từ

người hướng dẫn của bạn. Pha loãng dung dịch cho tới vạch định mức bằng

nước và trộn đều.

Đổ đầy buret bằng dung dịch chuẩn axit ascorbic.

Dùng pipet cho 10,00 mL dung dịch mẫu vào bình nón (erlen) 100 mL, thêm

40 mL nước và đun nóng cho tới gần sôi.

Phát hành PDF bởi Ths Nguyễn Thanh Tú

Đăng ký Word doc qua Zalo 0905779594 Email thanhtuqn88@gmail.com


Thêm 4-5 giọt axit sunfosalicylic 25% với vai trò như một chất chỉ thì vào

hỗn hợp dung nóng trên.

Chuẩn độ dung dịch bằng dung dịch ascorbic đến khi màu tím biến mất.

Trong quá trình chuẩn độ đặc biệt là gần điểm kết thúc, dung dịch phải nóng.

Bạn có thể cần phải đun nóng thêm nếu cần thiết. Gần điểm kết thúc, axit

ascorbic nên thêm rất từ từ.

Lặp lại thí nghiệm đến khí thu được 3 thể tích dung dịch dùng để chuẩn độ có

sai khác không quá 0,10 mL.

Tính thể tích trung bình dung dịch đã dùng để chuẩn độ.

Tính khối lượng Fe(III) trong mẫu dung dịch đã đưa cho bạn.

Chú ý: axit ascorbic, đặc biệt trong dung dịch nước, rất bền và bị oxi hóa

bằng oxy từ không khí. Do đó quá trình chuẩn hóa dung dịch axit ascorbic và

quá trình xác định Fe(III) bằng ascorbic phải được thực hiện trong 1 ngày.

Câu hỏi

Viết và cân bằng tất cả các phương trình phản ứng xảy ra trong quá trình xác

định Fe(III). Biết axit ascorbic C 6 H 8 O 6 bị oxi hóa thành axit dehydro ascorbic

C 6 H 6 O 6 .

Trong môi trường nào axit ascorbic thể hiện tính chất khử rõ rệt nhất?

Trong môi trường axit

Trong môi trường trung tính

Trong môi trường kiềm

Tính khử của axit ascorbic không phụ thuộc vào pH

5. Xác định tổng Fe bằng phương pháp chuẩn độ tạo phức.

5.1 Đổ đầy buret với dung dịch chuẩn EDTA.

5.2 Chuyển 10,00 mL dung dịch mẫu vào bình nón 100 mL bằng pipet. Thêm

5 mL HCl đặc và 2 mL HNO 3 đặc để oxi hóa Fe(II) có mặt trong mẫu thành

Fe(III). Đậy bình nón bằng thủy tinh trong, đun nóng cho đến sôi và tiếp tục

đun nóng khoảng 3-5 phút và tránh dung dịch bắn.

5.3 Làm nguội dung dịch và trung hòa cẩn thận nó bằng cách thêm từng giọt

dung dịch NH 3 10% đến khi màu của dung dịch chuyển từ màu vàng chanh

sang hơi vàng nâu và hơi đục.

5.4 Thêm 1-2 giọt HCl 2M để hòa tan chất kết tủa, sau đó thêm 0,5 mL HCl

2M, pha loãng đến 50 mL bằng nước cất và đun nóng đến gần sôi.

5.5 Thêm 4-5 giọt sunfosalicylic 25% với vai trò như một chất chỉ thị vào hỗn

hợp dun nóng trên.

5.6 Chuẩn độ dung dịch đến khi màu tím chuyển thành màu vàng rõ rệtt.

Trong quá trình chuẩn độ đặc biệt là gần điểm kết thúc, dung dịch phải nóng.

Bạn có thể cần phải đun nóng thêm nếu cần thiết. Gần điểm kết thúc, dung

dịch EDTA nên thêm rất từ từ.

Phát hành PDF bởi Ths Nguyễn Thanh Tú

Đăng ký Word doc qua Zalo 0905779594 Email thanhtuqn88@gmail.com


5.7 Lặp lại thí nghiệm đến khí thu được 3 thể tích dung dịch dùng để chuẩn

độ có sai khác không quá 0,10 mL.

5.8 Tính thể tích trung bình dung dịch đã dùng để chuẩn độ.

5.9 Tính tổng khối lượng Fe trong mẫu đã đưa cho bạn.

5.10 Tính khối lượng Fe(II) bằng hiệu giữa kết quả đạt được trong 5.9 và 4.8.

Câu hỏi

1. Viết và cân bằng tất cả các phương trình phản ứng xảy ra trong quá trình

xác định tổng Fe.

2. Một trong những điểm cốt lõi trong việc xác định Fe(III) bằng phương

pháp chuẩn độ tạo phức là phải tuyệt đối duy trì độ axit cho dung dịch. Lý do

nào ta phải làm như vậy?

Nếu đô axit quá thấp, Fe(OH) 3 sẽ kết tủa.

Nếu độ axit quá cao, phức của Fe(III) với sunfosalicylic sẽ không

tạo ra

Hướng dẫn

Nếu độ axit quá cao, phức của Fe(III) với EDTA sẽ không tạo ra

Nếu độ axit quá thấp/cao chất dùng chuẩn độ sẽ phân hủy.

Xác định Fe ở các trạng thái oxi hoá khác nhau bằng chuẩn độ

Vào phần 3

1. IO 3 – + 5I – + 6H + 3I 2 + 3H 2 O

2.

Vào phần 4

I 2 + C 6 H 8 O 6 2I – + C 6 H 6 O 6 + 2H +

■ metyl da cam + Na 2 S 2 O 3 (lấy dư)

1. 2 Fe 3+ + C 6 H 8 O 6 2 Fe 2+ + C 6 H 6 O 6 + 2H +

2.

Trong kiềm

Vào phần 5

1.

3 Fe 2+ + NO 3 – + 4 H + 3 Fe 3+ + NO + H 2 O

Phát hành PDF bởi Ths Nguyễn Thanh Tú

Đăng ký Word doc qua Zalo 0905779594 Email thanhtuqn88@gmail.com


2.

Fe 3+ + Y 4– (EDTA anion) FeY –

■ ở độ axit quá thấp Fe(OH) 3 kết tủa

■ ở độ axit quá cao phức của Fe(III) với axit sulfosalicylic không hình

thành được

■ ở độ axit quá cao phức của Fe(III) với axit EDTA không hình thành

được

Bài thực hành số 3: ( Trích bài chuẩn bị Icho 40)

Trong bài này, dùng phép chuẩn độ bằng complexon bạn sẽ tìm một kim loại.

EDTA tạo phức chất bền với hầu hết các ion kim loại hóa trị (II) và (III).

M 2+ + H 2 Y 2– = MY 2– + 2 H + M 3+ + H 2 Y 2– = MY – + 2 H +

M là kim loại, Y 4- là anion tạo thành từ EDTA.

Khi có một lượng dư ion kim loại chưa tạo phức, chúng liên kết với

phân tử chất chỉ thị. Phản ứng kết thúc, tất cả các ion hình thành phức chất

EDTA và phân tử chất chỉ thị được giải phóng làm thay đổi màu sắc. Vì thế,

điểm cuối của phản ứng là khi thêm EDTA vào mà không làm thay đổi màu

sắc của dung dịch. Mỗi lần phải chuẩn độ cho đến khi màu ổn định. Mẫu

chuẩn độ đầu tiên có thể dùng để so sánh.

Đầu tiên mẫu kim loại được hòa tan bằng axit nitric . Sau khi pH của

dung dịch thu được khoảng bằng 2 thì chuẩn độ với EDTA. Một cách đo khác

là hòa tan oxit của kim loại và dung dịch lại được chuẩn độ với EDTA. (Oxit

được chuẩn bị trước bằng cách làm bay hơi dung dịch axit nitric đã hòa tan

kim loại , rồi nung bã rắn thu được)

Nhiều dung dịch đã dùng có tính axit, xử lý các dung dịch này phải cẩn thận

đúng mức.

Chuẩn độ kim loại

Cân chính xác 150 mg kim loại chưa biết cho vào bình chuẩn độ. Cẩn

thận thêm vào 3 cm 3 axit nitric đặc rồi làm trong tủ hút. Hòa tan xong mất

khoảng 10 phút. Pha loãng dung dịch đến 50 cm 3 và rồi quay trở lại bàn làm

việc. Đầu tiên thêm vào 8 cm 3 dung dịch amoniăc 5%, rồi tiếp tục thêm từng

giọt dung dịch amoniăc cho đến khi thành phần chất trong bình chuẩn độ bắt

đầu vẫn đục do kết tủa hydroxit. Thêm ngay vào 5 cm 3 dung dịch axit nitric

10%. Thêm vào hai nhúm chất chỉ thị Methylthymol xanh rắn. Chuẩn độ với

Phát hành PDF bởi Ths Nguyễn Thanh Tú

Đăng ký Word doc qua Zalo 0905779594 Email thanhtuqn88@gmail.com


EDTA 0,0500 mol/dm 3 cho đến khi có màu vàng ổn định. Lặp lại khi cần

thiết.

Chuẩn độ oxit kim loại

Cân chính xác 1,000 g oxit, hòa tan vào 5 cm 3 axit nitric đặc, rồi pha

loãng dung dịch đến 100 cm 3 trong bình định mức. Chuyển 20,00 cm 3 vào

bình chuẩn độ và pha loãng đến 50 cm 3 . Bắt đầu thêm từng giọt dung dịch

amoniăc 5% cho đến khi xuất hiện kết tủa. Thêm ngay vào 5 cm 3 dung dịch

axit nitric 10%. Thêm vào hai nhúm chất chỉ thị Methylthymol xanh rắn.

Chuẩn độ với EDTA 0,0500 mol/dm 3 cho đến khi có màu vàng ổn định. Lặp

lại khi cần thiết.

a/ Bằng tính toán, hãy xác định kim loại.

b/ Xác định công thức oxit.

Lưu ý : những hiện tượng rõ ràng nhất chưa hẳn đã quyết định được kết quả

mà kết quả đó tạo ra các giá trị dự kiến về mặt lý thuyết.

Thuốc thử Nồng độ Nhóm R Nhóm S

Muối natri EDTA 0.05 mol/dm 3 36/38 26-36

axit Nitric 65 % 35 23-26-36-45

axit Nitric 10 % 35 23-26-36-45

Ammoniac 5 % 34-50 26-36/37/39-45-61

Methylthymol xanh

Phát hành PDF bởi Ths Nguyễn Thanh Tú

Đăng ký Word doc qua Zalo 0905779594 Email thanhtuqn88@gmail.com


3.1. Kiến nghị

PHẦN 3: KIẾN NGHỊ VÀ KẾT LUẬN

3.1.1. Những công việc đã thực hiện

1. Hệ thống kiến thức về các nguyên tố nhóm VII.B và nhóm VIIIB.

2. Xây dựng, sưu tầm hệ thống câu hỏi lý thuyết và bài tập vận dụng cho các

nguyên tố nhóm VII.B và VIII.B.

3.1.2. Những tác dụng của đề tài

Đối với học sinh: Đề tài này đã được chúng tôi sử dụng giảng dạy cho

học sinh khối chuyên Hóa và học sinh ôn thi học sinh giỏi tỉnh, khu

vực,trong đội tuyển quốc gia.

Thông qua đề tài này, góp phần giúp học sinh nắm được một cách hệ

thống kiến thức về kim loại chuyển tiếp quan trọng, tiếp xúc với các bài

tập về kim lọai chuyển tiếp.

Đối với giáo viên: Đề tài này cũng là một tư liệu trong giảng dạy, đặc

biệt là đối với các giáo viên trẻ. Qua đề tài này, các giáo viên tiếp tục

củng cố, bổ xung, xây dựng nhiều bài tập hay về phần kim loại chuyển

tiếp phục vụ cho việc ôn luyện học sinh giỏi trong các kỳ thi.

3.2. Kết luận

Trong quá trình biên soạn, tôi không tránh khỏi những hạn chế về mặt kiến

thức nên sẽ có vấn đề có thể còn chưa thật chặt chẽ. Tôi rất mong sự góp ý

của các đồng nghiệp để chúng tôi hoàn thiện đề tài này và đóng góp vào bộ tư

liệu giảng dạy của chúng tôi thêm phong phú.

Phát hành PDF bởi Ths Nguyễn Thanh Tú

Đăng ký Word doc qua Zalo 0905779594 Email thanhtuqn88@gmail.com


TÀI LIỆU THAM KHẢO

[1] Nguyễn Duy Ái, Đào Hữu Vinh, Tài liêụ giáo khoa chuyên hóa học 10,

Tập 1, NXB Giáo dục, 2001.

[2] Nguyễn Đức Vận, Hóa học vô cơ, Tập 2, NXB Khoa học kỹ thuật, 2000.

[3] Đề thi chọn HSG Quốc gia các năm.

[4] Đề thi chọn đội tuyển học sinh thi Olympic

[5] Bài tập hóa học vô cơ trong những kỳ thi Olympic hóa học.

[6] Hóa học Vô cơ – Tập 3 – Hoàng Nhâm

[7] Bài tập đại cương và vô cơ – Nguyễn Duy Ái, Đào Hữu Vinh

[8] Các đề thi chọn HSG Quốc Gia, đề thi chọn đội tuyển dự thi Quốc tế, đề

thi Olympic Hóa học Quốc tế.

[9] Cao Cự Giác (2004), Bài tập lý thuyết và thực nghiệm Hóa học, tập 1.

NXB Giáo dục, Hà Nội.

[10] Hội hóa học Việt Nam(2000,2002), Olympic hóa học Việt Nam và quốc

tế tập I.

[11] F.Cotton –G.Wilkinson. Người dịch Lê Mậu Quyền, Lê Chí Kiên.

(1984), [12] Đề thi khu vực Đồng Bằng và Duyên Hải Bắc Bộ các năm.

[13] Đề thi olympic 30/4 các năm.

[14] http://chemistry.about.com/

Phát hành PDF bởi Ths Nguyễn Thanh Tú

Đăng ký Word doc qua Zalo 0905779594 Email thanhtuqn88@gmail.com


Chuyên đề: XÂY DỰNG HỆ THỐNG CÂU HỎI VÀ BÀI TẬP NHÓM VIIB VÀ

VIIIB

Phần 1: MỞ ĐẦU

I. Lí do chọn chuyên đề.

Đào tạo và bổi dưỡng nhân tài được quan tâm từ thời xã hội phong kiến. Ngày nay,

nhiệm vụ đào tạo và bồi dưỡng học sinh giỏi được coi là một nội dung mũi nhọn trong việc

nâng cao dân trí, đào tạo nguồn lực, bồi dưỡng nhân tài cho nhà trường nói riêng, cho địa

phương nói chung.

Phần hóa học nguyên tố là một nội dung quan trọng ở chương trình hóa học THPT,

chiếm phần lớn nội dung chương trình lớp 10, 11, 12 và là một chuyên đề nâng cao trong

việc bồi dưỡng học sinh giỏi Hóa trường THPT Chuyên. Tuy nhiên phần nguyên tố kim loại

chuyển tiếp và đặc biệt là nhóm VIIB và VIIIB mới chỉ được đề cập ở cuối chương trình lớp

12 và còn sơ lược. Trong các kì thi chọn học sinh giỏi cấp tỉnh, cấp quốc gia, bài tập về hóa

học nguyên tố, mà thường gặp là bài tập về các kim loại chuyển tiếp nhóm VIIB, VIIIB và

hợp chất ngày càng được đề cập nhiều hơn vì đây là một vấn đề hay và cần thiết cho các em

học sinh khi nghiên cứu, tìm hiểu sâu về hóa học. Mặt khác các bài tập nâng cao trong các

tài liệu hóa THPT còn ít, do đó đã gây ra rất nhiều khó khăn cho các giáo viên dạy đội tuyển

cũng như các em học sinh khi tham gia đội tuyển Hóa THPT.

Để giải quyết những khó khăn trên của giáo viên cũng như học sinh với mong muốn

có thêm tài liệu bổ ích và phù hợp với công việc giảng dạy trong nhà trường, đồng thời bồi

dưỡng học sinh giỏi chúng tôi sưu tầm và xây dựng chuyên đề : “Xây dựng hệ thống câu hỏi

và bài tập nhóm VIIB và VIIIB”.

II. Mục đích nghiêncứu

Hệ thống hóa lí thuyết và xây dựng hệ thống câu hỏi và bài tập vận dụng các nguyên

tố nhóm VIIB và VIIB để làm tài liệuphụcvụ cho giáo viên trường chuyên giảng dạy, ôn

luyện, bồi dưỡng học sinh giỏi các cấp và làm tài liệu học tập cho học sinh chuyên hoá.

Ngoài ra còn là tài liệu tham khảo cho giáo viên môn hóa học và học sinh yêu thích môn

hóa học nói chung.

III. Nhiệmvụ

Nghiên cứu chương trình hóa học phổ thông nâng cao và chuyên hóa học, phân tích

các đề thi học sinh giỏi cấp tỉnh, khu vực, cấp quốc gia, quốc tế và đi sâu về nội dung liên

quan kim loại nhóm VIIB và nhóm VIIIB (chủ yếu xây dựng bài tập về Mn Fe, Co, Ni).

Sưu tầm, lựa chọn trong tài liệu giáo khoa, sách bài tập cho học sinh, trong các tài

Phát hành PDF bởi Ths Nguyễn Thanh Tú

Đăng ký Word doc qua Zalo 0905779594 Email thanhtuqn88@gmail.com

1


liệu tham khảo. Các đề thi học sinh giỏi các cấp có nội dung liên quan; phân loại, xây dựng

các bài tập lí thuyết và tínhtoán.

Đề xuất phương pháp xây dựng và sử dụng hệ thống bài tập dùng cho việc giảng dạy,

bồi dưỡng học sinh giỏi các cấp ở trường THPTchuyên.

IV.Điểm mới của chuyênđề

- Chuyên đề đã xây dựng được hệ thống lí thuyết cơ bản có mở rộng và nâng cao một cách

hợp lí và hệ thống bài tập, phân loại rõ ràng các dạng bài tập về kim loại chuyển tiếp để làm

tài liệu phục vụ cho học sinh và giáo viên trường chuyên học tập. giảng dạy, ôn luyện, bồi

dưỡng trong các kì thi học sinh giỏi các cấp. Ngoài ra còn là tài liệu tham khảo mở rộng và

nâng cao cho giáo viên môn hóa học và học sinh yêu thích môn hóa học nói chung.

Phát hành PDF bởi Ths Nguyễn Thanh Tú

Đăng ký Word doc qua Zalo 0905779594 Email thanhtuqn88@gmail.com

2


Phần 2: NỘI DUNG

Chương 1: SƠ LƯỢC VỀ NGUYÊN TỐ KIM LOẠI NHÓM VIIB VÀ NHÓM VIIIB

I. SƠ LƯỢC VỀ NGUYÊN TỐ KIM LOẠI NHÓM VIIB

Nhóm VIIB gồm các nguyên tố Mangan (Mn), Tecnexi (Tc) và Reni (Re)

I. 1. Đặc điểm chung của các nguyên tố nhóm VIIB

Bảng I. 1: Đặc điểm của các nguyên tố nhóm VIIB.

Nguyên tố (E) Mn Tc Re

Số thứ tự 25 43 75

Cấu hình electron nguyên tử Ar 3d 5 4s 2 Kr 4d 5 5s 2 Xe4f 14 5d 5 6s 2

I 1 7,43 7,28 7,79

Năng lượng

I

Ion hoá (eV)

2 15,63 15,26 13,1

I 3 33,69 29,5 26,0

Bán kính nguyên tử (A 0 ) 1,30 1,36 1,37

M 2+ (A 0 ) 0,91 0,95 -

Bán kính Ion

M 3+ (A 0 ) 0,70 - -

M 4+ (A 0 ) 0,52 0,72 0,72

M 7+ (A 0 ) 0,46 0,57 0,57

Thế điện cực chuẩn E 0 (V) -1,18(Mn 2+ /Mn) +0,4(Tc +2 /Tc) +0,3(Re 3+ /Re)

Số oxi hoá đặc trưng ( bền) +2, +4, +7 +7 +4, +7

Độ âm điện 1,55 1,9 1,9

% nguyên tử trong vỏ trái đất

0,09 -

10 -7

Nhiệt độ nóng chảy ( 0 C) 1244 2140 3180

Nhiệt độ sôi ( 0 C) 2080 4900 5900

Khối lượng riêng (g/cm 3 ) 7,47 11,5 20,5

Nhận xét:

Mangan, Tecnexi và Reni có cấu hình electron giống nhau đều là (n-1)d 5 ns 2 nên có tính

chất tương tự nhau.Tuy nhiên,Tc và Re giống nhau nhiều hơn so với mangan vì chúng có

bán kính nguyên tử tương đương nhau. Nguyên nhân của hiện tượng này là có sự nén

Lantanit, nên bán kính ít có sự biến đổi.

Với số lớn electron hoá trị, những nguyên tố nhóm VIIB tạo nên hợp chất có nhiều số

oxi hoá khác nhau, từ 0 đến +7. Cấu hình electron bền d 5 thể hiện ở năng lượng ion hoá thứ

ba, tương đối cao hơn tổng năng lượng Ion thứ nhất và thứ hai. Tuy nhiên, việc mất 2

electron ns của nguyên tử biến thành cation kim loại chỉ đặc trưng ở Mn còn Tc và Re có

khuynh hướng tạo nên hợp chất với số oxi hoá cao hơn, nhất là số oxi hoá +7. Đó là do sự

tăng độ bền của liên kết cộng hoá trị làm tăng độ bền của anion chứa nguyên tố có số oxi

hoá cao. Ví dụ anion TcO - 4 bền hơn anionMnO - 4.

Những số oxi hoá của Mn là +2,+3, +4,+6, +7; Tc có số oxi hoá là +4, +7; Re có số oxi

hoá đặc trưng là +3, +4, +5, +7.

Mangan (Mn)

3

Phát hành PDF bởi Ths Nguyễn Thanh Tú

Đăng ký Word doc qua Zalo 0905779594 Email thanhtuqn88@gmail.com


Mangan có số oxi hoá bền nhất là Mn +2 , Mn +4 ,Mn +7 . Trong môi trường axit MnO - 4 có

tính ôxi hoá rất mạnh nhưng trong môi trường kiềm tính oxi hoá của MnO - 4 giảm hẳn so

với môi trường axit, MnO - 4 có thể bị khử xuống Mn +2 nhưng trong môi trường kiềm,

MnO - 4 chỉ bị khử xuống Mn +6 (MnO 4 2- ) hoặc Mn +4 ( MnO 2 ).

- Trong môi trường axit hoặc bazơ thì Mn đều dễ bị oxi hoá lên Mn +2 .

- Các hợp chất số oxi hoá +5, +6,+3 của mangan không bền trong môi trườngkiềm Mn +5 ,

Mn+6 cũng không bền. Trong môi trường kiềm, Mn +2 dễ bị khử lên Mn +4 trong MnO 2 bởi

các chất oxi hoá mạnh.

Tecnexi (Tc)

- Số oxi hoá bền của Tc trong môi trường trung tính là Tc+7, Tc+4

- Tính oxi hoá của TcO 4- yếu hơn rất nhiều so với MnO 4- ở cùng mức oxihoá +7 nên

TcO 4- bền hơn MnO 4- , điều này được thể hiện thông qua giá trị thế điện cực

E 0 (MnO 4- /MnO 2 ) =1,7(V) > E 0 (TcO 4- /TcO 2 ) = 0,70(V).

Reni (Re)

- Trong môi trường axit Re +7 , Re +4 bền, còn Re +6 , Re +3 kém bềnhơn.

- Trong môi trường kiềm ReO 3 có tính khử, dễ bị ngay cả oxi phân tử của khôngkhí oxi

hoá theo phảnứng

4K 2 ReO 4 + O 2 + 2H 2 O → 4 KReO 4 + 4 KOH E o pư = 0,489V >0

Do đó, trạng thái số oxi hoá +7 của Re là bền hơn của Mn. Tuy nhiên, do trong môi

trường kiềm E 0 (ReO 4 - / ReO 3 ) = -0,89 < E o ( ReO 3 / ReO 2 ) =-0,446 nên dễ bị tự oxi hoátự

khử chuyển thành Re +7 và Re +4

I. 2. Tính chất vậtlý

Mangan, tecnexi và reni là những kim loại màu trắng bạc. Dạng bề ngoài của mangan

giống với sắt, của tecnexi giống với platin nhưng mangan cứng và khó nóng chảy hơn sắt.

Tuỳ theo phương pháp điều chế, mangan tạo ra ở 4 dạng thù hình:

Mangan điều chế bằng phương pháp nhiệt nhôm tồn tại ở 2 dạng -Mn và - Mn

Dạng -Mn tồn tại ở nhiệt độ thường có khối lượng riêng là 7,21g/cm 3 kết tinh theo

mạng lập phương tám khối phức tạp.

Dạng - Mn tồn tại ở nhiệt độ thường, có khối lượng riêng là 7,29g/cm 3 , kết tinh theo

mạng lập phương phức tạp.

Nếu Mangan được kết tủa bằng phương pháp điện phân mangan tồn tại ở dạng -Mn

bền trng khoảng 1070 – 1130 0 C , có khối lượng riêng là 7,21g/cm 3 kết tinh theo mạng tứ

phương. Dạng thù hình tồn tại ở nhiệt độ cao hơn 1130 0 C là dạng - Mn , kết tinh theo hệ

lập phương tám khối.

Thù hình -Mn - Mn -Mn - Mn

Tồn tại Nhiệt độ thường (1070 0 C) (1130 0 C) (>1130 0 C)

Các dạng -Mn, - Mn đều cứng và giòn, -Mn thì mềm và dẻo. Dưới đây là các hằng

số vật lý quan trọng của Mn, Tc, Re.

4

Phát hành PDF bởi Ths Nguyễn Thanh Tú

Đăng ký Word doc qua Zalo 0905779594 Email thanhtuqn88@gmail.com


Bảng I.2: Hằng số vật lý quan trọng của các kim loại Mn, Tc, Re

Nhiệt

Ki Nhiệt Nhiệt

thăng Tỉ Độ

m độ độsôi

loại nóng ( 0 hoa khối cứng

C)

chảy( 0 (kJ/mol)

(thang

C)

Maxơ)

Độ dẫn

điện

(Hg=1)

5

Phát hành PDF bởi Ths Nguyễn Thanh Tú

Đăng ký Word doc qua Zalo 0905779594 Email thanhtuqn88@gmail.com

Cấu

trúc

tinh thể

Mn ()1244 2080 280 7,44 5-6 5 Phức tạp

Tc 2140 4900 649 11,49 - - Lục phương

Re 3180 5900 777 21,04 7,4 4,5 Lục phương

Nhận xét:

- Mangan, tecnecxi là kim loại rất khó nóng chảy và khó sôi. Sự tăng nhiệt độ nóng chảy,

nghiệt độ sôi, nhiệt thăng hoa và độ cứng trong nhóm Mn- Tc- Re được giải thích bằng sự

tăng độ bền của liên kết trong tinh thể kim loại chủ yếu bằng số liên kết cộng hoá trị được tạo

nên từ số electron độc thân ở obitand của các nguyên tử Mn, Tc, Re là tối đa. Về nhiệt độ

nóng chảy reni chỉ thua vonfram là kim loại khó nóng chảy nhất nên reni là nguyên liệu rất tốt

để làm dây tóc bóng đèn điện, bền hơnvofram.

- Mangan tinh khiết dễ cán và dễ rèn nhưng khi chứa tạp chất nó trở nên giòn và cứng.

Mangan và reni tạo nên hợp kim với nhiều kimloại.

-Trong thiên nhiên, Mn có đồng vị 55 Mn chiếm 100%. Tc là nguyên tố nhân tạo, các đồng vị

có tính phóng xạ, re có 14 đồng vị trong thiên nhiên là 185 Re (37,07%), 187 Re (62,93%).

I. 3. Tính chất hoáhọc

Từ Mn đến Re, hoạt tính hoá học của các nguyên tố giảm xuống. Mangan là kim loại

tương đối hoạt động, còn tecnexi và reni là kim loại kém hoạt động. Điều này được giải thích

bằng sự tăng nhiệt thăng hoa một cách rõ rệt từ Mn đến Re mặc dù tổng năng lượng Ion hoá

thứ nhất và thứ hai của chúng không khác nhau nhiều.

Mangan dễ bị oxi không khí oxi hoá nhưng màng oxit Mn 2 O 3 được tạo nên lại bảo vệ

cho kim loại không bị oxi hoá tiếp tục kể cả khi đun nóng, các kim loại Tecnexi và reni bền

trong không khí, ở dạng bột Mangan, Tecnexi và reni tác dụng với oxi.

3Mn +2O 2 → Mn 3 O 4

4Tc + 7O 2 → 2Tc 2 O 7

4Re + 7O 2 → Re 2 O 7

Mangan và reni đều phản ứng trực tiếp với lưu huỳnh, selen, telu tạo ra các hợp chất

MnS, MnSe, MnSe 2 , ReSe 2 …

Mangan hóa hợp trực tiếp với nito tạo ra Mn 3 N 2 ở khoảng 600-1000 0 C

Với flo, Clo, Mangan và reni tạo nên MX 2 chúng tác dụng với phốt pho, các bon, Silic.

Trong đó, tương tác của Tc và Re xảy ra ở nhiệt độ cao hơn so với Mn. Nhờ tác dụng dễ dàng


với các nguyên tố không kim loại ở nhiệt độ cao nên Mangan còn có vai trò của chất loại oxi

trong luyện kim.Ở trạng thái phân bố nhỏ Mangan tác dụng được với nước giải phóng Hiđrô,

Tc và Re không có khả năng đó.

Mn + 2H 2 O → Mn(OH) 2 +H 2 .

Mn tan trong các axit loãng không có tính oxi hóa như HCl, H 2 SO 4 tạo ra H 2 nhưng Tc

và Re không có khả năng đó.

Mn + H 2 SO 4 → MnSO 4 +H 2 .

Mn tan trong H 2 SO 4 đặc tạo ra SO 2 , phản ứng sảy ra nhanh khi đun nóng.

Mn + 2H 2 SO 4 → MnSO 4 + SO 2 +H 2 O.

Mn phản ứng với HNO 3 tạo ra NO

3Mn + 8HNO 3 3Mn(NO 3 ) 2 + 2NO +4H 2 O

Tc và Re phản ứng mạnh với HNO 3 và H 2 SO 4 đặc tại ra hợp chất ứng với hóa trị bền

của Tc (VII) và Re (VII)

Mn không phản ứng với kiềm

Khác với Mangan và tecnexi, kim loại Reni tan trong Hiđrôpeoxit tạo thành axit

perenic

2Re + 7H 2 O 2 2HReO 4 +6H 2 O.

I.4. Hợp chất của nguyên tố nhóm VIIB Đặc điểm chung

Các hợp chất của mangan với oxi hoá thấp, phổ biến nhất là các hợp chất mangan (II).

Các hợp chất này tương đối bền, trong môi trường axit rất khó oxi hoá lên các số oxi hoá cao.

Mangan cũng hình thành các muối đơn với số oxi hoá+3 trong đó mangan thể hiện là các ion

Mn 3+ nhưng muối này dễ tự oxi hoá khử tạo thành các ion Mn 2+ và Mn(IV). Mangan với số

oxi hoá +1 chỉ tồn tại trong các phức chất. Số oxi hoá của các mangan oxit càng tăng tính

bazơ càng giảm và tính axít tăng. Ở trạng thái số oxi hoá + 4 do mangan (IV) oxit rất khó tan

nên tính bazơ cũng như tính axít thể hiện rất yếu.

Ở mức oxi hoá +5, +6, +7 các oxit của mangan chỉ thể hiện tính axít. Các muối tương

ứng là mangannat (VI), và pemanganat (mangannatVII).

Các hợp chất của tecnexi giống tính chất của các hợp chất reni hơn là giống tính chất

của các hợp chất mangan.

Tecnexi hình thành các hợp chất với số oxi hoá +4, +5 và +7.

Ở các số oxi hoá thấp reni không hình thành các cation đơn giản trong dung dịch mà

hình thành các phức chất. Reni với số oxi hoá +7 tồn tại trong axit perenic (HReO 4 ) các muối

perenat tương tự như axit pemanganic và các muối pemanganat. Ngoài ra reni còn có các

renat với số oxi hoá +6, +5, +4.

Re cũng hình thành các hợp chất dưới dạng anion Re - tương tự anion halogenua. Các

nguyên tố nhóm VII B không tạo thành các hợp chất với hidro. Riêng mangan có khả năng

hấp thụ hidro một cách hạn chế.

I.4.1. Hợp chất với số oxi hóa +2.

Các hợp chất Mn(II) tương đối bền và phổ biến hơn so với Tc, Re. Các hợp chất Tc(II)

6

Phát hành PDF bởi Ths Nguyễn Thanh Tú

Đăng ký Word doc qua Zalo 0905779594 Email thanhtuqn88@gmail.com


và Re(II) không đặc trưng.

Số phối trí đặc trưng của Mn(II) là bằng 6 ứng với dạng lai hoá sp 3 d 2 của nguyên tử và

sự phân bố bát diện đều của các liên kết. Các hợp chất Mn(II) thuận từ và chứa 5 election độc

thân (trừ các xianua).

Các hợp chất Mn(II) là những chất tinh thể. Số phối trí của Mn(II) trong các tinh thể

thường bằng 6. Một số hợp chất gồm hai nguyên tố đều dễ kết tinh có tính đồng hình. Ví dụ:

MnO, MnS có cấu trúc kiểu NaCl, còn MnF 2 có cấu trúc kiểu rutin.

Đa số các hợp chất Mn(II) đều dễ tan trong nước, ít tan là MnO, MnS, MnF 2 , Mn(OH) 2 ,

MnCO 3 và Mn 3 (PO 4 ) 2 . Khi tan trong nước các muối Mn(II) phân ly tạo phức chất aquơ dạng

[Mn(OH 2 ) 6 ] 2+ làm cho dung dịch có màu hồng. Các tinh thể hidrat của Mn(II) như Mn(NO 3 ) 2 .

6H 2 O, MnSiF 6 . 6H 2 O cũng có màu như vậy.

Trong các tinh thể hiđrat có số phân tử H 2 O bé hơn, ngoài những nhóm OH 2 ra, các

anion thành phần cũng đóng vai trò phối tử.

Mangan (II) oxit MnO

Là chất bột nàu xám lục, có mạng lưới tinh thể kiểu NaCl, có thành phần biến đổi từ

MnO đến MnO 1,5 và nóng chảy ở 1780 0 C. Không tan trong nước, không phản ứng với

nước. ở trạng thái tinh thể hoàn toàn bền trong không khí, nhưng ở dạng bột dễ bị oxi hoá

tạo thành các oxit cao như MnO 2 , Mn 2 O 3 ,Mn 3 O 4

2MnO + O 2 →2MnO 2

Tan trong axit tạo thành muối Mn(II):

MnO + 2HCl → MnCl 2 + H 2 O

Bị H 2 khử thành kim loại ở nhiệt độ rấtcao:

t

MnO + H 2

0

Mn +H 2 O

Điều

chế:

M

t

nCO 3

0

MnO + CO 2

t

MnC 2 O 4

0

MnO + CO 2 + CO

Hoặc khử các oxít cao của Mangan bằng H 2 hay C ở nhiệt độ cao:

Mn 3 O 4 + H 2 → 3MnO + H 2 O

Mangan (II) hidroxit Mn(OH) 2

Là chất kết tủa màu trắng không tan trong nước nhưng tan khi có mặt muối amoni.

Là một bazo yếu dễ tan trong axit tạo muối Mn (II)

Dễ bị oxi hóa: 2Mn(OH) 2 + O 2 + 2 H 2 O → 2Mn(OH) 4

Mangan (II) clorua MnCl 2

Ở trạng thái khan tạo ra tinh thể hình phiến màu hồng. Nóng chảy ở 650 0 C và trong

luồng H 2 . MnCl 2 bay hơi ở 119 0 C:

MnCl 2 + H 2 O → Mn(OH)Cl + HCl

Có tính khử: 4MnCl 2 + O 2 + 4H 2 O → 2Mn 2 O 3 + 8HCl

Điều chế: MnCO 3 + 2HCl → MnCl 2 + CO 2 +H 2 O

MnO 2 + 4HCl → MnCl 2 + Cl 2 +2H 2 O

Mangan (II) sunfat MnSO 4

7

Phát hành PDF bởi Ths Nguyễn Thanh Tú

Đăng ký Word doc qua Zalo 0905779594 Email thanhtuqn88@gmail.com


Là chất rắn màu trắng ở trạng thái khan. Khi kết tinh từ dung dịch nước tạo ra tinh thể

màu hồng khác nhau phụ thuộc vào hàm lượng nước kết tinh.

MnSO 4 .7H 2 O MnSO 4 .5H 2 O MnSO 4 .4H 2 O MnSO 4 .2H 2 O

Điều chế: Kim loại, oxit, hiđroxit, muối CO 2- 3 của Mn(II) + H 2 SO 4

Hoặc 2MnO 2 + 2H 2 SO 4 → MnSO 4 + 2H 2 O + O 2

Mangan (II) cacbonat

Là chất bột màu trắng, mịn như lông tơ, không tan trong nước (Tt = 1.10 -10 ở 25 0 C). Khi

đun nóng ở 100 0 C bị phân huỷ: MnCO 3 → MnO + CO 2

Để trong không khí ẩm dễ bị oxi hoá thành Mn 2 O 3 màu thẫm

Mangan sunfua

Điều chế: Muối Mn(II) + S 2- MnS hồng thẫm.

Khi để lâu (không có không khí) tạo MnS dạng khan màu xanh. Để trong khôngkhí:

MnS + O 2 + 2H 2 O → S + MnO 2 . 2H 2 O

Không tan trong nước (Tt = 2,5 .10 -10 )

Nhận xét: Trong các phản ứng không thay đổi số oxi hoá thì sự chuyển thành phức chất

cation đặc trưng nhất đối với chúng: MnO + 2H 3 O + + 3H 2 O → [Mn(H 2 O) 6 ] 2+ còn với kiềm

chúng chỉ phản ứng khi đun nóng khá mạnh và lâu Mn(OH) 2 + 4OH - → [Mn(OH) 6 ] 4-

Tuy nhiên tất cả các hiđroxo mangannat (II) đều tự phân huỷ hoàn toàn trong dung dịch nước

nên ở điều kiện thường Mn, oxit, hiđrôxít Mn(II) không tác dụng với kiềm.

- Từ sơ đồ Latime ta thấy trong môi trường axít, trạng thái oxi hoá +2 là bền nhất của

mangan. Muối mangan chỉ bị oxi hoá bởi những chất oxi hoá mạnh như PbO 2 , NaBiO 3 ,

(NH 4 ) 2 S 2 O 8 oxi hoá thành MnO - 4 màu tím.

I.4.2. Hợp chất với số oxi hóa +3.

Mangan III oxit

- Là chất bột màu đen không tan trong nước. Khi đun nóng trong không khí : Mn 2 O 3 (950 –

1100 0 C)→ Mn 3 O 4 và MnO

- Dịch thể Mn 2 O 3 có kiến trúc không phải lập phương mỗi nguyên tử Mn được bao quanh

bởi 4 nguyên tử O vớid Mn-O :1,96A 0 ,hai nguyêntửO:d : 0-0 2,052,25A 0

- Tác dụng với axítđặc:Mn 2 O 3 + H 2 SO 4đ → Mn 2 (SO 4 ) 3 + 3H 2 O

- Tác dụng với axitloãng: Mn 2 O 3 + H 2 SO 4loãng → MnO 2 + MnSO 4 + H 2 O

- Mn 2 O 3 tạo phức chất của Mn(III) khi tan trong HF, HCN, kết hợp với axít MO.Mn 2 O 3

kiểuspinen.

Điều chế: Nung MnO trong không khí ở 550 0 C -900 0 C

Mangan (III) hiđrôxit

- Mangan (III) hiđrôxit không có thành phần ứng đúng công thức Mn(OH) 3 mà là

hiđratMn 2 O 3 .xH 2 O

0

Mn 2 O 3 .xH 2 O 1 000 C Mn 2 O 3 .H 2 O(MnOOH)

MnOOH (monohiđrôxit) là chất dạng tinh thể màu nâu gần như đen, không tan trong nước ở

365 - 400 0 C mất nước thành Mn 2 O 3 .

+ Tác dụng với axít loãng MnO 2 + Mn(II)

+ Với axit hữu cơ Mn(III) bền

Điều chế: MnCO 3 (huyền phù trong nước) + Cl 2 hoặc KMnO 4

3MnCO 3 + Cl 2 + H 2 O → 2MnOOH + MnCl 2 + 3CO 2

8

Phát hành PDF bởi Ths Nguyễn Thanh Tú

Đăng ký Word doc qua Zalo 0905779594 Email thanhtuqn88@gmail.com


MnF 3

Mn 3+ không bền trong dung dịch dễ bị phân huỷ: 2Mn 3+ + 2H 2 O MnO 2 + Mn 2+ + 4H +

Cation Mn 3+ được làm bền trong những phứcchất

- Dạng tinh thể đơn tà màu đỏ, phân huỷ trên 600 0 C thành MnF 2 và F 2 , dễ bị thuỷ phân theo

phảnứng:2MnF 3 + 2H 2 O → MnO 2 + MnF 2 + 4HF

- Dư HF: kết tinh ở dạng MnF 3 .2H 2 O màu đỏ thắm, dễ tạo nên với florua kim loại kiềm

phức chất màu đỏ thẫm như: K[MnF 4 ],K 2 [MnF 5 ]

Điều chế: 2MnI 2 + 3F 2 → 2MnF 3 +2I 2

Mangan III sunfat

Dạng tinh thể màu lục, hút ẩm mạnh, bị thuỷ phân. Phân huỷ ở300 0 C:

2 Mn 2 (SO 4 ) 3 → 4MnSO 4 + 2SO 3 + O 2

Điều chế:4MnO 2 +6H 2 SO 4 → 2Mn 2 (SO 4 ) 3 +6H 2 O + O 2

Mn(CH 3 COO) 3

Dạng tinh thể màu nêu, hút ẩm mạnh, tự thuỷ phân.

Điều chế: dùng Cl 2 hay KMnO 4 oxi hoá Mn(CH 3 COO) 3 trong a xít axetic băng và nóng

I.4.3. Hợp chất với số oxi hóa +4.

Đối với Mn(IV) hợp chất bền là oxit MnO 2 và hiđrôxit Mn(OH) 4 . Các dẫn xuất phức

manganat (IV) kiểu MnF 2- 6 và MnCl 2- 6 cũng tương đối bền, trong khi đó MnF 4 và MnCl 4 lại

dễ bị phân huỷ.

Số phối tử cao nhất của Mn(IV) bằng 6. Những sự nghiên cứu hoá từ chỉ rằng các ion kiểu

[MnHal 6 ] 2- là thuận từ và chứa 3 electron chưa ghép đôi, tương ứng với sự tham gia tạo

thành liên kết của các obitan d 2 sp 3 của nguyên tử trung tâm.Mn(IV) cũng có số phối trí 6

trong các tinh thể đioxit

Mangan đioxit MnO 2

Là chất bột màu đen có thành phần không hợp thức. KhiđunnóngMnO 2 500

0 C Mn 2 O 3

900

0 C Mn 3 O 4 khôngtan trong nước, khi đun nóng với H 2 SO 4 đặc nóng tạo ra O 2 2MnO 2 +

2H 2 SO 4 →2MnSO 4 + O 2 + 2H 2 O

Là chất lưỡng tính. Khi tan trong dung dịch axít theo phản ứng ôxy hoá vì muối Mn 4+ kém

bền: MnO 2 +4HCl → MnCl 2 + Cl 2 +2H 2 O

tan trong kiềm đặc: 2Mn +4 O 2 + 6KOH → K 3 Mn +5 O 4 + K 3 [Mn +3 (OH) 6 ]

MnO 2 có cả tính oxi hoá mạnh và tính khử

Điều chế: Mn(NO 3 ) 2 → MnO 2 +2NO 2

hoặc oxi hoá muối Mn(II) trong môi trường kiềm bằng Cl 2 , HOCl, Br 2 hay điện phân hỗn hợp

MnSO 4 và H 2 SO 4 . MnSO 4 + 2H 2 O → MnO 2 + H 2 SO 4 + H 2

Mangan (IV) hiđroxít

Có màu nâu sẫm, không tan trong nước, có tính lưỡng tính giống MnO 2

Muối Mangan (IV)

MnF 4

Mn 4+ bị thuỷ phân mạnh trong dung dịch nước tạo thành MnO 2 nhưng được làm bền hơn

trong các phức chất. Là chất rắn màu xanh xám, dễ phân huỷ thành MnF 3 và F 2 nên là chất o

xi hoámạnh.

9

Phát hành PDF bởi Ths Nguyễn Thanh Tú

Đăng ký Word doc qua Zalo 0905779594 Email thanhtuqn88@gmail.com


Điều chế : Khi hoà tan MnO 2 trong dung dịch HF đậm đặc.

MnCl 4

Là kết tủa màu nâu đỏ hoặc đen, tồn tại ở nhiệt đô thấp, phân huỷ thành MnCl 2 và Cl 2 ở -

10 o C, ta trong dung môi hữucơ.

Điều chế: bằng cách thêm hỗn hợp CHCl 3 và CCl 4 vào dung dịch màu lục được tạo nên khi

sục khí HCl qua huyền phù MnO 2 trong ete ở - 70 o C.

Mn(SO 4 ) 2

Kết tủa màu đen, tan trong a xit Sunfuric đậm đặc cho dung dịch màu nâu. Khá bền trong a

xit Sunfuric nhưng bị nước phânhuỷ

Điềuchế: 3MnSO 4 + 2KMnO 4 + 8H 2 SO 4 → 5Mn(SO 4 ) 2 + K 2 SO 4 + 8H 2 O

Nói chung các hợp chất Tetrahalogenua của Mangan (IV) kém bền trong nước nhưng

dễ kết hợp với halogenua kim loại kiềm tạo lên những phức chất có màu vàng và bền hơn như

M[MnX 5 ] và M 2 [MnX 6 ] trong đó (M = K, Rb, NH + 4 và X = F, Cl)

I.4.4. Hợp chất với số oxi hóa +6.

Mangan (VI) chỉ biết được trong ion mangarat (MnO 2- 4 ) có màu lục thẫm.

Natrimanganat (Na 2 MnO 4 ) và kalimanganat (K 2 MnO 4 ) là những chất ở dạng tinh thể màu lục

đen, phân huỷ trên 500 o C: 2K 2 Mn +6 O 4 → 2K 2 Mn +4 O 3 +O 2

Manganat kim loại kiềm tan và bền trong dung dịch kiềm nhưng tự phân huỷ trong các

môi trường trung tính và a xit theo phản ứng: 3MnO 2- 4 + 2H 2 O → 2 MnO - 4 + MnO 2 + 4 OH -

vì E o (MnO 2- 4 / MnO 2 ) = 2,26(V) > E o ( MnO - 4 / MnO 2- 4 ) =0,564 (V) nên khi để lâu trong

không khí chứa CO 2 hoặc khi pha loãng bằng nước thì màu lục thẫm trở thành màu tím (của

MnO - 4 ) và kết tủa đen xuất hiện (MnO 2 ).

Muối Manganat là chất oxi hoá mạnh, phản ứng với những chất khử ở trong dung dịch

xảy ra tương tự như Pemanganat. Trong môi trường kiềm sẽ bị khử đến MnO 2 , còn trong môi

trường a xít tạo ra muối Mn(II)

K 2 MnO 4 + 2H 2 S +2H 2 SO 4 → 2S + MnSO 4 + K 2 SO 4 + 4H 2 O

K 2 MnO 4 + 2 Fe(OH) 2 + 2H 2 O → MnO 2 + 2Fe(OH) 3 + 2 KOH

Nhưng khi tác dụng với chất oxi hoá mạnh hơn, manganat thể hiện tính khử

2K 2 MnO 4 +Cl 2 → 2KMnO 4 + 2KCl

Điềuchế: 2MnO 2 + 4KOH + O 2 → 2K 2 MnO 4 +2H 2 O

I.4.5. Hợp chất với số oxi hóa +7.

Oxit pemanganic (Mn 2 O 7 )

Oxit pemanganic (Mn 2 O 7 ) ở nhiệt độ thấp là chất ở dạng tinh thể màu lục thẫm, bền ở

dưới - 5 0 C, nóng chảy ở 6 0 C biến thành chất lỏng giống dầu có màu đỏ thẫm. ở 10 0 C, nó phân

huỷ nổ:Mn 2 O 7 → 2MnO 2 + O 3

Tan trong nước tạo thành dung dịch axít pemanganic nên còn được gọi là anhiđrit

pemanganic là chất oxi hoá rất mạnh, tác dụng với nhiều chất vô cơ và hữu cơ

2Mn 2 O 7 + 2(C 2 H 5 ) 2 O + 9O 2 → 4MnO 2 + 8CO 2 + 10H 2 O

Điều chế: KMnO 4 + H 2 SO 4 → HMnO 4 + KHSO 4

2HMnO 4 H sSO 4 Mn 2 O 7 +H 2 O

10

Phát hành PDF bởi Ths Nguyễn Thanh Tú

Đăng ký Word doc qua Zalo 0905779594 Email thanhtuqn88@gmail.com


Axít pemanganic (HMnO 4 )

Axít pemanganic (HMnO 4 ) chỉ biết được trong dung dịch nước, có màu tím đỏ, tương

đối bền trong dung dịch loãng nhưng phân huỷ khi dung dịch có nồng độ trên 20%.

2HMnO 4 → 2MnO 2 + O 3 + H 2 O

Axít pemanganic là axít mạnh, muối của nó là pemangan (MnO - 4). Muối pemanganat

bền hơn axít, đồng hành với peclorat nhưng dễ phân huỷ hơn khi đun nóng. Những tinh thể

hiđrat như LiMnO 4 .3H 2 O, NaMnO 4 .3H 2 O... tan nhiều trong nước trong khi các muối khan

NH 4 MnO 4 , KMnO 4 ,... tan íthơn.

Axít pemanganic và muối pemanganat đều là chất oxi hoá mạnh.

Điều chế: Mn 2 O 7 + H 2 O → 2HMnO 4

Kali pemanganat (KMnO 4 ) là chất ở dạng tinh thể màu tím đen, đồng hình với KClO 4 ,

BaSO 4 và BaCrO 4 . Tan trong nước cho dung dịch có màu tím- đỏ, có độ tan biến đổi tương

đối nhiều theo nhiệt độ nên tinh chế được dễ dàng khi kết tinh lại. Ngoài ra nó còn có thể tan

trong amoniắc lỏng, pyriđin, rượu vàaxeton.

Trên 200 0 C, phân huỷ theo phản ứng:

2KMnO 4 → K 2 MnO 4 + MnO 2 + O 2

Kalipemanganat có tính oxi hoá mạnh nên được dùng làm chất oxi hoá trong tổng hợp

vô cơ và hữu cơ, dùng để tẩy trắng vải, dầu, mỡ sát trùng trong y học và đời sống. Khả năng

oxi hoá của KMnO 4 phụ thuộc mạnh vào môi trường của dung dịch.

Axít: MnO - 4 + 8H + + 5e → Mn 2+ +4H 2 O E 0 =1,51V

Trung tính: MnO - 4 + 2H 2 O + 3e → MnO 2 +4OH - E 0 = 0,588V

Kiềm: MnO - 4 + e→MnO 2- 4 E 0 = 0,56V

Trong dung dịch axít ion MnO - 4 có thể oxi hoá nhiều chất như HCl, H 2 S, PH 3 , Na 2 SO 3 ,

FeSO 4 , HCOOH,... và biến thành Mn 2+ như:

2KMnO 4 + 5Na 2 SO 3 + 3H 2 SO 4 → 2MnSO 4 + K 2 SO 4 + 5Na 2 SO 4 + 3H 2 O

2KMnO 4 + 5H 2 O 2 + 4Na 2 SO 4 → 2MnSO 4 + 2KHSO 4 + 8H 2 O + 5O 2

Ngay khi không có chất khử, dung dịch KMnO 4 không bền, có thể phân huỷ theo phản

ứng: 4MnO - 4 + 4H + → 3O 2 + 4MnO 2 + 2H 2 O

Phản ứng xảy ra chậm, trong dung dịch axít nhưng khá rõ rệt. Trong dung dịch trung

tính hay kiềm yếu và trong bóng tối, phản ứng xảy ra gần như không đáng kể. ánh sáng thúc

đẩy phản ứng phân huỷ đó nên cần đựng dung dịch KMnO 4 chuẩn trong lọ thuỷ tinh có màu

thẫm.

Điều chế: Điện phân dung dịch K 2 MnO 4 với các điện cực bằng thép

2K 2 MnO 4 + 2H 2 O → 2KMnO 4 + 2KOH + H 2

Như vậy, qua sơ đồ thế điện cực oxi hoá khử của Mn, chúng ta nhận thấy:

- Ion MnO - 4 bị khử đến Mn 2+ 4

trong môi trường axít đến MnO 2 trong môi trườngtrung tính và

đến MnO 2- trong môi trường kiềm. Khả năng oxi hoá của các ion MnO - 4 và MnO 2- 4 đến

MnO 2 trong môi trường axít đều lớn hơn trong môi trường kiềm. Ngay trong môi trường axít,

khi tăng nồng độ của ion H + , hoạt tính oxi hoá của ion MnO - 4 cũng tăng lên

- Có lẽ rằng trong môi trường axít MnO 2- 4 và MnO - 4 đã được proton hoá tạo thành HMnO 4 và

11

Phát hành PDF bởi Ths Nguyễn Thanh Tú

Đăng ký Word doc qua Zalo 0905779594 Email thanhtuqn88@gmail.com


HMnO - 4 và có thể cả H 2 MnO 4 (giả thiết). Những phân tử và anion một mặt có cấu tạo kém

đối xứng hơn so với anion tứ diện đều MnO - 4 nên dễ tiếp xúc với chất khử hơn và quá trình

phản ứng được đẩy mạnh. Mặt khác, chúng đều kém bền, dễ mất nước tạo thành anhiđrit có

cấu tạo còn kém đối xứng hơn nữa so với phân tử axít hay anion tứ diện đều cho nên càng dễ

tiếp xúc hơn với chất khử và phản ứng càng được đẩy mạnh hơn. Đó là lý do chung làm cho

oxi axít có tính oxi hoá mạnh hơn muối của nó và anhiđrit axít có tính oxi hoá mạnh hơn axít

ở cùng điều kiện.

Nhận xét chung về các hợp chất của Mn

Mangan có khả năng tạo được oxít và các hiđrôxit ứng với bậc oxi hoá từ thấp đến cao

và sự biến đổi bậc oxi hoá ảnh hưởng đến tính chất của chúng.

Mn +2 O Mn +3 2O 3 Mn 4 O 2 Mn +6 O 3 Mn +7 2O 7

Tính a xít tăng dần

Mn(OH) 2 Mn(OH) 3 Mn(OH) 4 H 2 MnO 4 HMnO 4

Tính a xit tăng dần

Tính a xít tăng theo dãy trên có thể giải thích trong khuôn khổ sự tương quan điện tích

và bán kinhs của các ion. Khi chuyển từ Mn(II) đến Mn(VII), bán kính của ion giảm dần, điện

tích của ion tăng dần như sau:

Ion R ion (A 0 )

Mn 2+ 0,91

Mn 3+ 0,70

Mn 4+ 0,52

Mn 7+ 0,46

Làm cho mật độ điện tích (+) của nguyên tố trung tâm Mn tăng dần, lực hút của Mn

với O tăng lên, độ dài liên kết Mn – O giảm xuống Độ bền liên kết Mn – O tăng lên Độ

phân cực của liên kết Mn – O giảm Khả năng tách H + tăng dần, H + càng dễ bị tách ra nên

tính a xít tăng dần. Mặt khác, do sự tăng số oxi hoá từ Mn(II) đến Mn(VI) cũng giống như sự

tăng số oxi hoá của Cl trong các oxi a xít làm cho tính a xít của các hiđrôxit của chúng tăng

dần, tính bazơ giảm dần.

Mn + (OH) 2 Mn +3 (OH) 3 Mn +4 (OH) 4 H 2 Mn +6 O 4 HMn +7 O 4

Bazơ Bazơ yếu Lưỡng tính Axít Axít mạnh

Sự tăng số oxi hoá Mn(II) lên Mn(VII) làm cho độ âm điện tăng vì vậy hiệu độ âm

điện của Mn và o ( = 0 - Mn ) giảm dần làm cho độ phân cực của liên kết Mn – O giảm

nên mối liên kết O – H dễ bị tách ra, tính axít tăngdần.

Ở trạng thái hoá trị cao nhất ứng với số thứ tự nhóm, Mangan có tính chất hoá học gần

với tính chất hoá học của Clo ở cùng nhóm VII nhưng khác phân nhóm. ở đây, cả Clo và

mangan đều sử dụng cả 7 electron hoá trị để hình thành liên kết hoá học, cả Clo (VII) và

mangan (VII) đều có cấu hình electron của khítrơ.

Cl(VII): 1s 2 2s 2 2p 6 Mn(VII):1s 2 2s 2 2p 6 3s 2 3p 6

12

Phát hành PDF bởi Ths Nguyễn Thanh Tú

Đăng ký Word doc qua Zalo 0905779594 Email thanhtuqn88@gmail.com


Sự giống nhau về cấu hình electron dẫn tới sự giống nhau về tính chất của các hợp chất

ứng với hoá trị VII của mangan và Clo. Chẳng hạn Mn 2 O 7 và Cl 2 O 7 đều là chất lỏng ở điều

kiện thường, đều kém bền, đều là anhiđrit của a xítmạnh.Hai axít pemanganic HMnO 4 và

pecloric HClO 4 và các muối tương ứng chẳng hạn KMnO 4 và KClO 4 đều là những chất oxi

hoá mạnh. Trái lại, ở trạng thái oxi hoá càng thấp, tính chất hoá học của Mangan càng khác

xa tính chất hoá học của Cl. Đó là vì ở trạng thái oxi hoá thấp, ion mangan còn lại các

electron (n – 1)d còn đối với Clo thì còn lại các electron s và p. Vì vậy các hợp chất của Clo

và Mangan ứng với các trạng thái hoá trị thấp có tính chất khácnhau.

Ví dụ: Cl 2 O: trạng thái khí là anhiđrit của axit hipoClorơ HClO

Độ bền của ion MnO - 4 lớn hơn độ bền của ion MnO 2- 4 cũng được giải thích trong

phạm vi của thuyết obitan phân tử như sau:

Trong ion MnO - 4 chứa 24 electron hoá trị gồm: Mn: 7 electron của nguyên tử Mn (3d 5 4s 2 ), 16

electron của 4 nguyên tử O (2p 4 ) và 1 electron điện tích của ion.

Ion MnO 2- 4 chứa đến 25 electron có điện tích lớn hơn điện tích của ion MnO - 4 một đơn vị. Sự

phân bố electron hoá trị trên các obitan phân tử của ion MnO - 4 và MnO 2- 4 tương ứng với các

cấu hình sau:

MnO - 4 : [ lk ] 8 [ lk ] 10 [] 6

MnO 2- 4 : [ lk ] 8 [ lk ] 10 [] 6 [ plk ] 1

Như vậy, khác với MnO - 4 , trong ion MnO 2- 4 có một electron chiếm obitan phân tử (MO) phản

liên kết nên ion MnO 2- 4 phải kém bền hơn ion MnO - 4. Các ion MnO 3- 4 , MnO 4- 4 còn kém bền

hơn nữa vì có 2 và 3 electron chiếm MO phản liên kết. Tính không bền của số phối trí 4 trong

hợp chất Mn(IV) cũng được giải thích như vậy.

I. 5. Điềuchế

I.5.1. Điều chếMangan.

Mangan được điều chế bằng phương pháp nhiệt nhôm từ MnO hoặc Mn 3 O 4 :

3Mn 3 O 4 +8Al

t 0

4Al O +9Mn

2 3

Cũng có thể điều chế bằng phương pháp nhiệt Silic:

MnO 2 +Si

t 0

Mn+SiO . 2

Trong công nghiệp Mangan được điều chế bằng phương pháp điện phân muối Sunfat.

Mangan tinh khiết được điều chế bằng cách điện phân dung dịch MnCl 2 với catốt bằng

thuỷ ngân . Mangan hoà tan trong thuỷ ngân tạo thành hỗn hợp Mn - Hg. Chưng cất hỗn hợp

trong chân không tách được Mn và thu hồi lại Hg.

I.5.2. Điều chếtecnexi.

Người ta điều chế một lượng nhỏ tecnexi (vài mg) trong lò phản ứng hạt nhân khi bắn

phá oxit molipđen bằngnơtron

Để điều chế lượng lớn hơn, Tc được tách ra từ chất thải của lò phản ứng nguyên tử .

Tecneti là một nguyên tố mảnh trong những sản phảm phân chi hạt nhân của uran, sau khi

tách Tc ra dưới dạng kết tủa NH 4 TcO 4 , người ta khử NH 4 TcO 4 bằng khí H 2 ở nhiệt độ cao sẽ

t

được bột tecnexi kim loại: 2NH 4 TcO 4 + 7 H 2

0

2 Tc + 8H 2 O + 2NH 3

I.5.3. Điều chế Reni

13

Phát hành PDF bởi Ths Nguyễn Thanh Tú

Đăng ký Word doc qua Zalo 0905779594 Email thanhtuqn88@gmail.com


Reni là nguyên tố trơ hơn mangan nên có thể điều chế bằng phản ứng khử reni oxit hay amoni

perenat ( NH 4 ReO 4 ) với hiđrô ở 673 – 873 0 K.

2NH 4 ReO 4 +4H 2

t 0

Re+8H O+N 2 2

Amoni perenat (NH 4 ReO 4 ) được điều chế từ sản phẩm thu hồi của lò đốt molipđenrit.

Khí Hiđrô cũng coá thể tác dụng với Kaliperenat tạo thành bột kimloại màu xám. Sau khi

phản ứng kết thúc người ta phải xử lý sản phẩm với axit loãng để loại kalihiđrôxit.

t

2KReO 4 + 7H 2

0

2KOH +2Re+6H 2 O.

Renni còn được điều chế bằng nhiệt phân các muối halogenua hoặc khử muối sunfua

II. SƠ LƯỢC VỀ NGUYÊN TỐ KIM LOẠI NHÓM VIIIB

Nhóm VIIIB bao gồm 9 nguyên tố xếp trong 3 cột: sắt (Fe), ruteni (Ru) và osmi (Os); coban

(Co), rođi (Rh) và iriđi (Ir); niken (Ni), padi (Pd) và platin (Pt). Dựa vào đặc điểm giống

nhau của các nguyên tố mà chúng được chia ra làm 2 họ: họ sắt gồm Fe, Co, Ni và họ platin

gồm Ru, Rh, Pd, Os, Ir và Pt. Trong chuyên đề này chỉ xét các nguyên tố họ sắt.

II.1. Đặc điểm chung nhómVIIIB

Trong chuyên đề này, chỉ xét các nguyên tố họ sắt, chúng có một số đặc điểm sau:

Đặc điểm Fe Co Ni

Số thứ tự 26 27 28

Cấu hình e - hoá trị 3d 6 4s 2 3d 7 4s 2 3d 8 4s 2

Bán kính nguyên tử R(A 0 ) 1,26 1,25 1,24

Bán kính ion R 2+ (A 0 ) 0,80 0,78 0,74

Bán kính ion R 3+ (A 0 ) 0,67 0,64 -

Năng lƣợng ion hoá I 1 (eV) 7,90 7,86 7,5

Thế điện cực chuẩn E 0 M 2+ /M(V) - 0,44 - 0,28 - 0,23

Thế điện cực chuẩnE 0 M 3+ /M 2+ (V) +0,77 +1,81 +2,1

Fe, Co và Ni có vỏ electron ngoài cùng giống nhau: 4s 2 , bán kính nguyên tử giảm dần theo

chiều tăng số electron điền vào các orbital 3d, do có cùng số lớp electron như nhau, khi điện

tích hạt nhân tăng, các electron được hút mạnh hơn làm giảm bán kính nguyên tử.

Trạng thái oxi hóa đặc trưng của Fe, Co, Ni là +2 và +3.

II.2 Tính chất hóa học

Fe, Co và Ni là những kim loại hoạt động hóa học trung bình, hoạt tính giảm từ Fe đến Ni.

Ở điều kiện thường, không có hơi ẩm, Fe, Co và Ni không tác dụng rõ rệt ngay với những

nguyên tố phi kim điển hình như O 2 , S, Cl 2 , Br 2 vì nó có màng oxit bảo vệ. Nhưng khi đun

nóng thì phản ứng xảy ra mãnh liệt.

Fe, Co và Ni bền với khí F 2 ở nhiệt độ cao vì florua của chúng không bay hơi, trong khi đó

kim loại Fe phản ứng dễ dàng với khíCl 2 .

Với N 2 , cả 3 kim loại tác dụng ở nhiệt độ không cao lắm tạo Fe 2 N, CoN và Ni 3 N 2 . Những

nitrua này phân hủy ở nhiệt độ cao hơn, nhưng trong kim loại vẫn còn lại một lượng nitơ đáng

kể ở dạng dung dịchrắn.

Cả 3 kim loại tác dụng với S khi đun nóng nhẹ tạo những hợp chất không hợp thức có thành

phần gần với MS (M = Fe, Co,Ni).

14

Phát hành PDF bởi Ths Nguyễn Thanh Tú

Đăng ký Word doc qua Zalo 0905779594 Email thanhtuqn88@gmail.com


Với khí CO thì Fe, Co và nhất là Ni tác dụng trực tiếp tạo cacbonyl kim loại.

Fe + 5CO → [Fe(CO) 5 ]

Co + 8CO → [Co(CO) 8 ]

Ni + 4CO →[Ni(CO 4 )]

Fe, Co, Ni tinh khiết đều bền với không khí và nước. Nhưng Fe có chứa tạp chất bị ăn mòn

dần bởi hơi ẩm, CO 2 và O 2 trong không khí tạo gỉ sắt. 2Fe + 3/2 O 2 + nH 2 O →Fe 2 O 3 .nH 2 O

II. 3. Các hợp chất của Fe, Co, Ni

II.3.1.Hợp chấtcacbonyl

Các nguyên tố họ sắt tạo được các hợp chất M(0) bằng liên kết cho nhận.

II.3.1.1.Sắt pentacacbonyl(Fe(CO) 5 )

Fe(CO) 5 ) là chất lỏng màu vàng, hóa rắn ở -20 0 C và sôi ở 103 0 C, rất độc. Phân tử có tính

nghịch từ, nguyên tử Fe trong phân tử có cấu hình 3d 8 và ở trạng thái lai hóadsp 3 . Fe(CO) 5

không tan trong nước nhưng tan trong rượu, ete, axeton, benzen. Trong dung dịch ete, bị

phân hủy ở nhiệt độ thường bởi tia tử ngoại. 2Fe(CO) 5 → Fe 2 (CO) 9 + CO

Phân hủy khi đun nóng ở 200-250 0 t

C không có không khí: Fe(CO) 5

0

Fe+5CO

Trong dung dịch ete, Fe(CO) 5 tác dụng mãnh liệt với axit H 2 SO 4 đặc

Fe(CO) 5 + H 2 SO 4 → FeSO 4 + 5CO + H 2

và tác dụng với halogen tạo Fe(CO) 5 X 2 kém bền dễ chuyển thành Fe(CO) 4 X 2

Tác dụng với dung dịch kiềm mạnh và đặc tạo H 2 Fe(CO) 4 tự bốc cháy trong không khí.

Fe(OH) 5 + Ba(OH) 2 → H 2 Fe(CO) 4 + BaCO 3

Khi đun nóng ở 45 0 C với khí NO dưới áp suất, NO có thể thay thế hoàn toàn CO tạo sắt

tetranitrozylFe(NO) 4 .

II.3.1.2.Coban octacacbonyl (Co 2 (CO) 8 )

Co 2 (CO) 8 : tinh thể trong suốt, màu đỏ - da cam. Phân tử 2 nhân, có tính nghịch từ, có cấu tạo:

Mỗi nguyên tử Co tạo nên 6 liên kết: 4 liên kết ( cho - nhận từ CO, 1 liên kết cho - nhận từ

electron d của Co đến MO (trống của CO và 1 liên kếtCo-Co).

Do có số lẻ electron nên Co tạo hợp chất cacbonyl ở dạng đime [Co(CO) 4 ] 2 . Co 2 (CO) 8 nóng

chảy ở 51 0 t

C, trên nhiệt độ đó thì phân hủy: 2Co 2 (CO) 8

0

Co 4 (CO) 12 + 4 CO

Trên 60 0 C thì phân hủy thành kim loại Co và CO (do Co 4 (CO) 12 phân hủy). Tan trong rượu

và ete nhưng bị nước phân hủy:

3Co 2 (CO) 8 + 4H 2 O → 4HCo(CO) 4 + 2Co(OH) 2 + 8CO

Tác dụng với dung dịch kiềm:

6Co 2 (CO) 8 + 8NaOH → 8HCo(CO) 4 + 4Na 2 CO 3 + Co 4 (CO) 12

(HCo(CO) 8 : axit tetracacbonyl cobantic - chất lỏng màu vàng, hóa rắn ở -26,2 0 C và sôi ở

10 0 C).

II. 3.1.3. Niken tetracacbonyl (Ni(CO) 4 )

Ni(CO) 4 : chất lỏng không màu, rất dễ bay hơi và rất độc. Phân tử có cấu hình từ diện đều.

Phân tử có tính nghịch từ, nguyên tử Ni trong phân tử có cấu hình 3d 10 và lai hóa sp 3 .

Ni(CO) 4 hóa rắn ở - 23 0 C và sôi ở 43 0 C. Dưới tác dụng của tia tử ngoại hoặc khi đun nóng ở

180-200 0 C, nó phân hủy hoàn toàn thành kim loại Ni và CO.

15

Phát hành PDF bởi Ths Nguyễn Thanh Tú

Đăng ký Word doc qua Zalo 0905779594 Email thanhtuqn88@gmail.com


Không tan trong nước nhưng tan trong ete, clorofom, benzen. Trong không khí, Ni(CO) 4 bị

oxi hoá dần thành NiO và CO 2 : 2Ni(CO 4 ) + 5O 2 → 2NiO + 8CO 2

Dễ dàng tác dụng với halogen: Ni(CO) 4 + Cl 2 → NiCl 2 + 4CO

Không tác dụng với dung dịch axit loãng và kiềm nhưng tác dụng mạnh với axit đặc H 2 SO 4

và HNO 3 tạo muối Ni 2+ (có thể gây nổ)

Tương tự Mg(OH) 2 , các M(OH) 2 tan trong dung dịch đặc của muối NH + 4 :

M(OH) 2 + 2NH 4 Cl đặcnóng → MCl 2 + 2NH 3 + 2H 2 O

Co(OH) 2 và Ni(OH) 2 tan được trong dung dịch NH 3 tạo phức:

Co(OH) 2 + 6NH 3 → [Co(NH 3 ) 6 ](OH) 2 (vàng)

Ni(OH) 2 + 6NH 3 → [Ni(NH 3 ) 6 ](OH) 2 (chàm)

Muối của axit mạnh như Cl - , NO - 3 , SO 2- tan 4 dễ trong nước tạo các ion bát diện

[M(H 2 O) 6 ] 2+ có màu đặc trưng: [Fe(H 2 O) 6 ] 2+ màu lục nhạt, [Co(H 2 O) 6 ] 2+ màu đỏ hồng,

[Ni(H 2 O) 6 ] 2+ màu lục. Các muối của axit yếu như S -2 , CO 2- 3 , CN - , C 2 O 2- 3-

4 , PO 4 khó tan

trongnước.

II.3.2. Sự tạophức

Các ion M 2+ tạo nhiều phức chất, độ bền của các phức tăng theo chiều giảm bán kính ion từ

Fe 2+ đến Ni 2+ (Fe 2+ : 0,74A 0 ; Co 2+ : 0,72 A 0 ; Ni 2+ :0,69 A 0 ).

Các M 2+ đều tạo phức bát diện với số phối trí là 6. Ion Fe 2+ ít có khuynh hướng tạo phức tứ

diện hơn Co 2+ và Ni 2+ . Co 2+ tạo được nhiều phức tứ diện nhất do những phức đó có cấu hình

electron bền ((d*)4 ((d*)3. Ngoài phức tứ diện, Ni 2+ còn tạo được phức hình vuông với phối

tử trường mạnh.

* Phức amoniacat: Các muối M 2+ khan kết hợp với khí NH 3 tạo muối phức amoniacat chứa

ion bát diện [M(NH 3 ) 6 ] 2+ . Amoniacat sắt (II) kém bền, trong nước bị phân hủy tạo hidroxit.

Ví dụ: [Fe(NH 3 ) 6 ]Cl 2 + 2H 2 O → Fe(OH) 2 + 2NH 4 Cl + 4NH 3

[Co(NH 3 ) 6 ] 2+ có màu nâu vàng, [Ni(NH 3 ) 6 ] 2+ có màu tím.

Trong dung dịch, [Co(NH 3 ) 6 ] 2+ dễ bị oxi hoá bởi oxi không khí:

4[Co(NH 3 ) 6 ] 2+ + O 2 + 2H 2 O → 4[Co(NH 3 ) 6 ] 3+ + 4OH -

* Phức xianua:

[Fe(CN) 6 ] 4- : màu vàng, [Co(CN) 6 ] 4- : màu đỏ, [Ni(CN) 6 ] 2- : phức hình vuông. [Fe(CN) 6 ] 4-

làphứcbềnnhấtcủaFe +2 ,còn[Co(CN) 6 ] 4- kémbền,dễbịoxihóatrong không khí.

4K 4 [Co(CN) 6 ] + O 2 + 2H 2 O → 4K 3 [Co(CN) 6 ] + 4KOH

2K 4 [Co(CN) 6 ] + 2H 2 O → 2K 3 [Co(CN) 6 ] + 2KOH +H 2

Phức xianua được tạo ra khi cho muối M( +2 ) tác dụng với dung dịch xianua kim loại kiềm,

ban đầu tạo kết tủa M(CN) 2 , sau đó kết tủa tan trong xianua dư tạo phức

Vídụ: FeSO 4 + 2KCN → Fe(CN) 2 + K 2 SO 4

Fe(CN) 2 + 4KCN →K 4 [Fe(CN) 6 ]

kali feroxianua

II.3.3. Hợp chất M +2

Từ Fe +2 đến Ni +2 độ bền tăng,

Hợp chất M +2 có tính khử mạnh, giảm từ Fe +2 đến Ni +2 .

16

Phát hành PDF bởi Ths Nguyễn Thanh Tú

Đăng ký Word doc qua Zalo 0905779594 Email thanhtuqn88@gmail.com


Các ion Fe 2+ , Co 2+ và Ni 2+ tạo nên nhiều phức chất, độ bền của những phức chất đó tăng

lên theo chiều giảm của bán kính ion từ Fe 2+ (0,74Å), Co 2+ (0,72Å) đến Ni 2+ (0,69Å). Cả ba

ion đều tạo nên phức bát diện với số phối trí 6. Ion Fe 2+ ít có khuynh hướng tạo nên phức chất

tứ diện hơn các ion Co 2+ và Ni 2+ . Trong các kim loại chuyển tiếp, ion Co 2+ tạo nên số phức

chất tứ diện nhiều nhất. Sở dĩ như vậy là vì những phức chất tứ diện đó có cấu hình electron

bền (π * d ) 4 (σ * d ) 3 . Ngoài số ít phức chất tứ diện được tạo nên với phối tử trường yếu, ion

Ni 2+ còn tạo nên nhiều những phức chất hình vuông với phối tử trườngmạnh.

II.3.3. Hợp chất M +3

Trạng thái oxi hóa + 3 kém đặc trưng dần từ Fe đến Ni. Số hợp chất Fe +3 gần tương đương

với số hợp chất của Fe +2 trong hợp chất đơn giản cũng như trong phức chất. Co +3 có trong

nhiều phức chất bền nhưng có rất ít trong hợp chất đơn giản kém bền. Ni +3 không tạo muối

đơn giản và có rất ít phức chất.

Các ferit MFeO 2 (M: kim loại kiềm) thủy phân mạnh trong dung dịch.

Ví dụ: NaFeO 2 + 2H 2 O → Fe(OH) 3 ↓ + NaOH

Các M(OH) 3 tan dễ dàng trong dung dịch axit, Fe(OH) 3 tạo muối Fe 3+ còn Co(OH) 3 và

Ni(OH) 3 là chất oxi hóa mạnh nên khi tan trong axit HCl giải phóng Cl 2 , trong các axit khác

giải phóng khí O 2 và tạo muối Co 2+ ,Ni 2+ .

Vídụ: 2Ni(OH) 3 + 6HCl → 2NiCl 2 + Cl 2 +6H 2 O

Muối M +3

Đa số muối Fe +3 dễ tan trong nước cho ion [Fe(H 2 O) 6 ] 3+ . Khi kết tinh từ dung dịch, muối Fe +3

thường ở dạng hidrat có màu sắc như FeCl 2 .6H 2 O màu nâu vàng, Fe 2 (SO 4 ) 3 .10H 2 O màu

vàng...

Muối Fe +3 thủy phân mạnh hơn muối Fe +2 , dung dịch có màu vàng nâu và phản ứng axit

mạnh.

[Fe(H 2 O) 6 ] 3+ + H 2 O → [Fe(OH)(H 2 O) 5 ] 2+ + H 3 O +

[Fe(OH)(H 2 O) 5 ] 2+ + H 2 O → [Fe(OH) 2 (H 2 O) 4 ] + + H 3 O +

Khi thêm kiềm hoặc đun nóng dung dịch, phản ứng thủy phân xảy ra đến cùng tạo Fe(OH) 3 .

Dung dịch muối dễ dàng bị khử bởi các chất khử như I - , S 2- , Sn 2+ , S 2 O 2- 3 ...

Vídụ: Fe 2 (SO 4 ) 3 + 6KI → 2FeI 2 + I 2 +3K 2 SO 4

2FeCl 3 + H 2 S → 2FeCl 2 + S + 2HCl

Phức chất của M +3

Fe +3 và Co +3 tạo nên khá nhiều phức, đa số là phức bát diện như [FeF 6 ] 3- , [Fe(SCN) 6 ] 3- ,

[Fe(CN) 6 ] 3- , [Co(NH 3 ) 6 ] 3+ , [Co(CN) 6 ] 3- , [Co(NO 2 ) 6 ] 3- ...

* Những phức quan trọng như:

Kali ferixianua (K 3 [Fe(CN) 6 ]): là một thuốc thử thông dụng, dễ tan trong nước cho dung

dịch màu vàng, là hợp chất rất độc. [Fe(CN) 6 ] 3- thường được dùng để nhận biết ion Fe 2+ trong

dung dịch:FeCl 2 + K 3 [Fe(CN) 6 ] → KFe[Fe(CN) 6 ] + 2KCl

màu xanh chàm (xanh tuabin)

Natri hexanitrocobantat (Na 3 [Co(NO 2 ) 6 ]): cũng là một thuốc thử thông dụng, dễ tan trong

nước, được dùng để định lượng K + , Rb + và Cs + vì những hợp chất K 3 [Co(NO 2 ) 6 ],

17

Phát hành PDF bởi Ths Nguyễn Thanh Tú

Đăng ký Word doc qua Zalo 0905779594 Email thanhtuqn88@gmail.com


Rb 3 [Co(NO 2 ) 6 ] và Cs[Co(NO 2 ) 6 ] là kết tủa màu vàng ít tan trong nước, rượu và ete.

So sánh hợp chất M +2 và M 3+

Các hợp chất M +2 bền hơn các hợp chất M +3 .

Từ Fe +2 đến Ni +2 độ bền tăng và từ Fe +3 đến Ni +3 độ bền giảm, do sự tăng độ bền của cấu hình

electron theo thứ tự 3d 6 (Fe 2+ ) - 3d 7 (Co 2+ ) - 3d 8 (Ni 2+ ).

Hợp chất M +2 có tính khử mạnh, giảm từ Fe +2 đến Ni +2 . Hợp chất M +3 có tính oxi hóa mạnh

tăng từ Fe +3 đến Ni +3 . M +3 có khả năng tạo phức bền nhiều hơn so với M +2 .

18

Phát hành PDF bởi Ths Nguyễn Thanh Tú

Đăng ký Word doc qua Zalo 0905779594 Email thanhtuqn88@gmail.com


Chương 2. MỘT SỐ BÀI TẬP KIM LOẠI NHÓM VIIB VÀ NHÓM VIIIB

I. Bài tập lý thuyết.

I.1. Nhóm VIIB

Bài 1

a) Từ MnO bằng phương pháp nào có thể thu được Mn(OH) 2 biết rằng MnO

không tan trongnước?

b) Bằng phản ứng nào chứng minh rằng Mn(OH) 2 có tínhkhử?

Bài 2

Viết phương trình của các phản ứng sau:

1) MnSO 4 + KClO 3 + KOH (nóng chảy)→

2) MnSO 4 + PbO 2 + HNO 3 →

3) MnSO 4 + Br 2 + NaOH→

4) MnBr 2 + H 2 O 2 + KOH→

5) MnSO 4 + CaOCl 2 + NaOH→

Bài 3: Mô tả công thức cấu tạo của MnCl 3 .4H 2 O biết rằng hợp chất đó có cấu hình tám mặt

trong đó 4 phân tử nước cũng tham gia hình thành liên kết.

Bài 4

a) MnO 2 là chất oxi hóa mạnh nhưng khi tác dụng với chất oxi hóa mạnh hơn thì MnO 2

thể hiện tính khử. Tìm dẫn chứng để minh họa cho kết luậnđó.

b) Viết phương trình của các phản ứngsau:

1) MnO 2 + HCl→

2) MnO 2 + NaOH đặc→

3) MnO 2 + H 2 SO 4 →

Bài 5:

a) Từ MnO 2 bằng phản ứng nào có thể thu được: MnCl 2 , KMnO 4 ,Mn 2 O 7 ?

b) Từ MnO 2 điều chế Ba(MnO 4 ) 2 . Viết các phương trình phảnứng.

Bài 6

a) Các ion MnO 2- 4 và MnO - 4 bền trong môi trường nào? Giải thích nguyênnhân.

b) Thêm từ từ từng giọt dung dich NaOH cho đến môi trường kiềm vào một dung dịch

KMnO 4 sau đó cho thêm từng giọt H 2 SO 4 loãng cho đến môi trường axit. Hãy nêu các quá

trình xẩy ra trong quá trình trên và giải thích nguyênnhân.

Bài 7:

a) ViếtphươngtrìnhphảnứngmôtảtínhoxihóavàtínhkhửcủaK 2 MnO 4 .

b) Có thể thu được H 2 MnO 4 bằng phương pháp cho H 2 SO 4 đặc tác dụng với muối

K 2 MnO 4 đượckhông?

Bài 8:

a) Từ phản ứng giữa KMnO 4 với K 2 SO 3 hãy mô tả tính oxi hóa của KMnO 4 trong các môi

trường axit, bazơ, trungtính.

b) Có thể điều chế HMnO 4 bằng cách cho H 2 SO 4 tác dụng với muối tương ứng

được không?

Bài 9:

a) Tính chất của Mn 2 O 7 ? So sánh với tính chất củaCl 2 O 7 ?

19

Phát hành PDF bởi Ths Nguyễn Thanh Tú

Đăng ký Word doc qua Zalo 0905779594 Email thanhtuqn88@gmail.com


b) Phương pháp điều chế Mn 2 O 7 ? So sánh với phương pháp điều chếCl 2 O 7 ?

Bài 10:Viết các phương trình phản ứng sau:

1) KMnO 4 + MnCl 2 →

2) K 2 MnO 4 + Cl 2 →

3) KMnO 4 + KI + H 2 SO 4 →

4) KMnO 4 + KI + H 2 O→

5) KMnO 4 + FeSO 4 + H 2 SO 4 →

Bài 11:Viết các phương trình phản ứng sau đây dưới dạng phân tử:

1) Mn 2+ + ClO - + OH - →

2) MnO - 4 + NO - 2 + H + →

3) MnO - 4 + Fe + H + →

4) Mn 2 -

+ BrO 3 + H 2 O →

5) MnO - 4 + H 2 O 2 + OH - →

Bài 12: So sánh các phản ứng sau đây:

1) KMnO 4 + HCl→

2) K 2 Cr 2 O 7 + HCl→

3) PbO 2 + HCl→

Phản ứng nào xảy ra nhanh hơn? Muốn điều chế một lượng nhỏ khí Clo nên dùng phản ứng

nào?

Bài 13:Có ba dung dịch K 2 CrO 4 , K 2 MnO 4 , K 2 SO 4 cho tác dụng lần lượt với H 2 SO 4 , NaOH.

Nêu hiện tượng và giải thích các hiện tượng đó?

HƯỚNG DẪN TRẢ LỜI:

Bài 1

a) Chuyển MnO thành MnSO 4 hoặc MnCl 2 , sau đó cho dung dịch muối Mn 2+ tác dụng với

kiềm thu được kết tủa Mn(OH) 2 mầutrắng.

b) Có thể dùng phản ứng: 2Mn(OH) 2 + O 2 (không khí) + 2H 2 O → Mn(OH) 4

Bài 2

3MnSO 4 + 2KClO 3 + 12KOH → 3K 2 MnO 4 + 2KCl + 6H 2 O +3K 2 SO 4

2MnSO 4 + 5PbO 2 + 6HNO 3 →2HMnO 4 + 3Pb(NO 3 ) 2 + 2PbSO 4 +2H 2 O

MnSO 4 + 2Br 2 + 8NaOH→ Na 2 MnO 4 + 4H 2 O + 4NaBr +Mn 2 SO 4

MnBr 2 + 2H 2 O 2 + 4KOH→ K 2 MnO 4 + 4H 2 O +K 2 SO 4

MnSO 4 + CaOCl 2 + 2NaOH→ MnO 2 + Na 2 SO 4 + CaCl 2 + H 2 O

Bài 5

a) Có thể cho MnO 2 tác dụng với HCl đặc thu được MnCl 2 :

MnO 2 + 4HCl → MnCl 2 + Cl 2 +2H 2 O

Nung hỗn hợp MnO 2 + KClO 3 + KOH rắn phản ứng tạo ra K 2 MnO 4, hòa tan, lọc dung dịch

nước lọc có K 2 MnO 4 . Axit hóa dung dịch K 2 MnO 4 thu được KMnO 4 . Đun nóng dung dịch ở

80 0 C, sau đó làm nguội, tinh thể KMnO 4 xuất hiện.

Muốn thu được Mn 2 O 7 , cho H 2 SO 4 đặc tác dụng với tinh thể KMnO 4 :

2KMnO 4 + H 2 SO 4 → 2HMnO 4 + K 2 SO 4

2HMnO 4 → Mn 2 O 7 + H 2 O

b) Nung hỗn hợp Ba(OH) 2 và MnO 2 trong không khí:

2Ba(OH) 2 + 2MnO 2 + O 2 → 2BaMnO 4 + 2H 2 O

3BaMnO 4 + 2H 2 O → Ba(MnO 4 ) 2 + MnO 2 +2Ba(OH) 2

Ba(OH) 2 + CO 2 → BaCO 3 + H 2 O

lọc và rửa sản phẩm, Ba(MnO 4 ) 2 còn lại trong dung dịch.

Bài 7:

a) Có thể bằng các phảnứng:

20

Phát hành PDF bởi Ths Nguyễn Thanh Tú

Đăng ký Word doc qua Zalo 0905779594 Email thanhtuqn88@gmail.com


K 2 MnO 4 + 2H 2 S + 2H 2 SO 4 → 2S + MnSO 4 + K 2 SO 4 + 4H 2 O

2K 2 MnO 4 + Cl 2 →2KMnO 4 + 2KCl

4K 2 MnO 4 + O 2 + 2H 2 O → 4KMnO 4 + 4KOH

b) H 2 MnO 4 không bền nhanh chóng bị phân hủy:

K 2 MnO 4 + H 2 SO 4 → H 2 MnO 4 + K 2 SO 4

2H 2 MnO 4 → 2HMnO 4 + MnO 2 +2H 2 O

Bài 10

2KMnO 4 + 3MnCl 2 + 2H 2 O → 5MnO 2 + 2KCl + 4HCl

2K 2 MnO 4 + Cl 2 →2KMnO 4 + 2KCl

2KMnO 4 + 10KI + 3H 2 SO 4 → 2MnSO 4 + 6K 2 SO 4 + 5I 2 + 8H 2 O

2KMnO 4 + 6KI + 4H 2 O → 2MnO 2 + 3I 2 + 8KOH

2KMnO 4 + 10FeSO 4 + 8H 2 SO 4 →5Fe 2 (SO 4 ) 3 + 2MnSO 4 + K 2 SO 4 + 8H 2 O

Bài 11

1) 2MnCl 2 + 4KClO + 8KOH → 2K 2 MnO 4 + 8KCl + 4H 2 O

PT dạng ion: 2Mn 2+ + 4ClO - +8OH - → 2MnO 4 2- + 4Cl - + 4H 2 O

2) 2MnO 4 - + 5NO 2 - + 6H + → 2Mn 2+ + 5NO 3 - + 3H 2 O

3) 3MnO 4 - + 5Fe + 24H + → 3Mn 2+ + 5Fe 3+ + 12H 2 O

4) 5Mn 2+ + 2BrO 3 - + 4H 2 O → 5MnO 2 + Br 2 + 8H -

5) 2MnO 4 - + H 2 O 2 + 2OH - → 2MnO 4 2- + O 2 + 2H 2 O

I.2. Sắt, coban, niken.

Bài 1

a) Đặc điểm cấu tạo nguyên tử củasắt?

b) Người ta đã biết được các mức oxi hóa nào của Sắt? Lấy ví dụ các hợp chất ứng với

các mức oxi hóađó.

c) Với sắt, mức oxi hóa nào bền nhất? Tạisao?

Bài 2

a) Viết phương trình phản ứng và nêu rõ các điều kiện khi cho Fe tác dụng với O 2 ,

Cl 2 , S, H 2 O, H 2 SO 4 loãng, H 2 SO 4 đặc.

b) Sắt có bị ăn mòn không khi để trong không khí có chứa SO 2 , H 2 ,CO 2 ?

Bài 3

a) HãygiảithíchsựhìnhthànhliênkếttrongphântửhợpchấtFe(CO) 5 .Phương pháp điều chế và ứng

dụng củaFe(CO) 5

b) Trong dung dịch nước, ion Fe 2+ có tính khử mạnh nhất là trong môi trường kiềm; ion Fe 3+

có tính oxi hóa mạnh nhất trong môi trường axit. Hãy lấy ví dụ để minhhọa.

c)Có phản ứng xảy ra không khi cho dung dịch FeCl 3 tác dụng với dung dịch KBr, KI?

Bài 4: Viết phương trình của các phản ứng sau:

1) Fe 2 (SO 4 ) 3 + Na 2 SO 3 + H 2 O→

2) FeSO 4 + HNO 3 + H 2 SO 4 →

3) FeSO 4 + HNO 3 →

4) FeSO 4 + KMnO 4 + H 2 SO 4 →

5) FeCl 3 + Na 2 CO 3 + H 2 O →

Bài 5:Viết phương trình của các phản ứng sau đây dưới dạng ion:

1) FeSO 4 + K 2 Cr 2 O 7 + H 2 SO 4 →

2) FeSO 4 + HClO 3 + H 2 SO 4 → HCl +…

3) FeSO 4 + KBrO 3 + H 2 SO 4 →

4) K 4 [Fe(CN) 6 ] + KMnO 4 + H 2 SO 4 →

5) 5) K 4 [Fe(CN) 6 ] + H 2 O 2 + H 2 SO 4 →

21

Phát hành PDF bởi Ths Nguyễn Thanh Tú

Đăng ký Word doc qua Zalo 0905779594 Email thanhtuqn88@gmail.com


Bài7:Viết phương trình của các phản ứng sau dưới dạng

phân tử: Fe 3+ + H 2 S→

Fe 3+ + I - →

Fe 3+ + S 2 O 3 - →

Fe 3+ + SO 3 2- + H 2 O→

Fe 2+ + Br 2 + H + →

Bài 8:

Cho dung dịch FeCl 2 tác dụng với KCN đặc, dư thu được dung dịch A. Cho A tác dụng lần

lượt với FeSO 4, Fe 2 (SO 4 ) 3 đặc, AgNO 3 thì tương ứng được kết tủa B màu trắng, kết tủa C

xanh đậm và kết tủa D màu trắng. Nếu cho A tác dụng với dung dịch KMnO 4 trong môi

trường axit mạnh thì thu được dung dịch E, dung dịch này tác dụng với FeCl 2 thì kết tủa G

màu xanh tạo thành, còn nếu cho E tác dụng với Pb(OH) 2 trong KOH thì thu được kết tủa F

màu nâu và dung dịch A.

Viết phương trình ion các phản ứng xẩy ra.Cho biết từ tính của hợp chấtA.

HƯỚNG DẪN TRẢ LỜI

Bài 3

a) Phân tử được hình thành theo cơ chế "cho - nhận" nhờ các obital lai hóa dsp 3 của

nguyên tửFe.

3d 4s 4p

và các cặp electron của 5 phân tử CO

b) Fe(CO) 5 điều chế bằng cách nung bột sắt trong dòng khí CO ở 150 - 200 0 C với áp suất

khoảng100at.Fe + 5CO → Fe(CO) 5

Bài 4

Fe 2 (SO 4 ) 3 + Na 2 SO 3 + H 2 O → 2FeSO 4 + Na 2 SO 4 + H 2 SO 4

6FeSO 4 + 2HNO 3 + 3H 2 SO 4 → 3Fe 2 (SO 4 ) 3 + 2NO + 4H 2 O

3FeSO 4 + 10HNO 3 → 3Fe(NO 3 ) 3 + NO + 3H 2 SO 3 + 2H 2 O

2FeCl 3 + 3Na 2 CO 3 + 3H 2 O → 2Fe(OH) 3 + 6NaCl + 3CO 2

Bài 5

6Fe 2+ + BrO - 3 + 6H + → 6Fe 3+ + Br - + 3H 2 O

5[Fe(CN) 6 ] 4- + MnO - 4 + 8H + → 5[Fe(CN) 6 ] 3- + 4H 2 O + Mn 2+

2[Fe(CN) 6 ] 4- + H 2 O 2 + 2H + → 2[Fe(CN) 6 ] 3- + 2H 2 O

Bài 8

a. Các phương trình phản ứng xẩy ra :

Fe 2+ + 6CN - → [Fe(CN) 4- 6 ]

[Fe(CN) 4- 6 ] + 2Fe 2+ → Fe 2 [Fe(CN) 6 ] ↓ trắng

3[Fe(CN) 4- 6 ] + 4Fe 3+ → Fe 4 [Fe(CN) 6 ] 3 ↓ Xanh đậm.

[Fe(CN) 4- 6 ] + 4Ag + → Ag 4 [Fe(CN) 6 ] ↓ trắng

5[Fe(CN) 6 4- ] + MnO 4 - + 8H + → Mn 2+ + 4H 2 O + 5[Fe(CN) 6 3- ]

2[Fe(CN) 6 3- ] + 3Fe 2+ → Fe 3 [Fe(CN) 6 ] 2 ↓ Xanh

Hoặc :

22

Phát hành PDF bởi Ths Nguyễn Thanh Tú

Đăng ký Word doc qua Zalo 0905779594 Email thanhtuqn88@gmail.com


K + + [Fe(CN) 6 3- ] + Fe 2+ → KFe[Fe(CN) 6 ] ↓ Xanh

2[Fe(CN) 6 3- ] + Pb(OH) 2 + 2OH - → 22[Fe(CN) 6 4- ] + 2H 2 O + PbO 2 ↓ nâu

b. Phức A là phức nghịchtừ.

II. Bài tập tìm chất.

Bài 1: A, B, C, D, E đều là những nguyên tố nằm cạnh nguyên tố F trong bảng tuần hoàn.

NguyêntửkhốicủaBlớnhơnAvàC.NguyêntửkhốicủaDlớnhơnnguyêntửkhốicủa F là 31 đvC và

lớn hơn B là 49 đvC. Chênh lệch nguyên tử khối của D và E là 2 đvC. B và E thuộc cùng một

nhóm. A và C có trong acquy C-A, trong đó đơn chất C phản ứng với hydroxide của A, tạo

thành hai hydroxidekhác.

a) Xác định các nguyên tốA-F.

b) Viết phương trình của phản ứng giữa đơn chất C với hydroxide củaA.

HD

a) A – Ni; B – Co; C – Fe; D – Os; E– Ir; F – Rh

b) Fe + 2Ni(OH) 3 = 2Ni(OH) 2 +Fe(OH) 2

Bài 2: Một chất rắn màu trắng X tham gia một loạt các thí nghiệm trong đó X bị đốt thành tro

dưới tác dụng của các luồng khí vào khác nhau. Kết quả thí nghiệm được thống kê ở bảng

sau:

Thí nghiệm số Khí vào Sự chênh lệch khối lượng

mẫu so với ban đầu

1 N 2 -37,9

2 NH 3 -51,7

3 O 2 -31,0

4 HCl +9,5

5 HCl + Cl 2 -100

Trong tất cả các thí nghiệm thì trong hỗn hợp sau phản ứng ngoài khí ban đầu còn có một khí

chưa biết Y. Ở thí nghiệm số 5 xuất hiện một hợp chất màu đỏ nâu Z ngưng tụ khi tiến hành

bước làm lạnh trong thí nghiệm.

a) Sửdụngcácgiátrịchoởbảngtrên,xácđịnhcácchấtđượckýhiệubằngchữcái.

b) Viết các phản ứng xảy ra trong thínghiệm.

c) Cho biết cấu trúc của Z trong phakhí.

HD

a) X là FeCO 3 , Y là CO 2 , Z làFeCl 3

b) Các phản ứng sau đây đã xảyra

FeCO 3 = FeO + CO 2

3FeCO 3 + 2NH 3 = 3Fe + 3CO 2 + 3H 2 O

4FeCO 3 = 2Fe 2 O 3 + 4CO 2

FeCO 3 + 2HCl = FeCl 2 + CO 2 + H 2 O

2FeCO 3 + 4HCl + Cl 2 = 2FeCl 3 + 2CO 2 + 2H 2 O

c) Ở pha hơi thì sắt (III) clorua tồn tại ở dạng dime(FeCl 3 ) 2

Bài 3: HợpchấtAlàmộtoxit,vàhợpchấtDlàmộtmuốisunfat.Sửdụngcácphảnứngđãcân bằng cùng

23

Phát hành PDF bởi Ths Nguyễn Thanh Tú

Đăng ký Word doc qua Zalo 0905779594 Email thanhtuqn88@gmail.com


một số dữ kiện dưới đây hãy xác định các chất từ A – D. Khẳng định cáccâu trả lời bằng

tínhtoán.

A + 3NaOCl + 4NaOH → 2B + 3NaCl + 2H 2 O (pH > 7) (1)

4B + 6H 2 O → 2A·H 2 O + 8NaOH + 3O 2 (pH = 7) (2)

D + 3Na 2 O 2 → C + Na 2 SO 4 + O 2 (3)

3C + 5H 2 O → A + B + 10NaOH (4)

Màu của dung dịch chất B là tím đỏ đậm.

Nếu 0,10 g hợp chất C được hòa tan trong 100 mL nước cất thì pH của dung dịch đo được là

12,2 (B tan hoàn toàn).Xác định các chất từ A –D.

HD

A gần như chắc chắn là một oxit kim loại do nó không tan trong nước (phương trình 4). Giờ

ta có thể cho rằng A chứa x nguyên tử oxy còn B – y. Nếu chúng ta đếm số nguyên tử oxy

trong phản ứng 1 hay 2, chúng ta sẽ có biểuthức: 2y = x + 5

Từ phương trình 1 và 4 thấy rõ rằng hợp chất C không chứa lưu huỳnh. Bằng cách đếm số

nguyên tử oxy trong phương trình 3, ta có thể kết luận được C chứa 4 nguyên tử oxy.

Như vậy từ phương trình 4 ta sẽ có biểu thức khác x + y = 7

Kết quả là x = 3 và y = 4.

Giờ chúng ta đã biết được A ở dạng M 2 O 3 , B là Na 2 MO 4 , C là Na 4 MO 4 và D là MSO 4 . Sử

dụng những dữ liệu đã cho ta có thể tính được số mol NaOH có trong dung dịch: n(NaOH) =

c·V = 10 (pH-14)·0,1 = 0,00157 mol. Vậy

n(Na 4 MO 4 ) = 0,000472 mol và M(Na 4 MO 4 ) = 211.85 g/mol.

M là Fe. A – Fe 2 O 3 B – Na 2 FeO 4 C – Na 4 FeO 4 D – FeSO 4

Bài 4: Có thể tổng hợp bột sắc tố đỏ tươi C bằng cách trộn dung dịch hai muối A, B. Muối B

được dùng trong sản xuất kính và xà phòng, có thể thu được khi nhiệt phân soda. Muối A là

một chất hút ẩm lưỡng nguyên tố (chứa 54.6 % X), có thể tạo thành từ phản ứng

củamộtkimloạiY(thuộcbộbanguyêntốsắt)vớihalogenkhíX 2 .Cũngcóthểtổnghợp sắc tố C bằng

các đun nóng dung dịch muối A, D hoặc E (các muối này đều chứa cùng

mộtkimloại)vàhợpchấtF.MuốiDchứamộttrongcácanioncótrongnướccườngtoan.

AniontrongmuốiEcónguồngốctừaceticacid.HợpchấtFlàchấthữucơđầutiênđược tổng hợp từ các

nguyên liệu vô cơ. Trong công nghiệp, F được sản xuất bằng phảnứng của NH 3 vàCO 2 .

a) Xác định kim loại Y và công thức, tên gọi các chất A-F,X 2 .

b) Hoàn thành các phương trình phản ứngsau:

phân huỷsoda

X 2 + Y→A

A + B → C

2NH 3 + CO 2 → F + ….

HD

Đặt công thức muối A là YX n .

Do halogen là chất khí nên muối là flouride hoặc chloride. Không có muối fluoride nào của

bộ ba nguyên tố sắt (Fe, Co, Ni) thoả mãn các điều kiện bài toán. Chỉ có thể làmuối chloride.

24

Phát hành PDF bởi Ths Nguyễn Thanh Tú

Đăng ký Word doc qua Zalo 0905779594 Email thanhtuqn88@gmail.com


Y là Co.

A – CoCl 2 ; B – Na 2 CO 3 ; C – CoCO 3; D – Co(NO 3 ) 2 ; E – Co(CH 3 COO) 2 ; F - (NH 2 ) 2 CO.

X 2 – Cl 2 , Y – Co,

b) 2NaHCO 3 →Na 2 CO 3 + H 2 O + CO 2 ↑

Co + Cl 2 →CoCl 2

CoCl 2 + Na 2 CO 3 → CoCO 3 ↓ + 2NaCl

2NH 3 + CO 2 → (NH 2 ) 2 CO + H 2 O

Bài 5:Sắc tố tím K – với cation có chứa 19.2 % nito; 75.28 % nguyên tố Z và 5.52 % hydro–

đượcdùngtrongmỹphẩm.Sắctốnàyđượctạothànhtừphảnứngcủaaxit M, muối axit N (chứa

12.18 % nitrogen) và oxit nâu đen O (chứa 63.19 % Z). Muối N

cóaniontrongM.CácloạinướcgiảikhátthườngchứamộtlượngnhỏaxitM.Trongsắc tố K, kim loại

có số oxi hoá +3 và phopho đạt tới số oxi hoá caonhất.

1. Xác định kim loại Z và công thức sắc tố K nếu biết anion của K chứa O vàP.

2. Xác định công thức, tên gọi của các chất M-O,Z.

HD

Đặt công thức oxit O là Z 2 O

Chỉ có n = 4, Z là Mn, oxit MnO 2 thoả mãn

Thành phần của cation K:

Cation của sắc tố là

q

Anion của K chứa phopho và oxy, nên ta có thể dự đoán công thức của K là Mn(NH 4 )P 2 O 7

M – H 3 PO 4 ,

N – NH 4 H 2 PO 4 . Trong N:

O – MnO 2 , Z – Mn.

Bài 6

Cho dung dịch NaOH từ từ vào dung dịch CoCl 2 (hồng) thu được kết tủa màu hồng. Tiếp tục

cho dung dịch NH 3 đến dư vào thì kết tủa màu hồng ta và dung dịch thu được có màu vàng.

Cho hỗn hợp axeton và amonithioxianua và thu được dung dịch có màu xanh.

Mặt khác, cho dung dịch H 2 O 2 vào dung dịch màu vàng thì thu được dung dịch có màu hồng

thẫm. Thêm tiếp dung dịch H 2 SO 4 loãng vào thì thấy có khí bay ra. Viết phương trình phản

ứng xảy ra.

HD:

Co 2+ + 2OH - Co(OH) 2 (hồng)

Co(OH) 2 (hồng) +6NH 3 [Co(NH 3 ) 6 ] 2+ (màu vàng) + 2OH - .

[Co(NH 3 ) 6 ] 2+ (màu vàng) + 4SCN - [Co(SCN) 4 ] 2- +6NH 3

25

Phát hành PDF bởi Ths Nguyễn Thanh Tú

Đăng ký Word doc qua Zalo 0905779594 Email thanhtuqn88@gmail.com


Mặt khác, cho dung dịch H 2 O 2 vào dung dịch [Co(NH 3 ) 6 ] 2+ (màu vàng)

H 2 O 2 + 2[Co(NH 3 ) 6 ] 2+ (màu vàng) 2[Co(NH 3 ) 6 ] 3+ (hồng thẫm) + 2OH -

[Co(NH 3 ) 6 ] 3+ (hồng thẫm) + 6H + Co 3+ + 6NH + 4

4Co 3+ +2H 2 O 4Co 2+ + O 2 +4H +

Bài 7: Cho X là dung dịch muối sunfat

HD

(1) X tác dụng với dung dịch KCN cho dung dịch A màu vàngnhạt

(2) A bị Cl 2 oxi hóa tạo thành dung dịchB

(3) Đun sôi dung dịch B trong KOH đặc lại thu được dung dịchA

(4) A tác dụng với X tạo ra kết tủa C màutrắng.

(5) A tác dụng với Fe 2 (SO 4 ) 3(đặc) tạo ra kết tủa D màu xanhchàm.

(6) B tác dụng với dung dịch KI tạo thànhA.

(7) Trong dung dịch KOH loãng, B oxi hóa Cr(OH) 3 thànhK 2 CrO 4 .

(8) B tác dụng với dung dịch X tạo ra chất E có màu xanh Tuabun. Xác định các

chất X, A, B, C, D, E và viết các phương trình phảnứng.

FeSO 4 ; A: K 4 [Fe(CN) 6 ]; B: K 3 [Fe(CN) 6 ]; C: Fe 2 [Fe(CN) 6 ];

D: Fe 4 [Fe(CN) 6 ] 3 ; E: Fe 3 [Fe(CN) 6 ] 2 .

(1) FeSO 4 + 6 KCN K 4 [Fe(CN) 6 ] +K 2 SO 4

(2) 2 K 4 [Fe(CN) 6 ] + Cl 2 2 K 3 [Fe(CN) 6 ] + 2KCl

(3) 4 K 3 [Fe(CN) 6 ] + 4 KOH đặc 4K 4 [Fe(CN) 6 ] + O 2 + 2H 2 O

(4) K 4 [Fe(CN) 6 ] + 2 FeSO 4 Fe 2 [Fe(CN) 6 ]↓ trắng + 2K 2 SO 4

(5) K 4 [Fe(CN) 6 ] + 2 Fe 2 (SO 4 ) 3 đặc Fe 4 [Fe(CN) 6 ] 3 ↓ xanhchàm + 6K 2 SO 4

(6) 2 K 3 [Fe(CN) 6 ] + 2 KI 2K 4 [Fe(CN) 6 ] +I 2

(7) 2 K 3 [Fe(CN) 6 ] + 5 KOH loãng + Cr(OH) 3 3 K 4 [Fe(CN) 6 ] + K 2 CrO 4 +

4H 2 O

(8) 2 K 3 [Fe(CN) 6 ] + 3 FeSO 4 Fe 3 [Fe(CN) 6 ] 2 ↓ xanhTuabun + 3K 2 SO 4

III. Bài tập phức chất.

III.1. Sơ lược về phức chất.

Phức chất là những chất trong đó nguyên tử hay ion kim loại liên kết với một nhóm

phân tử hay ion

Một muối được tạo nên bởi một ion phức và (phổ biến là) một ion thường trong mạng

tinhthể.

Một phân tử trung hòa trong đó tổng điện tích của tất cả các thành phần tham gia vào

liên kết phối trí bằngkhông.

Số liên kết phối trí (số cặp electron tham gia liên kết) tạo nên bởi các ligan

vàiontrungtâm gọilàsốphốitrí.VídụnhưsốphốitrícủaionCo 3+ ,Ni 2+ trong các phức chất

[Co(NH 3 ) 6 ]Cl 3 , [Co(NH 3 ) 4 Cl 2 ]Cl và [Ni(NH 3 ) 6 ](NO 3 ) 2 bằng 6.

Phổ biến nhấtlà cấu trúc bát diện: các phối tử nằm trên sáu đỉnh của hình bát diện, chất

tạo phức nằm tại tâm.

26

Phát hành PDF bởi Ths Nguyễn Thanh Tú

Đăng ký Word doc qua Zalo 0905779594 Email thanhtuqn88@gmail.com


Ví dụ:

[

FeF 6 ] 3- [Co(en) 3 ] 3+

F

N

F

Fe

N

Co

N

F

F

N

N

N

Đồng phân là những chất có thành phần hóa học giống nhau nhưng cấu trúc khác

nhau dẫn đến tính chất khác nhau. Phức chất có 2 loại đồng phân chính là đồng phân cấu

tạo và đồng phân lập thể. Có 3 loại đồng phân cấu tạo thường gặp trong phức chất: Đồng

phân ionhóa, Đồng phân liênkết, Đồng phân phốitrí

Ví dụ: CoCl 2 (NO 2 ).5NH 3 có hai đồng phân liên kết:

[Co(NH 3 ) 5 NO 2 ]Cl 2 (màu vàng)

[Co(NH 3 ) 5 ONO]Cl 2 (màu đỏ)

[Co(NH 3 ) 4 Cl 2 ] + có hai đồng phân hình học:

Ví dụ: cis_[Co(en) 2 Cl 2 ] + có hai đồng phân quang học:

Khả năng tạo phức của nhóm VIIB: Theo chiều tăng số oxi hóa khả năng tạo phức anion

tăng, tạo phức cation giảm. Ion Mn 2+ có khả năng tạo nên nhiều phức chất nhưng hằng số bền

27

Phát hành PDF bởi Ths Nguyễn Thanh Tú

Đăng ký Word doc qua Zalo 0905779594 Email thanhtuqn88@gmail.com


của những phức chất đó không lớn hơn so với hằng số bền của phức chất các kim loại hóa trị

hai khác (Fe, Co, Ni, Cu) vì ion Mn 2+ có bán kính lớn nhất trong các kim loại hóa trị hai và

năng lượng làm bền bởi trường tinh thể của các phức chất của Mn 2+ đều bằng số không. Muối

mangan (III) có thể tạo một sốphức như K[MnF 4 ], K 2 [MnF 5 ], Na 2 [MnCl 5 ], K 2 [MnCl 5 ]. Những

phức chất thường gặp của mangan (III) là: M 3 [Mn(CN) 6 ] (M=Na + , K + , NH + 4 ) là chất dạng tinh

thể màuđỏ, K 3 [Mn(C 2 O 4 ) 3 ].3H 2 O là tinh thể màu đỏ - tím, mangan (III) axetylaxetonat

[Mn(C 5 H 4 O 2 ) 3 ] là tinh thể màu đen nhánh, [Mn(EDTA)] - bền vớinước.

- Phức chất của reni (III) cũng khá khác thường như [Re 2 X 8 ] 2- trong đó độ dài của liên kết

Re-Re là 2,24Å bị rút ngắn hơn nhiều so với độ dài của liên kết trong kim loại. Bởi vậy,

người ta cho rằng liên kết kim loại – kim loại trong anion phức đó là liên kết bốn giữa Re với

Re, gồm một liên kết σ, hai liên kết π và một liên kếtδ.

- Tecneti (IV) và reni (IV) có thể tạo phức chất với halogen dạng M 2 [ReX 6 ]

hayM 2 [TcX 6 ].

Khả năng tạo phức của nhóm VIIIB: Các nguyên tố nhóm VIIIB (Fe, Co, Ni và họ Platin)

đều có khuynh hướng tạo phức, đặc trưng nhất là phản ứng tạo phức với NH 3 , với CO và cả

với NO. Các nguyên tố này tạo phức cation bền hơn phứcanion.

- Các ion Fe 2+ , Co 2+ và Ni 2+ tạo nên nhiều phức chất, độ bền của những phức chất đó tăng

lên theo chiều giảm của bán kính ion từ Fe 2+ (0,74Å), Co 2+ (0,72Å) đến Ni 2+ (0,69Å). Cả ba

ion đều tạo nên phức bát diện với số phối trí 6. Ion Fe 2+ ít có khuynh hướng tạo nên phức chất

tứ diện hơn các ion Co 2+ và Ni 2+ . Trong các KLCT, ion Co 2+ tạo nên số phức chất tứ diện

nhiều nhất. Sở dĩ như vậy là vì những phức chất tứ diện đó có cấu hình electron bền

(π * d ) 4 (σ * d ) 3 . Ngoài số ít phức chất tứ diện được tạo nên với phối tử trường yếu, ion Ni 2+ còn

tạo nên nhiều những phức chất hình vuông với phối tử trườngmạnh.

- Sắt (III) tạo nên nhiều phức chất. Đa số phức chất đó có cấu hình bát diện như

M 3 [FeF 6 ], M 3 [Fe(SCN) 6 ], M 3 [Fe(CN) 6 ], một số rất ít có cấu hình tứ diện như M[FeCl 4 ] trong

đó M là kim loại kiềm. Những phức chất bát diện thường có spin cao, trừ những phức chất

tạo nên với phối tử trường mạnh có spin thấp, ví dụ như [Fe(CN) 6 ] 3- ,[Fe(phen) 3 ] 3+ .

- Nhiều phức chất của Co(III) cũng đã được biết đến. Hầu hết chúng đều có cấu hình bát

diện còn phức chất tứ diện rất hiếm. Tất cả những ion phức bát diện như [Co(NH 3 ) 6 ] 3+ ,

[Co(CN) 6 ] 3- , [Co(NO 2 ) 6 ] 3- đều nghịch từ trừ một ngoại lệ duy nhất là ion thuận từ [CoF 6 ] 3- với

4 electron độcthân.

III.2. Một số bài tập phức chất.

Bài 1. Gọi tên theo danh pháp quốc tế của các phức sau:

K 2 [NiCl 4 ] ; [Cr(NH 3 ) 6 ](NO 3 ) 3 ; [Co(NH 3 ) 5 CO 3 ]Cl; [Pt(NH 3 ) 2 (H 2 O)(OH)]NO 3 .

HD

Phức chất

Tên gọi

K 2 [NiCl 4 ]

Kali tetracloro nikenat (II)

[Cr(NH 3 ) 6 ](NO 3 ) 3

Hexaammin crom (III) nitrat

[Co(NH 3 ) 5 CO 3 ]Cl

Monocacbonato pentaammin coban (III) clorua

Phát hành PDF bởi Ths Nguyễn Thanh Tú

Đăng ký Word doc qua Zalo 0905779594 Email thanhtuqn88@gmail.com

28


[Pt(NH 3 ) 2 (H 2 O)(OH)]NO 3 Monohiđroxo monoaqua điammin platin (II) nitrat

Bài 2

a) TronghaichấtK 4 [Fe(CN) 6 ]vàFeSO 4 chấtnàocótínhkhửmạnhhơn?Tạisao?

b) Hai chất K 4 [Fe(CN) 6 ]và K 3 [Fe(CN) 6 ] chất nào có tính oxi hóa? Chất nào có tính khử?

c) Viết phương trình phản ứng khi cho K 3 [Fe(CN) 6 ] tác dụng với H 2 O 2 trong môi

trườngKOH, H 2 O 2 trong dung dịchHCl.

HD

Trong dung dịch nước ion Fe 2+ ở dạng ion phức [Fe(H 2 O) 6 ] 2+ có độ bền kém hơn ion phức

[Fe(CN) 6 ] 4- , nên [Fe(H 2 O) 6 ] 2+ có tính khử mạnhhơn.

c) 2K 3 [Fe(CN) 6 ] + H 2 O 2 +2KOH→2K 4 [Fe(CN) 6 ] + O 2 +2H 2 O

2K 4 [Fe(CN) 6 ] + H 2 O 2 + 2HCl → 2K 3 [Fe(CN) 6 ] + 2KCl + 2H 2 O

Bài 3: Coban tạo ra được các ion phức :

CoCl 2 (NH 3 ) 4 + (A), Co(CN) 6 3- (B), CoCl 3 (CN) 3 3- (C),

1. Viết tên của (A), (B),(C).

2. Theo thuyết liên kết hoá trị, các nguyên tử trong B ở trạng thái lai hoánào?

3. Các ion phức trên có thể có bao nhiêu đồng phân lập thể? Vẽ cấu trúc củachúng.

4. Viết phương trình phản ứng của (A) với ion sắt (II) trong môi trườngaxit.

HD

1. Tên của các ionphức:

A. Điclorotetraammincoban(III);

B. Hexaxianocobantat(III);

C. Triclorotrixianocobantat(III).

Co(CN) 6 3- . Co : d 2 sp 3 ; C :sp ; N : không ở vào trạng thái lai hoá hoặc ở

trạng thái lai hoásp.

3. Ion phức (A) có 2 đồngphân:

a. Ion phức (A) có 2 đồngphân:

H 3 N

H 3 N

Cl

Co

Cl

NH 3

NH 3

H 3 N

H 3 N

Cl

Co

NH 3

Cl

NH 3

Phát hành PDF bởi Ths Nguyễn Thanh Tú

Đăng ký Word doc qua Zalo 0905779594 Email thanhtuqn88@gmail.com

28


b. Ion phức (B) không có đồngphân:

NC

NC

CN

Co

CN

CN

CN

c. Ion phức (C) có 2 đồngphân:

NC

Cl

Cl

Co

CN

CN

Cl

NC

Cl

CN

Co

CN

Cl

Cl

CoCl 2 (NH 3 ) 4 + + Fe 2+ +4H + → Co 2+ + Fe 3+ + 2 Cl - + 4 NH 4

+

Bài 4: Hai chất A và B chứa anion phức bát diện có cùng thành phần nguyên tố nhưng chúng

khác nhau về momen từ ( = [n(n +2)] 1/2 trong đó n là số electron không cặp đôi): A = 0, B =

1,72D. Khi cho 20mL dung dịch 0,1M của A tác dụng với 1,3240g Pb(NO 3 ) 2 thì tạo thành 1,2520g

kết tủa trắng và trong dung dịch chỉ còn lại muối kali. Khi cho 1,2700g FeCl 3 vào một lượng dư

dung dịch của A thì tạo thành 1,6200g kết tủa trắng C (51,85% khối lượng là sắt). Khi để ra ngoài

không khí C trở thành xanh lơ và chuyển thành D. Dung dịch của B tác dụng với FeCl 2 tạo thành

ngay một kết tủa xanh lơ E có thành phần giống hệt D.

a) Các chất A, B, C, D, E là những chất gì. Tính gía trị của n đối với chất B.

b) Viết các phương trình phản ứng.

c) Sự khác nhau giữa D và E là gì

HD

a) n(Pb(NO 3 ) 2 ) : n(A) = 1,3240/331:0,1 .0,02 = 2:1 Anion trong A là X 4-

2Pb 2+ + X 4- = Pb 2 X

4.10 -3 2.10 -3 2.10 -3

M(Pb 2 X) = 1,252/2.10 -3 = 626 M(X 4- ) = 212

2Fe 2+ + X 4- = Fe 2 X

0,01 0,005

n(FeCl 2 ) = 0,01; M(FeX 2 ) = 324

n(Fe) = 324.0,5185/56 = 3; C là Fe 2 [FeY 6 ]

1,72 = [n(n+2)] 1/2 n 1; = 0; Fe 3+ .

Vậy Y là CN; A là K 4 [Fe(CN) 6 ]; B: K 3 [Fe(CN) 6 ]; C: Fe 2 [Fe(CN) 6 ]; D và E: KFe[Fe(CN) 6 ]

b) K 4 [Fe(CN) 6 ] + 2Pb(NO 3 ) 2 = Pb 2 [Fe(CN) 6 ] + 4KNO 3

K 4 [Fe(CN) 6 ] + 2FeCl 2 = Fe 2 [Fe(CN) 6 ] + 4KCl

2Fe 2 [Fe(CN) 6 ] + 2K 4 [Fe(CN) 6 ] + O 2 + H 2 O = 4KFe[Fe(CN) 6 ] + 4KOH

K 3 [Fe(CN) 6 ] + FeCl 2 = KFe[Fe(CN) 6 ] + 2KCl

c) KFe 2+ [Fe 3+ (CN) 6 ] và KFe 3+ [Fe 3+ (CN) 6 ] chỉ là cùng một hợp chất.

29

Phát hành PDF bởi Ths Nguyễn Thanh Tú

Đăng ký Word doc qua Zalo 0905779594 Email thanhtuqn88@gmail.com


Bài 5: Ion [Mn(CN) 6 ] 3- có 2 electron độc thân, ion [MnBr 4 ] 2- có 5 electron độc thân, ion

[Ni(CN) 4 ] 2- không có electron độc thân. Dựa vào thuyết liên kết hoá trị (thuyết VB), hãy

viết cấu hình electron (dưới dạng ô lượng tử) của các ion phức trên, cho biết kiểu lai hoá

và cấu trúc hình học củachúng.

Hướng dẫn giải:

[Mn(CN) 6 ] 3- : d 2 sp 3 , bát diện d 2 sp 3

[Mn(CN) 6 ] 3- 6CN -

[MnBr 4 ] 2- : sp 3 , tứ diện

[Ni(CN) 4 ] 2- : dsp 2 , vuông phẳng dsp 2

[Ni(CN) 4 ] 2-

4CN -

Bài 6:(Đề thi chọn đội tuyển thi Olympic Quốc tế năm 2010)

Cho sơ đồ phản ứng

FeSO 4

KCNđặc , dư

FeCl 2 (dd) A (dd)

Fe 2 (SO 4 ) 3 đặc

AgNO 3

KMnO 4 , H +

B kết tủa trắng

C kết tủa xanh đậm

D kết tủa trắng

FeCl 2

E(dd)

Pb(OH) 2 ,KOH

G kết tủa xanh

A + F kết tủa nâu

30

Phát hành PDF bởi Ths Nguyễn Thanh Tú

Đăng ký Word doc qua Zalo 0905779594 Email thanhtuqn88@gmail.com


1. Viết phương trình ion của các phản ứng xảy ra theo sơ đồtrên.

2. Hãy cho biết từ tính của hợp chất A, dùng thuyết lai hóa để giảithích.

Hướng dẫn giải:

1. Các phương trình phảnứng:

Fe 2+ + 6 CN - [Fe(CN ) ] (A)

[Fe(CN

) 4

6

4 6

- ] + 2 Fe 2+ Fe 2 [Fe(CN) 6 ] trắng (B)

3 [Fe(CN ) 4

6

] + 4 Fe 3+ Fe 4 [Fe(CN) 6 ] 3 xanh đậm (C)

[Fe(CN ) ] + 4 Ag + Ag 4 [Fe(CN) 6 ] trắng (D)

4 6

5 [Fe(CN

)

6

] + Mn O

4

+ 8 H + Mn 2+ + 4 H 2 O + 5 [Fe(CN ) 3

6

] (E)

2 [Fe(CN ) 3

6

] + 3 Fe 2+ Fe 3 [Fe(CN) 6 ] 2 xanh

Hoặc K + + [Fe(CN ) ]+ Fe 2+ KFe[Fe(CN) 6 ] xanh

(G)

3

6

2[Fe(CN ) ]+Pb(OH) 2 +2OH - 2 [Fe(CN ) ] + 2H 2 O + PbO 2 (F)

3 6

4 6

2. Cấu hình electron của Fe 2+ là[Ar]3d 6 4s 0 4p 0 4d 0

3d 6 4s 0 4p 0 4d 0

Vì CN - là phối tử trường mạnh, do đó khi tạo phức với Fe 2+ , 4 electron độc thân trên 4 obitan 3d

của Fe(II) bị ghép đôi, giải phóng 2 obitan 3d trống. Hai obitan này lai hóa với 1 obitan 4s và 3

obitan 4p, tạo thành 6 obitan lai hóa d 2 sp 3 hướng về 6 đỉnh của hình bát diện đều. Mỗi obitan lai

hóa này xen phủ với một obitan tự do có hai electron củaCN - , tạo ra 6 liên kết cho nhận, hình thành

phức [Fe(CN ) ] lai hóa trong, có cấu trúc bátdiện.

4 6

Phức này nghịch từ vì có tổng spin bằng không:

Bài 7:

a) Momen từ của dung dịch (nước) Fe(III) giảm từ ~6,0 BM (Bohr magneton) xuống ~1,8 BM ,

khi thêm lượng dư CN - vào dung dịch Fe(III). Hãy giải thích sự thay đổi momen từ nói trên theo

thuyết trường phối tử (hay thuyết trường tinhthể).

b) Cả Fe(II) và Fe(III) đều tạo thành phức chất bát diện bền với CN - . Bằng phương pháp phổ,

người ta đã xác định được độ dài liên kết Fe-C trong [Fe(CN) 6 ] 4- là 192 pm và trong [Fe(CN) 6 ] 3-

là 193 pm. Hãy giải thích sự khác biệt về độ dài liên kết đó theo thuyết obitan phântử.

6

31

Phát hành PDF bởi Ths Nguyễn Thanh Tú

Đăng ký Word doc qua Zalo 0905779594 Email thanhtuqn88@gmail.com


HD

a) Trong dung dịch nước, Fe(III) tạo phức bát diện [Fe(OH 2 ) 6 ] 3+ . Khi thêm CN - với lượng dư sẽ

tạo thành [Fe(CN) 6 ] 3- do phức này rất bền. Theo thuyết trường phối tử (thuyết trường tinh thể),

trong trường bát diện, mức năng lượng d bị tách thành 2 mức: t 2g (gồm d xy , d xz , d yz ) và e g (gồm

, ). [Fe(H 2 O) 6 ] 3+ là phức spin cao, có cấu hình , có 5e độcthân:

= 5,9 ≈ 6BM.

[Fe(CN) 6 ] 3- là phức spin thấp, có cấu hình

, có 1e độc thân vàcó:

= 1,73 (BM).

b) Giản đồ các mức năng lượng các MO của CN - :

Cấu hìnhelectron:

Trong phức [Fe(CN) 6 ] 3- , phối tử CN - , ngoài tạo liên kết σ với nguyên tử trung tâm kiểu L

→ M, còn có các MO-π * trống, có năng lượng tương đối cao, có thể tạo liên kết π với các obitan

có tính đối xứng π là và của nguyên tử trung tâm theo kiểu M → L (liên kết cho nhận

ngược). Nói cách khác, CN - có MO trống có năng lượng tương đối cao nên nó là phối tử π nhận

(tạo thành liên kết πcho).

Liên kết π có tác dụng tương hỗ với liên kết σ làm cho phức rất bền. Do Fe 3+ có mật độ điện tích

dương lớn hơn Fe 2+ nên sự chuyển electron theo kiểu M → L của Fe 3+ yếu hơn Fe 2+ , làm cho

liên kết M → L trong phức này của Fe(III) kém bền hơn trong phức của

Fe(II). Do vậy, liên kết Fe-C trong phức chất [Fe(CN) 6 ] 4- có độ dài ngắn hơn trong phức chất

[Fe(CN) 6 ] 3- .

Bài 8:(QG 2014)

1. Chất A là hợp chất có thành phần chỉ gồm nitơ và hiđro. Chất A được sử dụng làm nhiên liệu

cho tên lửa. Ở cùng điều kiện về nhiệt độ và áp suất, một thể tích hơi của A có khối lượng bằng

khối lượng của cùng một thể tích khíoxi.

a) Xác định công thức phân tử, công thức cấu tạo của A và cho biết trạng thái lai hóa của nitơ

trongA.

b) Người ta thực hiện thí nghiệm sau: cho 25,00 mL dung dịch A nồng độ 0,025M vào dung

dịch Fe 2 (SO 4 ) 3 dư, đun nóng, thu được dung dịch B và một chất khí X. Chuẩn độ 1/2 dung dịch

B trong môi trường axit, cần vừa đủ 12,40 mL dung dịch KMnO 4 . Biết rằng chuẩn độ 10,00 mL

32

Phát hành PDF bởi Ths Nguyễn Thanh Tú

Đăng ký Word doc qua Zalo 0905779594 Email thanhtuqn88@gmail.com


dung dịch H 2 C 2 O 4 0,05M (trong môi trường axit H 2 SO 4 ) cần vừa đủ 9,95 mL dung dịch KMnO 4

ở trên. Xác định chấtX.

2. NH 3 có khả năng phản ứng với nhiều ion kim loại chuyển tiếp. Alfred Werner (được giải

Nobel hóa học năm 1913) đã phân lập thành công một số phức chất giữa CoCl 3 và NH 3 , trong

đó có phức chất bát diện với công thức phân tử là CoCl 3 4NH 3 . Tùy thuộc vào điều kiện tổng

hợp, phức chất này có màu tím hoặc màu xanh. Khi cho lượng dư dung dịch AgNO 3 tác dụng

với dung dịch chứa 1 mol phức chất này đều thu được 1 mol AgCl kết tủa. Hãy xác định các

công thức có thể có của phức chất nêutrên.

HD

1. Gọi công thức của chất A là N x H y .

Ở cùng điều kiện về nhiệt độ và áp suất, một thể tích khí A có khối lượng bằng khối

lượng của cùng một thể tích khí oxi M A =M O = 32

14x + y.1 = 32 x= 2, y= 4 chất A là N 2 H 4 ( hiđrazin)

Công thức cấu tạo của N 2 H 4 :

Trong N 2 H 4 , cả hai nguyên tử N đều ở trạng thái lai hóa sp 3 , phân tử N 2 H 4 có thể coi là

2

sản phẩm thế một nguyên tử H trong NH 3 bằng nhóm NH 2

b) Tính nồng độ của dung dịchKMnO 4 :

2- -

- phản ứng chuẩn 10.0,05.2 độ: 5C 2 O 4 + 2 MnO 4 + 16 H + → 10 CO 2 + 2 Mn 2+ + 8H 2 O

C M ( ddKMnO )

0,0201( M )

4

N

5.9,95

2 H 4 + Fe 2 (SO 4 ) 3 dung dịch B + chất khí X

Do N 2 H 4 có tính khử, Fe 3+ bị khử về Fe 2+ dung dịch B có chứa Fe 2+ , chất khí X là một

hợp chất chứa N với số oxi hóa là x.

Phản ứng của dung dịch B với KMnO 4 :

5Fe 2+ +

- Số mol Fe 2+ trong dung dịch Blà:

Fe

MnO - + 8 H + → 5Fe 3+ + Mn 2+ + 4H 2 O

4

n 2+ 12,40.10 3 . 0,0201.5.2 = 2,492.10 -3 (mol)

33

Phát hành PDF bởi Ths Nguyễn Thanh Tú

Đăng ký Word doc qua Zalo 0905779594 Email thanhtuqn88@gmail.com


Số mol N 2 H 4 = 25.10 -3 . 0,025= 0,625.10 -3 (mol)

Trong phản ứng N 2 H 4 + Fe 2 (SO 4 ) 3 dung dịch B + chất khí X

Quá trìnhnhậnelectron

Quá trình nhườngelectron

Fe 3+ +1e Fe 2+ 2N -2 2N x + 2. (2+x)e

2,492.10 -3 mol 2,492.10 -3 mol

2.0,625.10 -3 mol 2.0,625.10 -3 .(2+x)

mol

Áp dụng bảo toàn electron: trong phản ứng oxi hóa khử số mol e nhận = số mol e nhường

2,492.10 -3 = 2.0,625.10 -3 .(2+x) 2+ x 2 x= 0

N -2 N 0 +2e vậy chất khí X làN 2 .

2. Vì 1 mol phức CoCl 3 .4NH 3 tác dụng dung dịch AgNO 3 (dư) tạo 1 mol AgCl → chỉ có 1

Cl - ở cầu ngoại trong phân tử phức:[Co(NH 3 ) 4 Cl 2 ]Cl.

Do phức [Co(NH 3 ) 4 Cl 2 ] + có cấu trúc bát diện nên có 2 đồng phân:

NH 3

Cl

Cl

Cl NH 3

Co

NH 3

H 3 N NH 3

Co

H 3N NH 3

NH 3

Dạngcis

Hai đồng phân này có màu sắc khác nhau (xanh và tím).

Cl

Dạngtrans

34

Phát hành PDF bởi Ths Nguyễn Thanh Tú

Đăng ký Word doc qua Zalo 0905779594 Email thanhtuqn88@gmail.com


IV. Bài tập cân bằng oxi hóa khử, pin điện.

IV.1. Lí thuyết cần nhớ.

1. Một số khái niệm

- Điện cực là hệ gồm có vật dẫn e tiếp xúc với vật dẫn ion

- Thế điện cực: Chọn điện cực chuẩn hiđro làm gốc

H + (1M)│H 2 (1 atm), Pt

Quy ước E 0 (2H+/H2) = 0 với mọi giá trị của T

Quy ước về sơ đồ điện cực: aOxh + ne → bKh

Phương trình Nerst: E (Oxh/Kh) = E 0 (Oxh/Kh) +

[ ]

[ ]

Thế điện cực tiêu chuẩn tại 25 o C :

Hệ số Nernst : RTln10/F = 0,05916 Volt tại 25 o C

Hằng số Faraday : F = 96450 C. mol -1

Hằng số khí : R = 8,314 J.mol -1 .K -1 = 0,08314 L. bar. mol -1 .K -1

O o C = 273,15 o K

Ở 25 0 C ta có: E (Oxh/Kh) = E 0 (Oxh/Kh) + , [ ]

[ ]

- Pin điện là một hệ gồm hai điện cực nhúng trong cùng một dung dịch hoặc hai dung dịch

khác nhau được nối với nhau bằng một cầu nối

Quy ước về sơ đồ pin điện:

Điện cực đóng vai trò là anot (nơi xảy ra sự oxi hóa) được viết bên trái

Điện cực đóng vai trò catot (nơi xảy ra sự khử) được viết bên phải

Ranh giới giữa điện cực và dung dịch là gạch dọc

Giữa hai chất rắn hoặc khí và dung dịch thì dùng dấu phẩy

Giữa hai dung dịch có cầu muối là 2 gạch dọc

2. Phương trình tổng quát trong pin

aOxh1 + bKh1 cOxh2 + dKh2

Phương trình Nerst: E pin = E 0 pin +

Ở 25 0 C ta có: E pin = E 0 pin + , [ ] [ ]

[ ] [ ]

E pin = E catot - E anot

3. Quan hệ giữa E và ∆G, ∆H, ∆S

∆G = -RTlnK = -nFE = ∆H - T∆S

LnK = => Ở 25 0 C ta có: LgK = ,

[ ] [ ]

[ ] [ ]

35

Phát hành PDF bởi Ths Nguyễn Thanh Tú

Đăng ký Word doc qua Zalo 0905779594 Email thanhtuqn88@gmail.com


IV.2. Bài tập

Bài 1: Cho giản đồ thế khử chuẩn của Mn trong môi trường axit:

- ? 2- +2,27V

+0,95V 3+ ? 2+

MnO MnO MnO Mn Mn

4 4 2

- 2-

3

2

1. Tính thế khử chuẩn của cặp: MnO 4 /MnO4

và Mn /Mn

2. Hãy cho biết các tiểu phân nào không bền với sự dị phân. Hãy tính hằng số cân bằng của

các phản ứng dị phân đó.

HD:

- 2-

3

2

1. Thế khử chuẩn của cặp: MnO 4 /MnO4

và Mn /Mn

2-

MnO 4 + 4H + + 2e MnO 2 + 2H 2 O(1)

-

MnO 4 + 4H + + 5e MnO 2 + 2H 2 O(2)

-

Lấy (2) trừ (1) ta có: MnO 4 + e

2- MnO4 (3)

0 0 0

6 5 4

0

E

6=+1,5V

MnO 3+

và Mn không bền với sự dị phân.

2.

2

4

MnO 4

2-

2MnO 4

2-

+1,7V

ΔG =ΔG -ΔG

-FE =-3FE -(-2FE )

E =+0,56V

0 0 0

3 2 1

0 0 0

3 2 1

0

3

MnO 2 + 4H + + e Mn 3+ + 2H 2 O (4)

MnO 2 + 4H + + 2e Mn 2+ + 2H 2 O(5)

Lấy (5) trừ (4) ta có: Mn 3+ + e Mn 2+ (6)

0 0 0

ΔG

6=ΔG5-ΔG4

-FE =-2FE -(-FE )

+ 4H + 0

+ 2e MnO 2 + 2H 2 O E

1

=+2,27V

-

2MnO 4 +2e

0

-E

3

=-0,56V

2-

3MnO 4 + 4H + -

2MnO 4 + MnO2 + 2H 2 O (7)

0 0 0 0

ΔG

7

=ΔG1 -ΔG

3

=-2FΔE

7

=-3,42F<0 nên phản ứng (7) tự diễn biến.

0

2ΔE7

lgK

7

= 57,966

0,059 K 7 = 9,25.10 57

Mn 3+ + e Mn 2+ 0

E

6

=+1,51V

Mn 3+ + 2H 2 O MnO 2 + 4H + 0

+ e -E

4

=+0,95V

2Mn 3+ + 2H 2 O MnO 2 + Mn 2+ +4H + (8)

0 0 0 0

ΔG

8

=ΔG

6-ΔG 4

=-FΔE

8

=-0,56F<0 nên phản ứng (8) tự diễn biến.

0

ΔE8

lgK

8= 9,492

0,059 => K 8 = 3,1.10 9

36

+1,23V

Phát hành PDF bởi Ths Nguyễn Thanh Tú

Đăng ký Word doc qua Zalo 0905779594 Email thanhtuqn88@gmail.com


Bài 2:Tính nồng độ ban đầu của HSO 4 - , biết rằng khi đo sức điện động của pin:

Pt I - 0,1M; I - 3 0,02M ║ MnO - 4 0,05M, Mn 2+ 0,01M, HSO - 4 C M Pt

ở 25 0 C được giá trị 0,824V.

0

0

Cho: E - 2+ = 1,51V; E - - = 0,5355V; K a (HSO - 4 ) = 1,0.10 -2 .

MnO 4/Mn

I 3/3I

HD:

-

Ở điện cực phải: MnO 4 + 8H + + 5e Mn 2+ + 4H 2 O

Ở điện cực trái: 3I - -

I 3 + 2e

E phải

0,0592 [MnO ].[H ] 0,0592 0,05.[H ]

E = lg = 1,51 + lg

5 [Mn ] 5 0,01

- + 8 + 8

0 4

2+

= - 2+

MnO 4/Mn

= - -

I 3/3I

E trái

-

0 0,0592 [I

3] 0,0592 0,02

E = lg = 0,5355 + lg = 0,574V

- 3 3

2 [I ] 2 (0,1)

E pin = E phải - E trái

0,0592 0,05.[H ]

0,824 = 1,51 + lg

5 0,01

+ 8

- 0,574

h = [H + ] = 0,054M

-

Mặt khác từ cân bằng: HSO 4 H + 2-

+ SO 4

[] C – h h h

37

K a = 10 -2

2

h

= Ka

C - h

Thay h = 0,054M , K a = 10 -2 ta được C - = 0,3456M

HSO 4

Bài3:

1. Cân bằng các phản ứng oxi hoá khử sau:

a) Na 2 SO 3 + KMnO 4 + NaHSO 4 → ? + ? + Na 2 SO 4 + H 2 O

b) Cu 2 FeS 3 + HNO 3 → ? + ? + Fe 2 (SO 4 ) 3 + N 2 O + H 2 O

c) C x H y O+KMnO 4 +HCl → CH 3 -CHO+CO 2 + ?+KCl+H 2 O

(Chobiết tỉ lệ sốmolgiữaCH 3 -CHOvớiCO 2 là1:1)

d) Cr 2 S 3 + Mn(NO 3 ) 2 + Na 2 CO 3 → Na 2 CrO 4 + Na 2 SO 4 + Na 2 MnO 4 + NO + CO 2

e) K 2 SO 3 + KMnO 4 + KHSO 4 → K 2 SO 4 + MnSO 4 + H 2 O

2. Cho pin sau : H 2 (Pt), P

H

=1atm/ H +

: 1M // MnO : 1M, Mn 2+ : 1M, H + : 1M / Pt

2

4

Biết rằng sức điện động của pin ở 25 0 C là 1,5V.

0

a) Hãy cho biết phản ứng thực tế xảy ra trong pin và tính E - 2+ ?

MnO 4 /Mn

b) Sức điện động của pin thay đổi như thế nào khi thêm một ít NaHCO 3 vào nửa trái

của pin?

Phát hành PDF bởi Ths Nguyễn Thanh Tú

Đăng ký Word doc qua Zalo 0905779594 Email thanhtuqn88@gmail.com


HD:

a) 5Na 2 SO 3 + 2KMnO 4 + 6NaHSO 4 → K 2 SO 4 + 2MnSO 4 + 8Na 2 SO 4 + 3H 2 O

c) 8Cu 2 SFeS 2 + 58HNO 3 → 12CuSO 4 + 4Cu(NO 3 ) 2 + 4Fe 2 (SO 4 ) 3 + 25N 2 O + 29H 2 O

d) 15C x

H y

O+(2x+3y-6)KMnO 4

+(6x+9y-18)HCl →

5xCH 3

-CHO+5xCO 2

+(2x+3y-6)MnCl 2

+(2x+3y-6)KCl+(-7x+12y-9)H 2

O

d) Cr 2 S 3 + 15Mn(NO 3 ) 2 + 20Na 2 CO 3 → 2Na 2 CrO 4 + 3Na 2 SO 4 + 15Na 2 MnO 4 + 30NO + 20CO 2

Cr 2 S 3 → 2Cr +6 + 3S +6 +30e x 1

Mn(NO 3 ) 2 +2e → MnO 2- 4 +2NO x 15

e) 5K 2 SO 3 + 2KMnO 4 + 6KHSO 4 → 9K 2 SO 4 + 2MnSO 4 + 3H 2 O

S +4 → S +6 + 2e x 5

Mn +7 + 5e → Mn +2 x 2

2. a. Phản ứng thực tế xảy ra trong pin:

Do E pin = 1,5 V > 0 nên cực Pt - (phải) là catot, cực hiđro - (trái) là anot do đó phản ứng

thực tế xảy ra trong pin sẽ trùng với phản ứng qui ước:

- Catot:

MnO + 8H + 4

+ 5e Mn 2+ + 4H 2 O

- Anot: H 2 2H + + 2e

=> phản ứng trong pin:

2MnO + 6H + 4

+ 5H 2 2Mn 2+ + 8H 2 O

0 0

0

* E pin = E 2

- E = 1,5 V

E 0 2

= 1,5 V

MnO 4 / Mn

b).Nếu thêm một ít NaHCO 3 vào nửa trái của pin sẽ xảy ra pư:

HCO - 3 + H + H 2 O + CO 2

0,059

H

H giảm nên E =

.lg giảm , do đó:

2H / H 2

2

E pin = (E 2 - E ) sẽ tăng

2H / H

o

o

1,51V

4 , / ; E 2 2, 26V

MnO 4 , H / MnO 2

; E

2 1,23V

MnO2 ,

H /

Mn

E o

4 , H 2O/

MnO2

MnO4 / MnO4

0

E

MnO 4/MnO4

K1

10

38

Phát hành PDF bởi Ths Nguyễn Thanh Tú

Bài 4: Kỳ thi chọn HSGQG 2011

o

1. Cho: E

2

a) Tính

MnO H Mn

E và 2

o

MnO

b) Nhận xét về khả năng oxy hóa của MnO 4 - trong môi trường axit, trung tính và bazơ.

Giải thích

2. Viết phương trình ion của phản ứng để minh họa khả năng oxy hóa của ion

pemanganat phụ thuộc vào pH của môi trường

HD:

1.

0

a) Tính E - và - 2-

MnO 4 , H 2 O/MnO 2

-

4

MnO4

/ Mn

MnO 4 / Mn

2

2H / H 2

P H 2

MnO + 8H + + 5e ⇌ Mn 2+ + 4H 2 O

5.1,51/ 0,0592

Đăng ký Word doc qua Zalo 0905779594 Email thanhtuqn88@gmail.com


Mn 2+ + 2H 2 O ⇌ MnO 2 ↓ + 4H + -1 2.1,23/ 0,0592

+ 2e K 10

2-

MnO 2 ↓ + 2H 2 O ⇌ MnO

4

+ 4H + -1 2.2,26 / 0,0592

+ 2e K3

10

-

MnO

4

+ e ⇌

-1 -1

4 1 2 3

2-

MnO 4

K 10

K K .K .K → E - 2-

-

4

4

0

MnO 4/MnO4

(E 0

MnO

-

/MnO

2- ) / 0,0592

4 4

= 5.1,51 – 2.(1,23+2,26) = 0,57 (V)

MnO + 8H + + 5e ⇌ Mn 2+ 5.1,51/ 0,0592

+ 4H 2 O K 10

Mn 2+ + 2H 2 O ⇌ MnO 2 ↓ + 4H + -1 2.1,23/ 0,0592

+ 2e K 10

4× H 2 O ⇌ OH - + H + 14

K 10

-

4

MnO + 2H 2 O + 3e ⇌ MnO 2 ↓ + 4OH - 0

5

39

w

K 10

2

1

2

(3E MnO

- ) / 0,0592

4/MnO2

-1 4

K 5

K

1.K 2

.K → 0

5.1,51

2.1,23 14.4.0,0592

w

E = = 0,59 (V)

MnO - 4 , H 2 O/MnO 2

3

0

b)

MnO

- + 2+

4, H /Mn

0

E > E >

MnO - 4 , H 2 O/MnO

- 2-

2 MnO /MnO

0

-

E → khả năng oxi hóa của MnO mạnh nhất

4 4

trong môi trường axit và yếu nhất trong môi trường bazơ, bởi vì:

- + 8

° 0,0592 [MnO 4][H ]

- + 2+ - + 2+

MnO 4, H /Mn MnO 4, H /Mn

2+

E E + lg

5 [Mn ]

do đó khi pH tăng, [H + -

] giảm, tính oxi hóa của MnO4

giảm.

2. Các phản ứng minh họa khả năng oxi hóa của ion pemanganat phụ thuộc vào pH của

môi trường:

-

MnO

4 + 5Fe 2+ + 8H + → Mn 2+ + 5Fe 3+ + 4H 2 O

2+

MnO + 3 Mn + 2H 2 O

-

2

4

-

2

4

t

5MnO 2 ↓ + 4H +

2-

MnO + SO + 2OH - 2- 2-

→ 2 MnO + SO + H 2 O

3

4

Bài 5:Pin điện hóa A tạo bởi 2 cặp oxi hóa - khử CrO 2- 4 / CrO - 2 và MnO - 4 / MnO(OH) 2 .

Pin điện hóa B, khi hoạt động có phản ứng xảy ra là:

+

H 3 AsO 4 + NH 3 → H2AsO

4 + NH 4

1. Hãy thiết lập sơ đồ pin A và pin B .

2. Tính hằng số cân bằng của phản ứngxảy ra trong pin A.

3.Tính E của pin A khi nồng độ của ion CrO 2- 4 là 0,010M; CrO - 2 là 0,030M; MnO - 4 là

0,200M.

biết

E

C = 0,025 M;

H3AsO4

0

0,

V ; E

1,

V

0

2

18

CrO4

/ Cr ( OH ) 3

HD:

1. Lập pin

a.Xét cặp CrO 4 2- / Cr(OH) 3

4

C NH 3

= 0,010 M pK ai(H3AsO 4)

= 2,13; 6,94; 11,50; pK + 9, 24

a(NH 4 )

695 ;

MnO4

/ MnO(

OH ) 2

4

Phát hành PDF bởi Ths Nguyễn Thanh Tú

Đăng ký Word doc qua Zalo 0905779594 Email thanhtuqn88@gmail.com


CrO 4

2-

+ 4H 2 O + 3e Cr(OH) 3 + 5OH - 1

K 10

1

3E /0,0592

Cr(OH) 3 CrO - 2 + H + + H 2 O K = 10 -14

H + + OH - H 2 O K -1 w = 10 14

CrO 2- 4 + 2H 2 O+ 3e CrO - 2 + 4OH - 1

1

K K .K.K 10

2 1 w

3E /0,0592

E o CrO 4 2- / CrO 2 - = E o CrO 4 2- / Cr(OH) 3 = - 0,18V < E o MnO 4 - / MnO(OH) 2 nên có sơ đồ

pin:

(-) Pt | CrO 4 2- , CrO 2 - , OH - || MnO 4 - , H + , MnO(OH) 2 | Pt (+)

b.Phản ứng xảy ra trong pin được tổ hợp từ các cân bằng sau:

H 3 AsO 4

H + -

+ H AsO

NH 3 +H +

+

NH 4

2 4

H 3 AsO 4 + NH 3

- +

H2AsO 4 + NH 4 K (*)

Như vậy các cân bằng trên đều liên quan đến quá trình cho - nhận H + , do đó có thể

chọn điện cực hiđro để thiết lập pin. Vì giá trị thế của điện cực hiđro ( E + ) phụ thuộc

vào [H + ]:

+ 2

0,0592 [H ]

E + = lg

2H /H2

2 p

H2

40

2H /H 2

nên điện cực platin nhúng trong dung dịch H 3 AsO 4 (có [H + ] lớn hơn) có thế dương hơn, sẽ

là catot. Ngược lại điện cực platin nhúng trong dung dịch NH 3 sẽ là anot. Vậy ta có sơ đồ

pin:

(-) Pt(H 2 ) │ NH 3(aq) ║ H 3 AsO 4(aq) │ Pt (H 2 ) (+)

p H 2

= 1atm p H = 1atm

2

2. Tính K của phản ứng xảy ra trong pin A

-

MnO 4 + 4H + + 3e

MnO(OH) 2 + H 2 O K 1 = 10 3.1,695/0,0592

-

CrO 2 + 4OH -

2-

CrO 4 + 2H 2 O + 3e K -1 2 = (10 3.(-0,18)/0,0592 ) -1

4 | H 2 O

H + + OH - K w = 10 -14

-

MnO 4 + CrO - 2 + H 2 O

2-

MnO(OH) 2 + CrO 4 K = K 1 .K -1 2 .(K w ) 4 = 10 39

3.Tính E pin (A):

E pin(A) = E o

0,0592

[MnO

4

].[CrO

2

]

pin + lg

2

3 [CrO ]

Tính E o pin dựa vào K phản ứng ta có E o pin =

E pin (A) = 0,77 +

0,0592

lg

3

4

0,2.0,03

0,01

39.0,0592

3

= 0,7656V

= 0,77V

Bài6:Trộn 50ml dung dịch H 2 SO 4 2M, 50ml dung dịch FeBr 2 0,2M và 100ml dung dịch

Phát hành PDF bởi Ths Nguyễn Thanh Tú

Đăng ký Word doc qua Zalo 0905779594 Email thanhtuqn88@gmail.com


KMnO 4 0,04M được dung dịch A.

a.Xác định giá trị pH của dung dịch A.

b. Xác định thế của điện cực Pt được nhúng trong dung dịch A.

c. Điện cực hiđro p(H 2 )=1 atm được nhúng trong dung dịch CH 3 COOH 0,010 M

được ghép (qua cầu muối) với điện cực Pt được nhúng trong dung dịch A. Hãy viết sơ đồ

pin và viết phương trình phản ứng xảy ra trong pin.

Cho: pK a (HSO - 4 ) 2,00 ; pK a (CH 3 COOH) 4,76; (RT/F) ln = 0,0592lg ;

E 0 (Fe 3+ /Fe 2+ ) = 0,77V; E 0 (MnO - 4 /Mn 2+ ) = 1,51V;E 0 (Br 2 /Br - ) = 1,085V;

HD:

a. Nồng độ ban đầu các chất sau khi trộn:

C (KMnO 4 ) =0,02M; C (FeBr 2 ) =0,05M; C (H 2 SO 4 ) =0,5M;

H 2 SO 4 -> H + +

-

HSO 4

0,5

- 0,5 0,5

KMnO 4 -> K + +

-

MnO 4

0,02

- 0,02 0,02

FeBr 2 -> Fe 2+ + 2Br -

0,05

- 0,05 0,05

Do E 0 (Fe 3+ /Fe 2+ ) = 0,77V < E 0 (Br 2 /Br - ) = 1,085V< E 0 (MnO - 4 /Mn 2+ ) = 1,51V

Nên các phản ứng xảy ra theo thứ tự:

5Fe 2+ -

+ MnO 4 + 8H + -> 5Fe 3+ + Mn 2+ + 4H 2 O K 1 = 10 62,5 >>

Bđ 0,05 0,02 0,5

Sau - 0,01 0,42 0,05 0,01

10Br - -

+ 2MnO 4 + 16H + -> 5Br 2 + Mn 2+ + 8H 2 O K 2 = 10 71,8 >>

Bđ 0,1 0,01 0,42

Sau 0,05 - 0,34 0,025 0,02

Vì K 1 , K 2 rất lớn nên nồng độ MnO - 4 , Fe 2+ là rất không đáng kể.

TPGH : Fe 3+ = 0,05M; Mn 2+ = 0,02M; H + = 0,34M; K + = 0,02M; Br 2 = 0,025M;

HSO - 4 = 0,5M; Br - = 0,05M.

Xét cân bằng:

-

HSO 4 = H + +

2-

SO 4 Ka = 10 -2

0,5 0,34

[] 0,5 – x 0,34 + x x

Ka = x.(0,34+x)/(0,5 - x) => x = 0,0137M => [H + ] = 0,3537M

Phát hành PDF bởi Ths Nguyễn Thanh Tú

Đăng ký Word doc qua Zalo 0905779594 Email thanhtuqn88@gmail.com

41


pH A = 0,4514

b. Thế của điện cực Pt nhúng vào dung dịch A được tính theo cặp Br 2 /Br - :

Br 2 + 2e = 2Br -

Có E (Br 2 /Br - ) = E 0 (Br 2 /Br - ) + (0,0592/2). Lg([Br 2 ]/[Br - ] 2 )

=> E (Br 2 /Br - ) = 1,085 + (0,0592/2). Lg(0,025/[0,05] 2 ) = 1,115V

c. Xác định thể của điện cực hiđro:

. Cực Hiđro: 2 H + + 2e = H 2

CH 3 COOH = H + + CH 3 COO – ; K a = 10 -4,76

C 0,01

[ ] 0,01 - x x x

x 2 /(0,01-x) = 10 -4,76 x = [H + ] = 4,08.10 -4 M pH = 3,39

E (H 2 /CH 3 COOH) = - 0,0592 pH = - 0,0592 3,39 = - 0,2006 (V)

*Ta có E (Br 2 /Br - ) > E (H 2 /1=2H + ) => điện cực Pt nhúng trong dung dịch A là cực

dương; điện cực hiđro là cực âm.

*Sơ đồ pin: (anot) (-) (Pt) H 2 (P H2 = 1 atm)/CH 3 COOH // dd A / Pt (+) (catot)

* Phản ứng xảy ra trong pin:

Catot: Br 2 + 2e → 2Br -

Anot: H 2 + 2CH 3 COO - → 2CH 3 COOH + 2e

Phản ứng xảy ra trong pin: H 2 + Br 2 + 2CH 3 COO - = 2CH 3 COOH + 2Br - .

Bài7:Có dung dịch X gồm Fe 2 (SO 4 ) 3 0,100M; FeSO 4 0,010M và NaCl 2M.

1/ Cần đặt điện thế tối thiểu là bao nhiêu để có quá trình oxi hóa và quá trình khử xảy ra

đầu tiên ở mỗi điện cực khi điện phân dung dịch X ở pH = 0.

2/ Điện phân 100 ml dung dịch X với cường độ dòng điện một chiều không đổi có I =

9,650A và trong thời gian 100 giây, thu được dung dịch Y.

a) Tính khối lượng dung dịch giảm trong quá trình điện phân.

b) Tính pH của dung dịch Y.

c) Lắp một pin điện gồm một điện cực hiđro tiêu chuẩn với một điện Pt nhúng vào dung

dịch Y. Tính sức điện động của pin khi pin bắt đầu phóng điện và viết sơ đồ pin.

(Giả thiết rằng H 2 O bay hơi không đáng kể và thể tích của dung dịch không thay đổi trong quá

trình điện phân)

Cho: E o (Fe 3+ /Fe 2+ ) = 0,771V; E o (2H + /H 2 ) = 0,00V; * β[Fe(OH)] 2+ = 10 -2,17 ;

* β[Fe(OH)] + = 10 -5,92 ; E o (Cl 2 /2Cl - ) = 1,36V

HD:

1/ Bán phản ứng đầu xảy ra ở mỗi điện cực là

+ Điện cực A (+): 2Cl - ⇌ Cl 2 + 2e

+ Điện cực K (-): Fe 3+ + 1e ⇌ Fe 2+

Trong dung dịch X có C(Fe 3+ ) = 0,2M; C(Fe 2+ ) = 0,01M; C(H + ) = 1M; C(Cl - ) = 2M; Na + ;

42

Phát hành PDF bởi Ths Nguyễn Thanh Tú

Đăng ký Word doc qua Zalo 0905779594 Email thanhtuqn88@gmail.com


SO 4 2- .

Thế khử của mỗi cặp ở mỗi điện cực là:

E a = E(Cl 2 /2Cl - ) = 1,36 + (0,0592/2)lgP1/2 2 = 1,342(V)

Ở pH = 0; không có quá trình proton hóa của ion kim loại, vì vậy ta có

E c = E(Fe 3+ /Fe 2+ ) = 0,771 + 0,0592lg0,2/0,01 = 0,848(V)

Vậy thế cần đặt vào để có quá trình oxi hóa ion Cl - và quá trình khử ion Fe 3+ là: V = 1,342

– 0,848 =0,494(V)

2/ a) Số mol e phóng ra hay thu vào trong quá trình điện phân là

ne = It/F = 9,65.100/96500 = 0,01 (mol)

Có các bán phản ứng:

Ở cực (+): 2Cl - Cl 2 + 2e (1)

n o 0,2

Ở cực (-): Fe 3+ + 1e Fe 2+ (2)

n o 0,02 0,001

Theo (1), (2) và giả thiết cho, thấy ion Cl - và Fe 3+ đều dư. Vậy khối lượng dung dịch giảm

là: m = mCl 2

= 71.0,01/2 = 0,355(gam)

2/b) Theo phần (a), cho thấy trong dung dịch Y có C(Fe 3+ ) = (0,02-0,01)/0,1=0,1(M);

C(Fe 2+ ) = (0,001+0,01)/0,1=0,11(M); C(Cl - ) = (0,2-0,01)/0,1=1,95(M); Na + ; SO 2- 4 . Có các

cân bằng:

Fe 3+ + H 2 O ⇌ Fe(OH) 2+ + H +* β[Fe(OH)] 2+ = 10 -2,17 (3)

Fe 2+ + H 2 O ⇌ Fe(OH) + + H + * β[Fe(OH)] + = 10 -5,92 (4)

H 2 O ⇌ H + + OH - K w = 10 -14 (5)

Do [Fe(OH)] 2+ .[H + ] ≃ 0,1.10 -2,17 >> [Fe(OH)] + .[H + ] ≃ 0,11.10 -5,92 >> K w

Vì vậy pH là do cân bằng (3) quyết định. Xét cân bằng:

Fe 3+ + H 2 O ⇌ Fe(OH) 2+ + H +* β[Fe(OH)] 2+ = 10 -2,17

C o 0,1

[ ] (0,1-x) x x

=> * β[Fe(OH)] 2+ = x 2 /(0,1-x) = 10 -2,17

Với 0<x<0,1 => x = 0,023

Vậy pH = - lg0,023 = 1,638

2/c) Theo kết quả tính ở phần (b) và cho thấy ion Fe 2+ tạo phức hiđroxo không đáng kể,

nên ta có:

E(Fe 3+ /Fe 2+ ) = 0,771 + 0,0592lg(0,1-0,023)/0,11 = 0,762(V)

Vậy E(pin) = E (cao) – E (thấp) = 0,762-0,00 = 0,762 (V)

Do E(Fe 3+ /Fe 2+ )> E(2H + /H 2 ), nên có sơ đồ pin là

43

Phát hành PDF bởi Ths Nguyễn Thanh Tú

Đăng ký Word doc qua Zalo 0905779594 Email thanhtuqn88@gmail.com


A(-) Pt, H 2 (1atm) H + (1M) Fe 2+ (0,11M); Fe 3+ (0,077M) Pt (+) K

Bài 8:1. Cho một ít vụn Cu vào dung dịch gồm CuSO 4 0,5M ; FeSO 4 1,0 M ; Fe 2 (SO 4 ) 3

0,25M .

Có cân bằng sau xảy ra: Cu (r) + 2Fe 3+ Cu 2+ + 2Fe 2+

- Hãy cho biết chiều của phản ứng ở 25 0 C ? Tìm hằng số cân bằng của phản ứng?

- Thay đổi nồng độ của Fe 2+ và Fe 3+ 3

[ Fe ]

, tính tỉ lệ tối thiểu để phản ứng đổi chiều?

2

[ Fe ]

Cho biết ở 25 0 C có E 2 0,34 V , E 3 2

0,77V

2. Ion MnO - 4 có thể oxi hoá ion nào trong các ion Cl - ,Br - ,I - ở các giá trị pH lần lượt bằng 1,

4, 6. Trên cở sở đó hãy dùng dung dịch KMnO 4 và dung môi chiết là CCl 4 nhận biết các

ion I - và Br - có trong hỗn hợp NaCl, NaBr, NaI.

0

0

Cho E

E O

0, 62V

; E O

1,08V

E 1,36V

2

1, 51V

Br / 2Br

Cl / 2Cl

I I

MnO H Mn

HD:

2 / 2

44

4 , /

1. [Cu 2+ ] = [Fe 3+ ]= 0,5M Cu (r) + 2Fe 3+ Cu 2+ + 2Fe 2+

Ta có

E

2

E

E

3

2

Fe /

2

Cu /

Cu / Cu Fe / Fe

0,5

0,77 0,059lg 0,752V

Fe

1

0,059

0,34 lg0,5 0,331V

Cu

2

E nên phản ứng xảy ra theo chiều thuận.

3 2 2

Fe / Fe Cu / Cu

0

nE 2(0,77 0,34)

lg K 14,576 → K = 3,767.10 14

0,059 0,059

3

3

[ Fe ]

[ Fe ]

Để đổi chiều phản ứng: 0,77 0,059lg 0,331→ > 3,6.10 -8 lần

2

2

[ Fe ]

[ Fe ]

-

2. MnO 4 + 8 H + + 5e → Mn 2+ + 4 H 2 O

8

0 [ MnO4

][ H ]

E 2

E lg

MnO4

/ Mn

2

[ Mn ]

Khi pH = 1 = 1,4156 (V) > E

2

EMnO

2

4 / Mn

Ion MnO - 4 có thể oxi hoá các ion Cl - ,Br -- ,I - .

Khi pH = 4 E 0

2 = 1,1324 (V) > E

1,08V

Br / 2Br

MnO 4 / Mn

0

Cl2

Ion MnO - 4 có thể oxi hoá các ion Br -- ,I - .

Khi pH = 6 E = 0,9436 (V) > E

O 2

I I

MnO 4 / Mn

2

1,36V

/ 2

Cl

2 / 2

Ion MnO 4 - chỉ có thể oxi hoá ion I - .

Ban đầu thực hành ở pH = 6 , dùng KMnO 4 với dung môi CCl 4 , I 2 được hình thành tan

trong dung môi có màu tím. Chiết lớp dung môi , thay lớp dung môi có pH = 4, thấy lớp

dung môi có màu vàng của Br 2 .

Bài 9: Kỳ thi lập đội tuyển quốc tế năm 2007

Dung dịch A gồm FeSO 4 0,020 M; Fe 2 (SO 4 ) 3 và H 2 SO 4 .

Phát hành PDF bởi Ths Nguyễn Thanh Tú

Đăng ký Word doc qua Zalo 0905779594 Email thanhtuqn88@gmail.com


a) Lấy chính xác 25,00 ml dung dịch A, khử Fe 3+ thành Fe 2+ ; chuẩn độ Fe 2+ trong hỗn hợp

(ở điều kiện thích hợp) hết 11,78 ml K 2 Cr 2 O 7 0,0180 M. Hãy viết phương trình ion của

phản ứng chuẩn độ. Tính nồng độ M của Fe 2 (SO 4 ) 3 trong dung dịch A.

b) Tính nồng độ M của H 2 SO 4 trong dung dịch A, biết dung dịch này có pH = 1,07.

c) Ghép cực Pt nhúng trong dung dịch A (qua cầu muối) với cực Ag nhúng trong dung

dịch AgNO 3 0,0190 M có thêm K 2 CrO 4 cho đến nồng độ 0,0100 M (coi thể tích được giữ

nguyên).

Hãy cho biết anot, catot và viết phương trình phản ứng xảy ra khi pin hoạt động. Tính sức

điện động của pin.

Cho pK a : HSO 4 - 1,99; Fe 3+ ( Fe 3+ + H 2 O ⇌ FeOH 2+ + H + ) 2,17;

Fe 2+ ( Fe 2+ + H 2 O ⇌FeOH + + H + ) 5,69.

Chỉ số tích số tan pK s của Ag 2 CrO 4 11,89.

E o : Fe 3+ / Fe 2+ 0,771 V; Ag + /Ag 0,799 V; (RT/F)ln = 0,0592 lg.

HD:

a) Phản ứng chuẩn độ Cr 2 O 2 7 + 6 Fe 2+ + 14 H + → 2 Cr 3+ + 6 Fe 3+ + 7 H 2 O

C

2 Fe = CFeSO 4

+ 2 C Fe 2

(SO 4

) 3

= 0,02 + 2C 1

C

2 Fe . 25,00 = 6 (CCr 2

O 2

7 . V Cr 2

O 2

7 → 25,00(0,020 + 2C1 ) = 6(0,0180 . 11,78)

C 1 = 0,01544 M hay C Fe 2

(SO 4

) 3

= 0,01544 M.

b) Trong dd A có: Fe 2+ 0,020 M; Fe 3+ 2C 1; H +

(C, M); HSO 4

(C, M); các cân bằng:

Fe 2+ + 2 H 2 O⇌FeOH + + H 3 O + K a1 = 10 -5,96 (2)

2 H 2 O⇌H 3 O + + OH - K w = 10 -14 (1)

Fe 3+ + 2 H 2 O⇌FeOH 2+ + H 3 O + K a2 = 10 -2,17 (3)

HSO 4

+ H 2 O⇌ SO 2 4

+ H 3 O + K a = 10 -1,99 (4).

So sánh ta thấy (3) và (4) là chủ yếu và tương đương nhau. Áp dụng định luật bảo toàn

proton, ta có [H 3 O + ] = C

H + [FeOH 2+ ] + [SO 2 4

] (a)

Từ (3) có [FeOH 2+ ] / [Fe 3+ ] = K a2 / [H 3 O + ]→ [FeOH 2+ ] / C

3 Fe = K a2 / K a2 + [H 3 O + ]

= 10 -2,17 / (10 -2,17 + 10 -1,07 ) → [FeOH 2+ ] = 0,0736 C

3 Fe = 0,0736 . 0,015445 . 2.

Tương tự, từ (4) có [SO 2 4

] / [HSO 4

] = K a / [H 3 O + ]

→ [SO 2

4

] / C HSO 4

= 10 -1,99 / (10 -1,99 + 10 -1,07 ) → [SO 2

4

] = 0,107 C;

P/ trình (a) trở thành [H 3 O + ] = C + 0,0736 C

3 Fe + 0,107 C (b).

Từ (b) C H 2

SO 4

= C = (10 -1,07 – 0,0736 . 0,03089) / 1,107 → C H 2

SO 4

= C = 0,07483 M.

3 2

c) E Pt = E Fe / Fe = E 0 Fe 3 / Fe 2 + 0,0592 lg([Fe 3+ ]/[Fe 2+ ])

Fe 3+ + 2 H 2 O⇌FeOH 2+ + H 3 O + 10 -2,17

C 0,03089

45

Phát hành PDF bởi Ths Nguyễn Thanh Tú

Đăng ký Word doc qua Zalo 0905779594 Email thanhtuqn88@gmail.com


[ ] 0,03089 – x x 10 -1,07

x .10 -1,07 / (0,03089 – x) = 10 -1,07 → x = 0,002273→ [Fe 3+ ] = 0,03089 – 0,002273

= 0,02862 M → [Fe 2+ ] = C Fe

2 = 0,020 M (vì Ka1 rất bé).

Vậy: E Pt = 0,771 + 0,0592 lg ( 0,0862 / 0,020) = 0,780 V.

2 Ag + + CrO 2 4

→ Ag 2 CrO 4 ↓

0,019 0,010

- 5. 10 -4

Ag 2 CrO 4 ↓ → 2 Ag + + CrO 2

4

K s = 10 -11,89

C 5.10 -4

[ ] 2x 5.10 -4 + x

( 2x ) 2 (5.10 -4 + x) = 10 -11,89 → 4x 3 + 2,0.10 -3 x 2 - 10 -11,89 = 0 → x = 2,08.10 -5

Có: [Ag + ] = 2x = 4,96.10 -5 M.

o

E Ag = E

Ag /

+ 0,0592 lg [Ag + ] = 0,799 + 0,0592 lg4,96.10 -5 = 0,544 V.

Ag

Vì E Ag < E Pt nên cực Ag là anot; cực Pt catot.

Phản ứng trong pin: anot 2 Ag + CrO 2 4

⇌ Ag 2 CrO 4 ↓ + 2e

catot 2x│ Fe 3+ + e ⇌ Fe 2+

2 Ag + CrO 2

4

+ 2Fe 3+ ⇌ Ag 2 CrO 4 ↓ + 2 Fe 2+

E pin = E Pt - E Ag = 0,780 – 0544 = 0,236 V.

Bài10: Kỳ thi chọn HSGQG 2012

Dung dịch X gồm K 2 Cr 2 O 7 0,010 M; KMnO 4 0,010 M; Fe 2 (SO 4 ) 3 0,0050 M và H 2 SO 4 (pH

của dung dịch bằng 0). Thêm dung dịch KI vào dung dịch X cho đến nồng độ của KI là

0,50 M, được dung dịch Y (coi thể tích không thay đổi khi thêm KI vào dung dịch X).

a) Hãy mô tả các quá trình xảy ra và cho biết thành phần của dung dịch Y.

b) Tính thế của điện cực platin nhúng trong dung dịch Y.

c) Cho biết khả năng phản ứng của Cu 2+ với I - (dư) ở điều kiện tiêu chuẩn. Giải thích.

d) Viết sơ đồ pin được ghép bởi điện cực platin nhúng trong dung dịch Y và điện cực

platin nhúng trong dung dịch gồm Cu 2+ , I - (cùng nồng độ 1 M) và chất rắn CuI. Viết

phương trình hoá học của các phản ứng xảy ra trên từng điện cực và xảy ra trong pin khi

pin hoạt động.

E 0 = 1,330 V; E 0 = 1,510 V; E 0

= 0,771 V;

0

E = 0,5355 V

Cho:

2

3+ 2+ 3+ 2+

Cr2O 7

/ Cr MnO 4 /Mn Fe /Fe

0

E

2+ = 0,153 V; pK 12; ở 25 o RT

C: 2,303 = 0,0592; Cr (z = 24).

Cu /Cu

s(CuI)

F

HD:

0 0 0 0

a) Do E

- 2+

= 1,51 V > E

2- 3+

= 1,33 V > E

3+ 2+

= 0,771V > E

- -

= 0,5355 V, nên các quá

MnO 4 /Mn Cr2 O 7 /Cr Fe /Fe I 3 /I

trình xảy ra như sau:

46

I 3 /I

Phát hành PDF bởi Ths Nguyễn Thanh Tú

Đăng ký Word doc qua Zalo 0905779594 Email thanhtuqn88@gmail.com


2-

2 7

2

-

MnO 4

+ 16 H + + 15 I - 2 Mn 2+ + 5 - 3

2O

0,01 0,5

- 0,425 0,01 0,025

Cr O + 14 H + + 9 I - 2 Cr 3+ + 3 I - 3

+ 7 H 2 O

0,01 0,425 0,025

- 0,335 0,02 0,055

2 Fe 3+ + 3 I - 2 Fe 2+ -

+ I

3

0,01 0,335 0,055

- 0,32 0,01 0,06

-

Thành phần của dung dịch Y: I 3

0,060 M; I - 0,32 M; Mn 2+ 0,01 M; Cr 3+ 0,02 M; Fe 2+ 0,01

M.

b)

-

I + 2 e 3 I -

3

0,0592 0,06

E - - = 0,5355 + .log

I 3 /I

3

2 (0,32)

= 0,54 V.

Do

0

- -

I 3 /I

0

E = 0,5355 V> E

2+

Cu

/ Cu

= 0,153 V nên về nguyên tắc Cu 2+ không oxi hóa được I - và phản

ứng: 2 Cu 2+ + 3 I - 2 Cu + -

+ I hầunhư xảy ra theo chiều nghịch.

3

Nhưng nếu dư I - thì sẽ tạo kết tủa CuI.

0 0

1

Khi đó E

2+

= E

2+ + 0,0592.log 0,863 V.

Cu /CuI Cu /Cu

K

0

Như vậy E

2+

= 0,863 V > E

Cu

/CuI

0

- -

I 3 /I

S(CuI)

= 0,5355 V

=> Cu 2+ sẽ oxi hóa được I - do tạo thành CuI:

2 Cu 2+ + 5 I - 2 CuI +

0

d) Vì E

2+

= 0,863 V > E

Cu

/CuI

- -

I 3 /I

47

-

I

3

= 0,54 V điện cực Pt nhúng trong dung dịch Y là anot,

điện cực Pt nhúng trong dung dịch gồm Cu 2+ , I - (cùng nồng độ 1 M), có chứa kết tủa CuI là

catot. Vậy sơ đồ pin như sau:

-

(-) Pt│ I 3

0,060 M; I - 0,32 M║CuI; Cu 2+ 1 M; I - 1 M │Pt (+)

Trên catot: Cu 2+ + I - + e CuI

Trên anot: 3 I - -

I 3

+ 2e

Phản ứng trong pin: 2 Cu 2+ + 5 I - -

2 CuI + I

3

Bài11: Kỳ thi chọn HSGQG 2012

1. Hoà tan hoàn toàn 0,8120 gam một mẫu quặng sắt gồm FeO, Fe 2 O 3 và 35% tạp chất trơ

trong dung dịch HCl (dư), thu được dung dịch X. Sục khí SO 2 vào dung dịch X, thu được

dung dịch Y. Dung dịch Y phản ứng vừa đủ với 22,21 ml dung dịch KMnO 4 0,10 M. Mặt

Phát hành PDF bởi Ths Nguyễn Thanh Tú

Đăng ký Word doc qua Zalo 0905779594 Email thanhtuqn88@gmail.com


khác, hoà tan hết 1,2180 gam mẫu quặng trên trong dung dịch HCl (dư) rồi thêm ngay dung

dịch KMnO 4 0,10 M vào dung dịch thu được cho đến khi phản ứng xảy ra hoàn toàn, thì hết

15,26 ml dung dịch KMnO 4 0,10 M.

a) Viết phương trình hóa học của các phản ứng xảy ra.

b) Tính thể tích SO 2 (ở điều kiện tiêu chuẩn) đã dùng và thành phần phần trăm theo khối

lượng của FeO, Fe 2 O 3 có trong mẫu quặng.

2. a) Tính pH của dung dịch Na 2 A 0,022 M.

b) Tính độ điện li của ion A 2- trong dung dịch Na 2 A 0,022 M khi có mặt NH 4 HSO 4 0,001

M.

Cho:

pK = 2,00; pK + = 9,24;

-

a(HSO 4 )

a(NH 4 )

pK

a1(H2A)

= 5,30;

48

pK

a2(H2A)

= 12,60.

HD:

1. a) FeO + 2 HCl FeCl 2 + H 2 O (1)

Fe 2 O 3 + 6 HCl 2 FeCl 3 + 3 H 2 O (2)

2 FeCl 3 + 2 H 2 O + SO 2 2 FeCl 2 + H 2 SO 4 + 2 HCl (3)

5 FeCl 2 + KMnO 4 + 8 HCl 5 FeCl 3 + MnCl 2 + KCl + 4 H 2 O (4)

5 SO 2 + 2 KMnO 4 + 2 H 2 O 2 H 2 SO 4 + 2 MnSO 4 + K 2 SO 4 (5)

(Lượng HCl dùng để hòa tan quặng không được cho quá dư, chỉ đủ để làm môi trường cho

phản ứng (4))

b) Từ (1) và (4) ta có:

n FeO (trong 1,2180 gam mẫu) = n 2

Fe = 5. n

= 5 . 0,10 . 15,26.10 -3 = 7,63.10 -3 (mol)

MnO 4

-3

7,63.10 . 0,8120

n FeO (trong 0,8120 gam mẫu) =

= 5,087.10 -3 (mol)

1,2180

m FeO (trong 0,8120 gam mẫu) = 72 . 5,087.10 -3 = 0,3663 (g)

và m

Fe2O

(trong 0,8120 gam mẫu) = 0,8120 . 0,65 – 0,3663 = 0,1615 (g)

3

n

Fe 2 O 3

(trong 0,8120 gam mẫu) = 0,1615 1,01.10 -3 (mol)

160

Tương tự, từ (3) và (5) ta có: nSO n

2 SO 2 (3)

nSO 2 (5)

1

Trong đó: n

SO 2 (3)

= . nFeCl

(trong 0,8120 gam mẫu) = n

3

Fe2O

(trong 0,8120 gam mẫu) = 1,01.10 -3 (mol)

3

2

5

n SO -

2 (5)

n

MnO 4 (5)

2

= 5 1

( n

MnO n )

4

- Fe

2 5

2

với: n 2

Fe = n FeO (trong 0,8120 gam mẫu) + 2.n

Fe2O

(trong 0,8120 gam mẫu)

3

n

SO 2 (5)

=

5 ( n 1

MnO - (n

FeO (trong 0,8120 gam mẫu) + 2.n

Fe

4

2O

(trong 0,8120 gam mẫu))

3

2 5

5

-3 1

-3 -3

n

SO 2 (5)

= 0,10 . 22,21.10 - (5,087.10 + 2 . 1,01.10 )

2

5

2.10-3 (mol).

Vậy: n SO

3,01.10 -3 (mol) V

2

SO 2

= 22,4 . 3,01.10 -3 = 0,0674 (lit)

Phát hành PDF bởi Ths Nguyễn Thanh Tú

Đăng ký Word doc qua Zalo 0905779594 Email thanhtuqn88@gmail.com


% FeO = 0,3663 .100

0,8120

= 45,11 %

% Fe 2 O 3 = 65 % – 45,11 % = 19,89 %

2. a) A 2- + H 2 O ⇌ HA - + OH - K b1 = 10 -1,4 (1)

HA - + H 2 O ⇌ H 2 S + OH - K b2 = 10 -8,7 (2)

H 2 O ⇌ H + + OH - K w = 10 -14 (3)

Vì K b1 .C >> K b2 .C >> K w pH của hệ được tính theo cân bằng (1):

A 2- + H 2 O ⇌ HA - + OH - K b1 = 10 -1,4

C 0,022

[ ] 0,022 - x x x

[OH - ] = x = 0,0158 (M) pH = 12,20

b) Khi có mặt NH 4 HSO 4 0,0010 M:

NH 4 HSO 4 NH 4

+ HSO

4

0,001 0,001

Phản ứng: HSO + A 2- →HA - +

4

2

SO K

4 1 = 10 10,6

0,001 0,022

- 0,021 0,001 0,001

NH 4

+ A 2- →HA - + NH 3 K 2 = 10 3,36

0,001 0,021 0,001

- 0,020 0,002 0,001

Hệ thu được gồm: A 2- 0,020 M; HA - 2

0,002 M; SO 0,001 M; NH

4

3 0,001 M.

Các quá trình xảy ra:

A 2- + H 2 O ⇌ HA - + OH - K b1 = 10 -1,4 (4)

NH 3 + H 2 O ⇌ NH 4

+ OH - K ' b

= 10 -4,76 (5)

HA - + H 2 O ⇌ H 2 A + OH - K b2 = 10 -8,7 (6)

2

SO + H

4 2 O ⇌ HSO 4

+ OH - K b = 10 -12 (7)

HA - ⇌ H + + A 2- K a2 = 10 -12,6 (8)

So sánh các cân bằng từ (4) đến (7), ta có: K b1 . C 2- >> K ' A b

. C

NH 3

>>K b2 . C - >> K

HA b . C

2

SO4

(4) chiếm ưu thế và như vậy (4) và (8) quyết định thành phần cân bằng của hệ:

A 2- + H 2 O ⇌ HA - + OH - K b1 = 10 -1,4

C 0,02 0,002

[] 0,02 - x 0,002 + x x

x = 0,0142 [HA - ] = 0,0162 (M)

49

-

Phát hành PDF bởi Ths Nguyễn Thanh Tú

Đăng ký Word doc qua Zalo 0905779594 Email thanhtuqn88@gmail.com


2

A

-

[HA ] 0,0162

α - = = = 0,7364 hay α

2

A

0,022 0,022 - = 73,64 %.

-

[OH ] + C - + C

+

(Hoặc α 2 -

HSO4 NH 0,0142 + 0,001 + 0,001

4

=

A

=0,7364

0,022 0,022

V. Một số bài toán khác.

Bài 1 (Chọn ĐTHSGQT 2013)

1. Tinh thể chất A được điều chế nhưsau:

- Hòa tan hoàn toàn bột kim loại X trong dung dịch H 2 SO 4 20%, dư. Đun nhẹ dung dịch cho

đến khi xuất hiện váng tinh thể chất B (dung dịch bão hòa1).

- Pha dung dịch (NH 4 ) 2 SO 4 bão hoà ở khoảng 70 o C (dung dịch 2). Trộn ngay hai dung dịch

1 và 2 với nhau. Sau khi làm lạnh dung dịch thu được, thấy tách ra tinh thể chất A màu

xanhnhạt.

Lấy 1,000 gam tinh thể A pha thành 50 ml dung dịch A. Chuẩn độ 10 ml dung dịch Abằng

dung dịch KMnO 4 0,01M (trong môi trường H 2 SO 4 ) thì thấy hết 10,20 ml.

Khi cho dung dịch A tác dụng với K 3 [Fe(CN) 6 ] thu được kết tủa màu xanh đậm; còn nếu

cho A tác dụng với dung dịch kiềm thì thu được kết tủa màu trắng xanh, sau chuyển sang

màu nâu đỏ.

a. XácđịnhcôngthứccủachấtA.Viếtphươngtrìnhhóahọccủacácphảnứngxảy ra.

b. NếutrongthínghiệmtrênthayH 2 SO 4 bằngHCl;(NH 4 ) 2 SO 4 bằngNH 4 Clthìkết

quả thí nghiệm có thu được muối kép không? Giải thích.

2. Hòa tan 2,000 gam tinh thể chất A (ở trên) vào nước, thêm axit H 2 SO 4 làm môi trường,

đunnóng,thêm từ từ dung dịch H 2 C 2 O 4 vào, thu được kết tủa D màu vàng. Lọc lấy kết tủa

D. Cho D tác dụng với dung dịch K 2 C 2 O 4 và H 2 O 2 thu được dung dịch Y. Đun dung dịch

Y cho bay hơi bớt nước, sau đó làm lạnh, thu được 1,566 gam tinh thể chất Z màu xanh.

Hiệu suất của cả quá trình đạt85%.

Lấy 0,361gam tinh thể Z pha thành 50 ml dung dịch Z. Chuẩn độ 10 ml dung dịch Zbằng

dung dịch KMnO 4 0,01M (trong môi trường H 2 SO 4 ) thì thấy hết 16,00 ml.

Xác định công thức phân tử của Z.

HD

1. a.KhichodungdịchAtácdụngvớiK 3 [Fe(CN) 6 ]thìthuđượckếttủamàuxanhđậm, còn nếu

cho A tác dụng với dung dịch kiềm thì thu được kết tủa màu trắng xanh, sau chuyển sang

màu nâu đỏ → trong dung dịch A có chứa ion Fe 2+ :

Fe 2+ + K + + [Fe(CN) 6 ] 3- → KFe[Fe(CN) 6 ] ↓ (xanh Tuabin)

Fe 2+ + 2OH - → Fe(OH) 2 ↓ (trắng xanh)

2Fe(OH) 2 + 1/2O 2 + H 2 O → 2Fe(OH) 3 (↓ nâu đỏ)

- Kim loại X là Fe. Hòa tan hoàn toàn bột kim loại X trong dung H 2 SO 4 20% dư:

Fe + H 2 SO 4 → FeSO 4 + H 2

- Khi trộn dung dịch bão hòa FeSO 4 và dung dịch (NH 4 ) 2 SO 4 bão hòa, làm lạnh dung dịch

sẽ thu được tinh thể muối kép (NH 4 ) 2 SO 4 . FeSO 4 . n H 2 O hoặc(NH 4 ) 2 Fe(SO 4 ) 2 .nH 2 O.

Phản ứng chuẩn độ:

5Fe 2+ + MnO - 4 + 8H + → 5 Fe 3+ + Mn 2+ + 4H 2 O

Số mol MnO - 4 = 0,01.10,2.10 -3 = 0,102.10 -3 (mol)

Số mol Fe 2+ trong 50ml dung dịch A là: (0,102.10 -3 .5.50)/10 = 2,55.10 -3 mol

Trong 1,00 gam tinh thể (NH 4 ) 2 SO 4 . FeSO 4 . n H 2 O có 2,55.10 -3 mol Fe

50

Phát hành PDF bởi Ths Nguyễn Thanh Tú

Đăng ký Word doc qua Zalo 0905779594 Email thanhtuqn88@gmail.com


→ số mol A là 2,55.10 -3 mol → M A = 1/(2,55.10 -3 ) ≈ 392 (g/mol)

Ta có, khối lượng mol của (NH 4 ) 2 Fe(SO 4 ) 2 .nH 2 O = 284 + 18n = 392 n = 6.

→ số phân tử nước kết tinh là 6;

công thức phân tử của tinh thể A là (NH 4 ) 2 SO 4 . FeSO 4 . 6 H 2 O.

b. Nếu trong thí nghiệm này thay H 2 SO 4 bằng HCl; (NH 4 ) 2 SO 4 bằng NH 4 Cl thì không thu

được muối kép, vì gốc Cl - không có khả năng tạo liên kết hidro, không làm cầu nối để tạo

muối kép, còn gốc SO 2 4 - có khả năng tạo liên kết hidro, làm cầu nối nên dễ tạo muối kép.

2. Cho dung dịch chất A (NH 4 ) 2 SO 4 . FeSO 4 . 6 H 2 O (có axit H 2 SO 4 làm môi trường) tác

dụng với H 2 C 2 O 4 , đun nóng, thu được kết tủaD:

Fe 2+ + C 2 O 4 2− → FeC 2 O 4 ↓ vàng

- Trong giai đoạn này cần có axit H 2 SO 4 làm môi trường để ion Fe 2+ không bị oxi

không khí oxi hóa thành ionFe 3+ .

- Khi cho FeC 2 O 4 tác dụng với H 2 O 2 có mặt lượng dư C 2 O 4 2− , Fe(II) bị oxi hóa thành

Fe(III), kết hợp với ion C 2 O 4 2− tạo thành phức sắt (III) oxalat. Gọi công thức của tinh thể

Z là: K x Fe(C 2 O 4 ) y (H 2 O) z

Nếu hiệu suất = 100%, thì số mol Fe trong tinh thể Z = số mol Fe trong tinh thể

A = 2/392=5,102.10 -3 (mol).

Vì n Z = n Fe và vì hiệu suất của cả quá trình là 85%

→ Số mol Z = số mol Fe. 85% == 4,3367.10 -3 (mol)

→ M Z = 1,566/(4,3367.10 -3 ) = 361 (g/mol)

- Phản ứng chuẩnđộ:

5C 2 O 4 2− +2MnO 4 − +16H + →10CO 2 +2Mn 2+ +8H 2 O

SốmolMnO - 4=0,01.16.10 -3 =0,16.10 -3 (mol)

Số mol C 2 O 4 2− trong 50ml dung dịch Y là: (0,16.10 -3 . 5. 50)/(2.10) = 2.10 -3 mol

Trong 0,361gam tinh thể Z ( 0,001mol) có 2.10 -3 mol C 2 O 4 2− → trong Z có 2 gốc C 2 O 4

2−

Áp dụng bảo toàn điện tích trong tinh thể Z: (1+).x + (3+).1 + 2.(2-) = 0 → x = 1 Công thức

phân tử của Z là KFe(C 2 O 4 ) 2 (H 2 O) z

Từ M Y =361 → z = 5 → Công thức phân tử của Z là KFe(C 2 O 4 ) 2 (H 2 O) 5 .

- Xác định công thức cấu tạo của Z:

Vì Z là phức chất của Fe(III) nên số phối trí của Fe thường là 6; dạng bền nhất, mỗi ion

C 2 O 4 2− có dung lượng phối trí là 2, vì vậy trong cầu nội, để cho nguyên tử trung tâm Fe có số

phối trí 6 thì trong cầu nội phải có 2 phân tử H 2 O. Công thức của Z là

K[Fe(C 2 O 4 ) 2 (H 2 O) 2 ].3H 2 O.

Bài 2: Có thể điều chế tinh thể FeCl 3 .6H 2 O theo cách sau: Hoà tan sắt kim loại vào trong

dung dịch axit clohidric 25%. Dung dịch tạo thành được oxi hóa bằng cách sục khí clo qua

cho đến khi cho kết qủa âm tính với K 3 [Fe(CN) 6 ]. Dung dịch được cô bay hơi ở 95 o C cho đến

khi tỉ trọng của nó đạt chính xác 1,695 g/cm 3 và sau đó làm lạnh đến 4 o C. Tách kết tủa thu

được bằng cách hút chân không rồi cho vào một dụng cụ chứa được niêm kín.

a. Viết các phản ứng dẫn đến sự kết tủaFeCl 3 .6H 2 O

b. Có bao nhiêu gam sắt và bao nhiêu ml dung dịch axit clohydric 36% (d=1,18g/cm 3 )

cần để điều chế 1,00kg tinh thể này. Biết rằng hiệu suất quá trình chỉ đạt65%

c. Đun nóng 2,752g FeCl 3 .6H 2 O trong không khí đến 350 o C thu được 0,8977g bã rắn.

Xác định thành phần định tính và định lượng của bãrắn.

HD:

a. Các phản ứng:

51

Phát hành PDF bởi Ths Nguyễn Thanh Tú

Đăng ký Word doc qua Zalo 0905779594 Email thanhtuqn88@gmail.com


Fe + 2HCl →FeCl 2 + H 2

2FeCl 2 + Cl 2 → 2FeCl 3

3FeCl 2 + 2K 3 [Fe(CN) 6 ] → Fe 3 [Fe(CN) 6 ] 2 + 6KCl

FeCl 3 + 6H 2 O → FeCl 3 .6H 2 O

Khi đun nóng thì FeCl 3 .6H 2 O phân huỷ theo phương trình sau:

FeCl 3 .6H 2 O → FeOCl + 5H 2 O + 6HCl

Khi nhiệt độ tăng thì FeOCl sẽ tiếp tục phân huỷ:

3FeOCl → FeCl 3 + Fe 2 O 3 (Hơi FeCl 3 bay ra)

Lượng FeCl 3 .6H 2 O trong mẫu là 10,18 mmol

Điều này ứng với khối lượng FeCl 3 là 107,3. 0,01018 = 1,092g FeOCl

Do khối lượng thu được của bã rắn bé hơn nên ta biết được FeOCl sẽ bị phân hủy một phần thành

Fe 2 O 3 . Khối lượng FeCl 3 mất mát do bay hơi là: 1,20mmol

Như vậy bã rắn cuối cùng chứa (0,01018 – 3.0,00120) = 6,58 mmol FeOCl và 1,20 mmol

Fe 2 O 3 .

Bài 3: Trong một bình kín dung tích không đổi chứa 50 (g) hỗn hợp gồm: a 1 (g) FeCO 3 chứa

a% tạp chất trơ và a 2 (g) FeS 2 cũng chứa a% tạp chất trơ và một lượng gấp 1,5 lần lượng cần

thiết không khí giàu oxi (70% N 2 và 30% O 2 về thể tích). Nung nóng bình để phản ứng xảy ra

hoàn toàn thu được hỗn hợp oxit B và hỗn hợp khí C, sau đó đưa nhiệt độ bình về trạng thái

ban đầu thấy áp suất trong bình vẫn như trước khi nung.

Lấy chất rắn trong bình cho vào ống sứ, đốt nóng rồi dẫn một luồng khí CO đi qua. Sau khi

kết thúc thí nghiệm, từ chất rắn còn lại trong ống sứ lấy ra được 17,92 (g) sắt, biết rằng chỉ có

80% sắt oxit bị khử thành sắt.

Tính % tạp chất trơ a và khối lượng a 1 ,a 2 .

HD

Gọi nFeCO

và n

3 FeS

lần lượt là x và y.

2

0

2FeCO 3 +1/2O 2 t Fe 2 O 3 +CO 2

x

x 4

x

2FeS 2 +11/2O 2 t Fe 2 O 3 +SO 2

11y

y

y/2 2y

4

x 11y

4 4

x 11y

banđan 1,5.

4

1,5.7 x 11y

x 11y

. 3,5.

3 4

4

n phanungg

n O 2

n N 2

0

Vậy hh C gồm các chất có số mol là: CO 2 x mol; SO 2 2y mol; O 2 dư 0,5;

x 11y

mol N 2

4

p 1 =p 2 nên n 1 = n 2 vậy x=y

Fe 2 O 3 CO 2Fe

52

Phát hành PDF bởi Ths Nguyễn Thanh Tú

Đăng ký Word doc qua Zalo 0905779594 Email thanhtuqn88@gmail.com


Từ (1) và (2) ta có x = y = 0,2

+ Tổng khối lượng FeS 2 và FeCO 3 là:

m FeCO + m FeS = 0,2 .120 + 0,2.116 = 47,2 (g)

3 2

+ Vì phần trăm tạp chất như nhau nên phần trăm nguyên chất như nhau:

a 1 = 24,58 (g) và a 2 = 5,42 (g)

a= 2,36 %

Bài 4: Cho 88,2 g hỗn hợp A gồm FeCO 3 , FeS 2 cùng lượng không khí (lấy dư 10% so với lượng

cần thiết để đốt cháy hết A) vào bình kín dung tích không đổi. Nung bình một thời gian để xảy

ra phản ứng, sau đó đưa bình về nhiệt độ trước khi nung, trong bình có khí B và chất rắn C (gồm

Fe 2 O 3 , FeCO 3 , FeS 2 ). Khí B gây ra áp suất lớn hơn 1,45% so với áp suất khí trong bình đó trước

khi nung. Hòa tan chất rắn C trong lượng dư H 2 SO 4 loãng, được khí D (đã làm khô); các chất

còn lại trong bình cho tác dụng với lượng dư dung dịch KOH thu được chất rắn E. Để E ngoài

không khí cho đến khi khối lượng không đổi, được chất rắn F. Biết rằng: Trong hỗn hợp A một

muối có số mol gấp 1,5 lần số mol của muối còn lại; giả thiết hai muối trong A có khả năng như

nhau trong các phản ứng; không khí chứa 20% oxi và 80% nitơ về thểtích.

a. Viết phương trình hóa học của các phản ứng xảyra.

b. Tính phần trăm khối lượng của các chất trong hỗn hợpF.

c. Tính tỉ khối của khí D so với khíB.

HD:

a. Phương trình hóa học của các phản ứng xảy ra:

4FeCO 3 + O 2 → 2Fe 2 O 3 +4CO 2 (1)

4FeS 2 + 11O 2 → 2Fe 2 O 3 +8SO 2 (2)

+ Khí B gồm: CO 2 , SO 2 , O 2 , N 2 ; chất rắn C gồm: Fe 2 O 3 , FeCO 3 , FeS 2 .

+ C phản ứng với dung dịch H 2 SO 4 loãng:

Fe 2 O 3 + 3H 2 SO 4 → Fe 2 (SO 4 ) 3 +3H 2 O (3)

FeCO 3 + H 2 SO 4 → FeSO 4 + H 2 O + CO 2 (4)

FeS 2 + H 2 SO 4 → FeSO 4 + S ↓ +H 2 S (5)

+ Khí D gồm: CO 2 và H 2 S; các chất còn lại gồm:FeSO 4 , Fe 2 (SO 4 ) 3 , H 2 SO 4 dư và S, khi tác dụng

với KOH dư:

2KOH + H 2 SO 4 → K 2 SO 4 +2H 2 O (6)

2KOH + FeSO 4 →Fe(OH) 2↓ +K 2 SO 4 (7)

6KOH + Fe 2 (SO 4 ) 3 →2Fe(OH) 3↓ +3K 2 SO 4 (8)

+ Kết tủa E gồm Fe(OH) 2 , Fe(OH) 3 và S, khi để ra không khí thì chỉ có phản ứng:

4Fe(OH) 2 + O 2 + 2H 2 O→4Fe(OH) 3 (9)

Vậy F gồm Fe(OH) 3 và S

- Nhận xét: So sánh hệ số các chất khí trong (1) và (2) ta thấy: áp suất khí sau phản ứng

tăng lên chứng tỏ lượng FeCO 3 có trong hỗn hợp A nhiều hơnFeS 2 .

b. Gọi a là số mol của FeS 2 số mol của FeCO 3 là 1,5a, ta có:

116.1,5a + 120a = 88,2 a =0,3.

+ Vậy trong A gồm : FeS 2 (0,3 mol), FeCO 3 (0,45 mol).

+ Nếu A cháy hoàn toàn thì cần lượng O 2 là : (0,45/4 + 11.0,3/4) = 1,03125 mol số mol N 2

là 4.1,03125 = 4,125 mol ; số mol không khí là (1,03125 + 4,125) = 5,15625 mol.

Vì hai muối trong A có khả năng như nhau trong các phản ứng nên gọi x là số mol FeS 2 tham

gia phản ứng (1) thì số mol FeCO 3 tham gia phản ứng (2) là1,5x.

53

Phát hành PDF bởi Ths Nguyễn Thanh Tú

Đăng ký Word doc qua Zalo 0905779594 Email thanhtuqn88@gmail.com


+ Theo (1), (2) và theo đề cho ta có : n B = (5,15625 + 0,375x)

+ Vì áp suất sau phản ứng tăng 1,45% so với áp suất trước khi nung, ta có :

(5,15625 + 0,375x) = 5,15625. 101,45/100 x = 0,2

Theo các phản ứng (1), ...(9) ta có chất rắn F gồm : Fe(OH) 3 (0,75 mol) và S (0,1 mol). Vậy

trong F có %Fe(OH) 3 = 96,17% ; %S =3,83%

- B gồm: N 2 (4,125 mol), O 2 (0,40625 mol), CO 2 (0,3 mol), SO 2 (0,4 mol)

M B = 32.

c. Khí D gồm CO 2 (0,15 mol), H 2 S (0,1 mol) M D = 40 d D/B = 1,25

Bài 5: Cho hỗn hợp X gồm MgO, FeO, Fe 2 O 3 và Fe 3 O 4 có số mol đều bằng nhau. Lấy m gam

X cho vào ống sứ chịu nhiệt, nung nóng rồi cho luồng khí CO đi qua ống, CO phản ứng hết,

toàn bộ khí CO 2 ra khỏi ống được hấp thụ hết vào bình đựng 100 ml dung dịch Ba(OH) 2

0,60M, thấy khối lượng dung dịch tăng so với dung dịch đầu là 1,665 gam. Chất rắn còn lại

trong ống sứ gồm 5 chất và có khối lượng là 21 gam. Cho hỗn hợp này tác dụng hết với dung

dịch HNO 3 , đun nóng được V lít khí NO (là sản phẩm khử duy nhất, ở 0 o C; 2atm).

Viết các phương trình phản ứng xảy ra và tính khối lượng m, V, số mol HNO 3 đem

dùng (biết lượng axit dư 20% so với ban đầu).

HD:

- Phản ứng oxit bị khử bởiCO:

3Fe 2 O 3 + CO 2Fe 3 O 4 +CO 2 (1)

Fe 3 O 4 + CO 3FeO+CO 2 (2)

FeO + CO Fe+CO 2 (3)

MgO + CO không phản ứng

- Viết phản ứng theo khí CO 2 lội vào dung dịchBa(OH) 2 :

CO 2 + Ba(OH) 2 BaCO 3 +H 2 O (4)

n o x 0,06

n s (x-0,06) 0 0,06

CO 2 + H 2 O + BaCO 3 Ba(HCO 3 ) 2 (5)

n o (x-0,06) 0,06

n s 0 (0,12-x)

Từ (4), (5) và giả thiết cho ta có:

mCO 2

– mCaCO 3

= 44x – 197(0,12-x) = 1,665 x = 0,105

Hoặc tính CO 2 theo hai phản ứng giữa CO 2 với Ba(OH) 2 tạo ra hai muối

Từ (1), (2), (3), theo bảo toàn khối lượng ta có

m + mCO = 21 + mCO 2

m + 28.0,105 = 21 + 44.0,105 m = 22,68 gam

+ Các phản ứng của MgO, Fe 2 O 3 , Fe 3 O 4 , FeO, Fe với dung dịch HNO 3 :

MgO + 2HNO 3 Mg(NO 3 ) 2 +H 2 O (6)

Fe 2 O 3 + 6HNO 3 2Fe(NO 3 ) 3 +3H 2 O (7)

3Fe 3 O 4 + 28HNO 3 9Fe(NO 3 ) 3 + NO+ 14H 2 O (8)

3FeO + 10HNO 3 3Fe(NO 3 ) 3 + NO+5H 2 O (9)

Fe + 4HNO 3 Fe(NO 3 ) 3 + NO+2H 2 O (10)

Tính V

Theo kết quả trên: m = 72x + 160x + 232x + 40x = 22,68 => x = 0,045 mol

54

Phát hành PDF bởi Ths Nguyễn Thanh Tú

Đăng ký Word doc qua Zalo 0905779594 Email thanhtuqn88@gmail.com


Từ (1), (2), (3), (8), (9), (10) và dựa vào bảo toàn electron ta có

n e(FeO, Fe 3 O 4 ) + n e(CO) = n e(NO)

0,045.1 + 0,045.1 + 0,105.2 = 3.V/22,4.2 V = 1,12 lít.

Tính nHNO 3

. Từ (6) (10), có số mol HNO 3 phản ứng là

nHNO 3

= 2nMg + 3nFe + nNO = 2.0,045 + 3.0,045.6 + 2.1,12/22,4 = 1 mol

Số mol HNO 3 đã dùng = 1/0,8 = 1,25 mol

Bài 6: Nung nóng một hỗn hợp muối rắn NaNO 3 và bột chì kim loại, người ta thu được một hỗn

hợp sản phẩm B. Nếu hoà tan B vào nước thì cần phải thêm 40 ml dung dịch KMnO 4 0,5 M

(được axit hoá bằng H 2 SO 4 ) màu tím-hồng mới bắt đầu xuất hiện trong dungdịch.

Nếu cho B vào một lượng dư dung dịch HNO 3 loãng, đun nóng, cô dung dịch đến cạn khô rồi hoà

tan sản phẩm thu được vào nước và thêm một lượng dư dung dịch Na 2 S, thì thu được 17,952 gam

kếttủa.

a. Viết các phương trình phản ứng xảyra.

b. Xác định thành phần phần trăm về khối lượng của hỗn hợp banđầu.

HD:

a. NaNO 3 +Pb NaNO 2 +PbO

5NO - -

2 + 2MnO 4 + 6H + 2Mn 2+ + 5NO - 3 + 3H 2 O

PbO + 2H + Pb 2+ +H 2 O

Pb + H + + NO - 3 Pb 2+ + NO +H 2 O

Pb 2+ + S 2- PbS

b. Từ khối lượng kết tủa => Số mol PbS = 17,952/ (32,07 + 207,2) =

0,075mol Số mol NaNO 2 = 2,5. 0,04.0,5 = 0,05mol

Vậy % khối lượng Pb = 0,075.207,2.100% / (207,2.0,075 + 85.0,05) = 78,5%

Bài 7: (QG 2006)Một hỗn hợp rắn A gồm kim loại M và một oxit của kim loại đó. Người ta

lấy ra 3 phần, mỗi phần có 59,08 gam A. Phần thứ nhất hoà tan vào dung dịch HCl thu được

4,48 lít khí hiđro; phần thứ hai hoà tan vào dung dịch của hỗn hợp NaNO 3 và H 2 SO 4 thu

được 4,48 lít khí NO; phần thứ ba đem nung nóng rồi cho tác dụng với khí hiđro dư cho đến

khi được một chất rắn duy nhất, hoà tan hết chất rắn đó bằng nước cường toan thì có 17,92 lít

khí NO thoát ra. Các thể tích khí đo ở điều kiện tiêuchuẩn.

Hãy tính khối lượng nguyên tử, cho biết tên của kim loại M và công thức oxit trong hỗn hợp

A.

HD

Kí hiệu số mol kim loại M có trong 59,08 gam hỗn hợp A là x (x > 0).

Giả thiết a): M có duy nhất một mức (hay số) oxi hoá là n+ :

Khi hoà tan 59,08 gam hỗn hợp A vào dung dịch HCl thu được khí hiđro theo phương trình:

M + nHCl → MCl n + 0,5 nH 2 (1)

xmol

0,5 nx mol

55

Phát hành PDF bởi Ths Nguyễn Thanh Tú

Đăng ký Word doc qua Zalo 0905779594 Email thanhtuqn88@gmail.com


Khi hoà tan 59,08 gam hỗn hợp A vào dung dịch của hỗn hợp NaNO 3 và H 2 SO 4 (cũng

chính là dung dịch HNO 3 ) ta thu được khí NO:

3 M + n NO 3 – +4nH + → 3 M n+ + nNO(k) + 2nH 2 O (2)

x mol

(nx : 3) mol NO

Theo đề bài có số mol H 2 bằng số mol NO (đều bằng 4,48 : 22,4 = 0,2

(mol)). Theo lập luận trên lại có 0,5 nx mol H 2 khác với (nx : 3) mol NO.

Vậy giả thiết a) này không phù hợp.

Giả thiết b): Xét M có hai mức (số) oxi hoá khác nhau:

*) Trong phản ứng (1), M có mức oxi hoá n+.

Từ liên hệ trên, ta thu được 0,5 nxmolH 2

*) Trong phản ứng (2), M có mức oxi hoá m+. Ta có:

(a)

-

3 M + m NO 3 + 4mH + → 3 M m+ + m NO (k) +2mH 2 O (2)

xmol

(mx : 3)mol

Số mol NO thu được làmx/3 mol

(b)Theo đề bài có số mol H 2 bằng số mol NO. Vậy từ ( a ) và ( b ) tacó:

(1/2) nx = (1/3) mx (c). Từ đây ta có: n/m = 2/3 = 4/6 = 6/9 = ...

(d)Ta đã biết các kim loại có số oxi hoá n hay m không vượt quá4+.

Vậy kim loại M được xét ở đây có đồng thời n = 2 và m = 3. Giả thiết b) là hợp lí.

c) Xác định M và oxit của nó:

c.1) Xét trường hợp M có số oxi hoá m = 3 trong oxít: hỗn hợp A gồm M và

M 2 O 3 . Với phản ứng M 2 O 3 + 3H 2 → 2M

+ 3H 2 O (3)

ta cũng thu được kim loại M. Vậy chất rắn duy nhất là kim loại M.

Khi tác dụng với nước cường toan (là chất oxi hoá rất mạnh) M chuyển thành

M 3+ trong phản ứng M + 3 HCl+HNO 3 →

MCl 3 + NO (k) + 2 H 2 O(4)

Theo (1) có 0,5 nx = 0,2 mà n = 2 vậy x = 0,2

Theo (4) tổng số mol M trong 59,08 g hỗn hợp A là:

n M = n NO = 17,92/22,4 = 0,8 (mol)

Biết số mol M ban đầu có trong 59,08 g A là x = 0,2. Vậy số mol M do phản ứng

(3) tạo ra là 0,8 - 0,2 = 0,6 (mol). Theo công thức M 2 O 3 thì 0,6 mol này tương ứng với

56

Phát hành PDF bởi Ths Nguyễn Thanh Tú

Đăng ký Word doc qua Zalo 0905779594 Email thanhtuqn88@gmail.com


số mol oxit là 0,6 : 2 = 0,3 (mol).

Kí hiệu khối lượng mol phân tử M là X, ta có phương

trình: 0,2 X + (2 X + 16 x 3) x 0,3 = 59,08. Vậy X = 55,85

(g/mol).

Suy ra nguyên tử khối của M là 55,85 ~ 56. Do đó M là Fe và oxit là Fe 2 O 3 .

c.2) Vấn đề được đặt ra tiếp theo là: Trong hỗn hợp A có oxit nào khác chứ không

phải Fe 2 O 3 ? Có một số cách trả lời câu hỏi này. Ta xét cách sauđây:

Kí hiệu số oxi hoá của Fe trong oxit này là z. Vậy công thức oxit là Fe 2 O z .

Theo kết quả tính ở trên, trong 59,08 gam hỗn hợp A có 0,2 mol Fe nên số gam

Fe 2 Oz là 59,08 - 0,2.55,85 = 47,91 (g) tương ứng với số mol được kí hiệu u.

Số mol NO do Fe từ Fe 2 O z tác dụng với nước cường toan tạo ra là

2 u =0,6→ u=0,3 (5)

Đưa kết quả này vào liên hệ về số gam Fe 2 O z , ta có:

0,3.(55,85 . 2 + 16z) = 47,91 → z= 3 (6)

Vậy Fe 2 O z là Fe 2 O 3

Kết luận: Hỗn hợp A gồm M là Fe, oxit chính là Fe 2 O 3 (không thể là oxit khác).

Bài 8:

Đểxácđịnhh 3 àmlượngcủacromvàsắttrongmộtmẫugồmCr 2 O 3 vàFe 2 O 3 ,ngườita đun nóng chảy

1,98 gam mẫu với Na 2 O 2 để oxi hóa Cr 2 O 3 thành CrO 2- 4 . Cho khối đã nung chảy vào nước,

đun sôi để phân huỷ hết Na 2 O 2 . Thêm H 2 SO 4 loãng đến dư vào hỗn hợp

thuđượcvàphathành100,00mL,đượcdungdịchAcómàuvàngdacam.Chodungdịch

KI(dư)vào10,00mLdungdịchA,lượngI - 3 (sảnphẩmcủaphảnứnggiữaI - vàI 2 )giải phóng ra phản

ứng hết với 10,50 mL dung dịch Na 2 S 2 O 3 0,40 M. Nếu cho dung dịch NaF (dư) vào 10,00 mL

dung dịch A rồi nhỏ tiếp dung dịch KI đến dư thì lượng I - giải phóng ra chỉ phản ứng hết với

7,50 mL dung dịch Na 2 S 2 O 3 0,40 M.

a. Viết các phương trình phản ứng xảyra.

b. Giải thích vai trò của dung dịchNaF.

c. Tính thành phần % khối lượng của crom và sắt trong mẫu ban đầu.

HD

57

Phát hành PDF bởi Ths Nguyễn Thanh Tú

Đăng ký Word doc qua Zalo 0905779594 Email thanhtuqn88@gmail.com


b.Vai trò của dd NaF, F - có mặt trong dung dịch tạo phức bền, không màu với Fe 3+ , dùng để

che Fe 3+ .

c.

Bài 9:

Nung hỗn hợp A gồm sắt và lưu huỳnh sau một thời gian được hỗn hợp rắn B. Cho B tác

dụng với dung dịch HCl dư, thu được V 1 lít hỗn hợp khí C. Tỉ khối của C so với hiđro bằng

10,6. Nếu đốt cháy hoàn toàn B thành Fe 2 O 3 và SO 2 cần V 2 lít khíoxi.

a. Tìm tương quan giá trị V 1 và V 2 (đo ở cùng điềukiện).

b. Tính hàm lượng phần trăm các chất trong B theo V 1 vàV 2 .

c. Hiệu suất thấp nhất của phản ứng nung trên là bao nhiêu phầntrăm.

d. Nếu hiệu suất của phản ứng nung trên là 75%, tính hàm lượng phần trăm các chất

trong hỗn hợp B.

HD

a. Fe +S t 0 FeS

58

Phát hành PDF bởi Ths Nguyễn Thanh Tú

Đăng ký Word doc qua Zalo 0905779594 Email thanhtuqn88@gmail.com


Thành phần B gồm có FeS, Fe và có thể có S.

FeS + 2HCl FeCl 2 + H 2 S

Fe + 2HCl FeCl 2 + H 2 .

Vậy trong C có H 2 S và H 2 . Gọi x là % của H 2 trong hỗn hợp C .

(2x+34(100-x))/100 = 10,6.2 = 21,2 x = 40%

Vậy trong C, H 2 = 40% theo số mol ; H 2 S = 60%. Đốt cháy B :

4FeS + 7O 2 2Fe 2 O 3 + 4SO 2

4Fe + 3O 2 2Fe 2 O 3

S + O 2 SO 2 .

Thể tích O 2 đốt cháy FeS là: (3V 1 /5) . (7/4) = 21V 1 /20.

Thể tích O 2 đốt cháy Fe là: (2V 1 /5) . (3/4) = 6V 1 /20.

Tổng thể tích O 2 đốt cháy FeS và Fe là: 21V 1 /20 + 6V 1 /20 = 27V 1 /20.

Thể tích O 2 đốt cháy S là: V 2 - (27V 1 /20) = V 2 - 1,35 V 1 .

Vậy V 2 ≥ 1,35 V 1

2V 1

x56x100

%Fe 5

70V 1

32(V 2 V 1 ) V 2 V 1

%

b. Nếu dư S so với Fe thì tính hiệu suất phản ứng theo Fe. Trường hợp này H = 60%.

Nếu dư Fe so với S tính hiệu suất phản ứng theo S. Trường hợp này H > 60%

Vậy hiệu suất thấp nhất của phản ứng nung trên là60%.

c. Nếu H = 75% có nghĩa là n FeS = 3n s dư. n FeS tỷ lệ 3V 1 /5 Vậy n S tỷ lệ vớiV 1 /5.

%FeS 5280V 1

5280V 1

64, 7%

75, 2V 1

32V 1

5

%Fe 2240V 1

27, 45%

81,6V 1

81, 6V 1

%S = 100 - (64,7+27,45) =7,85%

Bài 10: Theo lí thuyết khoáng pyrit có công thức FeS 2 , trong thực tế một phần ion được thay thế

bởi S 2– và công thức tổng quát của pyrit là FeS 2 – x . Như vậy, có thể coi pyrit như là hỗn hợp FeS 2 ,

FeS. Khi xử lý một mẫu khoáng với Br 2 trong KOH dư thì xảy ra phản ứng theo sơ đồ:

FeS 2 + Br 2 + KOH Fe(OH) 3 + KBr + K 2 SO 4 + H 2 O

FeS + Br 2 + KOH Fe(OH) 3 + KBr + K 2 SO 4 + H 2 O

Sau khi lọc, được chất rắn A và dung dịch B

Nung chất rắn A đến khối lượng không đổi thu được 0,2g Fe 2 O 3 .

Cho dư dung dịch BaCl 2 vào dung dịch B thu được 1,1087g kết tủa BaSO 4 .

1. Xác định công thức tổng quát của pyrit.

2. Cân bằng các phản ứng trên bằng phương pháp ion – electron.

3. Tính lượng Br 2 dùng để oxi hóa mẫu khoáng trên.

HD

1. Số mol Fe = 2 số mol Fe 2 O 3 = 2. = 0,00250 mol

Số mol S = số mol BaSO 4 = = 0,00475 mol

Tỉ lệ số mol S với số mol Fe trong công thức tổng pyrit = 1,9

Vậy công thức tổng quát của mẫu khoáng pyrit FeS 1,9 .

59

Phát hành PDF bởi Ths Nguyễn Thanh Tú

Đăng ký Word doc qua Zalo 0905779594 Email thanhtuqn88@gmail.com


2. FeS 2 + 19OH – Fe(OH) 3 + 2 + 8H 2 O + 15e

Br 2 + 2e 2Br

2FeS 2 + 38OH – + 15Br 2 2Fe(OH) 3 + 4 + 16H 2 O

2FeS 2 + 38KOH + 15Br 2 2Fe(OH) 3 + 4K 2 SO 4 + 30KBr + 16H 2 O

FeS + 11 OH – Fe(OH) 3 + 2 + 8H 2 O + 9e

Br 2 + 2e 2Br

2FeS + 22OH – + 9Br 2 2Fe(OH) 3 + 2 + 8H 2 O

2FeS + 22KOH + 9Br 2 2Fe(OH) 3 + 2K 2 SO 4 + 18KBr + 8H 2 O

3. Công thức tổng của pyrit FeS 2 – x = FeS 1,9 2 – x = 1,9

vậy x = 0,1 nghĩa là FeS 2 chiếm 90%, FeS chiếm 10%

Số mol Fe = số mol FeS 1,9 = 0,0025

Số mol mỗi chất trong mẫu khoáng pyrit:

Số mol FeS 2 : 0,9.0,0025 = 0,00225 mol

Số mol FeS: 0,1.0,0025 = 0,00025 mol

Khối lượng Br 2 dùng để oxi hóa mẫu khoáng trên là:

0,00225..160 + 0,00025..160 = 0,288(gam).

Bài 11: Cho mẫu chất chứa Fe 3 O 4 , Fe 2 O 3 và các tạp chất trơ. Hòa tan mẫu vào lượng dư dung dịch

KI trong môi trường axit (khử tất cả Fe 3+ thành Fe 2+ ) tạo thành dung dịch A. Pha loãng A đến thể

tích 50 ml. Lượng I 2 có trong 10 ml dung dịch A phản ứng vừa đủ với 5,5 ml dung dịch Na 2 S 2 O 3

1M (sinh ra S 4 O 2 6

). Lấy 25 ml mẫu dung dịch A khác, chiết tách I 2 , lượng Fe 2+ trong dung dịch

còn lại phản ứng vừa đủ với 3,2 ml dung dịch MnO 4

1M trong H 2 SO 4 .

a. Viết phương trình phản ứng xảy ra (dưới dạng phương trình ion thu gọn).

b. Tính phần trăm khối lượng Fe 3 O 4 và Fe 2 O 3 trong mẫu ban đầu.

HD

Phương trình phản ứng:

Fe 3 O 4 + 2I - + 8H + → 3Fe 3+ + I 2 + 4H 2 O (1)

Fe 2 O 3 + 2I - + 6H + → 2Fe 3+ + I 2 + 3H 2 O (2)

2S 2 O 2

+ I 3 2 → S 4 O 2

+ 6 2I- (3)

5Fe 2+ + MnO 4

+ 8H + → 5Fe 3+ + Mn 2+ + 4H 2 O (4)

(3) => n 1 1

I

n .0,0055.1 0,00275mol

2 (3)

2

2

S 2 O 3 2

(4) => n 2

5n 5.0,0032.1

0,016mol

Fe (4) MnO4

Đặt số mol Fe 3 O 4 và Fe 2 O 3 lần lượt là x và y, ta có:

3x + 2y = 0,016.2 = 0,032

x + y = 0,00275.5 = 0,01375

x = 0,0045 và y = 0,00925

%m Fe3O4 = 17,4% và %m Fe2O3 = 24,7%

60

Phát hành PDF bởi Ths Nguyễn Thanh Tú

Đăng ký Word doc qua Zalo 0905779594 Email thanhtuqn88@gmail.com


Bài 12:Trộn m gam hỗn hợp X gồm Mg, Fe, Fe 3 O 4 , Cu và CuO (trong đó nguyên tố oxi chiếm

12,82% theo khối lượng hỗn hợp X) với 7,05 gam Cu(NO 3 ) 2 , thu được hỗn hợp Y. Hòa tan hoàn

toàn Y trong dung dịch chứa đồng thời HCl; 0,05 mol KNO 3 và 0,1 mol NaNO 3 . Sau khi các phản

ứng xảy ra hoàn toàn, thu được dung dịch Z chỉ chứa muối clorua và 3,36 lít (đktc) hỗn hợp khí T

gồm N 2 và NO. Tỉ khối của T so với H 2 là 14,667. Cho Z phản ứng với dung dịch Ba(OH) 2 dư, kết

thúc các phản ứng thu được 56,375 gam kết tủa. Giá trị của m gần nhất với giá trị nào sau đây?

A.30,5. B.32,2. C.33,3. D.31,1.

Mg OH

Fe OH

BaOH

du

2

Mg

dd Z 56,375 g

Fe OH

Fe

hhX Fe3O4

% mO

12,82%

hhY

N2

: 0,05

Cu

NO

: 0,1

CuO

Cu NO3 : 0,0375

2

HCl

KNO3:0,05

Cu OH

NaNO3:0,1

2

BTNT N: n 2n

nKNO n 2 0,025

Cu NO

3 NaNO

n

3 N

n

2 NO

mol

NH4 3 2

Gs:

;

KL X

O X

m m n a

n 2 n 8 n 10 n 3 n 2 a 8.0,025 10.0,05 3.0,1 2 a 1

e O

NH N

4

2 NO

n

ne

2a

1

OH do X tao ra

m m m m

*

KL X OH X tao ra Cu OH 2 do Cu NO 3 2

tao ra

a

56,375 m

17 2 1 0,0375.98 1

16a

* % m

0,1282 2

O X

m 16a

m 27, 2

1 , 2 m m

16a 31, 2 gam

a

0,25

Bài 13: Hòa tan hết 27,2 gam hỗn hợp rắn X gồm Fe 3 O 4 , Fe 2 O 3 và Cu trong dung dịch chứa 0,9 mol

HCl, thu được dung dịch Y chứa 13,0 gam FeCl 3 . Điện phân Y đến khi ở catot bắt đầu có khí thoát

ra thì dừng điện phân, thấy khối lượng dung dịch giảm 13,64 gam. Cho dung dịch AgNO 3 dư vào

dung dịch sau điện phân, thấy khí NO thoát ra (sản phẩm khử duy nhất); đồng thời thu được m gam

kết tủa. Giá trị của m là:

A. 116,85. B. 118,64. C. 117,39. D. 116,31.

HD

Quy hỗn hợp thành Fe 2 O 3 , FeO và CuO với số mol lần lượt là a b và c.

61

Phát hành PDF bởi Ths Nguyễn Thanh Tú

Đăng ký Word doc qua Zalo 0905779594 Email thanhtuqn88@gmail.com

2

3

2


Ta có sơ đồ phản ứng:

FeCl 3

: 0,08

Fe2O 3

: a

CuCl 2

: c

X FeO : b HCl Y H

2

O

0,9mol

HCl

du

:

3abc

CuO : c

FeCl

2

: b

27,2g

Khi điện phân dung dịch Y đến khi catot thoát khí ⇒ FeCl 3 và CuCl 2 đã bị điện phân hết.

⇒ m Giảm = n FeCl3 ×35,5 + n CuCl2 ×135 = 13,64 gam n CuCl2 = 0,08 mol.

160a 72b 80c 27,2 FeCl 3

: 0,08

⇒ Ta có hệ phương trình 2a 0,08 FeCl 2

: 0, 2

c 0,08

CuCl 2

: 0,08

Bảo toàn Clo ⇒ n HCl dư = 0,1 mol.

●Tóm lại sau điện phân dung dịch chứa: n FeCl2 = 0,28 mol và n HCl = 0,1 mol.

Cho dung dịch sau điện phân + AgNO 3 ⇒ 3Fe 2+ + 4H + + NO – 3 → Fe 3+ + NO + 2H 2 O.

2+ 2+

⇒ n Fe bị mất đi = 0,1 × 3 ÷ 4 = 0,075 mol ⇒ n Fe còn lại = 0,28 – 0,075 = 0,205 mol.

⇒ n Ag = n 2+ Fe = 0,205 mol || n AgCl = n – Cl = 0,28×2 + 0,1 = mol.

⇒ m↓ = m Ag + m AgCl = 0,205×108 + 0,66×143,5 = 116,85

Bài 14: Hỗn hợp X gồm Mg, Fe, Fe 3 O 4 và CuO, trong đó oxi chiếm 20% khối lượng. Cho a gam

hỗn hợp X tan hết vào dung dịch Y gồm H 2 SO 4 1,32M và NaNO 3 0,8M, thu được dung dịch Zchứa

b gam các chất tan đều là muối trung hòa và 1,792 lít khí NO (ở đktc). Dung dịch Zphản ứng với

dung dịch KOH dư thấy có 68,32 gam muối trung hòa và 1,792 lít khí NO (ở đktc). Dung dịch

Zphản ứng với dung dịch KOH dư thấy có 68,32 gam KOH phản ứng. Biết các phản ứng đều xảy ra

hoàn toàn và 183a = 50b. Giá trị của b gần nhất với giá trị nào sau đây?

A. 120,00. B. 118,00. C. 115,00. D. 117,00.

HD

mMg, Fe,Cu 0,8a g ;mO

0, 2a g

. Đặt nH2 SO

1,32x n

4 NaNO

0,8x .

3

Mg, Fe,Cu ;O H SO :1,32x; NaNO : 0,8x NO : 0,08 H O Mg , Fe ,Cu ; Na : 0,8x; NH ;SO

2 ? 2 2

2 4 3 2 4 4

Mg , Fe ,Cu ; Na : 0,8x; NH ;SO :1,32x; NO KOH :1,22 K :1, 22; Na : 0,8x;SO :1,32x; NO

2 ? 2 2 2

4 4 3 4 3

n 2n 10n 4n n 0, 264x 0,0025a 0,032 mol .

O

H NH NO

4 NH 4

Bảo toàn nguyên tố nitơ:

n n n n n 0,536x 0,0025a 0,048

NO

NO 3 /Y NH4 NO 3 /Z NO 3 /Z

Bảo toàn điện tích: n n 2n 2 n 1, 22 0,8x 3,176x 0,0025a 0,048

K Na SO 4 NO 3

b m Mg, Fe,Cu m m m m 183,104x 0,91a 3,552

2

Na NH 4 SO4 NO3

183a 50183,104x 0,91a 3,552

Giải hệ có: x 0,5 mol;a=32 g b 117,12g

.

Bài 15:Hòa tan hoàn toàn 29,12 gam hỗn hợp gồm 0,08 mol Fe(NO 3 ) 2 , Fe, Fe 3 O 4 , Mg, MgO, Cu

và CuO vào 640 ml dung dịch H 2 SO 4 1M. Sau khi các phản ứng xảy ra hoàn toàn thu được dung

dịch X chỉ chứa các muối sunfat trung hòa và hỗn hợp hai khí là 0,14 mol NO và 0,22 mol H 2 . Cho

dung dịch X tác dụng với dung dịch Ba(OH) 2 dư, tạo ra kết tủa Y. Lấy Y nung trong không khí tới

62

Phát hành PDF bởi Ths Nguyễn Thanh Tú

Đăng ký Word doc qua Zalo 0905779594 Email thanhtuqn88@gmail.com


khối lượng không đổi thu được chất rắn có khối lượng giảm 10,42 gam so với khối lượng của Y.

Nếu làm khô cẩn thận dung dịch X thì thu được hỗn hợp muối khan Z (giả sử quá trình làm khô

không xảy ra phản ứng hóa học). Phần trăm khối lượng FeSO 4 trong Z gần nhất với giá trị nào sau

đây?

A. 18 B. 20 C. 24 D. 22

2

Fe

: x

FeOH

: x

Fe

NO : 0,08 Fe O

Fe

Fe

O Cu

29,12Mg

NH : 0,02

MgO

SO : 0,64

Cu

NO : 0,14

CuO

H 2

: 0, 22

BT:H

H2O : 0,38

BTKL

m 80,36g m m m m 18,56g

3

2

3 2

Fe

FeOH

2 3

2

3

Mg

BaOH du

MgO

2

t

ddX

Y Mg OH

2

2

CuO

3 4

H

2SO 4:0,64

Cu OH

BT:N

2

BaSO

4

4

BaSO

2

4 mgiam 10,42g

4

X KL X 2

SO4 NH4

n 2n 3n 2n 2n 2n n 1, 26 mol

2 3 2 2 2

OH X Fe Fe Mg Cu SO 4 NH 4

t

Y OH

FeOH Fe

2

OH

Chat ran H2O

3

x 1, 26

n n

x n

OH

Fe OH

H

Fe OH Fe OH

2

2O

2

3

2

x 1, 26

mchat ran giam

n

H2O m 10, 42 18 OH

17x x 0,115

FeOH2 FeOH3

2

0,115.152

%m

FeSO4X

.100% 21,75%

80,36

63

Phát hành PDF bởi Ths Nguyễn Thanh Tú

Đăng ký Word doc qua Zalo 0905779594 Email thanhtuqn88@gmail.com


I. KẾTLUẬN

PHẦN 3: KẾT LUẬN

Sau thời gian thực hiện nhiệm vụ nghiên cứu chúng tôi đạt được một số mục đích đề ra

của chuyên đề. Cụ thể:

1. Xây dựng được một số vấn đề lí thuyết cơ bản và nâng cao về kim loại nhóm VIIB và

VIIIB cho học sinh chuyên hoá nhằm giúp các em vận dụng để giải được các dạng bài

tập liên quan trong các kì thi học sinh giỏi khu vực và quốcgia…

2. Tiến hành xây dựng được hệ thống bài tập kim loại nhóm VIIB và VIIIB (với Fe, Co,

Ni) từ cơ bản đến nâng cao có kèm theo hướng dẫn giải giúp cho việc giảng dạy ở trường

chuyên và bồi dưỡng học sinh giỏi môn hóa học gồm:

Các bài tập cơ bản: những dạng bài này tương đối đơn giản, ngắn gọn giúp củng

cố chắc kiến thức lý thuyết làm cơ sở học nâng cao .

Phân một số dạng bài tập thường gặp trong các đề thi nhằm mục đích rèn kỹ năng,

kĩ sảo cho học sinh.

Thứ ba: bài tập mang tính chất tổng hợp trong đề thi HSG các năm giúp HS có thể

tự nghiên cứu lí thuyết và giải các bài tập nâng cao.

3. Đã áp dụng giảng dạy chuyên đề này trong quá trình ôn luyện đội tuyển thi Học sinh

giỏi cáccấp.

Kim loại chuyển tiếp và các dạng bài tập về mảng kim loại chuyển tiếp nói chung

và phần nhóm VIIB và VIIIB là một trong các phần phức tạp trong mảng kiến thức về

hoá học vô cơ do nội dung kiến thức phức tạp và rộng, trong khi yêu cầu về kiến thức đối

với đề thi cũng cao hơn nhiều so với nội dung trong sách giáo khoa chuyên hóa học. Từ

thực tế này, chúng tôi mong muốn được góp phần vào việc làm phong phú hơn các nội

dung kiến thức của phần kim loại chuyển tiếp, nhằm đưa các kiến thức đến gần hơn với

học sinh chuyên và HSG dự thi học sinh giỏi.

64

Phát hành PDF bởi Ths Nguyễn Thanh Tú

Đăng ký Word doc qua Zalo 0905779594 Email thanhtuqn88@gmail.com


DANH MỤC TÀI LIỆU THAM KHẢO

1. Đề thi chọn HSG Quốc gia cácnăm.

2. Đề thi chọn đội tuyển học sinh thiOlympic

3. Hóa học vô cơ – Tập 2 - Nguyễn ĐứcVận

4. Hóa học Vô cơ – Tập 3 – HoàngNhâm

5. Bài tập đại cương và vô cơ – Nguyễn Duy Ái, Đào HữuVinh

6. Tuyển tập đề thi HSG Quốc gia và đề thi chọn đội tuyển thi Olympic Quốc tế cácnăm.

7. http://chemistry.about.com/

8. http://edu.net.vn

65

Phát hành PDF bởi Ths Nguyễn Thanh Tú

Đăng ký Word doc qua Zalo 0905779594 Email thanhtuqn88@gmail.com


MỤC LỤC

NỘI DUNG

Trang

PHẦN 1: MỞ ĐẦU 1

I. Lí do chọn chuyên đề 1

II. Mục đích nghiên cứu 1

III. Nhiệm vụ 2

IV. Điểm mới của đề tài 2

PHẦN 2: NỘI DUNG 3

Chương 1: Sơ lược về nguyên tố kim loại nhóm VIIB và nhóm VIIIB 3

I: Sơ lược về nguyên tố kim loại nhóm VIIB 3

II. Sơ lược về nguyên tố kim loại nhóm VIIIB 14

Chương 2: Một số bài tập kim loại nhóm VIIB và nhóm VIIIB 19

I. Bài tập lý thuyết 19

I.1. Nhóm VIIB 19

II.2. Nhóm VIIIB (Sắt, Coban,Niken) 21

II.Bài tập tìm chất 22

III. Bài tập phức chất 26

II. 3.1. sơ lược về phức chất 26

II. 3.2. Một số bài tập phức chất 27

IV. Bài tập cân bằng oxi hóa khử, pin điện 35

IV.1. Lý thuyết cần nhớ 35

III.2. Bài tập 36

V. Một số bài toán khác 50

PHẦN 3: KẾT LUẬN 64

Tài liệu tham khảo 65

66

Phát hành PDF bởi Ths Nguyễn Thanh Tú

Đăng ký Word doc qua Zalo 0905779594 Email thanhtuqn88@gmail.com


67

Phát hành PDF bởi Ths Nguyễn Thanh Tú

Đăng ký Word doc qua Zalo 0905779594 Email thanhtuqn88@gmail.com


HỘI CÁC TRƯỜNG THPT CHUYÊN

KHU VỰC DUYÊN HẢI – ĐỒNG BẰNG BẮC BỘ

HỘI THẢO KHOA HỌC LẦN THỨ XI

MÔN HÓA HỌC

CHUYÊN ĐỀ

XÂY DỰNG HỆ THỐNG CÂU HỎI

VÀ BÀI TẬP NHÓM VIIB VÀ VIIIB

1

Phát hành PDF bởi Ths Nguyễn Thanh Tú

Đăng ký Word doc qua Zalo 0905779594 Email thanhtuqn88@gmail.com


MỤC LỤC

Phần 1. MỞ ĐẦU

* Lí do chọn đề tài……………………………………………………………………………4

* Mục đích của đề tài…………………………………………………………………………5

Phần 2. NỘI DUNG

A. CƠ SỞ LÝ THUYẾT

I. CÁC NGUYÊN TỐ NHÓM VIIB ………………………………………………………...6

I.1. Nhận xét chung về các nguyên tố nhóm VIIB……………………………………………6

I.2. Trạng thái thiên nhiên và thành phần các đồng vị………………………………………..9

I.3. Điều chế Mn, Tc, Re…………………………………………………………………….10

I.4. Tính chất vật lý của Mn, Tc, Re và ứng dụng…………………………………………..11

I.5. Tính chất hóa học của Mn, Tc, Re………………………………………………………11

II. CÁC NGUYÊN TỐ NHÓM VIII B……………………………………………………...13

II.1. Nhận xét chung về các nguyên tố nhóm VIIIB………………………………………...13

II.2. CÁC KIM LOẠI HỌ SẮT……………………………………………………………..15

II.2.1. Nhận xét chung về kim loại họ sắt…………………………………………………...15

II.1.2. Trạng thái thiên nhiên và thành phần các đồng vị……………………………………17

II.1.3. Điều chế Fe, Co, Ni…………………………………………………………………..18

II.1.4. Tính chất lý học của Fe, Co, Ni và ứng dụng………………………………………...20

II.1.5. Tính chất hóa học của Fe, Co, Ni…………………………………………………….21

II.1.6. Hợp chất cacbonyl của Fe, Co, Ni……………………………………………………26

II.1.7. Các xianua của Fe (II), Co (II), Ni (II) ………………………………………………28

II.1.8. Phức chất xianua của Fe (III) và Co (III) ……………………………………………30

II.3. CÁC KIM LOẠI HỌ PLATIN………………………………………………………...32

II.3.1. Ruteni ………………………………………………………………………………..33

II.3.2. Rođi ………………………………………………………………………………….34

II.3.3. Palađi ………………………………………………………………………………...35

II.3.4. Osimi…………………………………………………………………………………36

II.3.5. Iriđi…………………………………………………………………………………...36

II.3.6. Platin…………………………………………………………………………………36

Phát hành PDF bởi Ths Nguyễn Thanh Tú

Đăng ký Word doc qua Zalo 0905779594 Email thanhtuqn88@gmail.com

2


B. BÀI TẬP THAM KHẢO

I. Bài tập về Mn, Te, Re…………………………………………………………………….38

I.1. Bài tập lý thuyết, viết phương trình phản ứng, giải thích hiện tượng, điều chế, chuỗi phản

ứng, xác định công thức, ……………………………………………………………………38

I.2. Bài tập định lượng: xác định công thức, giản đồ Latimer, chuẩn độ, phức chất, ……....54

II. Bài tập về Fe……………………………………………………………………………..59

II.1. Bài tập lý thuyết, viết phương trình phản ứng, giải thích hiện tượng, điều chế, chuỗi

phản ứng, xác định công thức, ……………………………………………………………...59

II.2. Bài tập định lượng: xác định công thức, tinh thể, chuẩn độ, phức chất, ………………66

III. Bài tập về Co…………………………………………………………………………….72

III.1. Bài tập lý thuyết, viết phương trình phản ứng, giải thích hiện tượng, điều chế, chuỗi

phản ứng, xác định công thức, ……………………………………………………………...72

III.2. Bài tập về phức chất…………………………………………………………………..75

IV. Bài tập về Ni…………………………………………………………………………….84

IV.1. Bài tập lý thuyết, viết phương trình phản ứng, giải thích hiện tượng, điều chế, chuỗi

phản ứng, xác định công thức, ……………………………………………………………...84

IV.2. Bài tập tinh thể…………………………………………………………….. ………... 87

V. Bài tập về Ru, Rh, Pd, Re, Os, Ir, Pt ……………………………………………………89

* Bài tập về phức chất………………………………………………………………………89

* Bài tập tinh thể. …………………………………………………………………………..94

VI. Một số dạng bài tập phức chất thường gặp……………………………………………...97

VI.1. Dạng bài tập về đồng phân của phức chất ……………………………………………98

VI.2. Dạng bài tập thuyết VB và thuyết trường tinh thể trong phức chất …………………102

VI.3. Bài tập phản ứng trong phức chất …………………………………………………...106

Phần 3. KẾT LUẬN

Phát hành PDF bởi Ths Nguyễn Thanh Tú

Đăng ký Word doc qua Zalo 0905779594 Email thanhtuqn88@gmail.com

3


1. Lí do chọn đề tài

Phần 1. MỞ ĐẦU

Các kim loại chuyển tiếp nói chung và các kim loại nhóm VIIB, VIIIB nói riêng có những

tính chất quan trọng và đóng vai trò hữu ích trong công nghiệp và đời sống. Các kim loại

chuyển tiếp có thể tạo hợp chất có màu, có thể có nhiều trạng thái oxi hóa khác nhau, thể

hiện khuynh hướng tạo phức rõ rệt. Chúng có tính chất tuyệt vời làm chất xúc tác, palađi có

khả năng hấp phụ một lượng lớn hiđro (tới 900 thể tích hiđro trên một thể tích kim loại),

palađi cũng như reni, niken, ruteni, platin,… làm chất xúc tác cho nhiều phản ứng. Đồng vị

60

27Co được dùng rộng rãi trong y khoa để chữa bệnh ung thư. Các phức chất tâm ruteni đang

được nghiên cứu để tìm kiếm các tính chất chống ung thư. Reni được dùng trong công

nghiệp điện vì có độ dẫn điện cao, là nguyên liệu rất tốt để làm dây tóc bóng đèn điện, bền

hơn vonfram. Các kim loại chuyển tiếp được dùng để sản xuất những hợp kim có tính chất

đặc biệt, ví dụ hợp kim Re và Pb dùng làm pin nhiệt điện. Một số hợp kim của coban dùng

trong kỹ thuật quốc phòng và kỹ thuật tên lửa. Hợp kim palađi với bạc được dùng trong thiết

bị liên lạc, đặc biệt để chế tạo các tiếp điểm. Một hợp kim của ruteni với molypden có

tính siêu dẫn ở 10,6K. Iriđi tinh khiết được dùng để chế tạo một số dụng cụ khoa học, hợp

kim chứa 90% platin và 10% iriđi cũng được sử dụng cho mục đích này, từ hợp kim này

người ta chế tạo các mẫu chuẩn mét và kilogam quốc tế,…

Vì vậy việc nghiên cứu các kim loại chuyển tiếp nói chung và các kim loại nhóm VIIB,

VIIIB là hết sức cần thiết. Trong thực tế giảng dạy ở các trường chuyên, việc dạy và học

chuyên đề này gặp một số khó khăn:

- Đã có tài liệu giáo khoa dành riêng cho học sinh chuyên hóa, nhưng nội dung kiến thức lí

thuyết chưa đủ để trang bị cho học sinh, chưa đáp ứng được yêu cầu của kì thi học sinh giỏi

các cấp.

- Chưa có sách bài tập dành riêng cho học sinh chuyên hóa về chuyên đề này. Trong các tài

liệu giáo khoa chuyên hóa lượng bài tập rất ít, chưa thật sự rộng và sâu. Trong các tài liệu

tham khảo khác, bài tập dành cho giảng dạy và học tập của lớp chuyên còn nằm rải rác, chưa

phong phú và chưa được phân loại rõ ràng, chưa đủ để cho học sinh học tập, ôn luyện chuẩn

bị cho các kì thi học sinh giỏi các cấp.

Phát hành PDF bởi Ths Nguyễn Thanh Tú

Đăng ký Word doc qua Zalo 0905779594 Email thanhtuqn88@gmail.com

4


- Để khắc phục điều này, mỗi giáo viên dạy chuyên phải tự biên soạn nội dung chương trình

dạy và xây dựng hệ thống bài tập để phục vụ cho công việc giảng dạy của mình.

Xuất phát từ thực tiễn đó, là giáo viên trường chuyên, chúng tôi rất mong có được một

nguồn tài liệu có giá trị và phù hợp để giáo viên giảng dạy - bồi dưỡng học sinh giỏi các cấp

và cũng để cho học sinh có được tài liệu nghiên cứu, tham khảo.

* Mục đích của đề tài

Cung cấp những kiến thức cơ bản và chuyên sâu về các kim loại nhóm VIIB, VIIIB

Chọn lọc, sưu tầm các câu hỏi, bài tập, các đề thi để các em tự rèn luyện kĩ năng làm bài,

rèn luyện kĩ năng vận dụng các kiến thức vào giải các bài tập có liên quan.

Nội dung của chuyên đề bao gồm 2 phần:

A. Khái quát kiến thức cơ bản.

B. Hệ thống câu hỏi, bài tập.

Phát hành PDF bởi Ths Nguyễn Thanh Tú

Đăng ký Word doc qua Zalo 0905779594 Email thanhtuqn88@gmail.com

5


Phần 2. NỘI DUNG

A. CƠ SỞ LÝ THUYẾT

Giới thiệu chung về các nguyên tố chuyển tiếp:

Các nguyên tố chuyển tiếp đều có những đặc trưng sau:

- Tất cả chúng đều là kim loại.

- Trừ một số kim loại quí tương đối trơ hóa học, đa số các nguyên tố chuyển tiếp có độ

dương điện cao, nghĩa là có thế điện cực tương đối thấp (âm), nên chúng tan được trong các

axit vô cơ thông thường, giải phóng hiđro.

- Trừ một vài ngoại lệ, hầu hết các nguyên tố chuyển tiếp thể hiện nhiều mức oxi hóa khác

nhau.

- Một số lớn các hợp chất của các nguyên tố chuyển tiếp có tính thuận từ.

- Nhiều hợp chất của các nguyên tố chuyển tiếp có màu, nghĩa là có phổ hấp phụ electron.

- Các nguyên tố chuyển tiếp có khả năng tạo thành phức chất.

I. CÁC NGUYÊN TỐ NHÓM VIIB

I.1. Nhận xét chung về các nguyên tố nhóm VIIB

Phát hành PDF bởi Ths Nguyễn Thanh Tú

Đăng ký Word doc qua Zalo 0905779594 Email thanhtuqn88@gmail.com

6


Nhóm VIIB gồm các nguyên tố Mangan (Mn), Tecnexi (Tc) và Reni (Re) thuộc họ d ở các

chu kỳ 4; 5; 6 của bảng tuần hoàn.

Đặc điểm của các nguyên tố nhóm VIIB:

Nguyên tố Mn Tc Re

Số thứ tự 25 43 75

Cấu hình electron nguyên tử Ar3d 5 4s 2 Kr4d 5 5s 2 Xe4f 14 5d 5 6s 2

Năng lượng Ion I 1 7,43 7,28 7,79

hoá (eV)

I 2 15,63 15,26 13,1

I 3 33,69 29,5 26,0

Bán kính nguyên tử (A 0 ) 1,30 1,36 1,37

Bán kính ion

M 2+ ( A 0 )

0,91 0,95 -

M 3+ ( A 0 )

0,70 - -

Phát hành PDF bởi Ths Nguyễn Thanh Tú

Đăng ký Word doc qua Zalo 0905779594 Email thanhtuqn88@gmail.com

7


M 4+ ( A 0 )

0,52 0,72 0,72

M 7+ ( A 0 )

0,46 0,57 0,57

Thế điện cực chuẩn E 0 (V) -1,18 (Mn 2+ /Mn) +0,4 (Tc +2 /Tc) +0,3 (Re 3+ /Re)

Số oxi hoá đặc trưng (bền) +2, +4, +7 +7 +4, +7

% nguyên tử trong vỏ trái đất 0,09 - 10 -7

Cả ba nguyên tố có số electron phân bố ở các phân lớp ngoài cùng là: (n - 1)d 5 ns 2

(n - 1)d 5 ns 2

Mn, Tc, Re là những nguyên tố đa hóa trị, gây ra bởi các electron hóa trị. Bậc oxi hóa đặc

trưng của mangan là +2, +4 và +7, ngoài ra còn tạo ra những hợp chất ứng với bậc oxi hóa

+3, +4 và +6. Bậc oxi hóa đặc trưng của tecnexi và reni là +7. Theo chiều tăng của bậc oxi

hóa, khuynh hướng tạo ra anion phức tăng lên, còn khuynh hướng tạo ra cation phức giảm

xuống.

Tương tự các halogen (đặc biệt là với clo), mangan và reni tạo ra hợp chất Mn 2 O 7 và

Re 2 O 7 có tính chất tương tự Cl 2 O 7 ; các muối pemanganat ( MnO ), renat (

4

Re O ) đồng hình

4

với muối peclorat ( ClO ); cả hai axit pemanganic (HMnO

4

4 ) và axit pecloric (HClO 4 ) đều là

axit đơn chức và đều có tính oxi hóa mạnh.

Trong số các hợp chất của mangan, có một số giống với hợp chất của crom và của sắt.

2

Mangan và crom tạo ra oxit thấp (tính bazơ) và trioxit (tính axit). Muối cromat ( CrO ) đồng

4

2

hình với muối manganat ( MnO ). Mangan cũng tạo nên phèn tương tự như phèn sắt

4

K 2 SO 4 .Mn 2 (SO 4 ) 3 .24H 2 O; hoặc dạng muối kép như (NH 4 ) 2 SO 4 .MnSO 4 .6H 2 O tương tự muối

kép của sắt (II). Ngoài ra mangan cũng tạo ra các oxit MnO, Mn 2 O 3 , Mn 3 O 4 (MnO.Mn 2 O 3 )

tương tự các oxit của sắt.

Bán kính nguyên tử tăng từ Mn đến Re, nhưng không đáng kể, đặc biệt khi chuyển từ Tc

đến Re; do đó Tc và Re có tính chất gần nhau hơn so với Mn.

Dưới đây là sơ đồ thế điện cực của mangan và reni:

• Trong môi trường axit:

Phát hành PDF bởi Ths Nguyễn Thanh Tú

Đăng ký Word doc qua Zalo 0905779594 Email thanhtuqn88@gmail.com

8


• Trong môi trường kiềm:

Về mặt cấu tạo, các dạng thù hình của mangan đều kết tinh theo kiểu lập phương; còn reni

kết tinh theo kiểu lục phương.

Ví dụ: dạng thù hình γ - Mn tồn tại ở 1070 - 1130 0 C, có dạng cấu trúc mạng lập phương

tâm diện, có cạnh α = 3,862 A 0 , theo dạng cấu trúc của Cu. Còn dạng thù hình δ - Mn theo

dạng α - Fe, nghĩa là kết tinh theo kiểu lập phương tâm khối, có cạnh α = 3,081 A 0 .

I.2. Trạng thái thiên nhiên và thành phần các đồng vị

Khoáng vật chủ yếu của mangan là pirolusit (MnO 2 ); ngoài ra một số khoáng vật khác có

chứa mangan như bronit (Mn 2 O 3 ); manganit Mn 2 O 3 .H 2 O; và các sunfua như MnS; MnS 2 .

Tecnexi không có trong tự nhiên.

Phát hành PDF bởi Ths Nguyễn Thanh Tú

Đăng ký Word doc qua Zalo 0905779594 Email thanhtuqn88@gmail.com

9


Lượng reni ở trong vỏ quả đất có rất ít. Quặng giàu reni nhất là các molipđenit cũng chỉ

chứa khoảng 2.10 -3 % về khối lượng.

Trong cơ thể người, mangan có khoảng 4.10 -4 % chứa trong tim, gan và tuyến thượng thận,

ảnh hưởng đến sự trưởng thành của cơ thể và sự tạo máu.

Mangan có nhiều đồng vị từ 49 Mn đến 57 Mn, trong đó chỉ có 55 Mn là đồng vị thiên nhiên

chiếm 100%. Đồng vị phóng xạ bền nhất là 53 Mn có chu kỳ bán hủy là 140 năm và kém bền

nhất là 49 Mn có chu kỳ bán hủy là 0,4 giây.

Tecnexi là nguyên tố nhân tạo, các đồng vị đều có tính phóng xạ, trong đó đồng vị 99 Tc

bền nhất có chu kỳ bán hủy là 2,12.10 5 năm.

Reni có 14 đồng vị. Các đồng vị thiên nhiên là 185 Re (37,07%); 187 Re (62,93%); còn lại là

các đồng vị phóng xạ.

I.3. Điều chế Mn, Tc, Re

Mangan được điều chế bằng phương pháp nhiệt nhôm từ các oxit MnO hoặc Mn 3 O 4 :

3Mn 3 O 4 + 8Al

0

t

4Al 2 O 3 + 9Mn

Cũng có thể điều chế bằng phương pháp nhiệt silic:

0

t

MnO 2 + Si Mn + SiO 2

Trong công nghiệp mangan được điều chế bằng cách dùng cacbon để khử oxit mangan

trong lò điện.

Mangan cũng được điều chế bằng phương pháp điện phân dung dịch muối sunfat.

Mangan tinh khiết được điều chế bằng cách điện phân dung dịch MnCl 2 với catot bằng

thủy ngân. Mangan hòa tan trong thủy ngân tạo ra hỗn hống Mn-Hg. Chưng cất hỗn hống

trong chân không, tách được Mn và thu hồi lại Hg.

Người ta điều chế một lượng nhỏ tecnexi (vài mg) trong lò phản ứng hạt nhân khi bắn phá

molipđen bằng nơtron:

98

42 Mo + 1 0 n → 99

42 Mo +

99

42

99

Mo →

43Tc + β-

Người ta cũng điều chế tecnexi trong lò phản ứng hạt nhân khi phân hạch urani.

Người ta điều chế reni bằng cách dùng H 2 để khử amoni renat ở nhiệt độ cao:

2NH 4 ReO 4 + 4H 2

0

t

2Re + N 2 + 8H 2 O

10

Phát hành PDF bởi Ths Nguyễn Thanh Tú

Đăng ký Word doc qua Zalo 0905779594 Email thanhtuqn88@gmail.com


I.4. Tính chất vật lý của Mn, Tc, Re và ứng dụng

Tùy theo phương pháp điều chế, mangan tạo ra ở bốn dạng thù hình.

Mangan điều chế bằng phương pháp nhiệt nhôm, tồn tại ở hai dạng α-Mn và β-Mn. Dạng α

- Mn tồn tại ở nhiệt độ thường có khối lượng riêng là 7,21g/cm 3 . Dạng β - Mn tồn tại ở nhiệt

độ cao (742 - 1070 0 C) có khối lượng riêng là 7,29 g/cm 3 .

Nếu mangan được điều chế bằng phương pháp điện phân, mangan tồn tại ở dạng γ - Mn,

bền trong khoảng 1070 - 1130 0 C; có khối lượng riêng là 7,21 g/cm 3 . Dạng thù hình tồn tại ở

nhiệt độ cao hơn 1130 0 C là dạng δ - Mn:

α - Mn β - Mn γ - Mn δ –Mn

Tồn tại: t 0 thường 1070 0 C 1130 0 C > 1130 0 C

Các dạng α - Mn và β - Mn đều cứng và giòn; dạng γ - Mn thì mềm và dẻo.

Mn và Re có màu trắng bạc, còn Tc có màu xám.

Một số hằng số vật lý quan trọng của Mn, Tc, Re:

Tính chất Mn Tc Re

Độ âm điện 1,55 1,9 1,9

Khối lượng riêng (g/cm 3 ) 7,47 11,5 20,5

Nhiệt độ nóng chảy ( 0 C) 1244 2140 3180

Nhiệt độ sôi ( 0 C) 2080 4900 5900

Một lượng khá lớn mangan được dùng để điều chế hợp kim fero – mangan (60% - 90%

Mn và 40% -10% Fe) khi khử hỗn hợp sắt và quặng mangan. Loại hợp kim này rất bền và

cứng, thường dùng làm ổ bi, các bộ phận máy nghiền, làm đường ray xe lửa. Người ta cũng

dùng hợp kim của mangan để điều chế trực tiếp gang trắng trong lò cao. Hợp kim bronzơ -

mangan (95% Cu và 5% Mn) có độ bền cơ học cao.

Reni được dùng trong công nghiệp điện vì có độ dẫn điện cao (lớn hơn Hg 4,5 lần và thấp

hơn vonfram) là nguyên liệu rất tốt để làm dây tóc bóng đèn điện, bền hơn vonfram. Hợp

kim Re và Pb dùng làm pin nhiệt điện. Ngoài ra, reni làm chất xúc tác cho nhiều phản ứng

hóa học hữu cơ.

I.5. Tính chất hóa học của Mn, Tc, Re

Hoạt tính hóa học của kim loại trong nhóm giảm dần từ Mn đến Re do đó khả năng phản

ứng với các chất giảm dần.

Phát hành PDF bởi Ths Nguyễn Thanh Tú

Đăng ký Word doc qua Zalo 0905779594 Email thanhtuqn88@gmail.com

11


Mangan và reni không phản ứng trực tiếp với hiđro, nhưng khí H 2 tan được trong mangan

nóng chảy (70 - 100 cm 3 H 2 trong 100 gam Mn ở nhiệt độ 1244 0 C).

Reni không hòa tan được H 2 .

Trong không khí, mangan ở dạng khối rắn, không bị oxi hóa, ngay cả khi đun nóng vì

được bao bọc bởi một lớp oxit mỏng bảo vệ cho kim loại; nếu ở trạng thái vụn thì dễ bị oxi

hóa hơn; nhưng nói chung mangan rất khó phản ứng với oxi, tạo ra Mn 3 O 4 ở 940 0 C:

0

940 C

3Mn + 2O 2 Mn 3 O 4

Khi đun nóng bột reni trong khí quyển oxi ở 300 0 C tạo ra oxit Re 2 O 7 .

Mangan và reni đều phản ứng trực tiếp với lưu huỳnh, selen, telu tạo ra các hợp chất như

MnS, MnSe, MnSe 2 , ReSe 2 , MnTe, MnTe 2 ...

Mangan hóa hợp trực tiếp với nitơ tạo ra Mn 3 N 2 ở nhiệt độ khoảng 600 – 1000 0 C khi cho

bột mangan tác dụng với nitơ:

0

t

3Mn + N 2 Mn 3 N 2

Mangan và reni phản ứng trực tiếp với photpho khi nung nóng tạo ra các chất Mn 3 P 2 , MnP,

ReP...

Mangan hóa hợp trực tiếp với cacbon và silic tạo ra các hợp chất Mn 3 C, Mn 7 C 3 , Mn 3 Si,

MnSi ...

Cả mangan và reni đều phản ứng mạnh với các halogen tạo thành muối dạng MX 2 , ví dụ:

Mn + Cl 2 → MnCl 2

Mangan có thế điện cực đứng trước Zn, nên khi đun nóng phân hủy được H 2 O, đặc biệt khi

có tạp chất như cacbon, mangan dễ bị nước và không khí ẩm ăn mòn. Tecnexi và reni không

có khả năng đó.

Mn + 2H 2 O → Mn(OH) 2 + H 2

Mangan tan trong các axit loãng không có tính oxi hóa như HCl, H 2 SO 4 tạo ra H 2 . Tecnexi

và reni không có khả năng đó.

Mn + H 2 SO 4 loãng → MnSO 4 + H 2 ↑

Mangan tan trong H 2 SO 4 đặc tạo ra SO 2 , nếu H 2 SO 4 đặc nguội phản ứng xảy ra rất chậm,

nhưng khi đun nóng phản ứng xảy ra rất nhanh:

Mn + 2H 2 SO 4 đặc → MnSO 4 + SO 2 ↑ + 2H 2 O

Với HNO 3 tạo ra khí NO:

Phát hành PDF bởi Ths Nguyễn Thanh Tú

Đăng ký Word doc qua Zalo 0905779594 Email thanhtuqn88@gmail.com

12


3Mn + 8HNO 3 → 3Mn(NO 3 ) 2 + 2NO↑ + 4H 2 O

Mangan không thụ động khi ngâm trong HNO 3 đặc nguội.

Tecnexi và reni phản ứng mạnh với HNO 3 và H 2 SO 4 đặc tạo ra hợp chất ứng với hóa trị

bền: Tc (VII) và Re (VII)

3Tc + 7HNO 3 đặc → 3HTcO 4 + 7NO + 2H 2 O

2Re + 7H 2 SO 4 đặc → 2HReO 4 + 7SO 2 + 6H 2 O

Mangan không phản ứng với kiềm.

II. CÁC NGUYÊN TỐ NHÓM VIIIB

II.1. Nhận xét chung về các nguyên tố nhóm VIIIB

Nhóm VIIIB gồm 9 nguyên tố: sắt (Fe), coban (Co), niken (Ni), ruteni (Ru), rođi (Rh),

palađi (Pd), osimi (Os), iriđi (Ir), platin (Pt) thuộc họ d ở các chu kỳ 4; 5; 6 của bảng tuần

hoàn.

Khi so sánh tính chất vật lý và tính chất hóa học cơ bản của các nguyên tố nhóm VIIIB,

người ta thấy các nguyên tố sắt, coban và niken có tính chất tương tự nhau, nên được xếp

chung thành họ sắt; sáu nguyên tố còn lại được gọi là các kim loại họ platin theo tên của

nguyên tố phổ biến nhất trong số đó, bởi vì chúng có những tính chất giống nhau và giống

platin. Trong họ platin các nguyên tố sắp theo chiều thẳng đứng có tính chất giống nhau

nhiều hơn so với cách sắp xếp theo hàng ngang.

Về tính chất, các nguyên tố thuộc nhóm VIIIB có những nét chung như sau:

Phát hành PDF bởi Ths Nguyễn Thanh Tú

Đăng ký Word doc qua Zalo 0905779594 Email thanhtuqn88@gmail.com

13


- Đều có tính chất của kim loại, màu sắc từ xám đến xám trắng; rất khó nóng chảy và rất khó

bay hơi; thể tích nguyên tử thấp.

- Tất cả đều có khả năng hấp thụ hiđro trên bề mặt ít hoặc nhiều và gây ra hoạt tính cao của

hiđro (hiđro hoạt động).

- Tất cả đều có tác dụng xúc tác cho phản ứng hóa học vô cơ hoặc hữu cơ.

- Đều có khuynh hướng tạo phức, đặc trưng nhất là phản ứng tạo phức với NH 3 , với CO và

cả với NO.

- Có khả năng tạo ra nhiều hợp chất có hóa trị khác nhau và có thể dễ chuyển hóa từ trạng

thái hóa trị này đến trạng thái hóa trị khác.

- Đều tạo ra hợp chất có màu ngay cả ở trạng thái tự do (dạng hiđrat hóa).

- Hiđroxit của chúng đều có tính bazơ yếu, hoặc axit yếu, hoặc có tính lưỡng tính.

- Có ái lực yếu đối với oxi và giảm dần từ trái sang phải; nhưng lại có ái lực mạnh với lưu

huỳnh và tăng dần từ trái sang phải. Về mặt này, các nguyên tố nhóm VIIIB tương tự nhóm

IB.

Về cấu hình elcctron, nguyên tố nhóm VIIIB đều thuộc họ d mà nguyên tử lắp đầy dần các

obitan d ở lớp (n - l) (n là số thứ tự của chu kỳ).

Tinh thể các kim loại đó có cấu trúc theo mạng như sau:

Fe

Co

Ni

Mạng lập phương tâm khối Mạng lục phương Mạng lập phương tâm diện

a = 2,8664 0 A

Ru

Mạng lục phương

a = 2,7085 0 A

c = 4,2816

Os

0

A

Mạng lục phương

a = 1,5063 0 A

c = 4,0795

Rh

0

A

Mạng lập phương tâm diện

a = 3,8044 0 A

Ir

Mạng lập phương tâm diện

a = 3,5238 A

0

Pd

Mạng lập phương tâm diện

a = 3,8907 A

0

Pt

Mạng lập phương tâm diện

14

Phát hành PDF bởi Ths Nguyễn Thanh Tú

Đăng ký Word doc qua Zalo 0905779594 Email thanhtuqn88@gmail.com


a = 2,7353 0 A

c = 4,3191

0

A

a = 3,8938 0 A

a = 3,9239 0 A

II.2. CÁC KIM LOẠI HỌ SẮT

II.2.1. Nhận xét chung về kim loại họ sắt

Cả ba kim loại sắt, coban và niken - ở trạng thái kim loại cũng như ở trạng thái hợp chất

đều có những đặc điểm giống nhau về tính chất.

Cả ba kim loại đều tạo ra các muối có số oxi hóa +2; ngoài ra sắt tạo ra các muối có số oxi

hóa +3; coban (II) dễ dàng chuyển thành Co (III) nhưng thường gặp trong các hợp chất

phức; với niken thì chủ yếu tạo ra hợp chất Ni (II).

Khuynh hướng tạo ra hợp chất hóa trị I tăng từ sắt đến niken; niken dễ dàng tạo ra hợp

chất hóa trị I tương tự đồng.

2

Hóa trị cực đại của sắt là VI (trong ferat FeO ) tương tự mangan; trong khi đó hóa trị của

4

coban và niken là (IV) trong các hợp chất của đioxit dễ dàng bị phân hủy tách ra oxi.

Electron hóa trị

Một số đặc điểm của Fe - Co – Ni

Bán kính nguyên tử ( 0 A )

Bán kính ion M 2+ ( 0 A )

Bán kính ion M 3+ ( 0 A )

Năng lượng ion hóa I 1 (eV)

Thế điện cực được tóm tắt theo sơ đồ sau:

Trong môi trường axit:

Fe Co Ni

3d 6 4s 2

1,26

0,8

0,67

7,9

3d 7 4s 2

1,25

0,78

0,64

7,86

3d 8 4s 2

1,24

0,74

-

7,63

Phát hành PDF bởi Ths Nguyễn Thanh Tú

Đăng ký Word doc qua Zalo 0905779594 Email thanhtuqn88@gmail.com

15


Trong môi trường bazơ:

Về cấu tạo tinh thể:

Sắt có bốn dạng thù hình, có cấu tạo như hình dưới:

Mạng tinh thể các dạng thù hình của sắt

Ở điều kiện thường đến 770 0 C tồn tại dạng α - Fe có mạng lập phương tâm khối; ở 770 0 C

dạng α - Fe chuyển thành dạng β - Fe, mạng tinh thể không thay đổi nhưng độ dài giữa hai

nguyên tử tăng lên; đến 910 0 C chuyển thành dạng γ - Fe, mạng tinh thể thay đổi thành mạng

lập phương tâm diện; đến 1390 0 C lại chuyển thành mạng lập phương tâm khối là dạng δ -

Fe.

Phát hành PDF bởi Ths Nguyễn Thanh Tú

Đăng ký Word doc qua Zalo 0905779594 Email thanhtuqn88@gmail.com

16


Coban có hai dạng thù hình. Ở điều kiện thường đến 417 0 C tồn tại dạng α - Co có mạng

lục phương (a = 2,5063 A 0 , c = 4,0795 A 0 ); đến ~ 480 0 C tồn tại dạng β - Co có mạng lập

phương tâm diện (a = 3,5441 A 0 ).

Niken tồn tại hai dạng thù hình. Thấp hơn 250 0 C ở dạng α - Ni có mạng lục phương (a =

2,65 A 0 , c = 4,32 A 0 ); cao hơn 250 0 C chuyển thành dạng β – Ni với mạng lập phương tâm

diện (a = 3,523 A 0 ).

II.1.2. Trạng thái thiên nhiên và thành phần các đồng vị

Sắt thuộc các nguyên tố phổ biến nhất cấu tạo nên vỏ quả đất. Khoáng vật chủ yếu của sắt

là manhetit (Fe 3 O 4 ), hematit đỏ (Fe 2 O 3 ), hematit nâu [Fe 2 O 3 .2Fe(OH) 3 ]. Ngoài ra, một lượng

sắt khá lớn ở dạng khoáng chất xiđerit (FeCO 3 ) và cả trong quặng với lưu huỳnh, asen như

pirit FeS 2 ... tuy nhiên pirit không phải là chất dùng để điều chế sắt mà dùng điều chế H 2 SO 4 .

Hàm lượng của coban trong vỏ quả đất ít hơn nhiều so với sắt. Khoáng vật quan trọng của

coban là cobantin (CoAsS). Coban ở lẫn với các kim loại khác như Cu, Ni, Ag, Fe, Mn trong

các quặng đa kim.

Hàm lượng của niken trong vỏ quả đất nhiều hơn coban. Khoáng vật chủ yếu của niken là

penlađit là quặng sunfua của niken, đồng và sắt. Ngoài ra còn có các loại quặng khác như

gacnierit (NiSiO 3 .MgSiO 3 ), quặng smantit là quặng arsenua của Ni, Co, Fe.

Trong cơ thể động vật, sắt có trong hemoglobin, chức năng chính của phức chất này là

liên kết oxi phân tử và chuyển oxi đó vào các mô. Ngoài ra, còn có trong mioglobin là chất

protein dự trữ oxi trong cơ bắp; có trong phức chất feritin cũng là protein có chức năng tạo

nên những hợp chất khác chứa sắt cần cho cơ thể của sinh vật. Gan và lá lách là bộ phận

giàu sắt nhất trong cơ thể.

Coban thuộc nhóm nguyên tố quan trọng đối với con người, vitamin B 12 hay cobanamin là

phức chất của coban có chứa 4,5% Co về khối lượng, dùng để chữa bệnh thiếu máu.

Sắt có bốn đồng vị bền là 54 Fe (5,84%); 56 Fe (91,68%); 57 Fe (2,17%); 58 Fe (0,31%); trong

số các đồng vị phóng xạ thì đồng vị 55 Fe là bền hơn cả, có chu kỳ bán hủy là 2,9 năm. Coban

có tám đồng vị từ 54 Co đến 61 Co nhưng chỉ có 59 Co là đồngvị thiên nhiên (100%), số đồng vị

Phát hành PDF bởi Ths Nguyễn Thanh Tú

Đăng ký Word doc qua Zalo 0905779594 Email thanhtuqn88@gmail.com

17


còn lại đều là đồng vị phóng xạ trong đó bền nhất là 60 Co có chu kỳ bán hủy là 5,2 năm, kém

bền nhất là 54 Co với chu kỳ bán hủy là 0,18 giây.

Niken có 11 đồng vị từ 56 Ni đến 66 Ni, trong đó có năm đồng vị thiên nhiên là 58 Ni

(67,76%); 60 Ni (26,16%); 61 Ni (l,25%); 62 Ni (3,66%); 64 Ni (l,16%). Trong các đồng vị phóng

xạ thì đồng vị 59 Ni có chu bán hủy là 7,5.10 4 năm bền nhất, và đồng vị 65 Ni kém bền nhất có

chu kỳ bán hủy là 0,108 ngày đêm.

II.1.3. Điều chế Fe, Co, Ni

Sắt tinh khiết được điều chế bằng cách dùng H 2 tinh khiết để khử oxit sắt tinh khiết:

t

Fe 2 O 3 + 3H 0

2 2Fe + 3H 2 O

Quá trình khử xảy ra ở khoảng 175 - 270 0 C; ở nhiệt độ đó, sản phẩm thu được là loại sắt tự

cháy, khi tiếp xúc với không khí loại sắt này sẽ bị oxi hóa nhanh và nóng đỏ lên. Để thu

được sản phẩm có độ bền cao đối với không khí, quá trình khử phải thực hiện ở nhiệt độ cao

hơn khoảng 550 - 650 0 C; ở điều kiện đó, sắt thu được ở dạng bột mịn không còn có khả

năng tự cháy.

Quá trình khử xảy ra theo từng giai đoạn, ban đầu tạo ra Fe 3 O 4 , sau đó mới tạo thành Fe:

t

3Fe 2 O 3 + H 0

2 2Fe 3 O 4 + H 2 O

Nếu nung ở 573 0 C thì tạo ra oxit FeO:

t

Fe 2 O 3 + H 0

2 2FeO + H 2 O

Nếu quá trình khử xảy ra khi tăng nhiệt độ đến khoảng 700 - 1000 0 C không thu được dạng

bột mịn mà ở dạng sắt rèn tức là ở dạng khối chảy.

Có thể điều chế sắt bằng phương pháp nhiệt phân sắt pentacacbonyl Fe(CO) 5 :

0

t

Fe(CO) 5 Fe + 5CO↑

Sắt rất tinh khiết có thể điều chế bằng phương pháp điện phân dung dịch muối Fe (II)

chẳng hạn như FeSO 4 .FeCl 2 , với dương cực là tấm Fe - Cr, còn âm cực là sắt tinh khiết. Quá

trình điện phân phụ thuộc vào các yếu tố như pH của dung dịch, nồng độ của chất điện phân,

mật độ dòng của anot và catot.

Coban tinh khiết được điều chế bằng cách dùng H 2 để khử coban (II) oxit CoO hoặc các

oxit khác:

t

CoO + H 0

2 Co + H 2 O

Phát hành PDF bởi Ths Nguyễn Thanh Tú

Đăng ký Word doc qua Zalo 0905779594 Email thanhtuqn88@gmail.com

18


Khi nung đến 120 0 C quá trình khử đã bắt đầu xảy ra, đến 250 0 C thu được coban tự cháy.

Nếu nung đến khoảng 400 0 C, sản phẩm thu được ở dạng bột hoàn toàn bền đối với không

khí khô.

Sản phẩm thu được phụ thuộc vào nhiệt độ, chẳng hạn ở 450 0 C thu được dạng thù hình β -

Co (mạng lập phương), ở nhiệt độ thấp hơn 450 0 C sẽ thu được dạng α - Co (mạng lục

phương).

Coban tinh khiết có thể điều chế bằng phương pháp điện phân dung dịch CoSO 4 trong

nước với dương cực bằng thép không gỉ đã được xử lý bề mặt và âm cực bằng tấm chì tinh

khiết hóa học. Sản phẩm thu được có chứa 99,1% - 99,2% coban, tạp chất còn lại là niken.

Niken tinh khiết cũng được điều chế bằng các phương pháp như trên.

Khi dùng H 2 để khử oxit NiO:

t

NiO + H 0

2 Ni + H 2 O

nếu nung ở 270 - 280 0 C thu được niken tự cháy, còn ở 350 - 400 0 C hoặc cao hơn thu được

niken bột, hoàn toàn bền đối với không khí.

Niken bột cũng được điều chế bằng phương pháp điện phân dung dịch NiSO 4 trong nước

với dương cực bằng các lá (Fe - Ni), còn âm cực bằng Ni; pH của dung dịch chất điện phân

ở khoảng từ 4 - 4,5.

Từ tetracacbonyl Ni(CO) 4 :

0

t

Ni(CO) 4 Ni + 4CO↑

Ngoài các phương pháp đã nêu, các kim loại trên đều có thể điều chế bằng phương pháp

nhiệt nhôm thu được kim loại khối nóng chảy :

3Fe 3 O 4 + 8Al

3Co 3 O 4 + 8Al

0

2700 C

9Fe + 4Al 2 O 3

0

3300 C

9Co + 4Al 2 O 3

0

2450 C

3NiO + 2Al 3Ni + Al 2 O 3

Các phản ứng tỏa ra một lượng nhiệt khá lớn đủ để cho chất phản ứng nóng chảy. Sản

phẩm thu được không tinh khiết.

Như trên đã nêu, các kim loại Fe, Co, Ni ở trạng thái bột rất nhỏ tách ra từ các hợp chất có

khả năng tự cháy, nghĩa là tự bốc cháy trong không khí ở ngay nhiệt độ thường. Bột tự cháy

của sắt cũng có thể điều chế bằng cách nung cẩn thận Fe (II) oxalat:

Phát hành PDF bởi Ths Nguyễn Thanh Tú

Đăng ký Word doc qua Zalo 0905779594 Email thanhtuqn88@gmail.com

19


Fe(COO) 2

0

t

Fe + 2CO 2 ↑

Các hạt sắt điều chế bằng cách đó có đường kính khoảng 5.10 -8 cm do đó bề mặt tiếp xúc

rất lớn nên đã làm tăng tốc độ oxi hóa, tuy nhiên nguyên nhân quan trọng là do cấu trúc

mạng tinh thể của các hạt so với kiến trúc bền của kim loại đó.

II.1.4. Tính chất vật lý của Fe, Co, Ni và ứng dụng

Cả ba nguyên tố là những kim loại màu trắng có ánh kim; Fe và Co có màu xám, còn Ni có

màu trắng bạc.

Một số hằng số vật lý của Fe, Co, Ni:

Tính chất Fe Co Ni

Khối lượng riêng (g/cm 3 )

Nhiệt độ nóng chảy (t nc , 0 C )

Nhiệt độ sôi (t s , 0 C )

Độ dẫn điện (Hg =1)

Độ dẫn nhiệt (Hg =1)

Độ âm điện (theo Pauling)

tụ khi làm sạch nước, làm chất cầm màu, làm chất xúc tác trong hóa học hữu cơ. Các muối

20

7,9

1539

2740

10

10

1,8

Sắt và niken dễ rèn, dễ dát mỏng; coban cứng và giòn.

8,9

1495

2900

10

8

1,7

8,9

1455

2730

Ở điều kiện thường, Fe, Co, Ni đều là các chất sắt từ. Tuy nhiên, trong bốn dạng thù hình

của Fe, chỉ có α - Fe mới có tính sắt từ, nghĩa là tính sắt từ của sắt chỉ ở nhiệt độ đến 770 0 C,

trên nhiệt độ đó tính sắt từ của sắt biến mất mặc dù mạng tinh thể không đổi, sắt chỉ còn lại

tính thuận từ.

Coban có tính sắt từ ở nhiệt độ l075 0 C, còn niken ở 362 0 C; trên nhiệt độ đó tính sắt từ của

coban và niken sẽ mất.

Ngoài tính chất bị nam châm hút, dưới tác dụng của dòng điện các chất sắt từ sẽ trở thành

nam châm. Tuy nhiên, sắt nguyên chất chỉ tác dụng như một nam châm khi chịu tác dụng

của dòng điện, còn thanh thép khi đã nam châm hóa thì sau đó vẫn tác dụng như một nam

châm vĩnh cửu.

Sắt là kim loại quan trọng nhất đối với các ngành kỹ thuật và công nghiệp hiện đại. Cả ba

kim loại được dùng chủ yếu để tạo ra các hợp kim đặc biệt là các loại thép.

Nhiều hợp chất của sắt có ý nghĩa quan trọng trong thực tế như FeCl 3 dùng làm chất đông

Phát hành PDF bởi Ths Nguyễn Thanh Tú

Đăng ký Word doc qua Zalo 0905779594 Email thanhtuqn88@gmail.com

14

7

1,8


ferit của sắt (II) dùng trong kỹ thuật máy tính. Các oxit của sắt không những là nguồn điều

chế nhiều hợp chất của sắt mà còn là nguồn nguyên liệu quan trọng của ngành luyện kim

đen.

Hơn 3/4 lượng coban được sản xuất dùng để chế tạo thép và hợp kim đặc biệt. Coban

truyền cho thép tính cứng và tính chịu mòn nên loại thép có chứa coban dùng để chế tạo các

dụng cụ cắt gọt khác nhau.

Một số hợp kim của coban dùng trong kỹ thuật quốc phòng và kỹ thuật tên lửa. Coban là

vật liệu sắt từ, nhiều hợp kim của coban cũng có tính sắt từ. Hợp chất giữa coban và samari

(SmCo 5 ) và một số kim loại đất hiếm khác được dùng làm vật liệu để chế tạo nam châm vĩnh

cửu mạnh.

Đồng vị 60

27Co được dùng rộng rãi trong y khoa để chữa bệnh ung thư.

Niken có nhiều ứng dụng hơn coban, có nhiều ứng dụng trong nhiều ngành kinh tế quốc

dân, hơn 80% lượng niken được sản xuất, dùng trong ngành luyện kim.

Thép có chứa niken có độ bền cao về mặt hóa học và cơ học; niken được dùng để mạ lên

bề mặt các sản phẩm làm bằng vật liệu dễ bị ăn mòn.

Hợp kim monen có độ bền cao với môi trường ăn mòn, được sử dụng nhiều trong ngành

chế tạo máy cho công nghệ hóa học, đóng tàu ... Một số hợp kim khác của niken được dùng

để sản xuất các loại tiền thay cho bạc.

Niken và hợp kim của nó được dùng trong kỹ thuật hàng không và vũ trụ, trong công

nghiệp hóa chất dùng làm chất xúc tác.

Một điều cần chú ý là khi nói đến tính chất vật lý của sắt cần để ý đến thành phần tạp chất

có trong sắt với hai loại:

Sắt có độ tinh khiết cao: có chứa dưới 0,001% tạp chất, có từ tính tốt và dễ hàn, nhưng tính

chất cơ học không cao, độ bền kém hơn nhiều so với gang và các loại thép; không thể dùng

trong vật liệu chế tạo.

Sắt tinh khiết kỹ thuật còn gọi là thép điện kỹ thuật: có chứa 0,02 - 0,04% cacbon, ngoài ra

còn có các tạp chất khác như oxi, nitơ, lưu huỳnh, photpho có từ tính tốt và dễ hàn, có tính

cơ học cao, dùng làm vật liệu chế tạo.

II.1.5. Tính chất hóa học của Fe, Co, Ni

Phát hành PDF bởi Ths Nguyễn Thanh Tú

Đăng ký Word doc qua Zalo 0905779594 Email thanhtuqn88@gmail.com

21


Fe, Co, Ni là những kim loại hoạt động trung bình. Tác dụng được với nhiều đơn chất và

hợp chất. Hoạt tính hóa học giảm dần từ Fe đến Ni. Chẳng hạn, Fe dễ dàng bị oxi hóa trong

không khí ẩm; Co bền ở điều kiện thường, đun nóng đến 300 0 C thì bị oxi oxi hóa; còn niken

bắt đầu bị oxi hóa ở 500 0 C.

Cả ba kim loại đều không phản ứng trực tiếp với hiđro, nhưng ở trạng thái bột nhỏ và ở

nhiệt độ cao đều hấp thụ hiđro với lượng khá lớn. Các hợp chất với hiđro của chúng đều

không có thành phần xác định và điều chế bằng phương pháp gián tiếp. Các hiđrua đều bền

như: FeH, FeH 2 , FeH 3 ; CoH, CoH 2 ; NiH, NiH 2 .

Độ hòa tan của hiđro trong sắt nóng chảy phụ thuộc vào dạng thù hình của sắt. Dạng γ - Fe

là dung môi tốt hơn cả so với các dạng thù hình khác, ở 1540 0 C có thể hòa tan được 18 gam

hiđro trong 100 gam Fe; người ta cũng đã nghiên cứu thấy rằng sự giải hấp thụ bắt đầu ở 150

- 300 0 C.

Coban hấp thụ hiđro kém hơn nhiều. Ở 1200 0 C, 100 gam coban hấp thụ được 5,46 cm 3

hiđro. Coban điện phân hấp thụ kém, còn coban tấm hầu như không hấp thụ hiđro.

Đặc biệt là niken hấp thụ mạnh hiđro, khả năng đó cho phép dùng niken làm chất xúc tác

trong quá trình hiđro hóa các chất hữu cơ. Độ hòa tan của hiđro trong niken có khác nhau

phụ thuộc vào trạng thái rắn hay lỏng. Ở 1600 0 C, 100 gam niken hòa tan được 43 cm 3 hiđro.

Ở điều kiện thường sắt, coban và niken đều bền với không khí và nước.

Khi nung bột Fe trong không khí hoặc trong oxi tạo ra Fe 3 O 4 :

0

t

3Fe + 2O 2 Fe 3 O 4

Ở nhiệt độ cao, coban bị oxi hóa tạo ra Co 3 O 4 :

0

t

3Co + 2O 2 Co 3 O 4

Còn niken bị oxi hóa chậm hơn, bắt đầu ở 500 0 C tạo ra NiO:

0

t

2Ni + O 2 2NiO

Trong không khí ẩm, sắt có lẫn các tạp chất sẽ bị gỉ, tức là bị ăn mòn trên bề mặt. Lớp gỉ

tạo nên có thành phần chủ yếu là Fe 2 O 3 .xH 2 O, lớp gỉ này xốp và giòn nên không bảo vệ

được cho sắt. Phản ứng tạo gỉ chủ yếu theo phương trình:

4Fe + 3O 2 + 2xH 2 O → 2[Fe 2 O 3 .xH 2 O]

Phát hành PDF bởi Ths Nguyễn Thanh Tú

Đăng ký Word doc qua Zalo 0905779594 Email thanhtuqn88@gmail.com

22


Tốc độ tạo ra gỉ sắt lớn nhất ở các đầu mút, ở chỗ bẻ cong đặc biệt ở các vết cắt; còn ở

phần phẳng của tấm sắt bị ăn mòn ở mức độ nhỏ.

Cả ba kim loại đều phản ứng mạnh với các halogen. Khi đun nóng sắt kim loại với các

halogen thu được Fe (III) halogenua khan FeX 3 ; ví dụ:

0

t

2Fe + 3Cl 2 FeCl 3

Tuy nhiên khi nghiền bột iốt với bột sắt tạo ra sản phẩm có thành phần là Fe 3 I 8 (hay

2FeI 3 .FeI 2 ).

3Fe + 4I 2 → Fe 3 I 8

nghĩa là hỗn hợp có cả iotua Fe (III) và Fe (II). Nói cách khác, FeI 3 chỉ bền trong cân bằng

với lượng dư khá lớn FeI 2 :

2FeI 3

2FeI 2 + I 2

Coban và niken tác dụng trực tiếp với các halogen tạo ra muối ứng với số oxi hóa +2 của

kim loại. Ví dụ: khi đốt cháy coban hay niken trong luồng khí clo chỉ tạo ra CoCl 2 và NiCl 2 ;

tương tự như vậy với brom.

0

t

M + Cl 2 MCl 2 (M là Co, Ni)

nhưng với flo, coban tạo ra hỗn hợp CoF 2 + CoF 3 .

Khi nung hỗn hợp gồm S với các kim loại họ sắt tạo ra các sunfua:

Fe + S

0

t

FeS

Co và Ni cũng có phản ứng tương tự tạo ra CoS và NiS. Selen và telu phản ứng trực tiếp

với Fe, Co, Ni:

Với Fe tạo ra: FeSe, FeSe 2 ; FeTe, FeTe; trong đó FeSe và FeSe 2 là chất bán dẫn.

Với Co tạo ra: CoSe, CoSe 2 ; CoTe, CoTe 2 .

Với Ni tạo ra: NiSe, NiSe 2 ; NiTe, NiTe 2 .

Cả ba kim loại đều không phản ứng trực tiếp với nitơ. Các hợp chất như Fe 2 N, Fe 4 N;

Co 2 N, Co 3 N, Co 3 N 2 ; Ni 3 N, Ni 3 N 2 đều được điều chế bằng phương pháp gián tiếp và đều ít

bền. Tác dụng với photpho tạo ra dung dịch rắn ứng với các hợp chất có thành phần Fe 3 P,

Fe 2 P, FeP, FeP 2 ; Co 2 P, CoP, CoP 3 ; Ni 3 P, Ni 2 P, NiP 3 .

Fe, Co, Ni tạo ra hợp chất với cacbon ứng với thành phần M 3 C và M 2 C, trong đó quan

trọng nhất là Fe 3 C:

Phát hành PDF bởi Ths Nguyễn Thanh Tú

Đăng ký Word doc qua Zalo 0905779594 Email thanhtuqn88@gmail.com

23


3Fe + C

0

t

Fe 3 C

Fe 2 C được điều chế bằng cách nung Fe với C ở 250 0 C. Các hợp chất ứng với thành phần

như trên của Co và Ni được điều chế bằng phương pháp gián tiếp, chẳng hạn khi nung coban

bột với CO:

0

t

3Co + 2CO Co 3 C + CO 2

Fe tạo ra các xilixua quan trọng như FeSi 2 , FeSi, Fe 3 Si 2 và Fe 2 Si có trong thành phần của

ferosilic.

Coban tạo ra các hợp chất Co 3 Si, Co 2 Si, CoSi, CoSi 2 trực tiếp từ các nguyên tố, nóng chảy

ở nhiệt độ trong khoảng từ 1200 - 1400 0 C, ví dụ:

3Co + Si

0

t

Co 3 Si

Niken cũng phản ứng trực tiếp với silic tạo ra các silixua Ni 3 Si, Ni 2 Si, NiSi, NiSi 2 ; chẳng

hạn:

0

t

2Ni + Si Ni 2 Si

Nóng chảy trong khoảng từ 1000 - 1200 0 C

Fe, Co, Ni đều phản ứng trực tiếp với Bo:

Nung nóng bột Fe với B ở 1300 - 1400 0 C trong bầu khí trơ tạo ra FeB và FeB 2 :

0

t

2Fe + 3B FeB + FeB 2

Khi nung Co với bột B vô định hình trong ống thạch anh hàn kín tạo ra các hợp chất Co 3 B,

Co 2 B, CoB, CoB 2 .

Cũng tương tự cách điều chế như trên, niken tạo ra các hợp chất Ni 3 B, Ni 2 B, NiB, Ni 3 B 2 ,

Ni 4 B 3 .

Nước không chứa không khí hòa tan hầu như không ăn mòn sắt ở nhiệt độ thường, vì trên

bề mặt dễ tạo ra lớp mỏng Fe(OH) 2 bảo vệ cho kim loại. Nhưng khi cho hơi nước qua Fe

nung đỏ tạo ra Fe 3 O 4 :

0

t

3Fe + 4H 2 O Fe 3 O 4 + + 4H 2

Co và Ni không bị H 2 O ăn mòn ở nhiệt độ thường, nhưng khi cho H 2 O qua Ni nung đỏ tạo

ra NiO:

Ni + H 2 O

0

t

NiO + H 2

Phát hành PDF bởi Ths Nguyễn Thanh Tú

Đăng ký Word doc qua Zalo 0905779594 Email thanhtuqn88@gmail.com

24


Với dung dịch kiềm, ở điều kiện thường thực tế không tác dụng với cả ba kim loại. Tuy

nhiên, khi đun sôi bột sắt với dung dịch NaOH 50%, trong điều kiện không có không khí,

thu được dung dịch natri hipoferit:

Fe + 2NaOH + 2H 2 O Na 2 [Fe(OH)] 4 + H 2

Trong dãy thế điện cực, Co và Ni đứng giữa Fe và Sn nhưng gần Sn hơn nên cả hai kim

loại tan trong axit loãng chậm hơn Fe.

Sắt tác dụng với axit không có tính oxi hóa tạo ra muối Fe (II) và H 2 :

Fe + 2H 3 O + Fe 2+

+ H 2 ↑ + 2H 2 O

Sắt khử được các ion kim loại đứng sau sắt trong dãy thế điện cực:

Fe + Cu 2+ Cu + Fe 2+

Fe + 2Fe 3+ 3Fe 2+

H 2 SO 4 đặc nguội gây ra tính thụ động cho sắt, nhưng với H 2 SO 4 đặc nóng sẽ oxi hóa sắt

tạo ra muối Fe (III) và đồng thời bị khử đến SO 2 :

0

t

2Fe + 6H 2 SO 4 đặc Fe 2 (SO 4 ) 3 + 3SO 2 + 6H 2 O

Khi nhúng thanh sắt vào HNO 3 đặc nguội một thời gian, sau đó nhúng vào dung dịch chứa

ion Cu 2+ hoặc dung dịch axit loãng như HCl, H 2 SO 4 sắt sẽ mất khả năng phản ứng, nghĩa là

sắt đã bị thụ động hóa bởi HNO 3 đặc nguội. HNO 3 loãng, lạnh hoà tan sắt tạo ra sắt (II)

nitrat:

8Fe + 20HNO 3 loãng 8Fe(NO 3 ) 2 + 2NH 4 NO 3 + 6H 2 O

Trong dung dịch axit loãng như HCl, H 2 SO 4 , coban khó tan hơn so với sắt, tạo ra H 2 :

Co + H 2 SO 4 CoSO 4 + H 2

Coban phản ứng với H 2 SO 4 đặc nóng tạo ra SO 2 :

0

t

Co + 2H 2 SO 4 đặc CoSO 4 + SO 2 ↑ + 2H 2 O

Coban tan trong HNO 3 loãng, và cũng tương tự như sắt, coban cũng bị thụ động hóa bởi

HNO 3 đặc.

Các axit vô cơ loãng tác dụng với niken chậm hơn sắt tạo ra H 2 :

Ni + 2HCl NiCl 2 + H 2 ↑

nhưng dễ tan trong HNO 3 loãng:

3Ni + 8HNO 3 loãng 3Ni(NO 3 ) 2 + 2NO + 4H 2 O

Phát hành PDF bởi Ths Nguyễn Thanh Tú

Đăng ký Word doc qua Zalo 0905779594 Email thanhtuqn88@gmail.com

25


Tương tự Fe và Co, Ni cũng bị thụ động hóa bởi HNO 3 bốc khói.

II.1.6. Hợp chất cacbonyl của Fe, Co, Ni

Cũng tương tự như các nguyên tố Mn, Cr, V, Ti, các nguyên tố Fe, Co, Ni cũng có khả

năng phản ứng với cạcbon oxit tạo ra các hợp chất cacbonyl mà trong đó các nguyên tố trung

tâm này mang số oxi hóa "0".

Khi đun nóng bột sắt trong ống chứa CO ở 150 - 200 0 C ở áp suất khoảng 100 atm tạo ra

sắt pentacacbonyl:

0

t

Fe + 5CO Fe(CO) 5

Fe(CO) 5 là chất lỏng màu vàng nhạt (t nc = - 20 0 C; t s = l03 0 C) có cấu tạo hình chóp kép với

nguyên tử Fe ở trung tâm [d (Fe - C) = 1,84 A 0 ; d (C - O) = 1,15 A 0 ], ứng với obitan lai hóa dsp 3 của

nguyên tử sắt.

Khi đốt cháy hơi Fe(CO) 5 trong không khí tạo nên Fe 2 O 3 :

t

4Fe(CO) 5 + 13O 0

2 2Fe 2 O 3 + 20CO 2

Bị nhiệt phân hủy ở 140 0 C tạo thành sắt kim loại:

0

140 C

Fe(CO) 5 Fe + 5CO↑

Người ta dùng phản ứng này để điều chế sắt tinh khiết hóa học. Fe(CO) 5 không tan trong

nước, tan trong ete và benzen, trong dung dịch ete Fe(CO) 5 phản ứng với H 2 SO 4 theo

phương trình:

Fe(CO) 5 + H 2 SO 4 FeSO 4 + H 2 ↑+ 5CO↑

điều đó chứng tỏ sắt ở trong hợp chất đó có tác dụng như là sắt kim loại.

Fe(CO) 5 có tính chất oxi hóa và khử. Chẳng hạn trong amoniac lỏng, tác dụng với natri

kim loại thể hiện tính oxi hóa:

Fe(CO) 5 + 2Na Na 2 [Fe(CO) 4 ] + CO↑

và tác dụng với dung dịch kiềm trong ancol thể hiện tính khử:

Fe(CO) 5 + 4KOH K 2 [Fe(CO) 4 ] + K 2 CO 3 + 2H 2 O

Phát hành PDF bởi Ths Nguyễn Thanh Tú

Đăng ký Word doc qua Zalo 0905779594 Email thanhtuqn88@gmail.com

26


Các hợp chất M I 2 [Fe(CO)

4] dễ dàng bị thủy phân tạo ra hiđrocacbonyl H 2 [Fe(CO) 4 ] có tính

axit:

K 2 [Fe(CO) 4 ] + 2H 2 O H 2 [Fe(CO) 4 ] + 2KOH

Gốc [Fe(CO) 4 ] 2- có thể xem là dẫn xuất của sắt (-2).

Khi đun nóng bột coban trong khí quyển cacbon oxit ở 150 - 200 0 C và 250 atm tạo ra

coban octacacbonyl:

2Co + 8CO Co 2 (CO) 8

Co 2 (CO) 8 là hợp chất hai nhân, trong đó nguyên tử coban tạo ra sáu liên kết σ. Bốn liên kết

σ tạo ra bởi bốn cặp electron của bốn phân tử CO đặt vào bốn obitan tự do của coban; liên

kết σ thứ năm tạo ra từ một cặp electron d đặt vào obitan π của phân tử CO. Liên kết giữa Co

- Co tạo ra do sự ghép đôi bởi hai electron độc thân của hai nguyên tử coban:

Co 2 (CO) 8 là tinh thể có màu da cam, bắt đầu bị phân hủy ở 60 0 C:

0

t

Co 2 (CO) 8 2Co + 8CO↑

Khi đun nóng dưới áp suất của khí hiđro tạo ra hợp chất hidrua cacbonyl:

0

t

Co 2 (CO) 8 + H 2 2H[Co(CO) 4 ]

Chất này có màu vàng, t nc = - 33 0 C, tan trong dung dịch kiềm tạo thành muối như

K[Co(CO) 4 ], NH 4 [Co(CO) 4 ].

Đã biết được niken tetracabonyl Ni(CO) 4 là hợp chất của Ni (0) có cấu trúc tứ diện ứng với

dạng lai hóa sp 3 .

Phát hành PDF bởi Ths Nguyễn Thanh Tú

Đăng ký Word doc qua Zalo 0905779594 Email thanhtuqn88@gmail.com

27


Ni(CO) 4 là chất lỏng không màu (t nc = 19,3 0 C; t s = 43 0 C) dễ dàng điều chế bằng cách cho

CO tác dụng với bột niken ở 60 - 80 0 C:

Ni + 4CO

0

t

Ni(CO) 4

Ni(CO) 4 không tan trong nước, không phản ứng với axit loãng, kiềm loãng, dễ tan trong

các dung môi hữu cơ.

Khi đun nóng khoảng 180 - 200 0 C, Ni(CO) 4 bị phân hủy tạo ra Ni nguyên chất:

t

Ni(CO) 0

4 Ni + 4CO↑

Phản ứng trên dùng để điều chế niken tinh khiết.

II.1.7. Các xianua của Fe (II), Co (II), Ni (II)

Các hợp chất xianua trên đều tạo ra ở dạng kết tủa vô định hình màu vàng (Fe), màu đỏ

(Co) và màu xanh (Ni) khi cho KCN tác dụng với các muối.

Xianua của sắt (II) ở dạng tinh khiết chưa được biết. Khi cho ion Fe 2+ tác dụng với ion CN - ,

tạo thành phức [Fe(CN) 6 ] 4- kết tủa lắng xuống:

FeSO 4 + 2KCN Fe(CN) 2 + K 2 SO 4

Fe(CN) 2 + 4KCN K 4 [Fe(CN) 6 ]↓ (vàng)

Đã biết được nhiều muối dạng

I

M [Fe(CN) ] và thường kết tinh ở dạng hiđrat hóa, trong

4 6

đó quan trọng nhất là kali feroxianua K 4 [Fe(CN) 6 ] tách ra từ dung dịch ở dạng hiđrat tinh thể

Phát hành PDF bởi Ths Nguyễn Thanh Tú

Đăng ký Word doc qua Zalo 0905779594 Email thanhtuqn88@gmail.com

28


K 4 [Fe(CN) 6 ].3H 2 O có màu vàng, mất nước kết tinh ở 87 0 C, thường được gọi là muối vàng -

máu (vì trước kia người ta điều chế chất này từ máu bò).

Các feroxianua của kim loại kiềm và kiềm thổ đều dễ tan trong nước, trừ bari feroxianua

Ba 2 [Fe(CN) 6 ] là khó tan.

Nói chung, các hợp chất đó đều bền với không khí, với axit, với kiềm, tuy nhiên khi nung

nóng bị phân hủy và khi tác dụng với H 2 SO 4 đặc nóng tạo ra CO:

K 4 [Fe(CN) 6 ] + 6H 2 SO 4 + 6H 2 O 2K 2 SO 4 + 3(NH 4 ) 2 SO 4 + 6CO↑ + FeSO 4

Khi cho HCl đặc tác dụng với dung dịch đậm K 4 [Fe(CN) 6 ] tạo ra kết tủa trắng

H 4 [Fe(CN) 6 ] :

K 4 [Fe(CN) 6 ] + 4HCl H 4 [Fe(CN) 6 ]↓ + 4KCl

(axit feroxianhiđric)

Là axit mạnh ở hai chức đầu, sự phân ly tiếp theo thì yếu hơn (K 3 = 1.10 -3 ; K 4 = 5.10 -5 ). Dễ

tan trong nước và trong ancol.

Khi đun nóng với axit loãng tạo ra HCN (t s = 26 0 C ) tự do:

K 4 [Fe(CN) 6 ] + 3H 2 SO 4 FeSO 4 + 2K 2 SO 4 + 6HCN

còn khi tác dụng với CuSO 4 tạo ra kết tủa màu nâu socola Cu 2 [Fe(CN) 6 ]:

CuSO 4 + K 4 [Fe(CN) 6 ] Cu 2 [Fe(CN) 6 ]↓ + 2K 2 SO 4

(nâu)

Khi cho KCN tác dụng với dung dịch muối coban (II) hình thành kết tủa màu đỏ nâu

coban (II) xianua Co(CN) 2 :

CoSO 4 + 2KCN Co(CN) 2 ↓ + K 2 SO 4

Co(CN) 2 tan trong KCN dư tạo ra dung dịch màu vàng, sau đó đỏ dần và cuối cùng tạo ra

tinh thể màu tím có thành phần K 4 [Co(CN) 6 ] (kali cobantoxianua):

Co(CN) 2 + 4KCN K 4 [Co(CN) 6 ]↓

Hợp chất này bền trong điều kiện không có không khí, dễ tan trong nước cho dung dịch

màu đỏ, nhưng khi đun nóng dung dịch sẽ bị H 2 O oxi hóa (không có không khí):

K 4 [Co(CN) 6 ] + 2H 2 O 2K 3 [Co(CN) 6 ] + 2KOH + H 2 ↑

Phát hành PDF bởi Ths Nguyễn Thanh Tú

Đăng ký Word doc qua Zalo 0905779594 Email thanhtuqn88@gmail.com

29


Khi cho KCN tác dụng với muối niken (II) tạo ra Ni(CN) 2 kết tủa màu xanh sáng.

Khi cho dư KCN, kết tủa ban đầu sẽ tan tạo thành phức chất màu vàng K 2 [Ni(CN) 4 ], kết

tinh dạng monohiđrat K 2 [Ni(CN) 4 ].H 2 O màu da cam.

NiSO 4 + 2KCN Ni(CN) 2 ↓ + K 2 SO 4

Ni(CN) 2 + 2KCN K 2 [Ni(CN) 4 ]

Hợp chất này dễ mất nước và tan nhiều trong nước, bị axit mạnh phân hủy giải phóng

Ni(CN) 2 .

Khi thêm nhiều KCN vào dung dịch chứa ion [Ni(CN) 4 ] 2- , màu của dung dịch từ vàng

chuyển thành màu đỏ, do sự tạo thành con [Ni(CN) 6 ] 4- :

K 2 [Ni(CN) 4 ] + 2KCN K 4 [Ni(CN) 6 ]

II.1.8. Phức chất xianua của Fe (III) và Co (III)

Kali ferixianua (K 3 [Fe(CN) 6 ]) được điều chế bằng cách oxi hóa kali feroxianua

K 4 [Fe(CN) 6 ] bởi các chất như Cl 2 , KMnO 4 , HNO 3 hoặc H 2 O 2 :

2K 4 [Fe(CN) 6 ] + Cl 2 2K 3 [Fe(CN) 6 ] + 2KCl

Hợp chất này không thể điều chế bằng phương pháp như khi điều chế K 4 [Fe(CN) 6 ], nghĩa

là không thể cho muối Fe 3+ tác dụng với dung dịch KCN, vì sẽ tạo ra kết tủa Fe(OH) 3 do sự

thủy phân KCN tạo ra môi trường kiềm.

K 3 [Fe(CN) 6 ] là chất kết tinh màu đỏ - gạch, khi nghiền nhỏ chuyển thành chất bột màu

vàng, tan trong nước cho dung dịch màu vàng.

K 3 [Fe(CN) 6 ] là chất oxi hóa mạnh, đặc biệt hoạt động trong môi trường kiềm, kali

ferixianua có thể oxi hóa PbO thành PbO 2 ; hợp chất Cr 3+ 2

thành CrO ; H

4 2 S thành S:

2K 3 [Fe(CN) 6 ] + 2KOH + PbO 2K 4 [Fe(CN) 6 ] + H 2 O + PbO 2

2K 3 [Fe(CN) 6 ] + 2KOH + H 2 S 2K 4 [Fe(CN) 6 ] + 2H 2 O + S

Khi cho HCl đặc tác dụng với dung dịch bão hòa K 3 [Fe(CN) 6 ] tạo ra tinh thể màu đỏ nâu

là axit ferixian hiđric H 3 [Fe(CN) 6 ]:

K 3 [Fe(CN) 6 ] + 3HCl H 3 [Fe(CN) 6 ]↓ + 3KCl

Đun nóng K 3 [Fe(CN) 6 ] với dung dịch KOH đặc ( với tỉ lệ l: 1) tạo ra K 4 [Fe(CN) 6 ] và O 2 :

4K 3 [Fe(CN) 6 ] + 4KOH đặc 4K 4 [Fe(CN) 6 ] + O 2 + 2H 2 O

Với muối Fe 2+ , dung dịch K 3 [Fe(CN) 6 ] tạo ra kết tủa màu "xanh tuabun":

Phát hành PDF bởi Ths Nguyễn Thanh Tú

Đăng ký Word doc qua Zalo 0905779594 Email thanhtuqn88@gmail.com

30


3Fe 2+ + 2[Fe(CN) 6 ] 3- Fe 3 [Fe(CN) 6 ] 2 ↓

Ngược lại, các muối Fe 3+ lại tác dụng với dung dịch kali feroxianua K 4 [Fe(CN) 6 ] tạo ra kết

tủa màu "xanh phổ" còn gọi là "xanh beclin" hay "xanh prussian”:

4Fe 3+ + 3[Fe(CN) 6 ] 4- Fe 4 [Fe(CN) 6 ] 3 ↓

Trong ion [Fe(CN) 6 ] 3- có 17 electron hóa trị (12 electron của 6 nhóm CN và 5 electron của

ion trung tâm), có cấu hình: [σ lk ] 12 [π(d)] 5 , nhưng muốn có cấu hình bền [n(d)] 6 cần phải

thêm một electron, do đó ion [Fe(CN) 6 ] 3- là chất oxi hóa mạnh; tính oxi hóa của ion

[Fe(OH 2 ) 6 ] 3+ cũng giải thích tương tự; tuy nhiên, [Fe(CN) 6 ] 3- có tác dụng oxi hóa mạnh

trong môi trường kiềm, còn ion [Fe(OH 2 ) 6 ] 3+ - trong môi trường axit.

Kali cobantixianua (K 3 [Co(CN) 6 ) là những tinh thể màu vàng, thu được khi cho muối

Co 2+ tác dụng với KCN trong điều kiện có chất oxi hóa:

4K 4 [Co(CN) 6 ] + 4HCl + O 2 4K 3 [Co(CN) 6 + 4KCl + 2H 2 O

Nếu muối đó tác dụng với HNO 3 hay H 2 SO 4 sẽ thu được tinh thể không màu là axit

cobantixian hiđric:

2K 3 [Co(CN) 6 + 3H 2 SO 4 → 2H 3 [Co(CN) 6 ] + 3K 2 SO 4

Axit đó cũng có thể điều chế bằng cách cho K 3 [Co(CN) 6 ] tác dụng với CuSO 4 để tạo ra kết

tủa màu xanh là Cu 3 [Co(CN) 6 ] 2 , sau đó cho phân hủy với H 2 S:

2K 3 [Co(CN) 6 ] + 3CuSO 4 → Cu 3 [Co(CN) 6 ] 2 ↓ + 3K 2 SO 4

(xanh)

Cu 3 [Co(CN) 6 ] 2 ↓ + 3H 2 S →

2H 3 [Co(CN) 6 ] + 3CuS↓

Axit cobantixian hiđric là axit mạnh ba lần axit, kết tinh ở dạng hiđrat hóa

H 3 [Co(CN) 6 ].5H 2 O, trong dung dịch nước chỉ bền ở dưới 50 0 C. Khi đun nóng đến 120 0 C tạo

ra Co(CN) 3 màu xanh:

H 3 [Co(CN) 6 ]

0

t

Co(CN) 3 + 3HCN

Sắt (III) thioxianat (Fe(NCS) 3 ) hay sắt (III) sunfoxianua được tạo ra khi cho muối Fe 3+ tác

dụng với KNCS hay (NH 4 )NCS:

FeCl 3 + 3KNCS → Fe(NCS) 3 + 3KCl

là chất màu đỏ máu, có thể tách ra từ dung dịch ở dạng hiđrat tinh thể Fe(NCS) 3 .3H 2 O màu

đỏ thẫm, dễ tan trong nước. Màu đỏ máu của dung dịch chính là màu của ion [FeNCS] 2+ .

Phát hành PDF bởi Ths Nguyễn Thanh Tú

Đăng ký Word doc qua Zalo 0905779594 Email thanhtuqn88@gmail.com

31


II.3. CÁC KIM LOẠI HỌ PLATIN

Các kim loại họ platin gồm: ruteni (Ru), rođi (Rh), palađi (Pd), osimi (Os), iriđi (Ir), platin

(Pt) là nhóm các kim loại khá hiếm, chúng giống nhau về tính chất nên việc tách chúng là rất

khó khăn.

Trong thiên nhiên hầu như chỉ gặp các kim loại ở trạng thái tự do, tất cả chúng thường đi

với nhau, nhưng chưa bao giờ gặp trong các quặng sắt.

Một số đặc điểm của kim loại họ platin:

Ru Rh Pd Os Ir Pt

Electron hóa trị 4d 7 5s 1 4d 8 5s 1 4d 10 5d 6 6s 2 5d 7 6s 2 5d 9 6s 1

Bán kính nguyên tử, nm 0,134 0,134 0,137 0,135 0,138 0,138

Năng lượng ion hóa thứ nhất, I 1 7,37 7,46 8,34 8,5 8,9 8,9

Năng lượng ion hóa thứ hai, I 2 16,8 18,1 19,4 19 18,6 18,6

Khối lượng riêng, g/cm 3 12,4 12,4 12,0 22,4 21,5 21,5

Nhiệt độ nóng chảy, 0 C 2250 1960 1554 2450 1769 1769

Nhiệt độ sôi, 0 C 4200 3700 2940 4400 3800 3800

Các kim loại họ platin kém hoạt tính và rất bền với các tác dụng hóa học.

Ruteni, rođi, osimi và iriđi khó nóng chảy. Mặt dù khó kiếm và đắt đỏ, các kim loại này có

ứng dụng nhiều mặt bên cạnh platin, và ứng dụng kỹ thuật ngày càng tăng.

Các kim loại họ platin có thể thể hiện các mức oxi hóa khác nhau trong các hợp chất,

nhưng hợp chất trong đó mức oxi hóa của chúng bằng +4 rất điển hình. Đối với osimi và

ruteni thì hợp chất ứng với mức oxi hóa +8 cũng đặc trưng.

Osimi (VIII) oxit OsO 4 (osimi tetraoxit) là oxit bền nhất trong các oxit của nguyên tố này.

Nó tạo thành chậm ngay khi bảo quản osimi trong không khí và là những tinh thể màu vàng

nhạt, dễ nóng chảy (nhiệt độ nóng chảy 41 0 C). Hơi OsO 4 có mùi mạnh và rất độc.

OsO 4 khá tan trong nước, dung dịch của nó không có phản ứng axit với quỳ.

OsO 4 có tính oxi hóa mạnh, nó phản ứng mãnh liệt với các hợp chất hữu cơ, khi đó nó bị

khử đến osimi đioxit OsO 2 màu đen. OsO 4 được ứng dụng để nhuộm tiêu bản cho kính hiển

vi dựa trên cơ sở này.

Phát hành PDF bởi Ths Nguyễn Thanh Tú

Đăng ký Word doc qua Zalo 0905779594 Email thanhtuqn88@gmail.com

32


Ruteni (VIII) oxit RuO 4 (ruteni tetraoxit) là các tinh thể rắn màu vàng có ánh vàng kim

loại, nóng chảy ở 25,4 0 C và tan trong nước. RuO 4 ít bền hơn nhiều so với OsO 4 và bị phân

hủy thành RuO 2 và O 2 ở nhiệt độ ≈ 108 0 C.

Osimi (VIII) florua OsF 8 (osimi octaflorua) dưới dạng hơi không màu được điều chế bằng

cách hóa hợp trực tiếp osimi và flo ở 250 0 C, khi làm lạnh nó đông đặc thành các tinh thể

màu vàng chanh, có nhiệt độ nóng chảy ở -34,4 0 C.

OsF 8 thể hiện tính chất oxi hóa mạnh. Nó bị nước phân hủy dần thành osimi tetraoxit và

hiđro florua:

OsF 8 + 4H 2 O OsO 4 + 8HF

Tất cả các kim loại họ platin đều thể hiện khuynh hướng tạo phức rõ rệt.

II.3.1. Ruteni

Ruteni là kim loại màu trắng, cứng, nhiều hóa trị, các trạng thái oxi hóa của ruteni nằm

trong khoảng từ +1 tới +8, và trạng thái -2 cũng đã được biết đến, mặc dù các trạng thái +2,

+3 và +4 là phổ biến nhất.

Ruteni không bị xỉn đi ở nhiệt độ bình thường, nhưng bị oxi hóa dễ dàng trong không khí

để tạo thành ruteni tetraoxit (RuO 4 ), một tác nhân oxi hóa mạnh với các tính chất tương tự

như của osimi tetraoxit. Ruteni tetraoxit (RuO 4 ), tương tự như osimi tetraoxit, là dễ bay hơi,

có độc tính cao và có thể gây nổ nếu như nó tiếp xúc được với các vật liệu dễ cháy. Ruteni

không đóng vai trò sinh học gì nhưng có thể làm sạm màu da người, và có thể là chất gây

ung thư và tích lũy sinh học trong xương.

Ruteni hòa tan trong kiềm nóng chảy, không bị các axít ăn mòn nhưng bị các halogen ăn

mòn ở nhiệt độ cao.

Ruteni là cực hiếm và nó đứng thứ 74 về độ phổ biến trong số các kim loại trên trái đất.

Ruteni nguồn gốc tự nhiên là hỗn hợp của 7 đồng vị. Đồng vị phóng xạ ổn định nhất là

106 Ru với chu kỳ bán rã 373,59 ngày, 103 Ru với chu kỳ bán rã 39,26 ngày và 97 Ru có chu kỳ

bán rã 2,9 ngày.

Mười lăm đồng vị phóng xạ khác có nguyên tử lượng nằm trong khoảng từ 90 Ru tới 115 Ru.

Phần lớn trong số này có chu kỳ bán rã nhỏ hơn 5 phút, ngoại trừ 95 Ru (1,643 giờ) và 105 Ru

(4,44 giờ).

Phát hành PDF bởi Ths Nguyễn Thanh Tú

Đăng ký Word doc qua Zalo 0905779594 Email thanhtuqn88@gmail.com

33


Một lượng nhỏ ruteni có thể làm tăng độ cứng của platin và palađi nên ruteni được dùng

trong các hợp kim platin và palađi để chế tạo các tiếp điểm điện chống mài mòn. 0,1% ruteni

được bổ sung vào titan thì khả năng chống ăn mòn của titan tăng lên hàng trăm lần. Đôi khi

nó được tạo hợp kim với vàng trong nghề kim hoàn. Ngòi của bút máy cũng hay được bịt

đầu bằng hợp kim chứa ruteni. Ruteni cũng là chất xúc tác đa năng. Ruteni là thành phần của

các anôt oxit kim loại hỗn hợp (MMO) được dùng để bảo vệ catôt cho các kết cấu ngầm

dưới nước hay dưới đất và cho các điện cực điện phân trong các công nghệ sản xuất hóa chất

như clo từ nước muối. Một hợp kim của ruteni với molypden có tính siêu dẫn ở 10,6K. Các

phức chất tâm ruteni đang được nghiên cứu để tìm kiếm các tính chất chống ung thư. Các

phức chất này của ruteni, khác với các phức chất truyền thống của platin, thể hiện sự kháng

lại thủy phân cao hơn và tác động chọn lọc hơn đối với các khối u.

II.3.2. Rođi

Rođi là kim loại màu trắng bạc, cứng và bền, có hệ số phản xạ cao. Rođi được tìm thấy

trong quặng platin và được sử dụng trong các hợp kim với platin và như là một chất xúc tác.

Rođi kim loại thông thường không tạo ra oxit, ngay cả khi bị nung nóng. Rođi có điểm nóng

chảy cao hơn nhưng tỷ trọng riêng thấp hơn của platin. Nó không bị phần lớn các axit ăn

mòn: nó hoàn toàn không hòa tan trong axit nitric và chỉ hòa tan một chút trong nước cường

toan. Rođi ở dạng bột tan hoàn toàn trong axit sunfuric.

William Hyde Wollaston đã cô lập kim loại rođi từ quặng platin thô như sau: hòa tan

quặng trong nước cường toan, trung hòa dung dịch bằng natri hiđroxit. Sau đó kết tủa platin

dưới dạng cloroplatinat amoni bằng cách cho thêm clorua amoni. Nguyên tố palađi được loại

bỏ dưới dạng xyanua palađi sau khi xử lý dung dịch bằng xianua thủy ngân. Vật chất còn lại

là clorua rođi (III) có màu đỏ: rođi kim loại được cô lập thông qua việc khử bằng khí hiđrô.

Rođi nguồn gốc tự nhiên chỉ chứa một đồng vị ( 103 Rh). Hai mươi đồng vị phóng xạ khác

với nguyên tử lượng trong khoảng từ 93 Rh tới 117 Rh. Phần lớn trong số này có chu kỳ bán rã

nhỏ hơn 1 giờ, ngoại trừ 100 Rh (20,8 giờ) và 105 Rh (35,36 giờ).

Rođi kim loại trơ về mặt hóa học. Các hợp chất của rođi thông thường ít khi gặp nhưng nó

có độc tính và có khả năng gây ung thư. Các hợp chất này cũng có thể làm sạm màu da

người mặc dù nó không đóng vai trò sinh học nào ở người. Ở dạng kim loại thì nó không độc

hại gì.

Phát hành PDF bởi Ths Nguyễn Thanh Tú

Đăng ký Word doc qua Zalo 0905779594 Email thanhtuqn88@gmail.com

34


Ứng dụng chủ yếu của rođi là làm tác nhân tạo hợp kim để làm cứng platin và palađi. Các

hợp kim này được dùng trong các trục cuốn và ống lót của lò luyện để sản xuất các sợi thủy

tinh, các thành phần của cặp nhiệt điện, các điện cực cho bu gi của tàu bay và các nồi nấu

trong phòng thí nghiệm. Dùng làm vật liệu chế tạo tiếp điểm điện do điện trở thấp, điện trở

tiếp xúc thấp và ổn định cùng khả năng chống ăn mòn cao của nó. Lớp mạ rođi (do mạ

điện hay phủ hơi rođi) rất cứng và được dùng cho các thiết bị quang học. Kim loại này cũng

được dùng trong ngành kim hoàn và dùng trang trí. Nó được mạ điện trên vàng trắng để tạo

ra cho chúng bề mặt trắng có tính phản chiếu ánh sáng.

Rođi thuộc nhóm những kim loại khá hiếm, quý và đắt tiền nhất. Rođi đã từng được sử

dụng như là biểu tượng cho danh dự hay sự giàu có, khi các kim loại quý khác như bạc, vàng

hay platin được coi là chưa đủ. Năm 1979, Guinness Book of World Records tặng Paul

McCartney một đĩa mạ rođi vì họ coi ông là nhạc sĩ sáng tác bài hát cũng như là nghệ sĩ có

lượng bài hát được bán nhiều nhất mọi thời đại. Guinness cũng ghi nhận các đồ như "bút đắt

tiền nhất" hay "bàn cờ đắt tiền nhất" thế giới đều là những vật chứa rođi.

II.3.3. Palađi

Palađi là kim loại màu trắng bạc, nhẹ nhất trong các kim loại họ platin, mềm và dễ rèn

nhất. Nó có khả năng tuyệt vời hấp phụ một lượng lớn hiđro (tới 900 thể tích trên một thể

tích kim loại). Khi đó palađi vẫn giữ được dạng kim loại, nhưng thể tích tăng lên nhiều, trở

nên giòn và dễ tạo thành vết nứt. Tấm palađi bão hòa hiđro có hoạt tính rất cao, có thể

chuyển clo, brom, iot thành hiđro halogenua, khử muối sắt (III) thành muối sắt (II), muối

thủy ngân (II) thành muối thủy ngân (I), lưu huỳnh đi oxit thành hiđro sunfua.

Một số dụng cụ thí nghiệm cũng như chi tiết máy để tách các đồng vị của hiđro được chế

tạo bằng palađi. Hợp kim palađi với bạc được dùng trong thiết bị liên lạc, đặc biệt để chế tạo

các tiếp điểm. Trong máy điều chỉnh nhiệt và trong cặp nhiệt người ta sử dụng các hợp kim

của palađi với vàng, platin với rođi. Một số hợp kim của palađi được dùng trong trang sức và

trong nha khoa.

Palađi trên chất mang amian, sứ hoặc các chất mang khác là chất xúc tác cho nhiều phản

ứng oxi hóa – khử, được sử dụng trong phòng thí nghiệm cũng như trong công nghiệp khi

tổng hợp một số hợp chất hữu cơ. Chất xúc tác palađi được dùng để tinh chế hiđro khỏi các

vết oxi, cũng như tinh chế oxi khỏi các vết hiđro.

Phát hành PDF bởi Ths Nguyễn Thanh Tú

Đăng ký Word doc qua Zalo 0905779594 Email thanhtuqn88@gmail.com

35


Về phương diện hóa học palađi đặc trưng bằng hoạt tính lớn, khác với các kim loại platin

khác. Khi đun nóng đỏ nó hóa hợp với oxi tạo thành oxit PdO, tan trong axit nitric, trong

axit sunfuric đặc nóng và trong nước cường toan.

Giống như platin, mức oxi hóa +2 và +4 đặc trưng đối với palađi, hợp chất palađi (II) bền

hơn. Đa số muối palađi tan trong nước và bị thủy phân mạnh trong dung dịch. Palađi (II)

clorua PdCl 2 trong dung dịch rất dễ bị khử đến kim loại bằng một số chất khử thể khí, đặc

biệt cacbon (II) oxit, vì vậy nó được dùng để phát hiện cacbon oxit trong hỗn hợp khí. Nó

cũng được dùng làm chất chất xúc tác cho một số phản ứng oxi hóa khử.

II.3.4. Osimi

Osimi là kim loại nặng nhất trong các kim loại từng biết (khối lượng riêng 22,6g/cm 3 , nặng

hơn khoảng 0,2g/cm 3 so với iriđi, kim loại nặng thứ nhì). Osimi khá cứng nhưng cũng

khá giòn, nhiệt độ nóng chảy rất cao, khoảng 3030 0 C, đứng thứ 5 trong bảng xếp hạng các

nguyên tố có nhiệt độ nóng chảy cao nhất.

Trong hợp chất, osmi có số oxi hóa từ -2 đến +8. Trong đó, hợp chất có số oxi hóa +2; +3;

+4 và +8 là khá phổ biến.

OsO 4 (osimi tetraoxit) là hợp chất của osimi mà trong đó osimi có số oxi hóa +8. OsO 4 là

tinh thể màu vàng nhạt có mùi mạnh, tan trong nước, dễ bay hơi, được hình thành khi cho

bột osimi tiếp xúc với không khí.

Osimi có 7 đồng vị tự nhiên, trong đó có 6 đồng vị bền là: 184 Os, 187 Os, 188 Os, 189 Os, 190 Os

và 192 Os.

Kim loại này được dùng chủ yếu trong các hợp kim không gỉ dùng để bịt đầu các ngòi bút

hoặc các trụ bản lề dụng cụ.

II.3.5. Iriđi

Iriđi khác platin ở nhiệt độ nóng chảy rất cao và rất bền với các tác dụng hóa học khác

nhau. Các axit riêng biệt, cũng giống như nước cường toan đều không tác dụng với iriđi.

Ngoài ra iriđi có độ rắn lớn hơn platin nhiều.

Iriđi tinh khiết được dùng để chế tạo một số dụng cụ khoa học. Hợp kim chứa 90% platin

và 10% iriđi cũng được sử dụng cho mục đích này. Từ hợp kim này người ta chế tạo các

mẫu chuẩn mét và kilogam quốc tế.

II.3.6. Platin

Phát hành PDF bởi Ths Nguyễn Thanh Tú

Đăng ký Word doc qua Zalo 0905779594 Email thanhtuqn88@gmail.com

36


Tương tự như vàng, trong thiên nhiên platin gặp trong sa khoáng dưới dạng hạt nhỏ, luôn

luôn chứa tạp chất của các kim loại họ platin khác. Hàm lượng của platin trong vỏ trái đất

khoảng 5.10 -8 %.

Platin là kim loại óng ánh màu trắng, dễ rèn, không bị biến đổi trong không khí ngay khi

nung ở nhiệt độ cao. Các axit riêng biệt không tác dụng với platin. Platin tan trong nước

cường toan, nhưng khó hơn nhiều so với vàng.

Vì khó nóng chảy và rất bền về hóa học nên platin được dùng để chế tạo dụng cụ thí

nghiệm: chén, bát, chậu, …

Trong công nghiệp hóa học platin được dùng để chế tạo chi tiết máy bền với ăn mòn. Anot

platin được sử dụng nhiều trong quá trình sản xuất điện hóa (sản xuất axit pesunfuric,

peclorat, peborat). Platin được dùng rộng rãi làm chất xúc tác, đặc biệt khi thực hiện phản

ứng oxi hóa - khử. Nó là chất xúc tác dị thể đầu tiên đã biết từ đầu thế kỉ XIX. Hiện nay chất

xúc tác platin được dùng trong sản xuất axit sunfuric và axit nitric, trong việc làm sạch hiđro

khỏi tạp chất oxi và trong nhiều quá trình khác. Các bộ phận đun lò điện và dụng cụ đo nhiệt

độ (nhiệt kế điện trở và cặp nhiệt) được chế tạo bằng platin. Platin ở trạng thái phân tán hòa

tan một lượng lớn hiđro và oxi. Dựa vào khả năng hòa tan hiđro, người ta dùng platin để chế

tạo điện cực hiđro.

Trong đa số hợp chất, platin thể hiện mức oxi hóa +2, +4. Ở trạng thái này cũng như ở

trạng thái khác, nó có khả năng tạo thành hợp chất phức rõ rệt, hợp chất platin (IV) có giá trị

quan trọng hơn.

Khi hòa tan platin trong nước cường toan ta điều chế được axit hexacloplatinic H 2 [PtCl 6 ],

khi làm bay hơi dung dịch, nó thoát ra dưới dạng tinh thể nâu đỏ có thành phần

H 2 [PtCl 6 ].6H 2 O. Muối kali của axit này là một trong những muối kali ít tan nhất vì vậy được

dùng trong hóa học phân tích để phát hiện kali.

Khi đun trong dòng clo đến 360 0 C thì axit hexacloplatinic bị phân hủy thoát ra hiđro

clorua và tạo thành platin (IV) clorua PtCl 4 .

Nếu ta rót kiềm vào dung dịch H 2 [PtCl 6 ] thì thoát ra kết tủa Pt(OH) 4 (axit platinic) màu

nâu. Axit platinic hòa tan trong kiềm dư tạo thành muối.

Platin (II) clorua PtCl 2 được điều chế khi cho clo đi qua platin vụn. PtCl 2 có màu lục nhạt

và không tan trong nước.

Phát hành PDF bởi Ths Nguyễn Thanh Tú

Đăng ký Word doc qua Zalo 0905779594 Email thanhtuqn88@gmail.com

37


Ứng với phức chất của platin (II) là ví dụ muối của axit tetraxianoplatinic (II) H 2 [Pt(CN) 4 ].

Muối bari của axit này Ba[Pt(CN) 4 ] phát huỳnh quang rất rõ rệt khi tia tử ngoại và tia rơngen

tác dụng lên nó và dùng để phủ lên màn huỳnh quang trong chiếu điện bằng tia rơngen.

I. Bài tập về Mn, Te, Re

B. BÀI TẬP THAM KHẢO

I.1. Bài tập lý thuyết, viết phương trình phản ứng, giải thích hiện tượng, điều chế,

chuỗi phản ứng, xác định công thức, …

I.1.1. Cho biết sự biến đổi hoạt tính hóa học từ Mn đến Re? Viết phương trình phản ứng khi

cho Mn, Te, Re tác dụng với các chất sau:

- HCl loãng và đặc.

- H 2 SO 4 loãng.

- H 2 SO 4 đặc.

- HNO 3 đặc.

Hướng dẫn:

- Hoạt tính hóa học giảm từ Mn đến Re.

- Mn tan trong HCl và H 2 SO 4 loãng, Re và Tc thì không. Các kim loại Mn, Tc và Re phản

ứng với các axit HNO 3 và H 2 SO 4 đặc.

Phương trình phản ứng:

Mn + 2HCl → MnCl 2 + H 2 ↑

3Mn + 8HNO 3 → 3Mn(NO 3 ) 2 + 2NO↑ + 4H 2 O

3Tc + 7HNO 3 đặc → 3HTcO 4 + 7NO + 2H 2 O

2Re + 7H 2 SO 4 đặc → 2HReO 4 + 7SO 2 + 6H 2 O

I.1.2. Người ta đã biết được một số hợp chất của Re (VI) như ReF 6 có màu vàng nhạt; ReCl 6

có màu xanh; ReOCl 4 , ReOF 4 ; ReO 3 có màu đỏ. Các hợp chất đó đều có tính axit. Trừ ReO 3 ,

còn lại dễ dàng bị nước phân hủy. Reni trioxit không tác dụng với H 2 O và với axit, còn trong

kiềm xảy ra quá trình tự oxi hóa khử. Các dẫn xuất của Re (VI) thể hiện tính khử, dễ dàng bị

oxi hóa, ngay cả oxi của không khí. Viết các phương trình phản ứng minh họa.

Hướng dẫn:

3ReF 6 + 12H 2 O 2HReO 4 + Re(OH) 4 + 18HF

Phát hành PDF bởi Ths Nguyễn Thanh Tú

Đăng ký Word doc qua Zalo 0905779594 Email thanhtuqn88@gmail.com

38


3ReOF 6 + 9H 2 O 2HReO 4 + Re(OH) 4 + 12HF

ReO 3 + 4KOH 2KReO 4 + K 2 ReO 3 + 2H 2 O

4K 2 ReO 4 + O 2 + 2H 2 O 4KReO 4 + 4KOH

I.1.3. So sánh độ bền của các hợp chất Mn (VII), Tc (VII), Re (VII). Cho ví dụ một vài hợp

chất.

Hướng dẫn:

Theo dãy các hợp chất Mn (VII), Tc (VII), Re (VII) thì độ bền tăng dần. Chẳng hạn, với

Mn (VII) chỉ được biết Mn 2 O 7 và oxoflorua MnO 3 F, còn với Re (VII) đã điều chế một dãy

hợp chất ReF 7 , ReOF 5 , ReO 2 F 3 , ReO 3 F, Re 2 O 7 ; ví dụ:

Mn 2 O 7 có màu xanh thẫm t nc = 5,9 0 C.

Re 2 O 7 màu vàng

t nc = 296 0 C

Trong các dẫn xuất của Mn (VII) thì quan trọng nhất trong thực tế là KMnO 4 .

I.1.4. Các hợp chất Tc (VII), Re (VII) tương tự các hợp chất của Mn (VII). Các axit HTcO 4

và HReO 4 (axit petecnexic và axit perenic) đều là những axit mạnh, các dung dịch loãng

HTcO 4 và HReO 4 đều không có màu. Phương pháp chủ yếu để điều chế các axit này là cho

kim loại tương ứng tác dụng với HNO 3 30%. Các oxit tương ứng là Tc 2 O 7 và Re 2 O 7 đều là

chất rắn màu vàng, dễ tan trong nước. Các oxit này được điều chế bằng cách:

- đun nóng các axit HTcO 4 và HReO 4 .

- đun nóng bột Tc hay Re trong luồng khí oxi.

Viết các phương trình phản ứng minh họa.

Hướng dẫn:

3Re + 7 HNO 3 → 3HReO 4 + 7NO↑ + 2H 2 O

t

2HReO 0

4 Re 2 O 7 + H 2 O

t

2HTcO 0

4 Tc 2 O 7 + H 2 O

4Re + 7O 2

4Tc + 7O 2

0

t

2Re 2 O 7

0

t

2Tc 2 O 7

Re 2 O 7 + H 2 O → 2HReO 4

Tc 2 O 7 + H 2 O → 2HTcO 4

Phát hành PDF bởi Ths Nguyễn Thanh Tú

Đăng ký Word doc qua Zalo 0905779594 Email thanhtuqn88@gmail.com

39


I.1.5. Người ta có thể điều chế mangan bằng phương pháp điện phân dung dịch MnSO 4 .

Ngoài phương pháp điện phân có thể dùng phương pháp nào để điều chế mangan? Viết các

phương trình phản ứng.

Hướng dẫn:

Có thể điều chế Mn bằng phương pháp điện phân, nhiệt kim hoặc nhiệt silic:

2MnSO 4 + 2H 2 O → 2Mn + O 2 + 2H 2 SO 4

3Mn 3 O 4 + 8Al → 9Mn + 4Al 2 O 3

MnO 2 + Si → Mn + SiO 2

I.1.6. Mangan (II) oxit (MnO) được điều chế bằng cách:

a. Nhiệt phân Mn (II) oxalat hoặc cacbonat khi không có không khí (nung trong luồng khí

H 2 hoặc N 2 ).

b. Dùng H 2 hoặc CO để khử oxit bậc oxi hóa cao như MnO 2 , Mn 3 O 4 .

Viết các phương trình phản ứng.

Hướng dẫn:

a.

MnCO 3

0

t

MnO + CO 2 ↑

t

b. MnO 2 + H 0

2 MnO + H 2 O

I.1.7. MnO là chất tinh thể, màu xanh xám, t nc = 1780 0 C. Không tan trong nước, không phản

ứng với nước. Ở trạng thái tinh thể hoàn toàn bền trong không khí, nhưng ở dạng bột dễ bị

oxi hóa tạo thành các oxit cao như MnO 2 , Mn 2 O 3 . MnO bị H 2 khử thành kim loại (ở nhiệt độ

rất cao). MnO tan trong các axit tạo thành muối Mn (II).

Viết các phương trình phản ứng xảy ra.

Hướng dẫn:

6MnO + O 2 2Mn 3 O 4

t

MnO + H 0

2 Mn + H 2 O

MnO + 2HCl MnCl 2

+ H 2 O

40

Phát hành PDF bởi Ths Nguyễn Thanh Tú

Đăng ký Word doc qua Zalo 0905779594 Email thanhtuqn88@gmail.com


I.1.8.a. Từ MnO bằng phương pháp nào có thể thu được Mn(OH) 2 biết rằng MnO không tan

trong nước?

b. Bằng phản ứng nào chứng minh rằng Mn(OH) 2 có tính khử?

Hướng dẫn:

a. Chuyển MnO thành MnSO 4 hoặc MnCl 2 , sau đó cho dung dịch muối Mn 2+ tác dụng với

kiềm thu được kết tủa Mn(OH) 2 màu trắng.

b. Có thể dùng phản ứng :

2Mn(OH) 2 + O 2 (không khí) + 2H 2 O → 2Mn(OH) 4

I.1.9.a. Trong công nghiệp, MnSO 4 được điều chế bằng cách cho pirolusit (MnO 2 ) tan trong

H 2 SO 4 đặc nóng, hoặc nung hỗn hợp MnO 2 với FeSO 4 . Viết các phương trình phản ứng xảy

ra.

Hướng dẫn:

2MnO 2 + 2H 2 SO 4 đặc nóng → 2MnSO 4 + 2H 2 O + O 2 ↑

4MnO 2 + 4FeSO 4 → 4MnSO 4 + 2Fe 2 O 3 + O 2 ↑

b. MnSO 4 ở trạng thái khan hầu như là màu trắng. Khi kết tinh từ dung dịch nước tạo ra tinh

thể có màu hồng khác nhau phụ thuộc vào hàm lượng nước kết tinh: MnSO 4 .H 2 O;

MnSO 4 .4H 2 O; MnSO 4 .5H 2 O; MnSO 4 .7H 2 O. Trong tinh thể các sunfat đó, nhóm SO 4 đóng

vai trò cầu nối giữa các nguyên tử Mn (II). Viết công thức cấu tạo của MnSO 4 .4H 2 O.

Hướng dẫn:

c. Khi tác dụng với chất oxi hóa, muối mangan (II) thể hiện tính khử phụ thuộc vào môi

trường: trong môi trường kiềm mạnh: Mn 2+ →

2

MnO 4

; trong môi trường axit: Mn 2+ →

MnO 4 . Các dung dịch muối mangan (II) cũng đều bị oxi hóa bởi bột tẩy, hipoclorit, KMnO 4 ,

và những tác nhân oxi hóa khác tạo ra MnO 2 trong môi trường kiềm.

Hoàn thành các phương trình phản ứng sau:

3MnSO 4

+ 2KClO 3 + 12KOH nóng chảy →

Phát hành PDF bởi Ths Nguyễn Thanh Tú

Đăng ký Word doc qua Zalo 0905779594 Email thanhtuqn88@gmail.com

41


2MnSO 4

MnSO 4

Hướng dẫn:

+ 5PbO 2 + 6HNO 3 →

+ CaOCl 2 + 2NaOH →

2KMnO 4 + 3MnSO 4

+ 4KOH →

3MnSO 4 + 2KClO 3 + 12KOH nóng chảy → 3K 2 MnO 4 + 2KCl + 3K 2 SO 4 + 6H 2 O

2MnSO 4 + 5PbO 2 + 6HNO 3 → 2HMnO 4 + 3Pb(NO 3 ) 2 + 2PbSO 4 + 2H 2 O

MnSO 4 + CaOCl 2 + 2NaOH → MnO 2 ↓ + CaCl 2 + Na 2 SO 4 + H 2 O

2KMnO 4 + 3MnSO 4 + 4KOH → 5MnO 2 ↓ 3K 2 SO 4 + 2H 2 O

I.1.10.a. Viết phương trình phản ứng nhiệt phân Mn(NO 3 ) 2 , MnCO 3 .

b. Khi cho muối mangan (II) tác dụng với muối sunfua tan tạo ra kết tủa màu hồng thẫm.

Khi để lâu (không có không khí) tạo ra dạng khan MnS màu xanh. Khi để trong không khí,

bị oxi hóa chuyển dần thành màu nâu đen. Viết phương trình phản ứng.

Hướng dẫn:

t

a. Mn(NO 3 ) 0

2 MnO 2 + 2NO 2 ↑: đây là phương pháp điều chế MnO 2 nguyên chất.

0

t

MnCO 3 MnO + CO 2 ↑

b. Mn 2+ + S 2- → MnS

MnS + O 2 + 2H 2 O → S + MnO 2 .2H 2 O

I.1.11.a. Một số phức amoniacat của Mn (II) như [Mn(NH 3 ) 6 ]Cl 2 , [Mn(NH 3 ) 6 ](ClO 4 ) 2 dễ

dàng bị nước phân hủy tạo thành Mn(OH) 2 . Vì vậy các loại phức trên chỉ tồn tại ở trạng thái

rắn, hoặc trong dung dịch nhưng phải có lượng dư NH 3 rất lớn.

b. Mn(OH) 2 để trong không khí dần dần bị hóa nâu.

c. Trong môi trường kiềm, Mn(OH) 2 sẽ bị oxi hóa, ngay cả O 2 của không khí.

Viết các phương trình phản ứng xảy ra.

Hướng dẫn:

a. [Mn(NH 3 ) 6 ]Cl 2` + 2H 2 O

Mn(OH) 2 ↓ + 2NH 4 Cl + 4NH 3

b. 2Mn(OH) 2 ↓ + O 2 + 2H 2 O → 2Mn(OH) 4

hay 2Mn(OH) 2 ↓ + O 2 → 2[MnO 2 .H 2 O]

II IV

2 4

c. 6Mn(OH) 2 ↓ + O 2 → 2 Mn Mn O + 6H 2 O

Phát hành PDF bởi Ths Nguyễn Thanh Tú

Đăng ký Word doc qua Zalo 0905779594 Email thanhtuqn88@gmail.com

42


I.1.12.a. MnO 2 là chất oxi hóa mạnh nhưng khi tác dụng với chất oxi hóa mạnh hơn thì

MnO 2 thể hiện tính khử. Tìm dẫn chứng để minh họa cho kết luận đó.

b. Viết phương trình của các phản ứng sau:

MnO 2 + HCl →

MnO 2 + NaOH nóng chảy →

MnO 2 + H 2 SO 4 đặc nóng →

Hướng dẫn:

a. Ví dụ phản ứng:

3MnO 2 + KClO 3 + 6KOH → 3K 2 MnO 4 + KCl + 3H 2 O

2MnO 2 + 3PbO 2 + 6HNO 3 → 2HMnO 4 + 3Pb(NO 3 ) 2 + 2H 2 O

b.

MnO 2 + 4HCl → MnCl 2 + Cl 2 + 2H 2 O

MnO 2 + 2NaOH nóng chảy → Na 2 MnO 3 + H 2 O

hoặc MnO 2 + 4NaOH nóng chảy → Na 4 MnO 4 + 2H 2 O

MnO 2 + H 2 SO 4 đặc nóng → 2MnSO 4 + 2H 2 O + O 2 ↑

I.1.13.a. Từ MnO 2 bằng phản ứng nào có thể thu được: MnCl 2 , KMnO 4 , Mn 2 O 7 ?

b. Từ MnO 2 điều chế Ba(MnO 4 ) 2 . Viết các phương trình phản ứng.

Hướng dẫn:

a. Có thể cho MnO 2 tác dụng với HCl đặc thu được MnCl 2 :

MnO 2 + 4HCl → MnCl 2 + Cl 2 + 2H 2 O

Nung hỗn hợp MnO 2 + KClO 3 + KOH rắn phản ứng tạo ra K 2 MnO 4 , hòa tan, lọc dung

dịch nước lọc có K 2 MnO 4 . Axit hóa dung dịch K 2 MnO 4 thu được KMnO 4 . Đun nóng dung

dịch ở 80 0 C, sau đó làm nguội, tinh thể KMnO 4 xuất hiện.

Muốn thu được Mn 2 O 7 , cho H 2 SO 4 đặc tác dụng với tinh thể KMnO 4 :

2KMnO 4 + H 2 SO 4 → 2HMnO 4 + K 2 SO 4

2HMnO 4 → Mn 2 O 7 + H 2 O

b. Nung hỗn hợp Ba(OH) 2 và MnO 2 trong không khí:

2Ba(OH) 2 + 2MnO 2 + O 2 → 2BaMnO 4 + 2H 2 O

3BaMnO 4 + 2H 2 O → Ba(MnO 4 ) 2 + MnO 2 + 2Ba(OH) 2

Ba(OH) 2 + CO 2 → BaCO 3 + H 2 O

43

Phát hành PDF bởi Ths Nguyễn Thanh Tú

Đăng ký Word doc qua Zalo 0905779594 Email thanhtuqn88@gmail.com


lọc và rửa sản phẩm, Ba(MnO 4 ) 2 còn lại trong dung dịch.

I.1.14.a. MnO 2 là chất bột màu nâu đen, không tan trong nước. Khi đun nóng bị phân hủy và

khi đun nóng với H 2 SO 4 đặc nóng tạo ra O 2 .

b. MnO 2 và hiđroxit Mn(OH) 4 đều có tính lưỡng tính. Tác dụng với H 2 SO 4 đặc tạo ra muối

Mn(SO 4 ) 2 không bền, còn khi nung nóng chảy với kiềm tạo ra các hợp chất

I

M MnO và

2 3

I

M MnO .

4 4

Viết các phương trình phản ứng xảy ra.

Hướng dẫn:

t

a. 3MnO 0

2 Mn 3 O 4 + O 2 ↑

2MnO 2 + 2H 2 SO 4 đặc nóng → 2MnSO 4 + 2H 2 O + O 2 ↑

b. MnO 2 + 2NaOH nóng chảy → Na 2 MnO 3 + H 2 O

MnO 2 + 4NaOH nóng chảy → Na 4 MnO 4 + 2H 2 O

I.1.15. MnO 2 có tính oxi hóa mạnh và có cả tính khử.

a. Tính oxi hóa thể hiện trong các phản ứng:

- MnO 2 oxi hóa HCl đặc.

- Trong môi trường axit: MnO 2 oxi hóa muối Fe (II); oxi hóa axit oxalic; oxi hóa H 2 O 2 ; oxi

hóa axit sunfurơ thành Mn (II) đithionat.

b. Khi tác dụng với chất oxi hóa mạnh hơn, MnO 2 thể hiện tính khử.

- MnO 2 bị oxi hóa bởi KNO 3 , KClO 3 trong môi trường kiềm.

- Khi cho MnO 2 trộn với dung dịch KOH 5% đun nóng ở 250 0 C rồi cho không khí đi qua sẽ

tạo ra K 2 MnO 4 màu lục.

- Trong môi trường axit MnO 2 bị oxi hóa bởi PbO 2 tạo ra dung dịch màu tím.

Viết các phương trình phản ứng xảy ra.

Hướng dẫn:

MnO 2 + 4HCl → MnCl 2 + Cl 2 ↑ + 2H 2 O

MnO 2 + 2FeSO 4 + 2H 2 SO 4 → MnSO 4 + Fe 2 (SO 4 ) 3 + 2H 2 O

MnO 2 + (COOH) 2 + H 2 SO 4 → MnSO 4 + CO 2 + 2H 2 O

MnO 2 + 2H 2 SO 3 → MnS 2 O 6 + 2H 2 O

2MnO 2 + 4KOH + O 2 → 2K 2 MnO 4 + 2H 2 O

Phát hành PDF bởi Ths Nguyễn Thanh Tú

Đăng ký Word doc qua Zalo 0905779594 Email thanhtuqn88@gmail.com

44


Hoặc 3MnO 2 + 6KOH + KClO 3 → 3K 2 MnO 4 + KCl + 3H 2 O

MnO 2 + KNO 3 + 2KOH → K 2 MnO 4 + KNO 2 + H 2 O

2MnO 2 + 3PbO 2 + 6HNO 3 → 2HMnO 4 + 3Pb(NO 3 ) 2 + 2H 2 O

I.1.16.a. Viết phương trình phản ứng mô tả tính oxi hóa và tính khử của K 2 MnO 4 .

b. Có thể thu được H 2 MnO 4 bằng phương pháp cho H 2 SO 4 đặc tác dụng với muối K 2 MnO 4

được không?

Hướng dẫn:

a. Có thể bằng các phản ứng:

K 2 MnO 4 + 2H 2 S + 2H 2 SO 4 → 2S + MnSO 4 + K 2 SO 4 + 4H 2 O

2K 2 MnO 4 + Cl 2 →2KMnO 4 + 2KCl

4K 2 MnO 4 + O 2 + 2H 2 O → 4KMnO 4 + 4KOH

b. H 2 MnO 4 không bền nhanh chóng bị phân hủy:

K 2 MnO 4 + H 2 SO 4 → H 2 MnO 4 + K 2 SO 4

2H 2 MnO 4 → 2HMnO 4 + MnO 2 + 2H 2 O

I.1.17.a. Các muối

hay bazơ? Giải thích.

Hướng dẫn:

2

MnO 4

, MnO 4

,

Trong dung dịch tồn tại cân bằng sau:

2

TcO 4

,

4

3MnO 4 2- + 2H 2 O ⇌ 2MnO 4 - + MnO 2 + 4OH -

TcO 2

, Re O 4

, Re O

4

bền trong môi trường axit

từ cân bằng đó có thể thấy được các muối manganat bền trong môi trường kiềm, còn muối

pemanganat bền trong môi trường axit.

(Các muối

2

TcO 4

,

2

Re O 4

trong dung dịch nước cũng tồn tại cân bằng như trên).

b. Khi axit hóa dung dịch K 2 MnO 4 , hoặc khi đun nóng dung dịch K 2 MnO 4 thấy tạo ra dung

dịch màu tím và kết tủa màu nâu đen. Viết các phương trình phản ứng xảy ra.

Hướng dẫn:

Khi axit hóa dung dịch K 2 MnO 4 tạo ra axit H 2 MnO 4 nhưng lại không bền, phân hủy ngay

theo phương trình:

3H 2 MnO 4 → 2HMnO 4 + MnO 2 + 2H 2 O

3K 2 MnO 4 + 2H 2 O → 2KMnO 4 + MnO 2 ↓ + 4KOH

Phát hành PDF bởi Ths Nguyễn Thanh Tú

Đăng ký Word doc qua Zalo 0905779594 Email thanhtuqn88@gmail.com

45


I.1.18. Mn (VI) ứng với số oxi hóa trung gian, nên vừa có tính oxi hóa, vừa có tính khử. Viết

các phương trình phản ứng minh họa tính oxi hóa của Mn (VI) (trong môi trường axit, bazơ)

và phản ứng minh họa tính khử của Mn (VI).

Hướng dẫn:

K 2 MnO 4 + 2H 2 S + 2H 2 SO 4 → 2S↓ + MnSO 4 + K 2 SO 4 + 4H 2 O

K 2 MnO 4 + 2Fe(OH) 2 + 2H 2 O → MnO 2 ↓ + 2Fe(OH) 3↓ + 2KOH

2K 2 MnO 4 + Cl 2 → 2KMnO 4 + 2KCl

4K 2 MnO 4 + O 2 + 2H 2 O → 4KMnO 4 + 4KOH

I.1.19. Khi đun nóng một dung dịch gồm muối Mn (II) với PbO 2 và HNO 3 tạo ra axit

pemanganic. Khi làm lạnh một dung dịch đặc KMnO 4 và AgNO 3 thu được kết tủa màu đỏ

AgMnO 4 (Tt = 1,6.10 -3 ). Nếu cho BaCl 2 vào dung dịch có chứa AgMnO 4 thì AgCl sẽ kết tủa

(Tt = 1,78.10 -10 ), còn lại Ba(MnO 4 ) 2 trong dung dịch. Từ dung dịch bari pemanganat, cho

thêm một lượng H 2 SO 4 cần thiết, sau khi tách kết tủa BaSO 4 , dung dịch còn lại có chứa

HMnO 4 . Viết các phương trình phản ứng xảy ra.

Hướng dẫn:

2Mn(NO 3 ) 2 + 5PbO 2 + 6HNO 3 → 2HMnO 4 + 5Pb(NO 3 ) 2 + 2H 2 O

KMnO 4 + AgNO 3 → AgMnO 4 ↓ (đỏ) + KNO 3

2AgMnO 4 + BaCl 2 → Ba(MnO 4 ) 2 ↓ (đỏ) + 2AgCl↓

Ba(MnO 4 ) 2 + H 2 SO 4 → 2HMnO 4 + BaSO 4 ↓

I.1.20. Axit pemanganic HMnO 4 là một axit mạnh tương đương HCl và HNO 3 ; dung dịch

chỉ có thể cô đặc đến 20%, sau đó bắt đầu bị phân hủy. Anhiđrit tương ứng với HMnO 4 là

Mn 2 O 7 (là chất lỏng, màu xanh thẫm) thu được khi cho H 2 SO 4 đặc 90% tác dụng với

KMnO 4 . Khi đun nóng Mn 2 O 7 bị phân hủy. Viết các phương trình phản ứng xảy ra.

Hướng dẫn:

4HMnO 4 → 4MnO 2 ↓ + 3O 2 ↑ + 2H 2 O

2KMnO 4 + H 2 SO 4 → Mn 2 O 7 + K 2 SO 4 + H 2 O

t

2Mn 2 O 0

7 4MnO 2 ↓ + 3O 2 ↑

I.1.21. Kali pemanganat (KMnO 4 ) là một muối quan trọng nhất của axit pemanganic. Có thể

điều chế KMnO 4 bằng các cách:

- cho K 2 MnO 4 tác dụng với H 2 SO 4 .

Phát hành PDF bởi Ths Nguyễn Thanh Tú

Đăng ký Word doc qua Zalo 0905779594 Email thanhtuqn88@gmail.com

46


- cho clo tác dụng với K 2 MnO 4 .

- cho khí CO 2 tác dụng với dung dịch K 2 MnO 4 . Lọc tách MnO 2 , sau đó cô dung dịch nước

lọc. Kali pemanganat có độ hòa tan bé hơn kali sunfat nên kết tinh trước. Viết các phương

trình phản ứng xảy ra.

Hướng dẫn:

3K 2 MnO 4 + 2H 2 SO 4 → 2KMnO 4 + 2K 2 SO 4 + MnO 2 ↓ + 2H 2 O

2K 2 MnO 4 + Cl 2 → 2KMnO 4 + 2KCl

3K 2 MnO 4 + 4CO 2 + 2H 2 O → 2KMnO 4 + MnO 2 ↓ + 4KHCO 3

hay 3K 2 MnO 4 + 4CO 2 → 2KMnO 4 + MnO 2 ↓ + 2K 2 CO 3

I.1.22.a. Khi nung nóng KMnO 4 bị phân hủy ở 200 0 C và khi đun sôi với dung dịch kiềm tạo

ra K 2 MnO 4 và O 2 . Viết các phương trình phản ứng.

Hướng dẫn:

t

2KMnO 0

4 K 2 MnO 4 + MnO 2 ↓ + O 2 ↑

4KMnO 4 + 4KOH → 4K 2 MnO 4 + O 2 ↑ + 2H 2 O

b. KMnO 4 là chất oxi hóa mạnh. Viết các phương trình phản ứng minh họa tính oxi hóa của

KMnO 4 trong môi trường axit, môi trường trung tính hoặc kiềm loãng, môi trường kiềm.

Hướng dẫn:

Trong môi trường axit:

2KMnO 4 + 5K 2 SO 3 + 3H 2 SO 4 → 2MnSO 4 + 6K 2 SO 4 + 3H 2 O

Trong môi trường trung tính hoặc kiềm loãng:

Trong môi trường kiềm:

2KMnO 4 + 3K 2 SO 3 + H 2 O → 2MnO 2 ↓ + 3K 2 SO 4 + 2KOH

2KMnO 4 + K 2 SO 3 + 2KOH → 2K 2 MnO 4 + K 2 SO 4 + H 2 O

c. Trong dung dịch, KMnO 4 oxi hóa được các chất: muối Fe (II) thành muối Fe (III); sunfit

thành sunfat; axit oxalic tạo ra CO 2 và H 2 O (60 0 C); iotua thành iot; axit halogenhiđric thành

halogen tương ứng; nitrit thành nitrat; hiđro mới sinh thành H 2 O; hiđro sunfua thành lưu

huỳnh; thiosunfat thành sunfat; amoniac và muối amoni thành nitơ; muối stanit thành muối

stanat. Viết các phương trình phản ứng.

Hướng dẫn:

2KMnO 4 + 10[H] + 3H 2 SO 4 → 2MnSO 4 + K 2 SO 4 + 8H 2 O

Phát hành PDF bởi Ths Nguyễn Thanh Tú

Đăng ký Word doc qua Zalo 0905779594 Email thanhtuqn88@gmail.com

47


6KMnO 4 + 10NH 3 + 9H 2 SO 4 → 5N 2 ↑ + 6MnSO 4 + 3K 2 SO 4 + 24H 2 O

Trong môi trường axit, oxi hóa được cả H 2 O 2 :

2KMnO 4 + 5H 2 O 2 + 4H 2 SO 4 → 2MnSO 4 + 2KHSO 4 + 8H 2 O + 5O 2

KMnO 4 oxi hóa được muối Mn (II) thành MnO 2 :

3MnSO 4 + 2KMnO 4 + 2H 2 O → 5MnO 2 + K 2 SO 4 + 2H 2 SO 4

I.1.23. Mangan tạo các oxit và các hiđroxit ứng với bậc oxi hóa từ thấp đến cao (+2, +3, +4,

+6, +7). So sánh lực axit của các oxit và hiđroxit đó. Giải thích.

Hướng dẫn:

Tính axit tăng theo dãy trên cũng có thể giải thích trong khuôn khổ sự tương quan điện tích

và bán kính của các ion. Khi chuyển từ Mn (II) – Mn (VII), bán kính của ion giảm dần, điện

tích của ion tăng dần, nên khả năng tách ion H + tăng dần theo chiều từ Mn(OH) 2 đến

HMnO 4 .

I.1.24. Dựa vào thuyết obitan phân tử (MO) so sánh độ bền của ion MnO

4

Giải thích vì sao các ion

Hướng dẫn:

Độ bền của ion MnO

4

thuyết MO như sau:

3

MnO 4

;

4

MnO 4

không tồn tại.

lớn hơn độ bền của ion

Trong ion phức tứ diện, chẳng hạn như MnO 4

, CrO 4

,

và ion

2

MnO 4

.

2

MnO 4

, được giải thích trong phạm vi của

MnO ... thì các obitan của chất tạo

phức tham gia hình thành liên kết gồm 5 obitan (n-1)d; một obitan ns và ba obitan np, với

các obitan p của bốn phối tử.

Trong ion MnO 4

có 24 electron hóa trị gồm 7 electron của nguyên tử Mn (3d 5 4s 2 ), 16

electron của 4 nguyên tử oxi O (2p 4 ) và 1 electron do tích điện của ion. Trong ion

2

4

2

MnO

4

25 electron hóa trị đã tham gia hình thành liên kết. Sự phân bố electron trong các obitan

phân tử (MO) trong ion MnO

4

và ion

2

MnO 4

có cấu hình như sau:

4

MnO : (σ lk ) 8 (π lk ) 10 (π) 6 Phát hành PDF bởi Ths Nguyễn Thanh Tú

Đăng ký Word doc qua Zalo 0905779594 Email thanhtuqn88@gmail.com

48


2

MnO 4

: (σ lk ) 8 (π lk ) 10 (π) 6 (π plk ) 1

Như vậy, trong ion

2

MnO

4

có một electron hóa trị chiếm MO phản liên kết, do đó ion

2

MnO 4

kém bền hơn ion MnO 4

.

Tương tự như vậy, trong các ion MnO ;

3

4

4

MnO 4

có 2 và 3 electron chiếm MO phản liên kết

nên càng kém bền, và vì vậy chúng ta hiểu được tính không bền của số phối trí 4 trong hợp

chất Mn (IV) và cũng hiểu được vì sao các ion

3

MnO 4

;

4

MnO 4

không tồn tại.

Các mức năng lượng của phức chất tứ diện

I.1.25. Hoàn thành các phương trình của các phản ứng sau:

1) MnSO 4 + KClO 3 + KOH (nóng chảy) →

2) MnSO 4 + PbO 2 + HNO 3 →

3) MnSO 4 + Br 2 + NaOH →

4) MnBr 2 + H 2 O 2 + KOH →

5) MnSO 4 + CaOCl 2 + NaOH →

Hướng dẫn:

1) 3MnSO 4 + 2KClO 3 + 12KOH → 3K 2 MnO 4 + 2KCl + 6H 2 O + 3K 2 SO 4

2) 2MnSO 4 + 5PbO 2 + 6HNO 3 → 2HMnO 4 + 3Pb(NO 3 ) 2 + 2PbSO 4 + 2H 2 O

49

Phát hành PDF bởi Ths Nguyễn Thanh Tú

Đăng ký Word doc qua Zalo 0905779594 Email thanhtuqn88@gmail.com


3) MnSO 4 + 2H 2 O 2 + 4KOH → K 2 MnO 4 + 4H 2 O + K 2 SO 4

4) MnSO 4 + 2Br 2 + 8NaOH → Na 2 MnO 4 + 4H 2 O + 4NaBr + Mn 2 SO 4

5) MnSO 4 + CaOCl 2 + 2NaOH → MnO 2 + Na 2 SO 4 + CaCl 2 + H 2 O

I.1.26. Viết các phương trình phản ứng sau:

Hướng dẫn:

1) KMnO 4 + MnCl 2 →

2) K 2 MnO 4 + Cl 2 →

3) KMnO 4 + KI + H 2 SO 4 →

4) KMnO 4 + KI + H 2 O →

5) KMnO 4 + FeSO 4 + H 2 SO 4 →

1) 2KMnO 4 + 3MnCl 2 + 2H 2 O → 5MnO 2 + 2KCl + 4HCl

2) 2K 2 MnO 4 + Cl 2 → 2KMnO 4 + 2KCl

3) 2KMnO 4 + 10KI + 3H 2 SO 4 → 2MnSO 4 + 6K 2 SO 4 + 5I 2 + 8H 2 O

4) 2KMnO 4 + 6KI + 4H 2 O → 2MnO 2 + 3I 2 + 8KOH

5) 2KMnO 4 + 10FeSO 4 + 8H 2 SO 4 → 5Fe 2 (SO 4 ) 3 + 2MnSO 4 + K 2 SO 4 + 8H 2 O

I.1.27. Hoàn thành các phương trình phản ứng dạng ion sau đây:

Hướng dẫn:

1) Mn 2+ + ClO - + OH - →

2) MnO -

4

+ NO 2

+ H + →

3) MnO 4

+ Fe + H + →

4) Mn 2 + BrO 3

-

+ H 2 O →

5) MnO 4

+ H 2 O 2 + OH - →

1) 2Mn 2+ + 4ClO - +8OH - → 2 MnO + 4Cl - + 4H 2 O

2) 2 MnO 4

+ 5NO - 2 + 6H + → 2Mn 2+ + 5NO - 3 + 3H 2 O

3) 3 MnO 4

+ 5Fe + 24H + → 3Mn 2+ + 5Fe 3+ + 12H 2 O

4) 5Mn 2+ + 2BrO 3 - + 4H 2 O → 5MnO 2 + Br 2 + 8H -

5) 2 MnO 4

+ H 2 O 2 + 2OH - 2

→ 2 MnO 4

+ O 2 + 2H 2 O

2

4

Phát hành PDF bởi Ths Nguyễn Thanh Tú

Đăng ký Word doc qua Zalo 0905779594 Email thanhtuqn88@gmail.com

50


I.1.28. Cho sơ đồ chuyển hóa như hình bên:

M là kim loại, A, B, C, D, G là các hợp chất của

kim loại M.

A là bột oxit màu nâu đen, trong đó hàm lượng

kim loại là 63,2%. Trong tự nhiên, A tồn tại ở

dạng khoáng chất X, là nguồn chính để sản xuất

kim loại M và các hợp chất của nó.

a. Xác định kim loại M và các chất A, B, C, D, G. Xác định tên gọi của khoáng chất X, có

thành phần chính là oxit A.

b. Viết các phương trình phản ứng của các phản ứng xảy ra trong sơ đồ trên.

c. Tại sao C lại được gọi là “tắc kè hoa”? Viết phương trình phản ứng của C với K 2 SO 3

trong các môi trường axit (H 2 SO 4 ), trung tính và kiềm (KOH) (3 phương trình) và mô tả các

biến đổi quan sát được.

d. Cho biết B là một chất oxi hóa mạnh. Viết phương trình phản ứng của B với kali sunfit

trong môi trường axit sunfuric.

Hướng dẫn:

a. Trước tiên, xác định kim loại dựa trên hàm lượng trong oxit, có công thức M 2 O x .

% m O = 1 - 0,632 = 0,368 =

Với n = 4, M A = 55 (Mn)

16x

2M + 16x M A = 13,74x

A

Vậy M là Mn, A là MnO 2 , B là K 2 MnO 4 , C là KMnO 4 , G là MnSO 4 , D là MnS

Tên gọi của khoáng chất X: pyrolusite.

b. Phương trình phản ứng:

1) MnO 2 + KNO 3 + 2KOH → K 2 MnO 4 + KNO 2 + H 2 O

(A)

(B)

(hoặc 5MnO 2 + 2KNO 3 + 2KOH → 5K 2 MnO 4 + N 2 + 4H 2 O)

2) 3K 2 MnO 4 + 4CO 2 + 2H 2 O → 2KMnO 4 + MnO 2 + 4KHCO 3

(C)

(hoặc 3K 2 MnO 4 + 2CO 2 + 2H 2 O → 2KMnO 4 + MnO 2 + 2K 2 CO 3 )

3) 3K 2 MnO 4 + 2H 2 SO 4 → 2KMnO 4 + MnO 2 + 2K 2 SO 4 + 2H 2 O

51

Phát hành PDF bởi Ths Nguyễn Thanh Tú

Đăng ký Word doc qua Zalo 0905779594 Email thanhtuqn88@gmail.com


4) 2K 2 MnO 4 + Cl 2 → 2KMnO 4 + 2KCl

5) 8KMnO 4 + 5K 2 S + 12H 2 SO 4 → 9K 2 SO 4 + 8MnSO 4 + 12H 2 O

(G)

6) MnSO 4 + K 2 S → MnS↓ + K 2 SO 4

7) Mn + S → MnS (D)

c. Gọi C là "tắc kè hoa" bởi KMnO 4 biến đổi màu sắc trong các môi trường khác nhau.

2KMnO 4 + 5K 2 SO 3 + 3H 2 SO 4 → 2MnSO 4 + 6K 2 SO 4 + 3H 2 O (dung dịch không màu)

2KMnO 4 + 3K 2 SO 3 + H 2 O → 2MnO 2 + 3K 2 SO 4 + 2KOH (kết tủa nâu)

2KMnO 4 + K 2 SO 3 + 2KOH → 2K 2 MnO 4 + K 2 SO 4 + H 2 O (dung dịch xanh lục)

d. K 2 MnO 4 + 2K 2 SO 3 + 2H 2 SO 4 → MnSO 4 + 3K 2 SO 4 + 2H 2 O

HSQG - 2018

I.1.29. Cho sơ đồ chuyển hóa như hình bên:

Các hợp chất A 1 , A 2 , A 3 , A 4 , A 5 , A 6 đều chứa

nguyên tố kim loại A và nguyên tố oxi. Số oxi hóa

của A tăng từ +2 đến +7 trong các hợp chất từ A 1 ,

A 2 , A 3 đến A 4 . Hợp chất A 2 chỉ gồm 2 nguyên tố,

phần trăm khối lượng của oxi trong A 2 là 36,78%.

a. Xác định công thức phân tử của các chất từ A 1 đến A 6 . Viết phương trình hóa học của các

phản ứng xảy ra trong sơ đồ trên.

b. Trong phòng thí nghiệm, dung dịch A 4 thường được sử dụng trong các phép chuẩn độ oxi

hóa – khử. Giải thích (bằng các phương trình hóa học) tại sao:

- Khi thực hiện phép chuẩn độ này, người ta cho dung dịch A 4 vào buret, chất khử vào bình

tam giác mà không làm ngược lại.

- Dung dịch A 4 được bảo quản trong các bình tối màu.

c. Cho dung dịch A 4 tác dụng với AgNO 3 , thư được kết tảu màu đỏ X 1 . Cho BaCl 2 (vừa đủ)

vào dung dịch bão hòa của X 1 , thu được kết tủa trắng X 2 và dung dịch của X 3 . Khi cho

H 2 SO 4 loãng vào dung dịch của X 3 , thu được kết tủa trắng X 4 và dung dịch của X 5 . Đun

nóng dung dịch của X 5 thì thu được kết tủa A 2 . Còn nếu cho A 4 tác dụng với H 2 SO 4 đậm đặc

Phát hành PDF bởi Ths Nguyễn Thanh Tú

Đăng ký Word doc qua Zalo 0905779594 Email thanhtuqn88@gmail.com

52


thì thu được oxit X 6 là một chất oxi hóa rất mạnh. Xác định công thức phân tử các chất từ X 1

đến X 6 và viết phương trình hóa học của các phản ứng xảy ra trong các thí nghiệm trên. Biết

các hợp chất X 1 , X 3 , X 5 , X 6 đều chứa nguyên tố kim loại A.

Hướng dẫn:

a. Hợp chất A 2 có 2 nguyên tố gồm nguyên tố kim loại A và nguyên tố oxi, gọi công thức

của A 2 là A 2 O n .

Phần trăm khối lượng của oxi trong A 2 là 36,78% nên phần trăm khối lượng của A là

63,22%.

Có:

16n 36,78

2M 63, 22

→ M A = 13,75n

A

Với n = 4, M A = 55 (Mn)

Sơ đồ phản ứng:

Các phương trình hóa học:

0

t

(1) Mn(NO 3 ) 2 MnO 2 + 2NO 2

(2) K 2 MnO 4 + 2Fe(OH) 2 + 2H 2 O → MnO 2 + 2Fe(OH) 3 + 2KOH

(3) 3MnO 2 + KClO 3 + 6KOH

(4) 2K 2 MnO 4 + Cl 2 → 2KMnO 4 + 2KCl

0

t

3K 2 MnO 4 + KCl + 3H 2 O

t

(5) 2KMnO 0

4 K 2 MnO 4 + MnO 2 + O 2

Hoặc: 2KMnO 4 + K 2 SO 3 + 2KOH → 2K 2 MnO 4 + K 2 SO 4 + H 2 O

(6) 2KMnO 4 + 3K 2 SO 3 + H 2 O → 2MnO 2 + 3K 2 SO 4 + 2KOH

(7) 2KMnO 4 + 5K 2 SO 3 + 3H 2 SO 4 → 2MnSO 4 + 6K 2 SO 4 + 3H 2 O

53

Phát hành PDF bởi Ths Nguyễn Thanh Tú

Đăng ký Word doc qua Zalo 0905779594 Email thanhtuqn88@gmail.com


b. Giải thích:

(8) 3MnSO 4 + 2KMnO 4 + H 2 O → 5MnO 2 + K 2 SO 4 + 2H 2 SO 4

(9) MnSO 4 + 2NaOH → Mn(OH) 2 + Na 2 SO 4

(10) Mn(OH) 2 + H 2 O 2 → MnO 2 + 2H 2 O

- Nếu để KMnO 4 trong bình tam giác, khi chuẩn độ oxi hóa khử sẽ tạo thành Mn 2+ , Mn 2+

phản ứng với lượng dư MnO

4

Phương trình hóa học:

tạo thành kết tủa khó tan MnO 2 , gây sai số chuẩn độ.

2 MnO 4

+ 3Mn 2+ + 2H 2 O → 5MnO 2 ↓ + 4H +

- Dung dịch KMnO 4 để lâu trong các bình sáng màu sẽ bị phân hủy nhanh hơn dưới tác dụng

của ánh sáng:

t

2KMnO 0

4 K 2 MnO 4 + MnO 2 + O 2

3K 2 MnO 4 + 2H 2 O → 2KMnO 4 + MnO 2 + 4KOH

4KMnO 4 + 2H 2 O → 4MnO 2 + 3O 2 + 4KOH

c. Xác định các chất và viết phương trình hóa học:

(1) KMnO 4 + AgNO 3 → AgMnO 4 + KNO 3

(X 1 )

(2) 2AgMnO 4 + BaCl 2 → Ba(MnO 4 ) 2 + 2AgCl↓

(X 3 ) (X 2 )

(K s của AgMnO 4 là 1,6.10 -3 ; K s của AgCl là 1,7.10 -10 )

(3) Ba(MnO 4 ) 2 + H 2 SO 4 → 2HMnO 4 + BaSO 4

(X 5 ) (X 4 )

(4) HMnO 4 → 4MnO 2 + 3O 2 + 2H 2 O

(A 2 )

(5) 2KMnO 4 + H 2 SO 4 đặc → K 2 SO 4 + Mn 2 O 7 + H 2 O (tách nước của HMnO 4 )

I.2. Bài tập định lượng: xác định công thức, giản đồ Latimer, tinh thể, chuẩn độ, phức

chất, …

I.2.1. Dạng bài tập về giản đồ Latimer.

Cho giản đồ Latimer của các thế oxi hóa khử của mangan ở pH = 0 như sau:

MnO MnO MnO Mn Mn Mn

? 2 2,27V ? 3 1,5V 2

1,18V

4 4 2

+1,7V

+1,23V

54

Phát hành PDF bởi Ths Nguyễn Thanh Tú

Đăng ký Word doc qua Zalo 0905779594 Email thanhtuqn88@gmail.com


0 0

a. Từ giản đồ trên tính: E 2 ;E

3

b.

.

MnO 4 /MnO4 MnO 2 /Mn

2

MnO 4

; Mn 3+ có bền trong môi trường axit không? Giải thích.

Hướng dẫn:

a.

E

0

2

MnO 4 /MnO4

1,7.(7 4) 2, 27.(6 4)

0,56(V)

(7 6)

E

b.

0

3

MnO 2 /Mn

1, 23.(4 2) 1,5(3

2)

0,96 (V)

4 3

MnO MnO MnO Mn Mn Mn

0,56 2 2,27V 0,96 3 1,5V 2

1,18V

4 4 2

+1,7V

+1,23V

Nhận xét: trong môi trường axit

bên phải lớn hơn thế bên trái).

2

MnO 4

có thể tự chuyển hóa thành MnO

4

2

3 MnO 4

+ 4H + → 2 MnO 4

+ MnO 2 + 2H 2 O E 0 = 2,26 - 0,56 = 1,7V

và MnO 2 (vì thế

∆G 0 = -2,3RTlgK

K =

0

nE /0,059

2.1,7/0,059

10 K = 10 = 10 58

Do đó ion

pH = 0.

2

MnO 4

bị chuyển hóa hoàn toàn thành MnO

4

và MnO 2 trong môi trường axit ở

Tương tự Mn 3+ cũng không bền, dễ chuyển hóa thành Mn 2+ và MnO 2 (vì thế bên phải lớn

hơn thế bên trái).

I.2.2. Dạng bài tập về tinh thể

Bột reni bị đốt cháy trong không khí tạo thành oxit A chứa 76,9% kim loại. Khử A bằng

CO khi đun nóng tạo thành một reni oxit C khác. Ô mạng cơ sở của C là hình lập phương

(cho ở dưới) với độ dài cạnh a = 3,734.10 -10 m.

Phát hành PDF bởi Ths Nguyễn Thanh Tú

Đăng ký Word doc qua Zalo 0905779594 Email thanhtuqn88@gmail.com

55


Re

O

Một số phản ứng của các hợp chất reni được cho trong sơ đồ. Cần lưu ý rằng phân tử muối

nghịch từ G (33,08% Re và 6,95% K về khối lượng), chứa 2 nguyên tử reni và có liên kết

reni - reni.

a. Xác định công thức các chất A - G và viết các phương trình phản ứng.

b. Dự đoán B có tính axit mạnh, yếu hay trung bình?

c. Tính khối lượng riêng lí thuyết của tinh thể oxit С.

d. Xác định độ bội của liên kết reni - reni trong cấu trúc của G và xác định các kiểu liên kết

(, , ) được tạo thành trong hợp chất này.

Hướng dẫn:

a. Từ dữ kiện về hàm lượng nguyên tố, có thể xác định được công thức của A (Re x O y )

x : y = (76,9/186,2):(23,1/16) = 0,413/1,444 = 2 : 7 (Re 2 O 7 ).

Sự hyđrat hóa oxit tạo thành axit perenic HReO 4 (B). Có thể thấy rõ từ hình ảnh rằng các

nguyên tử reni được đặt ở đỉnh của khối lập phương, và các nguyên tử oxi thì nằm giữa các

cạnh. Như vậy, trong một 1 mạng cơ sở của oxit C thì có 8.1/8 = 1 nguyên tử reni và 12.1/4

= 3 nguyên tử oxi (ReO 3 ).

Oxit ReO 3 tan trong một bazơ tạo thành D là K 2 ReO 4 . Khi pha loãng với nước, dung dịch

renat (VI) có tính chất tương tự như manganat (VI), nghĩa là sẽ bị dị phân (tự oxi hóa - khử)

và màu sắc của các hợp chất reni tương tự như các hợp chất của mangan: E = ReO 2 , F =

KReO 4 .

Trong G, tỉ lệ K : Re = (6,95/39,1):(33,08/186,2) = 0,177/0,177 = 1:1. Tổng hàm lượng

của kali và reni nhỏ hơn 50%, do dó nhiều khả năng sẽ có 1 nguyên tố nặng nữa trong G.

Trong số các nguyên tố xuất hiện trong hệ phản ứng của chúng ta, brom là khả thi nhất.

Phát hành PDF bởi Ths Nguyễn Thanh Tú

Đăng ký Word doc qua Zalo 0905779594 Email thanhtuqn88@gmail.com

56


Khối lượng mol của G (ở dạng tối giản) là 186,2/0,3308 = 562,9 g/mol; 225,3 g/mol thuộc

về kali và reni; phần còn lại - tương đương 337,6 g/mol - thuộc về brom và các nguyên tố

nhẹ khác. Do 337,6/79,9 = 4,23 nên số nguyên tử brom tối đa chỉ có thể là 4. Phần còn lại,

tương đương với 337,6 - 4.79,9 = 18 g/mol 1 phân tử nước. Vậy, công thức đơn giản

của G có thể là KReBr 4 .H 2 O. Do G có 2 nguyên tử Re nên công thức phân tử là G =

K 2 [Re 2 Br 8 ].2H 2 O.

Công thức các chất:

А = Re 2 O 7 B = HReO 4 C = ReO 3 D = K 2 ReO 4

E = ReO 2 F = KReO 4 G = K 2 [Re 2 Br 8 ].2H 2 O

Phương trình phản ứng:

4Re + 7O 2 = 2Re 2 O 7

Re 2 O 7 + H 2 O = 2HReO 4

Re 2 O 7 + CO = 2ReO 3 + CO 2

ReO 3 + 2KOH = K 2 ReO 4 + H 2 O

3K 2 ReO 4 + 2H 2 O = 2KReO 4 + ReO 2 ↓ + 4KOH

2KReO 4 + 4H 3 PO 2 + 8HBr = K 2 [Re 2 Br 8 ].2H 2 O + 4H 3 PO 3 + 2H 2 O

b. B là axit mạnh, do có 3 nguyên tử oxi đầu mạch (hút electron mạnh).

c. Do ô mạng cơ sở của ReO 3 chứa 1 đơn vị phân tử 1 ReO 3 nên:

KLR (ReO 3 ) = m/V= (M(ReO 3 )) / (N A .a 3 ) = (1.0,2342 kg/mol)/(6,02.10 23 mol -1 .(3,734.10 -

10 m) 3 ) = 7472.5 kg/m 3

Cấu hình electron của Re +3

là 5d 4 6s 0 6p 0 . Chú ý đến tính chất nghịch từ của

K 2 [Re 2 Br 8 ].2H 2 O, có thể đề xuất rằng có sự ghép cặp của tất cả các electron độc thân của 2

nguyên tử reni cạnh nhau và tạo thành liên kết reni - reni bậc 4. Sự tạo thành liên kết này có

thể được biểu diễn như sau: Do các d-electron của reni xảy ra sự tạo thành liên kết kim loại -

kim loại và điều này đến từ số lượng và hình dạng các d-obitan, nên có thể rút ra một kết

luận rằng 1 liên kết được tạo thành bởi sự xen phủ các d-obitan dọc theo đường nối xuyên

qua các nguyên tử reni gọi đó là trục z), nghĩa là một liên kết σ. 2 liên kết khác được tạo

thành bởi sự xen phủ các d-obitan bên trục z, nghĩa là có 2 liên kết π. Cuối cùng, liên kết thứ

tư là sự xen phủ của các d-obitan trong mặt phẳng xy (liên kết δ). Do vậy, liên kết bậc 4

trong K 2 [Re 2 Br 8 ].2H 2 O có thể được mô tả là (1σ + 2π + 1δ).

Phát hành PDF bởi Ths Nguyễn Thanh Tú

Đăng ký Word doc qua Zalo 0905779594 Email thanhtuqn88@gmail.com

57


I.2.3. Dạng bài tập về chuẩn độ

HSQG - 2015

Một loại quặng chỉ chứa MnO 2 và tạp chất trơ. Cân chính xác 0,5000 gam quặng trên rồi

cho vào bình cầu có nhánh. Thêm từ từ vào bình này khoảng 50 ml dung dịch HCl đặc. Đun

nóng đến khi mẫu quặng tan hết, chỉ còn lại tạp chất trơ. Hấp thụ hoàn toàn khí Cl 2 thoát ra

bằng lượng dư dung dịch KI, thu được dung dịch X. Chuyển toàn bộ X vào bình định mức 250

ml, thêm nước cất đến vạch mức, lắc đều. Chuẩn độ 25,00 ml dung dịch này bằng dung dịch

chuẩn Na 2 S 2 O 3 0,05 M (chỉ thị hồ tinh bột) thì hết 22,50 ml.

a. Viết các phương trình hóa học xảy ra.

b. Tính hàm lượng % theo khối lượng của MnO 2 trong quặng trên.

Hướng dẫn:

2. a) Khử MnO 2 bằng lượng dư dung dịch HCl nóng:

MnO 2 + 4HCl → MnCl 2 + Cl 2 + 2H 2 O

Toàn bộ lượng Cl 2 thoát ra được hấp thụ vào dung dịch KI dư :

Cl 2 + 3KI → KI 3 + 2KCl

Chuẩn độ lượng KI 3 bằng dung dịch chuẩn Na 2 S 2 O 3 :

KI 3 + 2Na 2 S 2 O 3 → Na 2 S 4 O 6 + 2NaI + KI

b) Hàm lượng phần trăm về khối lượng MnO 2 trong quặng

Từ các phản ứng trên ta có:

1

n n n n

2

MnO Cl

2 2 I

Na S

2 2 2O3

Số mol Na 2 S 2 O 3 tiêu tốn để chuẩn độ 25,00 ml dung dịch X:

n

nI

Na2S2O3

22,50.0,05 1,125.10

3

( )

1000

mol

Số mol I 2 (dạng I 3 - ) có trong 250,0 ml dung dịch X:

3

1,125.10 .10

3

5,625.10 ( mol)

2

2

Số mol MnO 2 = Số mol I 2 (theo phương trình phản ứng) = 5,625.10 -3 (mol)

% Khối lượng MnO 2 :

3

5,625.10 .(55 16.2)

%mMnO

97,88%

2

0,5000

58

Phát hành PDF bởi Ths Nguyễn Thanh Tú

Đăng ký Word doc qua Zalo 0905779594 Email thanhtuqn88@gmail.com


II. Bài tập về Fe

II.1. Bài tập lý thuyết, giải thích hiện tượng, chuỗi phản ứng, xác định chất, …

II.1.1. Sắt (II) oxit là chất bột màu đen, được điều chế bằng cách dùng H 2 để khử Fe 2 O 3 ở

300 0 C, hoặc bằng cách nhiệt phân Fe (II) oxalat (chất rắn màu vàng) trong điều kiện không

có không khí. FeO dễ tan trong axit, không tan trong dung dịch kiềm, là oxit bazơ. Khi tan

trong axit loãng tạo ra ion [Fe(OH 2 ) 6 ] 2+ . Viết các phương trình phản ứng.

Hướng dẫn:

Fe 2 O 3 + H 2

0

300 C

2FeO + H 2 O

FeO + 2H 3 O + + 3H 2 O [Fe(OH 2 ) 6 ] 2+ (màu xanh nhạt)

II.1.2. Fe(OH) 2 kết tủa màu trắng xanh, dễ hóa nâu đỏ trong không khí. Fe(OH) 2 tan trong

dung dịch kiềm đặc nóng tạo ra hipoferit. Khi cho Sr(OH) 2 hay Ba(OH) 2 tác dụng lên

Fe(OH) 2 cũng tạo ra các hipoferit tương tự (Sr 2 [Fe(OH) 6 ] và Ba 2 [Fe(OH) 6 ]). Viết các

phương trình phản ứng.

Hướng dẫn:

4Fe(OH) 2 + O 2 + 2H 2 O 4Fe(OH) 3

Fe(OH) 2

+ 2NaOH đặc

0

t

Na 2 [Fe(OH) 4 ]

Fe(OH) 2

+ 2Sr(OH) 2 đặc

0

t

Sr 2 [Fe(OH) 6 ]

0

t

Fe(OH) 2 + 2Ba(OH) 2 đặc Ba 2 [Fe(OH) 6 ]

II.1.3. Sắt (II) sunfat là muối quan trọng nhất trong thực tế, tinh thể có thành phần

FeSO 4 .7H 2 O.

- Trong công nghiệp FeSO 4 được điều chế bằng cách dùng O 2 của không khí và H 2 O oxi hóa

pirit sắt ở nhiệt độ thường. Trong phòng thí nghiệm được điều chế bằng cách hòa tan sắt tinh

khiết trong H 2 SO 4 loãng.

- FeSO 4 khi kết tinh từ dung dịch nước tạo ra tinh thể màu xanh lam [Fe(OH 2 ) 6 ]SO 4 .H 2 O.

Khi nung nóng tạo ra muối khan màu trắng và khi nung nóng mạnh tạo ra Fe 2 O 3 .

- Khi FeSO 4 tác dụng với khí NO cho dung dịch màu nâu tối do tạo ra phức chất nitrozo Fe

(II) sunfat.

Phát hành PDF bởi Ths Nguyễn Thanh Tú

Đăng ký Word doc qua Zalo 0905779594 Email thanhtuqn88@gmail.com

59


- FeSO 4 bị oxi hóa chậm khi để trong không khí. Trong môi trường axit tạo ra muối Fe (III);

còn khi không có mặt axit tạo ra muối bazơ.

- FeSO 4 là chất khử tốt, khử được muối Ag + và muối Au 3+ thành kim loại. Khử được HgCl 2

thành Hg 2 Cl 2 .

Viết các phương trình phản ứng xảy ra.

Hướng dẫn:

2FeS 2 + 7O 2 + 2H 2 O 2FeSO 4 + 2H 2 SO 4

Fe + H 2 SO 4 FeSO 4 + H 2 ↑

0

4FeSO 4

t 2Fe 2 O 3 + 4SO 2 + O 2

Phản ứng trên được dùng để điều chế Fe 2 O 3 .

FeSO 4 + NO [FeNO]SO 4 (nâu tối)

4FeSO 4 + O 2 + 2H 2 O 4Fe(OH)SO 4

4FeSO 4 + O 2 + 2H 2 SO 4 2Fe 2 (SO 4 ) 3 + 2H 2 O

Ag + + Fe 2+ Ag + Fe 3+

Au 3+ + 3Fe 2+ Au + 3Fe 3+

2HgCl 2 + 2Fe 2+ Hg 2 Cl 2 + 2Fe 3+ + 2Cl -

II.1.4. Sắt (II) nitrat tạo ra khi hòa tan sắt trong HNO 3 loãng, lạnh. Tốt nhất là bằng phản

ứng trao đổi giữa FeSO 4 và Pb(NO 3 ) 2 . Kết tinh ở dạng hexahiđrat Fe(NO 3 ) 2 .6H 2 O màu xanh

sáng ở nhiệt độ thường, còn ở -10 0 C kết tinh ở dạng nonahiđrat Fe(NO 3 ) 2 .9H 2 O. Khi đun

nóng dung dịch sắt (II) nitrat bị phân hủy tạo thành muối bazơ sắt (III).

Viết các phương trình phản ứng xảy ra.

Hướng dẫn:

8Fe + 20HNO 3 8Fe(NO 3 ) 2 + 2NH 4 NO 3 + 6H 2 O

FeSO 4 + Pb(NO 3 ) 2 Fe(NO 3 ) 2 + PbSO 4 ↓

4Fe(NO 3 ) 2 + O 2 + 2H 2 O 4Fe(OH)(NO 3 ) 2

II.1.5. Khi nung sắt (III) hiđroxit, nói đúng hơn là oxit bị hiđrat hóa Fe 2 O 3 .nH 2 O, ở nhiệt độ

thấp hơn 650 0 C tạo ra chất rắn ở dạng bột màu đỏ nâu, nhưng nếu nung ở nhiệt độ cao hơn

tạo thành tinh thể màu xám đen không còn khả năng tan trong axit, tương tự như Cr 2 O 3 ,

Phát hành PDF bởi Ths Nguyễn Thanh Tú

Đăng ký Word doc qua Zalo 0905779594 Email thanhtuqn88@gmail.com

60


Al 2 O 3 . Fe 2 O 3 cũng có thể điều chế bằng cách nung FeSO 4 .7H 2 O; FeO hoặc một muối sắt (II)

của axit dễ bay hơi khác. Trong công nghiệp được điều chế bằng cách nung quặng pirit sắt.

- Bên cạnh tính chất chủ yếu là tính bazơ, Fe 2 O 3 còn có tính axit tạo thành muối ferit màu

vàng hoặc đỏ, khi nung hỗn hợp Na 2 CO 3 và Fe 2 O 3 .

- Khi nung với C, hoặc nung trong luồng khí CO, H 2 hoặc khí than đá, Fe 2 O 3 sẽ bị khử thành

Fe.

Viết các phương trình phản ứng xảy ra.

Hướng dẫn:

Fe 2 O 3 .nH 2 O

FeSO 4 .7H 2 O

0

t

0

t

Fe 2 O 3 + nH 2 O

FeSO 4 + 7H 2 O

2FeSO 4

0

t

Fe 2 O 3 + SO 2 ↑ + SO 3 ↑

4FeS 2 + 11O 2

0

t

2Fe 2 O 3 + 8SO 2 ↑ + SO 3 ↑

Fe 2 O 3 + Na 2 CO 3

0

t

2NaFeO 2 + CO 2 ↑

Fe 2 O 3 + 3C

0

t

2Fe + 3CO↑

Fe 2 O 3 + 3CO

0

t

2Fe + 3CO 2 ↑

0

t

Fe 2 O 3 + 3H 2 2Fe + 3H 2 O

II.1.6. Sắt (II, III) oxit (Fe 3 O 4 ) được điều chế bằng cách nung nóng Fe 2 O 3 ở nhiệt độ trên

1450 0 C. Khi nung sắt trong không khí tạo ra Fe 3 O 4 (cùng với các sản phẩm khác). Fe 3 O 4

cũng tạo ra khi cho luồng hơi H 2 O hoặc CO 2 qua sắt nung đỏ. Cũng có thể điều chế bằng

cách cho hỗn hợp hiđro và hơi nước đi qua Fe 2 O 3 nung nóng. Ngoài ra để điều chế dạng tinh

khiết người ta đã cho dung dịch FeSO 4 và Fe 2 (SO 4 ) 3 tác dụng với dung dịch KOH, đun sôi

thu được kết tủa Fe 3 O 4 , làm khô kết tủa trong khí quyển H 2 thu được sản phẩm với thành

phần Fe 3 O 4 .2H 2 O.

Viết các phương trình phản ứng xảy ra.

Hướng dẫn:

6Fe 2 O 3

0

t

4Fe 3 O 4 + O 2 ↑

0

3Fe + 2O 2

t Fe 3 O 4

3Fe + 4H 2 O

Fe 3 O 4 + 4H 2 ↑

61

Phát hành PDF bởi Ths Nguyễn Thanh Tú

Đăng ký Word doc qua Zalo 0905779594 Email thanhtuqn88@gmail.com


3Fe + 4CO 2

0

t

Fe 3 O 4 + 4CO↑

3Fe 2 O 3 + H 2

FeSO 4 +

0

400 C

2Fe 3 O 4 + H 2 O

Fe 2 (SO 4 ) 3 + 8KOH Fe 3 O 4 + 4K 2 SO 4 + 4H 2 O

II.1.7. Fe 3 O 4 là chất bột màu đen, nóng chảy ở 1540 0 C; nung ở nhiệt độ cao hơn, mất một

phần oxi tạo ra FeO. Ở trạng thái ẩm, dễ bị oxi hóa ngoài không khí tạo ra Fe 2 O 3 . Khi tác

dụng với ít axit tạo ra Fe 2 O 3 và dung dịch muối sắt (II), sau đó thêm tiếp axit đến dư, sẽ tan

hoàn toàn tạo nên muối sắt (II) và sắt (III). Fe 3 O 4 bị Al, H 2 hoặc CO khử thành kim loại.

Viết các phương trình phản ứng xảy ra.

Hướng dẫn:

2Fe 3 O 4 0 t cao

6FeO + O 2 ↑

4Fe 3 O 4 + O 2 6Fe 2 O 3

Fe 3 O 4 + 2HCl

Fe 3 O 4 + 8HCl

FeCl 2 + Fe 2 O 3 + H 2 O

FeCl 2 + 2FeCl 3 + 4H 2 O

0

3Fe 3 O 4 + 8Al

1100 C 9Fe + 4Al 2 O 3

II.1.8. Sắt (III) hiđroxit là chất kết tủa màu đỏ nâu được tạo ra khi cho một tác nhân kết tủa

như kiềm, amoniac, dung dịch cacbonat tác dụng với muối Fe (III). Khi đun nóng đến 500 -

700 0 C sẽ mất nước hoàn toàn biến thành Fe 2 O 3 .

- Bên cạnh tính chất chủ yếu là tính bazơ, Fe(OH) 3 còn thể hiện tính axit yếu (axit ferơ

HFeO 2 ) nên khi cho Fe(OH) 3 tác dụng với kiềm đặc nóng hoặc bằng cách nấu chảy với

Na 2 CO 3 hay K 2 CO 3 tạo thành các muối ferit NaFeO 2 hay KFeO 2 .

- NaFeO 2 hay các ferit khác đều bị thủy phân đến kiềm và Fe 2 O 3

Người ta cũng lợi dụng phản ứng trên để điều chế NaOH trong công nghiệp bằng cách nung

hỗn hợp Na 2 CO 3 và Fe 2 O 3 ở 1100 0 C, sau đó cho NaFeO 2 thủy phân.

Viết các phương trình phản ứng xảy ra.

Hướng dẫn:

FeCl 3 + 3NH 3 + 3H 2 O Fe(OH) 3 ↓ + 3NH 4 Cl

FeCl 3 + 3Na 2 CO 3 + 3H 2 O 2Fe(OH) 3 ↓ + 6NaCl + 3CO 2 ↑

2Fe(OH) 3

0

500 - 700 C

Fe 2 O 3 + 3H 2 O

Fe(OH) 3 + NaOH đặc

0

t

NaFeO 2 + 2H 2 O

62

Phát hành PDF bởi Ths Nguyễn Thanh Tú

Đăng ký Word doc qua Zalo 0905779594 Email thanhtuqn88@gmail.com


0

2Fe(OH) 3 + K 2 CO 3

t 2KFeO 2 + CO 2 ↑ + 3H 2 O

2NaFeO 2 + H 2 O 2NaOH + Fe 2 O 3

Na 2 CO 3 + Fe 2 O 3

0

1100 C

2NaFeO 2 + CO 2 ↑

II.1.9. Các muối Fe (III) được điều chế bằng cách oxi hóa các muối Fe (II) hoặc Fe(OH) 2

bằng axit tương ứng. Ví dụ hòa tan Fe(OH) 2 trong HNO 3

- Các muối Fe (III) đều dễ bị khử đến muối Fe (II) bởi các chất như hiđro mới sinh, SO 2 ,

SnCl 2 , H 2 S, KI và một số kim loại âm điện hơn như Zn... Viết các phương trình phản ứng

minh họa.

Hướng dẫn:

3Fe(OH) 2 + 10HNO 3 3Fe(NO 3 ) 3 + NO + 8H 2 O

Fe 3+ + [H] Fe 2+ + H +

2Fe 3+ + SO 2 + 2H 2 O 2Fe 2+ +

2Fe 3+ + Sn 2+ 2Fe 2+ + Sn 4+

2Fe 3+ + H 2 S 2Fe 2+ + S + 2H +

2Fe 3+ + Zn 2Fe 2+ + Zn 2+

2

SO 4

+ 4H +

2Fe 3+ + 2I - 2Fe 2+ + I 2

II.1.10. Sắt (III) sunfat được điều chế bằng các cách sau đây: hòa tan hiđroxit sắt (III) trong

H 2 SO 4 . Đun nóng FeSO 4 với H 2 SO 4 đặc. Đun nóng FeSO 4 với HNO 3 và H 2 SO 4 đặc. Chế hóa

Fe 2 O 3 với axit sunfuric đặc.

- Khi đun sôi dung dịch loãng, muối bazơ sẽ kết tủa. Khi đun nóng, sẽ mất dần nước kết tinh

tạo ra Fe 2 (SO 4 ) 3 khan và sau đó bị phân hủy.

- Sắt (III) sunfat có khả năng tạo ra muối kép dạng M.Fe(SO 4 ) 2 .12H 2 O được gọi là phèn -

sắt. Quan trọng hơn cả là phèn sắt - amoni NH 4 .Fe(SO 4 ) 2 .12H 2 O và phèn sắt - kali

K.Fe(SO 4 ) 2 .12H 2 O, được dùng làm chất cầm màu vải. Phèn được điều chế bằng cách oxi hóa

dung dịch FeSO 4 .7H 2 O bằng HNO 3 , cô dung dịch đến trạng thái bão hòa thì thêm một lượng

đồng phân tử amoni sunfat hoặc kali sunfat.

Viết các phương trình phản ứng xảy ra.

Hướng dẫn:

2Fe(OH) 3 + 3H 2 SO 4 Fe 2 (SO 4 ) 3 + 6H 2 O

Phát hành PDF bởi Ths Nguyễn Thanh Tú

Đăng ký Word doc qua Zalo 0905779594 Email thanhtuqn88@gmail.com

63


2FeSO 4

2FeSO 4

+ 2H 2 SO 4 đặc Fe 2 (SO 4 ) 3 + SO 2 ↑ + 2H 2 O

+ H 2 SO 4 đặc + 2HNO 3 đặc Fe 2 (SO 4 ) 3 + 2NO 2 ↑ + 2H 2 O

Fe 2 O 3 + 3H 2 SO 4 đặc

Fe 2 (SO 4 ) 3 + 3H 2 O

Fe 2 (SO 4 ) 3 + 2H 2 O 2Fe(OH)SO 4 + H 2 SO 4

0

Fe 2 (SO 4 ) 3

t Fe 2 O 3 + 3SO 3

6(FeSO 4 .7H 2 O) + 3H 2 SO 4 + 2HNO 3 3Fe 2 (SO 4 ) 3 + 2NO↑ + 46H 2 O

(NH 4 ) 2 SO 4 + Fe 2 (SO 4 ) 3 + 24H 2 O (NH 4 ) 2 SO 4 .Fe 2 (SO 4 ) 3 .24H 2 O

2

II.1.11. Hợp chất ferat FeO tạo ra khi oxi hóa kim loại Fe hoặc hợp chất của Fe (III) bằng

4

chất oxi hóa mạnh trong môi trường kiềm mạnh. Ví dụ khi nấu chảy Fe 2 O 3 với KNO 3 và

KOH, hoặc khi nấu chảy Fe 2 O 3 với Na 2 O 2 , hoặc cho clo tác dụng với huyền phù Fe(OH) 3

trong kiềm đặc. Các ferat có màu đỏ sẫm, ít bền trong dung dịch nước, khi đun nóng nhẹ giải

phóng ra oxi. Các ferat đều là chất oxi hóa mạnh, ví dụ oxi hóa amoniac ở điều kiện thường

tạo nitơ tự do. Khi cho ferat tác dụng với BaCl 2 tạo ra kết tủa màu đỏ tím của bari ferat. Viết

các phương trình phản ứng xảy ra.

Hướng dẫn:

Fe 2 O 3 + 3KNO 3 + 4KOH 2K 2 FeO 4 + 3KNO 2 + 2H 2 O

Fe 2 O 3 + 3Na 2 O 2 2Na 2 FeO 4

+ Na 2 O

0

3Cl 2 + 2Fe(OH) 3 + 10NaOH

t 2Na 2 FeO 4 + 6NaCl + 8H 2 O

4Na 2 FeO 4

+ 2H 2 O 4NaFeO 2 + 3O 2 ↑ + 4NaOH

K 2 FeO 4 + BaCl 2 BaFeO 4 ↓ + 2KCl

2K 2 FeO 4 + 2H 3 N KFeO 2 + N 2 ↑ + 2KOH + 2H 2 O

II.1.12. Khi nhỏ từng giọt kali feroxianua (K 4 Fe(CN) 6 ) vào dung dịch FeCl 3 , ta thu được kết

tủa xanh Prussian (một thành phần của mực đen và mực xanh dùng trong công nghiệp in) có

chứa 34,9% Fe theo khối lượng.

a. Cho biết công thức của kết tủa và viết phương trình phản ứng (M Fe = 55,8).

b. Sử dụng lý thuyết trường tinh thể, vẽ giản đồ tách obitan d cho các nguyên tử Fe trong

xanh Prussian.

c. Giải thích nguyên nhân sinh ra màu sắc của xanh Prussian?

Phát hành PDF bởi Ths Nguyễn Thanh Tú

Đăng ký Word doc qua Zalo 0905779594 Email thanhtuqn88@gmail.com

64


Hướng dẫn:

a. Kết tủa là Fe 7 (CN) 18 .14,5H 2 O

4Fe 3+ + 3[Fe(CN) 6 ] 4- Fe 3+ [Fe 3+ Fe 2+ (CN) 6 ] 3

b.

c. Hầu hết các sản phẩm nhuộm vô cơ đều chứa các ion tạo màu bằng cách hấp thụ chọn lọc

bức xạ điện từ do sự chuyển electron. Màu xanh đậm của xanh Prussian có liên quan đến

năng lượng cần thiết để chuyển electron từ Fe (II) đến Fe (III) thông qua nhóm CN - đóng vai

trò cầu nối

II.1.13. Moment từ của Na 2 [Fe(CN) 5 (NO)] bằng 0. Hãy xác định số oxi hóa của nguyên tử

sắt. Na 2 [Fe(CN) 5 (NO)] là tác nhân nhận biết S 2- bởi phản ứng tạo thành dung dịch màu tím.

Viết phương trình phản ứng dạng ion.

Hướng dẫn:

Fe +2

[Fe(CN) 5 (NO)] 2- + S 2- [Fe(CN) 5 (NOS)] 4-

II.1.14. Một chất rắn màu trắng X tham gia một loạt các thí nghiệm trong đó X bị đốt thành

tro dưới tác dụng của các luồng khí vào khác nhau. Kết qủa thí nghiệm được thống kê ở

bảng sau:

Thí nghiệm số Khí vào Sự chênh lệch khối lượng

mẫu so với ban đầu

1 N 2 -37,9

2 NH 3 -51,7

3 O 2 -31,0

4 HCl +9,5

5 HCl + Cl 2 -100,0

Phát hành PDF bởi Ths Nguyễn Thanh Tú

Đăng ký Word doc qua Zalo 0905779594 Email thanhtuqn88@gmail.com

65


Trong tất cả các thí nghiệm thì trong hỗn hợp sau phản ứng ngoài khí ban đầu còn có một

khí chưa biết Y. Ở thí nghiệm số 5 xuất hiện một hợp chất màu đỏ nâu Z ngưng tụ khi tiến

hành bước làm lạnh trong thí nghiệm.

a. Sử dụng các giá trị cho ở bảng trên hãy xác định các chất được ký hiệu bằng chữ cái.

b. Viết các phản ứng xảy ra trong thí nghiệm.

c. Cho biết cấu trúc của Z trong pha khí.

Hướng dẫn:

a. X là FeCO 3

Y là CO 2

Z là FeCl 3

b. Các phản ứng sau đây đã xảy ra

FeCO 3 = FeO + CO 2

3FeCO 3 + 2NH 3 = 3Fe + 3CO 2 + 3H 2 O

4FeCO 3 = 2Fe 2 O 3 + 4CO 2

FeCO 3 + 2HCl = FeCl 2 + CO 2 + H 2 O

2FeCO 3 + 4HCl + Cl 2 = 2FeCl 3 + 2CO 2 + 2H 2 O

c. Ở pha hơi thì sắt (III) clorua tồn tại ở dạng dime (FeCl 3 ) 2

II.2. Bài tập định lượng: xác định công thức, tinh thể, chuẩn độ, phức chất, …

II.2.1. Có thể điều chế tinh thể FeCl 3 .6H 2 O theo cách sau: Hoà tan sắt kim loại vào trong

dung dịch axit clohydric 25%. Dung dịch tạo thành được oxy hóa bằng cách sục khí clo qua

cho đến khi cho kết qủa âm tính với K 3 [Fe(CN) 6 ]. Dung dịch được cô bay hơi ở 95 0 C cho

đến khi tỉ trọng của nó đạt chính xác 1,695 g/cm 3 và sau đó làm lạnh đến 4 0 C. Tách kết tủa

thu được bằng cách hút chân không rồi cho vào một dụng cụ chứa được niêm kín.

a. Viết các phản ứng dẫn đến sự kết tủa FeCl 3 .6H 2 O

b. Có bao nhiêu gam sắt và bao nhiêu ml dung dịch axit clohiđric 36% (d = 1,18 g/cm 3 ) cần

để điều chế 1 kg tinh thể này. Biết rằng hiệu suất quá trình chỉ đạt 65%

Phát hành PDF bởi Ths Nguyễn Thanh Tú

Đăng ký Word doc qua Zalo 0905779594 Email thanhtuqn88@gmail.com

66


c. Đun nóng 2,752 gam FeCl 3 .6H 2 O trong không khí đến 350 0 C thu được 0,8977 gam bã rắn.

Xác định thành phần định tính và định lượng của bã rắn.

Hướng dẫn:

a. Các phản ứng:

b.

Fe + 2HCl = FeCl 2 + H 2

2FeCl 2 + Cl 2 = 2FeCl 3

3FeCl 2 + 2K 3 [Fe(CN) 6 ] = Fe 3 [Fe(CN) 6 ] 2 + 6KCl

FeCl 3 + 6H 2 O = FeCl 3 .6H 2 O

1000 = 3,7mol FeCl3 .6H 2 O

270,3

Như vậy cần

3,7.2.36,5

0,36.1,18.0,65

978ml

dung dịch HCl 36%

Khi đun nóng thì FeCl 3 .6H 2 O phân huỷ theo phương trình sau:

FeCl 3 .6H 2 O = FeOCl + 5H 2 O + 6HCl

Khi nhiệt độ tăng thì FeOCl sẽ tiếp tục phân huỷ:

3FeOCl = FeCl 3 + Fe 2 O 3 (hơi FeCl 3 bay ra)

Lượng FeCl 3 .6H 2 O trong mẫu là

2,752

270,3

= 10,18 mmol

Điều này ứng với khối lượng FeCl 3 là 107,3. 0,01018 = 1,092g FeOCl

Do khối lượng thu được của bã rắn bé hơn nên ta biết được FeOCl sẽ bị phân hủy một

phần thành Fe 2 O 3 . Khối lượng FeCl 3 mất mát do bay hơi là:

1,902 0,8977

162,2

= 1,20mmol

Như vậy bã rắn cuối cùng chứa (0,01018 – 3.0,00120) = 6,58 mmol FeOCl và 1,20 mmol

Fe 2 O 3 .

II.2.2. Hợp chất X là hiđroxit của kim loại M. Khi X được đun nóng (trong điều kiện không có

không khí) thì thu được chất rắn Y và hỗn hợp khí Z (ở 400K, 1 atm). Hợp chất Y chứa 27,6%

oxi về khối lượng. Hỗn hợp khí Z có tỉ khối so với He bằng 3,17.

a. Xác định công thức và tính phần trăm số mol của các khí có trong hỗn hợp Z.

b. Xác định công thức của X và Y.

67

Phát hành PDF bởi Ths Nguyễn Thanh Tú

Đăng ký Word doc qua Zalo 0905779594 Email thanhtuqn88@gmail.com


Hướng dẫn:

Cho: H = 1; O = 16; Cr = 52; Mn = 55; Fe = 56; Cu = 64; Zn = 65; Pb = 207.

a. Hiđroxit của kim loại M bị phân hủy khi đun nóng theo phương trình:

2M(OH) n (r) → M 2 O n (r) + nH 2 O (k)

Nếu oxit thu được không bền thì tiếp theo xảy ra 1 trong 2 khả năng sau:

- Khả năng 1: Oxit bị phân hủy tạo ra oxi và sản phẩm mà kim loại có số oxi hóa thấp hơn

M 2 O n → M 2 O m + (n-m)/2 O 2

- Khả năng 2: Oxit phản ứng với hơi nước làm tăng số oxi hóa của kim loại:

M 2 O n + (p-n) H 2 O → M 2 O p + (p-n) H 2

Ở 400K và 1 atm, nước ở trạng thái hơi do đó hỗn hợp Z có thể gồm O 2 và H 2 O hoặc gồm

H 2 và H 2 O.

Theo giả thiết: Khối lượng mol trung bình của hỗn hợp Z = 3,17.4 = 12,68 gam/mol →

hỗn hợp Z gồm H 2 và H 2 O. Gọi % số mol của H 2 trong Z là a, có phương trình:

2a + 18(100-a) = 12,68.100 → a = 33,33 → tỉ lệ mol của H 2 và H 2 O tương ứng là 1 : 2.

b. Đặt công thức của Y là M x O y , theo giả thiết có:

16y/Mx = 27,6/(100-27,6) → M = 20,985.(2y/x) với (2y/x) = 1, 2, 8/3, 3,...

2y/x 1 2 8/3 3 4 ...

M 20,985 41,971 55,96 62,956 83,942 ...

→ M = 55,96 56 là Fe và công thức của oxit là Fe 3 O 4 .

Công thức của X phải là Fe(OH) 2 .

Phản ứng: 3Fe(OH) 2

o

t

Fe 3 O 4 + 2H 2 O + H 2

II.2.3. Từ nhiệt độ phòng đến 1185K sắt tồn tại ở dạng Fe– với cấu trúc lập phương tâm

khối, từ 1185K đến 1667K ở dạng Fe– với cấu trúc lập phương tâm diện. Ở 293K sắt có

khối lượng riêng D = 7,874 g/cm 3 .

a. Hãy tính bán kính của nguyên tử Fe.

b. Tính khối lượng riêng của sắt ở 1250K (bỏ qua ảnh hưởng không đáng kể do sự dãn nở

nhiệt)

Thép là hợp kim của sắt và cacbon, trong đó một số khoảng trống giữa các nguyên tử sắt bị

chiếm bởi nguyên tử cacbon. Trong lò luyện thép (lò thổi) sắt dễ nóng chảy khi chứa 4,3%

Phát hành PDF bởi Ths Nguyễn Thanh Tú

Đăng ký Word doc qua Zalo 0905779594 Email thanhtuqn88@gmail.com

68


cacbon về khối lượng. Nếu được làm lạnh nhanh thì các nguyên tử cacbon vẫn được phân

tán trong mạng lưới lập phương tâm khối, hợp kim được gọi là martensite cứng và dòn. Kích

thước của tế bào sơ đẳng của Fe– không đổi.

c. Hãy tính số nguyên tử trung bình của C trong mỗi tế bào sơ đẳng của Fe– với hàm lượng

của C là 4,3%.

d. Hãy tính khối lượng riêng của martensite.

(Cho Fe = 55,847; C = 12,011; số N A = 6,022.10 23 )

Hướng dẫn:

a. Khối lượng mol nguyên tử Fe = 55,847 g/mol và khối lượng riêng D = 7,874 g/ cm 3 (ở

293K)

Vậy 1 mol Fe có thể tích là: V = m 55,847 = 7,093 g/cm 3 .

D 7,874

Mỗi tế bào lập phương có 2 nguyên tử Fe nên thể tích tế bào sơ đẳng là:

7,093.2

V 1 = = 2,356.10 –23 cm 3 .

23

6,022.10

Cạnh a của tế bào lập phương tâm khối:

a 3 = V 1 a =

3 23

2,356.10 = 2,867.10 –8 cm.

Ta đã biết với cấu trúc lập phương tâm khối: đường chéo của lập phương AC = a 3 = 4r

Vậy bán kính nguyên tử r của Fe: r =

8

a 3 2,867.10 . 3

= 1,241.10 –8 cm.

4 4

b. Ở 1250K sắt ở dạng Fe– với cấu trúc lập phương tâm diện.

– Khi đó đường chéo của một mặt là:

8

4r 4.1,241.10

a ' 2 4r a ' = 3,511.10 –8 cm.

2 2

– Thể tích tế bào sơ đẳng: V’= a’ 3 = (3,511.10 –8 ) 3 = 4,327.10 –23 cm 3 .

– Với cấu trúc lập phương tâm mặt mỗi tế bào có 4 nguyên tử Fe, do đó khối lượng riêng:

m 4.55,87

D' = 8,572 g/cm 3 .

23 23

V' 4,327.10 .6,022.10

69

Phát hành PDF bởi Ths Nguyễn Thanh Tú

Đăng ký Word doc qua Zalo 0905779594 Email thanhtuqn88@gmail.com


c. Trong 100 gam martensite có: 4,3 (g) C (0,36 mol) và 95,7 (g) Fe (1,71 mol). Nghĩa là

ứng với 1 nguyên tử Fe có 0,36/1,71 = 0,21 nguyên tử C.

– Mỗi tế bào sơ đẳng Fe– có 2 nguyên tử Fe tức là có trung bình 0,21.2 = 0,42 nguyên tử

C.

d. – Vì nguyên tử không chia sẻ được nên một cách hợp lý hơn ta nói cứ 12 tế bào sơ đẳng

có: (0,42.12) = 5 nguyên tử C

– Khối lượng mỗi tế bào sơ đẳng = tổng khối lượng của 2 nguyên tử Fe và 0,42 nguyên tử C.

55,487.2 12,022.0,42

– Vậy, m = = 1,938.10 –22 g

23 23

6,022.10 6,022.10

– Tỷ khối của martensite: D =

22

m 1,938.10

= 8,228 g/cm 3 .

23

V 2,356.10

II.2.4. HSGQG 2016

Nồng độ đường trong máu có thể được xác định bằng phương pháp Hagedorn – Jensen.

Phương pháp này dựa vào phản ứng của Na 3

[Fe(CN) 6 ] oxi hóa glucozơ thành axit gluconic.

Qui trình phân tích như sau: lấy 0,20 ml mẫu máu cho vào bình tam giác, thêm 5,00 ml dung

dịch Na 3

[Fe(CN) 6 ] (natri hexaxianoferat (III)) 4,012 mmol/l rồi đung cách thủy, thu được

dung dịch A. Thêm lần lượt các dung dịch KI dư, ZnCl 2 dư và CH 3 COOH vào dung dịch A.

Sau khi các phản ứng xảy ra hoàn toàn, lượng I 2 sinh ra được chuẩn độ bằng 4,00 mmol/l.

Giả thiết rằng các thành phần khác trong máu không ảnh hưởng đến kết quả thí nghiệm.

dung dịch Na 2 S 2 O 3

a. Viết các phương trình phản ứng xảy ra theo quy trình trên.

b. Tại sao không thể dùng các muối sắt (III) khác như FeCl 3 , Fe(NO 3 ) 3 , … để thay thế cho

muối phức Na 3

[Fe(CN) 6 ] trong thí nghiệm trên. Cho biết pH của máu là 7,4.

c. Tính hằng số cân bằng của phản ứng: 2[Fe(CN) 6 ] 3- + 3I - 2[Fe(CN) 6 ] 4- +

cho biết vai trò của ZnCl 2 trong quy trình trên.

-

I

3

, từ đó

d. Tính nồng độ (mg/ml) của glucozơ có trong mẫu máu, biết rằng phép chuẩn độ cần 3,28

ml dung dịch Na 2 S 2 O 3 để đạt tới điểm tương đương

Cho biết:

0

0

E = 0,5355 V; E = 0,771 V; pK s Fe(OH) 3 = 37, Fe(OH) 2 = 15,1

3+ 2+

3

Fe /Fe

I /I

Các phức [Fe(CN) 6 ] 3- và [Fe(CN) 6 ] 4- có hằng số bền tổng cộng lần lượt là 10 42 và 10 35 .

70

Phát hành PDF bởi Ths Nguyễn Thanh Tú

Đăng ký Word doc qua Zalo 0905779594 Email thanhtuqn88@gmail.com


Hướng dẫn:

a. Các phản ứng xảy ra:

C 6 H 12 O 6 + 2[Fe(CN) 6 ] 3- + 3OH - C 5 H 11 O 5 COO - + 2[Fe(CN) 6 ] 4- +2H 2 O (1)

2[Fe(CN) 6 ] 3- + 3I - 2[Fe(CN) 6 ] 4- +

I + 2S O 3I - +

-

3

2-

2 3

-

I

3

(2)

2-

S4O 6

(3)

b. Do pH của máu là 7,4 nên Fe 3+ sẽ kết tủa ở dạng Fe(OH) 3 và không có khả năng oxi hóa

glucozơ.

c. Tính hằng số cân bằng của phản ứng (2):

Fe 3+ + 6CN - [Fe(CN) 6 ] 3- K 2 =

Fe 2+ + 6CN - [Fe(CN) 6 ] 4- K 2 =

3-

Fe CN

6

3+ - 6

[ Fe ][C N ]

4-

Fe CN

6

2+ - 6

[ Fe ][C N ]

3+

E = E

0

+ 0,0592lg

Fe

3+ 2+

Fe /Fe Fe

3+ /Fe

2+

[ Fe

2+ ]

0

= E

3+ 2+

Fe /Fe

0

=

3+ 2+

Fe /Fe

0'

= E

3+ 2+

Fe /Fe

3-

- 6

Fe CN [CN ] K

6

+ 0,0592lg

4-

- 6

Fe CN [CN ] K

6

E + 0,0592lg K

3

Fe CN

+ 0,0592lg

FeCN

K + 0,0592lg Fe CN

2

FeCN

3-

6

4-

6

3

2

3-

6

4-

6

35

10

E = 0,771 + 0,0592lg

42

10

0'

Với:

3+ 2+

Fe /Fe

= 0,357V.

Đối với phản ứng (2): n = 2, K =

0

n E /0,0592

10 =

10

2( 0,3570,5355)

/0,0592

= 10 -6,03 = 9,33.10 -7 .

Vai trò của ZnCl 2 : Zn 2+ tạo kết tủa với [Fe(CN) 6 ] 4- làm cho phản ứng (2) xảy ra hoàn toàn

theo chiều thuận.

2K + + Zn 2+ + [Fe(CN) 6 ] 4- K 2 Zn[Fe(CN) 6 ]↓

d. Tính nồng độ của glucozơ:

Phát hành PDF bởi Ths Nguyễn Thanh Tú

Đăng ký Word doc qua Zalo 0905779594 Email thanhtuqn88@gmail.com

71


n

Ta có: n 2- = 3,28.10 -3 .4,00 = 13,13.10 -3 mmol

3-

[Fe(CN) 6 ]

S2O3

= 5,00. 10 -3 .4,012 = 20,06.10 -3 mmol

(3) n = ½. - n = ½.13,13.10 -3 = 6,56.10 -3 mmol

I

2-

3 S2O3

(1) n glucozơ = ½.(20,06.10 -3 - 13,13.10 -3 ) = 3,47.10 -3 mmol

C glucozơ = (3,47.10 -3 mmol .180(mg/mmol)/0,2 = 3,123 mg/ml.

III. Bài tập về Co

III.1. Bài tập lý thuyết, viết phương trình phản ứng, giải thích hiện tượng, điều chế,

chuỗi phản ứng, xác định công thức, …

III.1.1. Coban (II) oxit CoO là oxit bazơ, là chất rắn màu xanh, tạo ra khi nung Co(OH) 2 ,

CoCO 3 hay Co(NO 3 ) 2 trong bầu khí trơ hay trong chân không, nếu nung trong không khí sẽ

chuyển thành Co 3 O 4 . Khi nung rất mạnh, tất cả các oxit của coban đều chuyển thành CoO,

và khi nung trong luồng khí H 2 tất cả các oxit đó đều bị khử thành kim loại. Viết các phương

trình phản ứng minh họa.

Hướng dẫn:

t

Co(OH) 0

2 CoO + H 2 O

t

CoCO 0

3 CoO + CO 2 ↑

t

2Co(NO 3 ) 0

2 2CoO + 4NO 2 ↑ + O 2 ↑

t

3Co(OH) 2 + 1/2O 0

2 Co 3 O 4 + 3H 2 O

t

3Co(NO 3 ) 0

2 Co 3 O 4 + 6NO 2 ↑ + O 2 ↑

t

2Co 2 O 0

3 4CoO + O 2 ↑

CoO + H 2 ↑

0

t

Co + H 2 O

III.1.2. Co(OH) 2 kết tủa màu xanh bị oxi hóa chậm trong không khí chuyển thành màu

hung. Co(OH) 2 có tính lưỡng tính, nhưng nặng về tính bazơ, dễ tan trong axit, tan trong

dung dịch kiềm đặc nóng tạo thành cobantit. Khi cho Sr(OH) 2 hay Ba(OH) 2 tác dụng lên

Co(OH) 2 các hợp chất Sr 2 [Co(OH) 6 ] và Ba 2 [Co(OH) 6 ] cũng được hình thành.

Viết phương trình phản ứng.

Hướng dẫn:

4Co(OH) 2 + O 2 + 2H 2 O 4Co(OH) 3

72

Phát hành PDF bởi Ths Nguyễn Thanh Tú

Đăng ký Word doc qua Zalo 0905779594 Email thanhtuqn88@gmail.com


Co(OH) 2 + 2NaOH đặc

0

t

Na 2 [Co(OH) 4 ]

Co(OH) 2

+ 2Sr(OH) 2 đặc

0

t

Sr 2 [Co(OH) 6 ]

Co(OH) 2

+ 2Ba(OH) 2 đặc

0

t

Ba 2 [Co(OH) 6 ]

III.1.3. Coban (II) sunfat, Niken (II) sunfat được điều chế bằng cách cho kim loại, oxit,

hiđroxit tan trong H 2 SO 4 . Viết các phương trình phản ứng xảy ra.

- CoSO 4 kết tinh từ dung dịch ở dạng CoSO 4 .7H 2 O hay [Co(OH 2 ) 6 ]SO 4 .H 2 O

màu đỏ son. Để trong không khí không bị biến đổi, nhưng khi nung mạnh thì ban đầu mất

nước kết tinh trở thành dạng khan CoSO 4 màu đỏ và sau đó bị phân hủy thành Co 3 O 4 .

- Niken (II) sunfat tác dụng với khí NH 3 tạo thành amoniacat.

Viết các phương trình phản ứng xảy ra.

Hướng dẫn:

CoO + H 2 SO 4

Ni(OH) 2 + H 2 SO 4

CoSO 4 + H 2 O

NiSO 4 + 2H 2 O

t

3CoSO 0

4 Co 3 O 4 + SO 2 + 2SO 3

NiSO 4 + 6NH 3 [Ni(NH 3 ) 6 ] SO 4

III.1.4. Coban (II) nitrat được điều chế bằng cách hòa tan coban, oxit Co (II), Co(OH) 2 .

Coban (II) nitrat kết tinh ở trạng tinh thể hexahiđrat Co(NO 3 ) 2 .6H 2 O màu đỏ thẫm. Chảy rữa

ngoài không khí ẩm. Khi đun nóng sẽ nóng chảy ở gần 55 0 C rồi mất nước kết tinh, sau đó bị

phân hủy. Viết các phương trình phản ứng xảy ra.

Hướng dẫn:

Co(OH) 2 + 2HNO 3 → Co(NO 3 ) 2 + 2H 2 O

t

3Co(NO 3 ) 0

2 Co 3 O 4 + 6NO 2 ↑ + O 2 ↑

III.1.5. Coban (III) oxit được điều chế bằng cách nhiệt phân Co(NO 3 ) 2 . Coban (III) oxit

cũng được điều chế dạng tinh khiết bằng cách nung Co(OH) 2 trong lò điện ở 350 - 370 0 C.

Viết các phương trình phản ứng xảy ra.

Hướng dẫn:

t

4Co(NO 3 ) 0

2 2Co 2 O 3 + 8NO 2 ↑ + O 2 ↑

t

4Co(OH) 2 + O 0

2 2Co 2 O 3 + 4H 2 O

Để có sản phẩm thật tinh khiết người ta đã dùng phản ứng sau:

73

Phát hành PDF bởi Ths Nguyễn Thanh Tú

Đăng ký Word doc qua Zalo 0905779594 Email thanhtuqn88@gmail.com


4CoCl 2 + 4NH 4 Cl + 16NH 3 + O 2 4[Co(NH 3 ) 5 Cl]Cl 2 + 2H 2 O

2[Co(NH 3 ) 5 Cl]Cl 2 + 3H 2 O Co 2 O 3 + 4NH 3 ↑ + 6NH 4 Cl

III.1.6. Co 2 O 3 là chất bột màu nâu sẫm, nung đến gần 600 0 C tạo thành Co 3 O 4 là chất bột

màu đen. Nung ở 1300 0 C phân hủy tiếp tạo ra CoO. Co 2 O 3 cũng bị hiđro khử đến kim loại,

tan trong axit HCl tạo ra Cl 2 và với H 2 SO 4 tạo ra O 2 . Viết các phương trình phản ứng xảy ra.

Hướng dẫn:

t

6Co 2 O 0

3 4Co 3 O 4 + O 2 ↑

2Co 2 O 3

0

1300 C

4CO + O 2 ↑

t

Co 2 O 3 + 3H 0

2 2Co + 3H 2 O

Co 2 O 3 + 6HCl 2CoCl 2 + Cl 2 ↑ + 3H 2 O

2Co 2 O 3 + 4H 2 SO 4 4CoSO 4 + O 2 ↑ + 4H 2 O

III.1.7. Coban (II, III) oxit (Co 3 O 4 ) được tạo ra khi nung coban (II) nitrat. Cũng được tạo ra

khi nung CoO trong không khí

Co 3 O 4 là chất bột màu đen, bị H 2 khử thành kim loại khi nung nóng. Co 3 O 4 tác dụng với

axit HCl tạo ra muối Co (II) và clo. Viết các phương trình phản ứng xảy ra.

(Khác với Fe 3 O 4 , trong mạng tinh thể có ion kim loại hóa trị II và hóa trị III, trong Co 3 O 4

có ion kim loại hóa trị II và hóa trị IV, và được xem là một muối của Co 2+ tương tự Mn 3 O 4 :

Co 2 II [Co IV O 4 ]).

Hướng dẫn:

t

3Co(NO 3 ) 0

2 Co 3 O 4 + 6NO 2 ↑ + O 2 ↑

6CoO + O 2

0

t

2Co 3 O 4

t

Co 3 O 4 + 4H 0

2 3Co + 4H 2 O

Co 3 O 4 + 8HCl

3CoCl 2 + Cl 2 ↑ + 4H 2 O

III.1.8. Coban (III) hiđroxit là chất bột màu nâu đen, tạo ra khi oxi hóa Co(OH) 2 bằng oxi

của không khí tương tự Fe(OH) 2 , hoặc oxi hóa nhanh các muối Co (II) khi cho tác dụng với

các chất như NaOCl, Cl 2 , Br 2 , H 2 O. Co(OH) 3 là chất không tan trong nước (Tt : 4.10 -45 ), tan

trong axit tạo thành muối Co (III), trong kiềm đặc dư tạo ra muối hiđroxo, ví dụ:

K 3 [Co(OH) 6 ].

Phát hành PDF bởi Ths Nguyễn Thanh Tú

Đăng ký Word doc qua Zalo 0905779594 Email thanhtuqn88@gmail.com

74


Viết các phương trình phản ứng xảy ra.

Hướng dẫn:

4Co(OH) 2 + O 2 + 2H 2 O 4Co(OH) 3

2CoCl 2 + NaClO + 4NaOH + H 2 O 2Co(OH) 3 ↓ + 5NaCl

2CoCl 2 + H 2 O 2 + 4NaOH + H 2 O 2Co(OH) 3 ↓ + 4NaCl

Co(OH) 3 + 3HCl CoCl 3 + 3H 2 O

Co(OH) 3 + 3KOH K 3 [Co(OH) 6 ]

III.2. Bài tập về phức chất

III.2.1. HSGQG - 2014

NH 3 có khả năng phản ứng với nhiều ion kim loại chuyển tiếp. Alfred Werner (được giải

Nobel hóa học năm 1913) đã phân lập thành công một số phức chất giữa CoCl 3 và NH 3 ,

trong đó có phức chất bát diện với công thức phân tử là CoCl 3 .4NH 3 . Tùy thuộc vào điều

kiện tổng hợp, phức chất này có màu tím hoặc màu xanh. Khi cho lượng dư dung dịch

AgNO 3 tác dụng với dung dịch chứa 1 mol phức chất này đều thu được 1 mol AgCl kết tủa.

Hãy xác định các công thức có thể có của phức chất nêu trên.

Hướng dẫn:

Vì 1 mol phức CoCl 3 .4NH 3 tác dụng dung dịch AgNO 3 (dư) tạo 1 mol AgCl → chỉ có 1

Cl - ở cầu ngoại trong phân tử phức: [Co(NH 3 ) 4 Cl 2 ]Cl.

Do phức [Co(NH 3 ) 4 Cl 2 ] + có cấu trúc bát diện nên có 2 đồng phân:

NH 3

Cl

Cl

Cl

Co

NH 3

NH 3

H 3 N

H 3 N

Co

NH 3

NH 3

NH 3

Cl

Dạng cis

Dạng trans

Hai đồng phân này có màu sắc khác nhau (xanh và tím).

III.2.1. Xét phức chất sau: [Co(CN] 6 ] 3-

a. Gọi tên phức.

b. Vẽ giản đồ sự tách mức năng lượng trong phân tử phức.

c. Tính momen từ của phức ở đơn vị manheton Bohr.

75

Phát hành PDF bởi Ths Nguyễn Thanh Tú

Đăng ký Word doc qua Zalo 0905779594 Email thanhtuqn88@gmail.com


d. Tính năng lượng bền hóa trường tinh thể E bh

Biết thông số tách năng lượng phức bát diện: o = 401 kJ/mol; năng lượng ghép đôi: P =

251 kJ/mol).

(Năng lượng bền hóa phức chất phụ thuộc tổng năng lượng P + và được cho bởi công

thức: Ebh nt . E . .

2 t

n

2 e

Eg

x P với x là số cặp elecctron ghép đôi mới)

g g g

e. Khi thay thế ba phối tử bằng ba ion clo thì phức mới có công thức [CoCl 3 (CN) 3 ] 3- . Vẽ tất

cả các đồng phân không đối quang có thể có của phức này

Hướng dẫn:

a. [Co(CN] 6 ] 3- hexaxianocobantat(III)

b.

c. spin thấp; µ = 0

d. E bh = -6.2/5 o + 2 P = -460 kJ/mol

e.

III.2.2. Thêm dần dung dịch KCN vào dung dịch CoSO 4 lúc đầu thu được kết tủa mầu đỏ

nâu X, sau đó kết tủa này tan ra tạo thành dung dịch màu vàng, sau đó đỏ dần và cuối cùng

tạo ra tinh thể màu tím Y. Hợp chất này bền trong điều kiện không có không khí, dễ tan

trong nước cho dung dịch màu đỏ, khi đun nóng (không có không khí) dung dịch bị nước oxi

hóa. Nếu cho Y tác dụng với axit có mặt chất oxi hóa thu được chất màu vàng Z. Hãy viết

các phương trình phản ứng xảy ra trong thí nghiệm này.

b. Cho biết Z nghịch từ, dựa theo thuyết liên kết hóa trị (VB), hãy dự đoán cấu trúc phân tử

của Z ?

Phát hành PDF bởi Ths Nguyễn Thanh Tú

Đăng ký Word doc qua Zalo 0905779594 Email thanhtuqn88@gmail.com

76


c. Xét các phức sau: [Co(CN) 6 ] 3- , [Co(CO 3 ) 2 (NH 3 ) 2 ] - , [Co(CO 3 ) 3 ] 3- và [Co(NO 2 ) 6 ] 3- . Màu của

các ion phức này là xanh, vàng, cam và da trời. Hãy cho biết tên của từng phức và xác định

màu của chúng?

Hướng dẫn:

a. Các phản ứng xảy ra:

CoSO 4 + 2KCN Co(CN) 2 ↓ (X, nâu đỏ) + K 2 SO 4 .

Co(CN) 2 + 4KCN K 4 [Co(CN) 6 ] ↓ (Y, màu tím)

2K 4 [Co(CN) 6 ] + 2H 2 O 2K 3 [Co(CN) 6 ] +2KOH + H 2 ↑

4K 4 [Co(CN) 6 ] + 4HCl + O 2 4K 3 [Co(CN) 6 ] (Z, màu vàng) + 4KCl + 2H 2 O

b. K 3 [Co(CN) 6 ] (Z) chứa Co 3+ cấu hình d 6 , ion phức chất [Co(CN) 6 ] 3– nghịch từ do vậy sẽ lai

hóa trong, 4 e độc thân sẽ ghép đôi. Với số phối trí 6 sẽ phù hợp với dạng d 2 sp 3 , cấu trúc bát

diện. Học sinh có thể suy luận do CN - là phối tử trường mạnh.

c.

3d 6 4s 4p 6 cặp e nhận từ 6 CN -

Công thức Tên gọi Màu sắc

[Co(CN) 6 ] 3- Hexaxianocobantat (III) Vàng

[Co(NO 2 ) 6 ] 3- Hexa-N-nitritocobantat (III) Cam

[Co(CO 3 ) 3 ] 3- Tricacbonatocobantat (III) Xanh

[Co(CO 3 ) 2 (NH 3 ) 2 ] - Dicacbonatodiamincobantat (III) Xanh da trời

III.2.3. Coban tạo ra được các ion phức: CoCl 2 (NH 3 ) 4 + (X), Co(CN) 6 3- (Y).

a. Viết tên của X, Y.

b. Theo thuyết liên kết hóa trị, các nguyên tử trong Y ở trạng thái lai hóa nào?

c. Các ion phức trên có thể có bao nhiêu đồng phân lập thể? Vẽ cấu trúc của chúng.

d. Viết phương trình phản ứng của (X) với ion Fe 2+ trong môi trường axit.

Hướng dẫn:

a. Tên của các ion phức:

(X) Điclorotetraammincoban (III);

(Y) Hexaxianocobantat (III).

Phát hành PDF bởi Ths Nguyễn Thanh Tú

Đăng ký Word doc qua Zalo 0905779594 Email thanhtuqn88@gmail.com

77


b. Co(CN) 6 3- . Co: d 2 sp 3 ; C: sp; N: không ở vào trạng thái lai hóa hoặc ở trạng thái lai hóa

sp.

c. - Ion phức (X) có 2 đồng phân:

H 3 N

H 3 N

Cl

Co

Cl

NH 3

NH 3

H 3 N

H 3 N

Cl

Co

NH 3

Cl

NH 3

- Ion phức (Y) không có đồng phân:

NC

NC

CN

Co

CN

CN

CN

d. CoCl 2 (NH 3 ) 4 + + Fe 2+ + 4H + Co 2+ + Fe 3+ + 2Cl - + 4 NH

4

III.2.4. Phức chất rắn A màu vàng cam có tên là hexa amincoban (III) clorua, được tổng hợp

bằng cách trộn coban điclorua, amoniac đặc, amoni clorua và hiđro peoxit và sử dụng

cacbon hoạt tính làm xúc tác. Các nghiên cứu về cơ chế phản ứng cho thấy rằng ban đầu

phản ứng tạo thành CoNH

3

2+

6

, sau đó xảy ra phản ứng thế phối tử, tạo thành ion phức hai

tâm B 4+ với một phối tử mới dưới dạng liên kết cầu. Tiếp đó là sự phá vỡ liên kết cầu, đồng

thời 2 electron được chuyển sang phối tử, tạo ra 2 ion C 2+ . Cuối cùng, C 2+ tham gia vào phản

ứng thế phối tử trên bề mặt carbon hoạt hóa và kết hợp với các chloride ion tạo thành phức

chất rắn A. Viết phương trình phản ứng tổng và vẽ cấu trúc của B 4+ , C 2+ .

Hướng dẫn:

2CoCl 2 + 10NH 3 + 2NH 4 Cl + H 2 O 2 → 2Co(NH 3 ) 6 Cl 3 + 2H 2 O

Phát hành PDF bởi Ths Nguyễn Thanh Tú

Đăng ký Word doc qua Zalo 0905779594 Email thanhtuqn88@gmail.com

78


III.2.5. Dù các hợp chất của kim loại X đã được biết đến từ thời cổ đại nhưng nguyên nhân

của những vụ nhiễm độc thường xuyên của giới thợ mở trong quá trình khai tuyển quặng

chứa X vẫn là bí ẩn suốt một thời gian dài. Chính vì điều này mà người Nauy cổ cho rằng

nguyên nhân của những vụ nhiễm độc này là do linh hồn ma quỷ, và từ đó những người thợ

mỏ đã gọi quặng của kim loại X là “linh hồn”. Sau này, nguyên nhân được xác minh là bởi

asen cũng có trong quặng. Kim loại X đã được phân lập ở dạng tinh khiết vào năm 1735 bởi

nhà hóa học Thụy Điển G. Brandt.

Muối A được tạo thành trong phản ứng của kim loại X với nitric axit đặc. Khi đun nóng

27,44 gam muối khan A thì nó bị phân hủy tạo thành 11,24 gam chất B. Phản ứng của B với

oxi khi đun nóng tạo thành chất C (w(X) = 73,42 %). Phản ứng của chất A với natri hiđroxit

khi có mặt H 2 O 2 tạo thành hợp chất D (w(X) = 53,6 %, không có natri). Phản ứng của chất D

với sulfuric acid tạo thành hợp chất E (phản ứng này đi kèm với sự giải phóng khí không

màu). Khi thêm kali sunfat vào dung dịch E, sau đó làm bay hơi dung dịch tạo thành thì thu

được tinh thể của muối kép F (chứa 13,5 % X và 24,7 % H 2 O). Phản ứng của E với barium

chloride tạo thành muối H, chất này cũng có thể được tạo thành bởi phản ứng của C với

hydrochloric acid (phản ứng đi kèm với sự giải phóng chất khí). Phản ứng của H với natri

nitrit khi có axit axetic có thể tạo thành muối phức I (chứa 14,59% X, 20,81% N, 47,53% O)

có màu vàng tươi, được dùng trong hóa phân tích. Phản ứng này đi kèm với sự giải phóng

một chất khí không màu, có thể hóa nâu trong không khí.

a. Xác định các chất được kí hiệu ở trên và viết các phương trình phản ứng.

b. Trong phức I, nguyên tử trung tâm không chứa cặp electron chưa liên kết nào. Xác định

kiểu lai hóa của nguyên tử này và dạng hình học của anion muối I.

Phát hành PDF bởi Ths Nguyễn Thanh Tú

Đăng ký Word doc qua Zalo 0905779594 Email thanhtuqn88@gmail.com

79


c. Có thể dùng muối I để xác định những ion nào? Viết ít nhất một phương trình phản ứng

minh họa.

Hướng dẫn:

a. X là coban (Co).

Co + 4HNO 3 → Co(NO 3 ) 2 + 2NO 2 + 2H 2 O,

2Co(NO 3 ) 2 → 2CoO + 4NO 2 + O 2 ,

6CoO + O 2 → 2Co 3 O 4 ,

2Co(NO 3 ) 2 + 4NaOH + H 2 O 2 → 2Co(OH) 3 + 4NaNO 3

4Co(OH) 3 + 4H 2 SO 4 → 4CoSO 4 + O 2 + 10H 2 O,

CoSO 4 + K 2 SO 4 + 6H 2 O → K 2 Co(SO 4 ) 2·6H 2 O,

2CoSO 4 + H 2 SO 4 + O 3 → Co 2 (SO 4 ) 3 + O 2 + H 2 O,

CoSO 4 + BaCl 2 → CoCl 2 + BaSO 4 ↓,

Co 3 O 4 + 8HCl → 3CoCl 2 + Cl 2 ↑ + 4H 2 O,

CoCl 2 + 7NaNO 2 + 2CH 3 COOH → Na 3 [Co(NO 2 ) 6 ] + NO↑ + 2NaCl + 2CH 3 COONa + H 2 O.

А - Co(NO 3 ) 2 , В - CoO, С - Co 3 O 4 , D - Co(OH) 3 , E - CoSO 4 , F - K 2 Co(SO 4 ) 2·6H 2 O, G -

Co 2 (SO 4 ) 3 , H - CoCl 2 , I - Na 3 [Co(NO 2 ) 6 ].

b. Lai hóa d 2 sp 3 .

c. Ion kali. Ví dụ: Na 3 [Co(NO 2 ) 6 ] + 2KCl → K 2 Na[Co(NO 2 ) 6 ] + 2NaCl.

III.2.6. Năm 1912, Alfred Werner đã tổng hợp một số hợp chất phức đồng phân của coban,

được sử dụng để làm bằng chứng cho sự phù hợp về lí thuyết cấu trúc của các hợp chất phức

do chính ông đề xuất (Werner A., Ber., 45, 121 (1912)).

Một dung dịch chứa 10 gam CoCl 2 .6H 2 O trong 150 gam etylenđiamin (H 2 N-CH 2 -CH 2 -

NH 2 , en) 10% được để trong không khí trong nhiều giờ. Axit hóa dung dịch bởi axit

clohiđric rồi làm bay hơi cho đến khi bắt đầu xảy ra sự kết tinh, sau đó thêm amoni nitrat

vào dịch cái, thì có một lượng nhỏ tinh thể dạng phiến màu xanh lục (chất I) được tạo thành.

Lọc kết tủa, rồi thêm NaBr vào dịch lọc, thì một lượng lớn các tinh thể hình kim màu vàng

cam (chất III) được tạo thành. Dung dịch nước chứa 100 gam chất III được xử lí với một

lượng bạc tactrat (axit tactric: HOOC-CH (OH)-CH(OH)-COOH) vừa đủ (68,3 gam bạc

tactrat), thu được 2 đương lượng kết tủa của các ion halogenua. Bạc halogenua kết tủa được

lọc tách rồi làm bay hơi dung dịch. d-tactrat kết tinh (chất IV) sau khi làm bay hơi được lọc

Phát hành PDF bởi Ths Nguyễn Thanh Tú

Đăng ký Word doc qua Zalo 0905779594 Email thanhtuqn88@gmail.com

80


tách, và dịch lọc được làm nguội rồi chuyển thành một khối gelatin chứa l-tactrat (chất V).

Các tinh thể IV và V được nghiền riêng trong các cối giã với HBr đặc, hơi ấm. Lọc tách kết

tủa trong cả hai trường hợp, rồi tái kết tinh trong nước ấm, lần lượt thu được các chất VI và

VII. Tính chất của các chất được cho trong bảng sau:

STT

Hàm lượng, %

Co N Br (Cl) màu sắc

độ dẫn điện

phân tử

[]D (1%)

I 18,89 22,44 (22,72) xanh lục 105 - -

II 18,89 22,44 (22,72) tím 107 - -

III 11,06 15,77 44,97 vàng 415 - -

[M]D

IV 10,58 15,08 14,34 vàng 265 +98° +555°

V 10,58 15,08 14,34 vàng 273 - -

VI 11,44 16,32 46,54 vàng 420 +117° +602°

VII 11,44 16,32 46,54 vàng 418 -115° -592°

a. Xác định công thức các chất I - VII.

b. Viết phương trình điều chế I, III - VII.

c. Cho biết I và II, VI và VII thuộc loại đồng phân nào?

d. Với các hợp chất này, xác định: số oxi hóa của nguyên tử trung tâm; cấu hình electron của

coban ion; số phối trí của nguyên tử trung tâm; đa diện phối trí (hình đa diện tạo thành bởi

các nguyên tử phối trí). Vẽ sự sắp xếp (cấu trúc) các đồng phân với các nguyên tử phối trí

nằm ở đỉnh đa diện.

Hướng dẫn:

a. Theo dữ kiện trong bảng, thành phần chất I là:

Co : N : Cl = (18,89: 58,933): (22,44: 14,007): (22,72: 35,453) = 0,3205: 1,6021: 0,6408

=1,0: 4,99:1,99 = 1: 5: 2

Khối lượng mol M(I) = M(Co) : ω(NO) = 312 gam/mol. Công thức chất I gồm 1 Co 3+ , 2

phân tử etylenđiamin (tương ứng với 4 nguyên tử nitơ), 1 NO , 2 Cl - . Dựa vào dữ liệu về độ

dẫn điện, xác định được công thức chất I là [Co(en) 2 Cl 2 ](NO 3 ) (xanh lục). Công thức chất II

cũng tương tự: [Co(en) 2 Cl 2 ](NO 3 ) (tím).

-

3

Phát hành PDF bởi Ths Nguyễn Thanh Tú

Đăng ký Word doc qua Zalo 0905779594 Email thanhtuqn88@gmail.com

81


Công thức chất III: [Co(en) 3 ]Br 3 .3H 2 O

Công thức chất III phù hợp với dữ kiện về độ dẫn điện phân tử. Độ dẫn điện tăng (nhưng

không tuyến tính) theo số ion tạo thành trong quá trình phân li phức chất. Đây từng là

phương pháp chính để xác định công thức cầu phối trí ngoài cùng của Werner.

Từ 1 mol I có 2 mol ion được tạo thành, còn từ 1 mol chất III thì có 4. Thành phần các

chất IV và V là:

Co: N: Br = (10,58: 58,933): (15,08: 14,007): (15,08: 79,904) = 1,0: 6,0: 1,0.

Khối lượng mol M (IV) và M (V) = 58,933: 0,1058 = 557 gam / mol. Thành phần của IV

và V gồm 1 Co 3+ , 3 ethylenediamine, 1 Br - - tương đương với hợp phần Co(en) 3 Br = 319 -

vậy phần còn lại phải là tactrat ion ( C H O ) và 5 phân tử H 2 O.

2-

4 4 6

Độ dẫn diện cao hơn I nhưng thấp hơn III, do đó, trong quá trình phân li có 3 vi hạt được

tạo thành. Công thức của IV và V là [Co(en) 3 ]Br(C 4 H 4 O 6 ).5H 2 O Tương tự tìm được công

thức VI và VII là [Co(en) 3 ]Br 3 .2H 2 O

b. Điều chế chất III:

HCl

4CoCl 2 + 12en + O 2 4[Co(en) 3 ]Cl 3 + 2H 2 O

[Co(en) 3 ]Cl 3 + 3NaBr + 3H 2 O → [Co(en) 3 ]Br 3 .3H 2 O + 3NaCl (III)

Có một lượng nhỏ tạp chất tạo thành, khi thêm amoni nitrat thì tách ra ở dạng muối nitrat

màu xanh lục:

HCl

4CoCl 2 + 12en + O 2 4[Co(en) 2 Cl 2 ]Cl + 2H 2 O

[t-Co(en) 2 Cl 2 ]Cl + NH 4 NO 3 → [t-Co(en) 2 Cl 2 ]NO 3 + NH 4 NO 3 (I)

Điều chế chất IV - VII:

[Co(en) 3 ]Br 3 + Ag 2 С 4 H 4 O 6 → [Co(en) 3 ]Br(C 4 H 4 O 6 ) + 2AgBr↓

[d-Co(en) 3 ]Br(d-С 4 H 4 O 6 ) + 5H 2 O → [d-Co(en) 3 ]Br(d-С 4 H 4 O 6 ).5H 2 O↓ (IV)

[l-Co(en) 3 ]Br(l-С 4 H 4 O 6 ) + 5H 2 O → [l-Co(en) 3 ]Br(l-С 4 H 4 O 6 ).5H 2 O↓ (V)

[d-Co(en) 3 ]Br(d-С 4 H 4 O 6 ).5H 2 O + 2HBr → [d-Co(en) 3 ]Br 3 .2H 2 O + 3H 2 O + d-С 4 H 4 O 6 (VI)

[l-Co(en) 3 ]Br(l-С 4 H 4 O 6 ).5H 2 O + 2HBr → [l-Co(en) 3 ]Br 3 .2H 2 O +3H 2 O + l-С 4 H 4 O 6 (VII)

c. I và II là đồng phân hình học (cis/trans).

VI và VII là đồng phân quang học.

d. Số oxi hóa +3; Cấu hình electron: 1s 2 2s 2 2p 6 3s 2 3p 6 3d 6

Phát hành PDF bởi Ths Nguyễn Thanh Tú

Đăng ký Word doc qua Zalo 0905779594 Email thanhtuqn88@gmail.com

82


HSGQG - 2014

III.2.7. Trong dung dịch OH - 1,0M của [Co(NH 3 ) 5 Cl] 2+ tồn tại cân bằng:

[Co(NH 3 ) 5 Cl] 2+ + OH - [Co(NH 3 ) 4 (NH 2 )Cl] + + H 2 O.

Ở 25 o C, tại thời điểm cân bằng xác định được rằng ít nhất 95% phức chất tồn tại ở dạng

axit [Co(NH 3 ) 5 Cl] 2+ . Chứng minh [Co(NH 3 ) 5 Cl] 2+ là một axit rất yếu có

Hướng dẫn:

Ka

5,26.10 -16 .

[Co(NH 3 ) 5 Cl] 2+ (kí hiệu A) và [Co(NH 3 ) 4 (NH 2 )Cl] + (kí hiệu B) là axit và bazơ liên hợp:

[Co(NH 3 ) 5 Cl] 2+ + H 2 O [Co(NH 3 ) 4 (NH 2 )Cl] + + H 3 O + .

K =

a

+

[B][H

3O ]

[A]

[B] Ka

(1) → =

+

[A] [H O ]

3

[Co(NH 3 ) 5 Cl] 2+ + OH - [Co(NH 3 ) 4 (NH 2 )Cl] + + H 2 O (*)

-

(2)

[B]

K = (3)

[A][OH ]

Thế (2) vào (3) nhận được:

[B] K

K = =

-

[A][OH ] K

a

W

(4)

Theo giả thiết, với cân bằng (*) ở [OH - ] = 1,0M có ít nhất 95% phức tồn tại dạng axit, do vậy

có:

Phát hành PDF bởi Ths Nguyễn Thanh Tú

Đăng ký Word doc qua Zalo 0905779594 Email thanhtuqn88@gmail.com

83


[B] K 0,05

→ Ka

5,26.10 -16 .

a

K = [A][OH

- ] K

W 0,95.1,0

IV. Bài tập về Ni

IV.1. Bài tập lý thuyết, viết phương trình phản ứng, giải thích hiện tượng, điều chế,

chuỗi phản ứng, xác định công thức, …

IV.1.1. Niken (II) oxit NiO là chất rắn màu xanh, được tạo ra khi nhiệt phân Ni(OH) 2 ,

NiCO 3 , Ni(NO 3 ) 2 . NiO không tan trong nước, nhưng dễ tan trong axit. Hiđro dễ dàng khử

NiO thành kim loại. Viết các phương trình phản ứng minh họa.

Hướng dẫn:

Ni(OH) 2 NiO + H 2 O

NiCO 3 NiO + CO 2

Ni(NO 3 ) 2 NiO + 2NO 2 + 1/2O 2

0

t

NiO + H 2 Ni + H 2 O

IV.1.2. Ni(OH) 2 kết tủa màu xanh quả táo, không bị oxi hóa trong không khí. Có tính bazơ,

tan trong axit tạo thành muối tương ứng, tan trong dung dịch amoniac dư ra phức chất

amoniacat. Viết các phương trình phản ứng.

Hướng dẫn:

Ni(OH) 2 + 2HCl NiCl 2 + 2H 2 O

Ni(OH) 2 + 6NH 3

[Ni(NH 3 ) 6 ](OH) 2

IV.1.3. Niken (II) nitrat được điều chế bằng cách hòa tan niken kim loại, NiCO 3 trong

HNO 3 . Niken (II) nitrat kết tinh ở dạng hexahiđrat Ni(NO 3 ) 2 .H 2 O màu lam ngọc bích. Nhiệt

độ cao hơn 85,4 0 C mất dần nước kết tinh, và đến 300 0 C phân hủy còn lại Ni 2 O 3 . Ni 2 O 3 là

chất bột màu xám hoặc đen, do đó người ta dùng Ni(NO 3 ) 2 để tạo màu xám trong công

nghiệp đồ gốm. Ni 2 O 3 tạo ra khi nung trong không khí muối cacbonat hoặc nitrat niken (II) ở

300 0 C. Ni 2 O 3 là chất bột màu đen hoặc xám, cũng như Co 2 O 3 , Ni 2 O 3 là chất oxi hóa mạnh,

chẳng hạn tan trong HCl tạo ra khí Cl 2 . Viết các phương trình phản ứng xảy ra.

Hướng dẫn:

NiCO 3 + 2HNO 3 Ni(NO 3 ) 2 + CO 2 ↑ + H 2 O

Ni(NO 3 ) 2

0

t

Ni 2 O 3 + 4NO 2 ↑ + 1/2O 2 ↑

84

Phát hành PDF bởi Ths Nguyễn Thanh Tú

Đăng ký Word doc qua Zalo 0905779594 Email thanhtuqn88@gmail.com


4NiCO 3 + O 2

4Ni(NO 3 ) 2

0

t

2Ni 2 O 3 + 4CO 2 ↑

0

t

2Ni 2 O 3 + 8NO 2 ↑ + O 2 ↑

Ni 2 O 3 + 6HCl 2NiCl 2 + Cl 2 ↑ + 3H 2 O

IV.1.4. HSGQG - 2016

Một hợp kim gồm Cr, Fe, Co, Ni. Người ta phân tích hàm lượng các kim loại trong mẫu

hợp kim theo qui trình sau: cân 1,4 gam hợp kim, hòa tan hết vào dung dịch HNO 3 đặc nóng,

rồi thêm NaOH dư vào thu được dung dịch A và kết tủa B. Lọc tách kết tủa, rồi thêm dung

dịch H 2 O 2 dư vào dung dịch nước lọc, cô cạn. Lấy chất rắn thu được hòa tan hoàn toàn trong

dung dịch H 2 SO 4 loãng. Thêm một lượng dư KI vào dung dịch vừa thu được. Sau khi phản

ứng xảy ra hoàn toàn, chuẩn độ lượng I 2 sinh ra bằng dung dịch Na 2 S 2 O 3 0,2M thấy tốn hết

30ml. Kết tủa B được khuấy đều trong dung dịch NH 3 dư tới phản ứng hoàn toàn, thu được

kết tủa C và dung dịch D. Nung kết tủa C trong không khí ở 400 0 C đến khối lượng không

đổi thì thu được 0,96 gam chất rắn E. Thêm lượng dư KOH và K 2 S 2 O 8 vào dung dịch D, đun

nóng tới phản ứng hoàn toàn thì thu được một oxit màu đen F có khối lượng 0,81 gam và

dung dịch G. Hòa tan hết 0,81 gam chất F trong dung dịch HNO 3 , thu được dung dịch H và

100,8 ml khí không màu I (điều kiện tiêu chuẩn).

Viết các phương trình phản ứng xảy ra và xác định % về khối lượng các nguyên tố trong

mẫu hợp kim trên.

Hướng dẫn:

Hòa tan hợp kim Cr, Fe, Co, Ni trong dung dịch HNO 3

Cr + 6HNO 3 Cr(NO 3 ) 3 + 3NO 2 + 3H 2 O

Fe + 6HNO 3 Fe(NO 3 ) 3 + 3NO 2 + 3H 2 O

Co + 4HNO 3 Co(NO 3 ) 2 + 2NO 2 + 2H 2 O

Ni + 4HNO 3 Ni(NO 3 ) 2 + 2NO 2 + 2H 2 O

Thêm NaOH dư:

Cr(NO 3 ) 3 + 6NaOH Na 3 [Cr(OH) 6 ] + 3NaNO 3

Fe(NO 3 ) 3 + 3NaOH Fe(OH) 3 ↓ + 3NaNO 3

Co(NO 3 ) 2 + 2NaOH Co(OH) 2 ↓ + 2NaNO 3

Ni(NO 3 ) 2 + 2NaOH Ni(OH) 2 ↓ + 2NaNO 3

Phát hành PDF bởi Ths Nguyễn Thanh Tú

Đăng ký Word doc qua Zalo 0905779594 Email thanhtuqn88@gmail.com

85


Oxi hóa nước lọc bằng H 2 O 2 :

2Na 3 [Cr(OH) 6 ] + 3H 2 O 2 2Na 2 CrO 4 + 2NaOH + 8H 2 O

Hòa tan chất rắn thu được trong H 2 SO 4 loãng rồi chuẩn độ iot:

2Na 2 CrO 4 + H 2 SO 4 Na 2 Cr 2 O 7 + Na 2 SO 4 + H 2 O

Na 2 Cr 2 O 7 + 6KI + 7H 2 SO 4 Cr 2 (SO 4 ) 3 + 3I 2 + Na 2 SO 4 + 3K 2 SO 4 + 7H 2 O

2Na 2 S 2 O 3 + I 2 2Na 2 S 4 O 6 + 2NaI

n Cr = 1/3. n

Na 2S2O3

m Cr = 0,002.52 = 0,104 gam

Hòa tan kết tủa B:

= 0,2.30.10 -3 /3= 0,002 mol

Co(OH) 2 + 6NH 3 [Co(NH 3 ) 6 ](OH) 2

Ni(OH) 2 + 6NH 3 [Ni(NH 3 ) 6 ](OH) 2

Vậy kết tủa là Fe(OH) 3

t

2Fe(OH) 0

3 Fe 2 O 3 + 3H 2 O

Chất rắn sau khi nung là Fe 2 O 3 nặng 0,96 gam n

Fe2O3

Oxi hóa dung dịch D:

= 0,06 mol m Fe = 0,672 gam.

2[Co(NH 3 ) 6 ](OH) 2 + K 2 S 2 O 8 + 2KOH 2[Co(NH 3 ) 6 ](OH) 3 + 2K 2 SO 4

Kết tủa màu đen F là hợp chất Ni có số oxi hóa cao (III hoặc IV), do nó là sản phẩm tạo

thành khi đun [Ni(NH 3 ) 6 ](OH) 2 với K 2 S 2 O 8 . Các hợp chất này đều có tính oxi hóa rất mạnh,

khi tan trong dung dịch HNO 3 sẽ đóng vai trò là chất oxi hóa, nó sẽ oxi hóa nước (chất khử

duy nhất có trong hệ) và bị khử về Ni (II). Do đó khí không màu I là O 2 .

Xác định số oxi hóa của Ni trong hợp chất F:

Gọi số oxi hóa của Ni trong hợp chất F là n và số mol chất F (giả sử đơn nhân) là x thì số

mol O 2 là: (n-2).x/4 = 0,0045 mol, hay x = 0,018/(n-2).

Do đó phân tử lượng chất F là: 0,81.(n-2)/0,018 = 45.(n-2)

Với n = 3, M F = 9 < M Ni (loại)

Với n = 4, M F = 90: phù hợp với công thức NiO 2

Với n = 5: không tồn tại với Ni

Vậy công thức chất F là NiO 2 với số mol là: 0,81/90 = 0,009 mol.

Do vậy m Ni = 0,009.58 = 0,522 gam.

86

Phát hành PDF bởi Ths Nguyễn Thanh Tú

Đăng ký Word doc qua Zalo 0905779594 Email thanhtuqn88@gmail.com


Các phương trình phản ứng:

[Ni(NH 3 ) 6 ](OH) 2 + K 2 S 2 O 8 + 2KOH NiO 2 + 2K 2 SO 4 + 6NH 3 + 2H 2 O

2NiO 2 + 4HNO 3 2Ni(NO 3 ) 2 + 2H 2 O + O 2

Vậy khối lượng Co là: 1,4 – 0,104 – 0,672 – 0,522 = 0,102 gam.

Vậy % khối lượng của Cr là: 7,43%; Fe: 48%; Ni: 37,29%; Co: 7,29%.

IV.2. Bài tập tinh thể

IV.2.1.a. Biết rằng kim loại niken kết tinh theo một kiểu cấu trúc tinh thể A1 với độ dài tiếp

xúc giữa các nguyên tử là 249,2 pm. Tính khối lượng riêng của Ni ( Cho Ni = 58,69)

b. Biểu diễn sự sắp xếp các nguyên tử trong các mặt (100), (110) và (111).

Hướng dẫn:

a. Do kim loại Ni có cấu trúc kiểu A1 (lập phương tâm diện) nên các nguyên tử tiếp xúc với

nhau theo đường chéo của ô mạng lập phương.

a = 2 x249,2 pm = 352,4 pm

Ô mạng cơ sở Ni có 4 nguyên tử, do đó:

4M

D =

3

a N = 4. 58,69 g/mol

-10 3 23 -1

(352,4.10 cm) .6,022.10 mol

A

D = 8,91 g/cm 3

IV.2.2.a. Ô mạng cơ sở (tế bào cơ bản) của tinh thể NiSO 4 có 3 cạnh vuông góc với nhau, cạnh

a = 6,338 Å; b = 7,842 Å; c = 5,155 Å. Khối lượng riêng gần đúng của NiSO 4 là 3,9 g/cm 3 .

Tìm số phân tử NiSO 4 trong một ô mạng cơ sở và tính khối lượng riêng chính xác của

NiSO 4 .

Hướng dẫn:

a. a = 6,338.10 –8 cm; b = 7,842.10 –8 cm; c = 5,155.10 –8 cm

Phát hành PDF bởi Ths Nguyễn Thanh Tú

Đăng ký Word doc qua Zalo 0905779594 Email thanhtuqn88@gmail.com

87


Từ

D

n

NiSO4

m m n.MNiSO4

(1)

V a.b.c N .a.b.c

D .N .a.b.c

(2)

NiSO 4

A

MNiSO4

A

23 –8 –8 –8

3,9.6,02.10 .6,338.10 .7,842.10 .5,155.10

n = 3,888

154,76

Số phân tử NiSO 4 trong một ô mạng cơ sở phải là số nguyên n = 4

4.154,76

D

NiSO 4

(chính xác) =

23 –8 –8 –8

6,02.10 .6,338.10 .7,842.10 .5,155.10

= 4,012 (g/cm 3 )

IV.2.3. Niken (II) oxit có cấu trúc mạng tinh thể giống mạng tinh thể của natri clorua. Các ion O 2–

tạo thành mạng lập phương tâm mặt, các hốc bát diện có các ion Ni 2+ . Khối lượng riêng của niken(II)

oxit là 6,67 g/cm 3 .

Nếu cho niken (II) oxit tác dụng với liti oxit và oxi thì được các tinh thể trắng có thành

phần Li x Ni 1–x O:

x

2 Li 2O + (1 – x)NiO + x 4 O 2 Li x Ni 1–x O

Cấu trúc mạng tinh thể của Li x Ni 1–x O giống cấu trúc mạng tinh thể của NiO, nhưng một số

ion Ni 2+ được thế bằng các ion liti và một số ion Ni 2+ bị oxi hóa để bảo đảm tính trung hòa

điện của phân tử. Khối lượng riêng của tinh thể Li x Ni 1–x O là 6,21 g/cm 3 .

a. Vẽ một ô mạng cơ sở của niken(II) oxit.

b. Tính x (chấp nhận thể tích của ô mạng cơ sở không thay đổi khi chuyển từ NiO thành

Li x Ni 1–x O).

c. Tính phần trăm số ion Ni 2+ đã chuyển thành ion Ni 3+ và viết công thức thực nghiệm đơn

giản nhất của hợp chất Li x Ni 1–x O bằng cách dùng Ni(II), Ni(III) và các chỉ số nguyên.

Cho Li = 6,84; Ni = 58,69; O = 16; S = 32

Hướng dẫn:

a.

Phát hành PDF bởi Ths Nguyễn Thanh Tú

Đăng ký Word doc qua Zalo 0905779594 Email thanhtuqn88@gmail.com

88


Ion oxi (O 2- )

Ion niken (Ni 2+ )

b. Tính x:

Tính cạnh a của ô mạng cơ sở của NiO:

n.M

D = a 3 =

N .a

NiO

NiO 3

A

n.M

N .D

A

NiO

NiO

n = 4 (vì mạng là lập phương tâm diện)

4.74,69

23

3

a = 6,02.10 .6,67

a = 4,206.10 –8 cm

Theo đầu bài, ô mạng cơ sở của NiO và ô mạng cơ sở của Li x Ni 1–x O giống nhau, do đó:

D

x = 0,10

n.M

4. x.6,94 (1 x).58,69 16

6,21 =

23 8 3

6,02.10 .(4,206.10 )

Lix

Ni1

xO

LixNi1

xO 3

N

A.a

c. Thay x vào công thức Li x Ni 1–x O, ta có Li 0,1 Ni 0,9 O hay công thức là LiNi 9 O 10 . Vì phân tử

trung hòa điện nên trong LiNi 9 O 10 có 8 ion Ni 2+ và 1 ion Ni 3+ . Vậy cứ 9 ion Ni 2+ thì có 1 ion

chuyển thành Ni 3+ .

Phần trăm số ion Ni 2+ đã chuyển thành ion Ni 3+ là 1 100

9 % = 11,1%

Công thức thực nghiệm đơn giản nhất: LiNi(III)(Ni(II)) 8 O 10 .

V. Bài tập về Ru, Rh, Pd, Re, Os, Ir, Pt

* Bài tập phức chất

V.1. (HSGQG – 2014)

Năm 1965, các nhà khoa học đã tìm ra phương pháp cố định nitơ ở nhiệt độ phòng bằng

cách dẫn khí nitơ đi qua dung dịch pentaaminoaquơruteni (II) (A 1 ). Khi đó, nitơ sẽ thay thế

nước trong cầu nội của A 1 tạo phức chất mới A 2 . Phức chất A 2 có tính thuận từ.

a. Viết phương trình phản ứng xảy ra.

Phát hành PDF bởi Ths Nguyễn Thanh Tú

Đăng ký Word doc qua Zalo 0905779594 Email thanhtuqn88@gmail.com

89


b. Ru: [Kr]4d 7 5s 1 Ru – 2e → Ru 2+

b. Áp dụng thuyết liên kết hóa trị (VB), hãy mô tả liên kết trong phức A 2 và dự đoán cấu

trúc hình học của nó. Xác định hóa trị và số oxi hóa của ruteni trong phức chất A 2 .

Hướng dẫn:

a. Pentaaminoaquơruteni(II) - [Ru(NH 3 ) 5 H 2 O] 2+ phản ứng với nitơ:

[Ru(NH 3 ) 5 H 2 O] 2+ + N 2 = [Ru(NH 3 ) 5 (N 2 )] 2+ + H 2 O.

4d 7 5s 1 4d 6

Vì phức chất thuận từ, nên sự phân bố e trong AO 4d của Ru là:

AO 5s, các AO 5p và 2 AO 5d của Ru 2+ tham gia lai hóa sp 3 d 2 .

Trên mỗi nguyên tử N trong năm phân tử NH 3 và phân tử N 2 đều có cặp e tự do, tham gia

hình thành liên kết cho nhận với Ru 2+ .

Các AO sp 3 d 2 :

NH 3 NH 3 NH 3 NH 3 NH 3 N 2

Cấu trúc hình học của A 2 :

2+

NH 3

NH 3

H 3 N Ru N N

NH 3

NH 3

Phức chất A 2 có dạng bát diện.

Ruteni có hóa trị 6 và số oxi hóa là +2.

90

Phát hành PDF bởi Ths Nguyễn Thanh Tú

Đăng ký Word doc qua Zalo 0905779594 Email thanhtuqn88@gmail.com


V.2. [Ru(SCN) 2 (CN) 4 ] 4– là ion phức của ruteni. Cho biết dạng lai hóa của Ru trong ion phức

trên. Mô tả sự hình thành ion phức theo thuyết VB. Giải thích tại sao trong ion phức trên, liên

kết được hình thành giữa Ru và N của phối tử SCN – mà không phải là giữa Ru và S. Cho

biết phức có tính thuận từ hay nghịch từ, vì sao?

Hướng dẫn:

Ru 2+ có cấu hình electron [Kr]4d 6 5s 0 5p 0 . Kiểu lai hóa d 2 sp 3 (do CN - là phối tử trường

mạnh, Ru thuộc dãy thứ hai của kim loại chuyển tiếp), không có electron độc thân, nghịch

từ.

d 2 sp 3

[Ru(SCN) 2 (CN) 4 ] 4-

2 SCN - + 4CN -

So với S, N có độ âm điện lớn hơn và bán kính nguyên tử nhỏ hơn, do đó mật độ điện tích

âm trên nguyên tử N sẽ lớn hơn, ái lực phản ứng với ion dương Ru 2+ lớn hơn, vì vậy trong

phức chất P, liên kết phức được hình thành giữa Ru và N mà không phải là giữa Ru và S.

Hoặc: Ru 2+ là axit cứng, N là bazơ cứng.

Phức có tính nghịch từ vì trong ion phức không có electron độc thân.

V.3. So sánh độ bền của cặp phức chất sau và giải thích ngắn gọn: [Ru(bipy) 3 ] 3+ và

[Ru(py) 6 ] 3+

Hướng dẫn:

Sự tạo thành phức có phối tử đa càng có sự thuận lợi hơn về mặt entropy, do đó phức có

ligand nhiều càng bền hơn phức có liagnd ít càng. Vì thế [Ru(bipy) 3 ] 3+ bền hơn [Ru(py) 6 ] 3+

V.4. Có thể tách được rođi khỏi các kim loại quý khác bằng cách sau: Một mẫu bột rođi

được trộn với NaCl và đun nóng trong dòng khí clo. Bã rắn thu được chứa một muối A chứa

26,76 % rođi.. Bã rắn này sau đó được xử lý với nước dung dịch thu được đem lọc và cô bay

hơi thu được tinh thể B chứa 17,13% rođi. Tinh thể được làm khô ở 120 o C đến khối lượng

không đổi (khối lượng mất đi là 35,98%) rồi đun nóng tới 650 o C. Rửa bã rắn thu được bằng

nước cho kim loại rođi tinh khiết.

a. Xác định công thức cấu tạo của muối A.

b. Công thức của B là gì?

Phát hành PDF bởi Ths Nguyễn Thanh Tú

Đăng ký Word doc qua Zalo 0905779594 Email thanhtuqn88@gmail.com

91


c. Khi một lượng dư hiđro sunfua được sục qua dung dịch muối A thì tạo thành kết tủa C.

Hợp chất này có thành phần hợp thức chứa 47,59% lưu huỳnh. Xác định thành phần hóa học

của C.

d. Giải thích tại sao cần phải rửa bằng nước ở bước cuối cùng.

e. Viết phương trình hóa học cho các chuyển hóa ở câu trên.

Hướng dẫn:

a. A = Na 3 [RhCl 6 ]:

b. B = Na 3 [RhCl 6 ].12H 2 O

c. C = Rh 2 S 3 .3H 2 S

d. Để loại bỏ các muối tan (chủ yếu là NaCl).

e. 2Rh + 6NaCl + 3Cl 2 = 2Na 3 [RhCl 6 ]

Na 3 [RhCl 6 ].12H2O = Na 3 [RhCl 6 ] + 12H 2 O

2Na 3 [RhCl 6 ] = 2Rh + 6NaCl + 3Cl 2

2Na 3 [RhCl 6 ] + 3H 2 S = Rh 2 S 3 .3H 2 S + 6NaCl + 6HCl

V.5. Cisplatin Pt(NH 3 ) 2 Cl 2 là thuốc chữa khỏi ung thư tinh hoàn cho tay đua xe đạp người

Mỹ Lance Armstrong. Hãy viết công thức cấu tạo và cho biết từ tính của hợp chất này, giải

thích.

Hướng dẫn:

Phát hành PDF bởi Ths Nguyễn Thanh Tú

Đăng ký Word doc qua Zalo 0905779594 Email thanhtuqn88@gmail.com

92


Cisplatin (thuốc chữa ung thư)

d count = 10 -2 = 8

Vì có dạng mặt phẳng vuông

Nên giản đồ năng lượng được mô tả:

Đây là chất nghịch từ

(không có eletron độc thân)

V.6. Trong quá trình điện tinh luyện đồng, nhiều kim loại quý hiếm được tìm thấy trong bùn

anot. Một trong số đó là platin – được biết đến nhiều với các phức chất vuông phẳng. Cho sơ

đồ sau:

a. Xác định các chữ cái A, B, C, D tương ứng với phối tử nào sau đây: py (pyridine), NH 3 ,

Cl - - -

, NO 2 ? Biết ảnh hưởng của các phối tử (qua hiệu ứng trans) giảm dần theo thứ tự : NO 2

> Cl - > py > NH 3

b. Vẽ công thức các phức chất IV, V, VI.

c. Phức chất VI có thể có bao nhiêu đồng phân?

Hướng dẫn:

a. A : Cl- ; B : NO 2 - ; C : NH 3 ; D : py

b. phức IV: phức V: phức VI:

Phát hành PDF bởi Ths Nguyễn Thanh Tú

Đăng ký Word doc qua Zalo 0905779594 Email thanhtuqn88@gmail.com

93


c. Phức VI có 3 đồng phân

Py NH 3 Py NO 2 Py NH 3

Pt

Pt

Pt

Cl NO 2 Cl NH 3

O 2 N

Cl

* Bài tập tinh thể.

V.7. Hợp kim là dung dịch rắn - rắn có thành phần biến đổi. Trong một hợp kim thay thế,

các nguyên tử của các nguyên tố khác nhau cùng tạo nên các nút mạng trong lưới tinh thể.

Tuy nhiên, nếu các nguyên tử có kích thước khác nhau, các nguyên tử có kích thước nhỏ có

thể chiếm vị trí các khoảng trống trong mạng lưới tinh thể các nguyên tử lớn hơn. Các hợp

kim này, được gọi là tinh thể xâm nhập, các tinh thể này thường thu được khi cho các á kim

(B, H, O, N, C, …) hòa tan trong kim loại. Paladi có cấu trúc tinh thể lập phương tâm mặt,

trong đó có hai loại hốc tứ diện và bát diện, bán kính nguyên tử paladi là 137pm

a. Tính bán kính nguyên tử lớn nhất có thể lọt vào khe tạo ra tinh thể xâm nhập mà không

phá vỡ mạng lưới tinh thể (của paladi).

b. Một điều hiển nhiên ai cũng biết là hidro có thể hòa tan rất tốt trong kim loại Pd, hãy tính

phần trăm theo khối lượng của hidro trong hợp kim với paladi tạo ra tinh thể xâm nhập, cho

biết công thức thực nghiệm ứng với tinh thể này.

Hướng dẫn:

a. Giả sử nguyên tử Pd được kí hiệu là X trên hình vẽ, nguyên tử xâm nhập kí hiệu là M.

Với lổ hổng bát diện

(2rx) 2 = (r M

+ rx) 2 + (r M

+ r X

) 2

rX 2= r

M

+ rX r

M

= ( 2 -1) rX

= 56,75pm

Với lổ hổng tứ diện

nên

Một đường thằng đi từ các cạnh chia góc tứ

diện ra làm hai phần. Độ dài của mỗi cạnh là

2 r X

. Khoảng cách từ một đỉnh của tứ diện

đến tâm của nó là r M

+ r X

.

Góc lúc này là 109,5°/2.

sin θ = r x

/ (r M

+ r X

)→ sin (109,5°/2)· (r M

+ r X

) = r X

Phát hành PDF bởi Ths Nguyễn Thanh Tú

Đăng ký Word doc qua Zalo 0905779594 Email thanhtuqn88@gmail.com

94


→ 0.816 r M

= 0,184 r X

→ r M

/r X

= 0,225

→ r M

= 30,825pm

b. Trong một ô mạng cơ sở có 4 nguyên tử Pd (1/8 nguyên tử 8 đỉnh và ½ nguyên tử ở 6

mặt).

Số lỗ hổng tứ diện là 8 x 1 = 8;

bát diện là: 1x1 + (1/4)x 12 = 4

Vậy trong một ô mạng cơ sở có 4 nguyên tử Pd

Tất cả các hốc tứ diện và bát diện đều được lấp bởi nguyên tử hidro --> só nguyên tử H

bằng 12

CT: Pd 4 H 12 PdH 3

% H = 2,75%

V.8. Tinh thể ReO 3 thuộc hệ lập phương, trong đó ion Re 6+ chiếm các vị trí đỉnh của hình lập

phương, ion O 2- chiếm vị trí trung điểm tất cả các cạnh của ô mạng.

Biết bán kính: O 2- = 0,126nm, Re 6+ = 0,061 nm.

Nguyên tử khối: O =16; Re = 186

a. Hãy vẽ một ô mạng cơ sở của tinh thể ReO 3 , tính số nguyên tử Re và O trong một ô mạng

và khối lượng riêng của tinh thể ReO 3 (gam/cm 3 )

b. Cation Li + có thể xâm nhập vào mạng lưới tinh thể ReO 3 ở ngay nhiệt độ phòng. Cation

Li + có kích thước lớn nhất bằng bao nhiêu để khi xâm nhập vào mạng lưới tinh thể ReO 3

không làm thay đổi kích thước của ô mạng tinh thể ?

Hướng dẫn:

a. Vẽ ô mạng ReO 3

Phát hành PDF bởi Ths Nguyễn Thanh Tú

Đăng ký Word doc qua Zalo 0905779594 Email thanhtuqn88@gmail.com

95


- Trong một ô mạng có

Số ion Re 6+ = 8.1/8 = 1

Số ion O 2- = 12.1/4 = 3

- Độ dài cạnh a= 2(r Re + r O ) = 0,374nm

- Khối lượng riêng của tinh thể:

M (186 16.3) / 6,02.10

D

V

23

ô mang

3

7,43 gam / cm

7 3

ô mang

(0,374.10 )

b. Tính bán kính ion lạ

Ion Li + xâm nhập sẽ chiếm vị trí trung tâm của ô mạng cơ sở. Do vậy khoảng cách từ tâm

đến trung điểm mỗi cạnh tương ứng:

r 2 r a / 2 0, 265 nm

O

r

Li

Li

0,139nm

V.9. Kim loại platin có cấu trúc kiểu A1 (còn gọi là kiểu lập phương tâm diện), với hằng số

mạng là a = 392,3 pm. Xác định khối lượng riêng và bán kính nguyên tử của platin. (Cho Pt

= 195)

Hướng dẫn:

Do kim loại Pt có cấu trúc kiểu A1 (lập phương tâm diện) nên có 4 nguyên tử trong mỗi ô

mạng lập phương. Khối lượng riêng của Pt là:

4M

D =

3

a N = 4.195 g/mol

-10 3 23 -1

(392, 3. 10 c m) 6,022.10 mol

A

D = 21,45 g/cm 3 .

Phát hành PDF bởi Ths Nguyễn Thanh Tú

Đăng ký Word doc qua Zalo 0905779594 Email thanhtuqn88@gmail.com

96


Trong cấu trúc lập phương tâm diện, các nguyên tử tiếp xúc với nhau theo đường chéo của

ô mạng lập phương, do đó: a = 2R 2

R = a/2

2 = 392,3/2 2 = 138,7 pm.

VI. Một số dạng bài tập phức chất thường gặp

Cho biết số phối trí phổ biến trong phức chất của các ion kim loại nhóm VIIB, VIIIB? Cho

ví dụ một số công thức phức của từng kim loại.

Hướng dẫn:

Số phối trí phổ biến trong phức chất của các ion kim loại nói chung, cũng như các ion kim

loại nhóm VIIB, VIIIB là 4 và 6.

Ion kim loại Số phối trí Ví dụ

Mn 2+ 4, 6 [MnBr 4 ] 2- ,

Mn 3+ 6 [Mn(CN) 6 ] 3- ,

Re 7+ 4 [ReO 4 ] -

[Mn(OH 2 ) 6 ] 2+ , [Mn(CN) 6 ] 4-

Fe 2+ 6 [Fe(CN) 6 ] 4- , [Fe(H 2 O) 6 ] 2+ , [Fe(bpy) 3 ] 2+

Fe 3+ 6 [Fe(H 2 O) 6 ] 3+ , [Fe(CN) 5 (NO)] 2+

[Fe(C 2 O 4 ) 3 ] 3- , [Fe(CN) 6 ] 3- , [FeF 6 ] 3- , [FeI 2 (C 2 O 4 ) 2 ] 3-

Co 2+ 4, 6 [Co(CN) 4 ] 2- , [CoCl 4 ] 2- , [CoF 6 ] 4- , [Co(CN) 6 ] 4- ,

[Co(en)(NH 3 ) 2 Cl 2 ]

Co 3+ 6 [CoCl(NO 2 )(NH 3 ) 4 ] + , [Co(NH 3 ) 6 ] 3+ , [Co(NH 3 ) 5 Br] 2+ ,

[Co(NH 3 ) 4 (NO 2 ) 2 ] + , [Co(NH 3 ) 5 NO 2 ] 2+ , [Co(en) 2 Cl 2 ] + ,

[Co(NSSN)Cl 2 ] +

[CoF 6 ] 3- , [Co(CO 3 ) 2 (NH 3 ) 2 ] - , [Co(CO 3 ) 3 ] 3- và [Co(NO 2 ) 6 ] 3-

[Co(NH 3 ) 3 (SCN) 3 ]

Ni 0 4 [Ni(CO) 4 ], [Ni(CO) 3 (py)],

Ni 2+ 4, 6 [Ni(CN) 4 ] 2- , [NiCl 4 ] 2- ,

[Ni(NH 3 ) 6 ] 2+ , [Ni(en) 3 ] 2+

Ru 2+ 6 [Ru(NH 3 ) 6 ] 2+ , [Ru(NH 3 ) 5 (H 2 O)] 2+ , [Ru(en) 3 ] 2+ ,

[Ru(bipy) 3 ] 2+

Phát hành PDF bởi Ths Nguyễn Thanh Tú

Đăng ký Word doc qua Zalo 0905779594 Email thanhtuqn88@gmail.com

97


[RuCl 2 (dppe) 2 ], [Ru(NH 3 ) 4 Cl 2 ]

[Ru(SCN) 2 (CN) 4 ] 4–

Ru 3+ 6 [Ru(H 2 O) 2 (C 2 O 4 ) 2 ] - , [Ru(DMSO)(imidazole)Cl 4 ] – ,

[Ru(CN) 6 ] 3-

[Ru(H 2 O) 6 ] 3+ , [Ru(bipy) 3 ] 3+ , [Ru(py) 6 ] 3+ ,

Ru 4+ 6 [RuBrCl 2 (NH 3 ) 3 ] +

Rh + 4 [RhI 2 (CO) 2 ] -

Rh 3+ 6 [Rh(Cl) 2 (NH 3 ) 4 ] + , [Rh(NH 3 ) 5 Cl] 2+

[RhCl 3 (H 2 O) 3 ], [Rh(py) 3 Cl 3 ]

Pd 2+ 4 [PdCl 4 ] 2- ,

[Pd(NH 3 ) 4 ] 2+

[Pd(SCN) 2 (NH 3 ) 2 ], [Pd(NCS) 2 {P(CH 3 ) 3 } 2 ],

[PdBrCl(PEt 3 ) 2 ]

Os 4+ [OsF 6 ] 2- ,

Ir + 4 [IrCl(CO)(PPh 3 ) 2 ]

Ir 3+ 6 [Ir(Br) 2 (CH 3 )(CO)(PPh 3 ) 2 ], [Ir(Cl) 3 (PEt 3 ) 3 ]

Ir 4+ 6 [IrCl 6 ] 2- ,

Pt 2+ 4 [PtCl 2 (NH 3 ) 2 ], [Pt(Ph 2 PCH 2 CH 2 PPh 2 ) 2 ]Cl 2

[PtCl 4 ] 2- , [PtCl 3 (NO 2 )] 2- ,

[Pt(NH 3 ) 4 ] 2+ , [PtCl 3 (NH 3 )] - , [Pt(NH 3 ) 2 (H 2 O) 2 ] 2+

Pt 4+ 6 [PtCl 2 (NH 3 ) 2 (OH 2 ) 2 ] 2+ , [Pt(CN) 2 (en) 2 ] 2+ , [PtBr 2 (en) 2 ] 2+ ,

[PtCl 2 (NH 3 ) 4 ] 2+ , [Pt(en) 3 ] 4+

[PtCl 2 Br 2 (NO 2 ) 2 ] 2- , [PtCl 6 ] 2- , [PtCl 5 (NH 3 )] - ,

Ngoài ra cũng gặp các số phối trí khác: 5, 9.

Ví dụ: Fe(CO) 5 , Mn(CO) 5 , [Ni(CN) 5 ] 3- , [Ru(NCS) 2 (SCN) 3 ] 3- , [ReH 9 ] 2- .

VI.1. Dạng bài tập về đồng phân của phức chất:

VI.1.1.a. Hãy viết một đồng phân ion hoá của phức [Co(NH 3 ) 4 (NO 2 ) 2 ]Br.

Hướng dẫn:

[Co(NH 3 ) 4 (NO 2 )Br]NO 2

Phát hành PDF bởi Ths Nguyễn Thanh Tú

Đăng ký Word doc qua Zalo 0905779594 Email thanhtuqn88@gmail.com

98


b. Viết 2 đồng phân hiđrat hóa của phức [Co(H 2 O) 6 ]Cl 3 .

Hướng dẫn:

[Co(H 2 O) 5 Cl]Cl 2 .H 2 O và [Co(H 2 O) 4 Cl 2 ]Cl.2H 2 O

c. Viết các đồng phân liên kết của phức [Pt(SCN) 2 (dppe)].

Biết phối tử dppe là phối tử 2 càng và có công thức cấu tạo

bên:

Hướng dẫn:

NCS P SCN P

Pt

Pt

SCN P SCN P

d. Viết đồng phân cis-trans của phức: [PtCl 2 (NH 3 ) 2 ]; [Co(NH 3 ) 4 Cl 2 ]

Hướng dẫn:

e. Hãy cho biết phức [Co(en) 3 ]I 3 có bao nhiêu đồng phân lập thể? (en là phối tử etylen

điamin).

Hướng dẫn:

2 đồng phân đối quang

N

N

N

N

Co

N

N

N

N

Co

N

N

N

N

99

Phát hành PDF bởi Ths Nguyễn Thanh Tú

Đăng ký Word doc qua Zalo 0905779594 Email thanhtuqn88@gmail.com


[Co(en) 3 ] 3+

g. Hãy vẽ tất cả các đồng phân liên kết và đồng phân hình học của phức chất sau:

[Co(ONO) 2 (NH 3 ) 2 (en)] + . Chỉ ra đồng phân nào có đối quang?

Hướng dẫn:

ONO

ONO

NO 2

N

Co

NH 3

N NH 3

N

Co

NH 3

N NH 3

N

Co

NH 3

N NH 3

ONO

NO 2

NO 2

ONO

ONO

NO 2

NO 2

N

Co

NH 3

N ONO

N

Co

NH 3

N NO 2

N

Co

NH 3

N ONO

N

Co

NH 3

N NO 2

NH 3

NH 3

NH 3

NH 3

NH 3

NH 3

NH 3

N

N

Co

ONO

ONO

N

Co

NO 2

N ONO

N

Co

NO 2

N NO 2

NH 3

NH 3

NH 3

4 đồng phân sau bất đối

ONO

ONO

NO 2

NO 2

N

Co

NH 3

N ONO

N

Co

NH 3

N NO 2

N

Co

NH 3

N ONO

N

Co

NH 3

N NO 2

NH 3

NH 3

NH 3

NH 3

h. Hãy vẽ các đồng phân có thể có của phức [Co(en) 2 (NO 2 )Cl] + .

Hướng dẫn:

Chú ý: tính cả đồng phân liên kết

i. Hãy vẽ các đồng phân có thể có của phức [Pt(SCN) 2 (dmen)]

)

100

Phát hành PDF bởi Ths Nguyễn Thanh Tú

Đăng ký Word doc qua Zalo 0905779594 Email thanhtuqn88@gmail.com


(cấu hình vuông phẳng).

Biết phối tử dmen là phối tử 2 càng và có công thức cấu tạo

bên:

Hướng dẫn:

N *

N

Pt

SCN NCS

N *

N

Pt

NCS SCN

N *

N

Pt

NCS NCS

N *

N

Pt

SCN SCN

Câu 2: Viết các đồng phân phối trí có thể có của hợp chất có công thức phân tử

Pt 2 (NH 3 ) 4 Cl 6 .

Hướng dẫn:

([Pt II (NH 3 ) 4 ][Pt IV Cl 6 ]

[Pt II (NH 3 ) 3 Cl][Pt IV Cl 5 ((NH 3 )]

[Pt II (NH 3 ) 2 Cl 2 ][Pt IV Cl 4 ((NH 3 ) 2 ]

[Pt II (NH 3 )Cl 3 ][Pt IV Cl 3 ((NH 3 ) 3 ]

[Pt II Cl 4 ][Pt IV Cl 2 ((NH 3 ) 4 ]

Câu 3: Một số phức của kim loại chuyển tiếp có khả năng điều trị bệnh ung thư. Gần đây

người ta tìm ra được phức của Ru (III) là [Ru(DMSO)(imidazole)Cl 4 ] – . Phức của

đimetylsunfoxit [(CH 3 ) 2 SO] (DMSO) rất đặc biệt vì phối tử DMSO có thể liên kết với kim

loại qua cả nguyên tử O và S. Hãy cho biết tổng số đồng phân lập thể và đồng phân cấu tạo

của phức [Ru(DMSO)(imidazole)Cl 4 ] – ?

Hướng dẫn:

4 đồng phân (cis, trans và đồng phân liên kết).

VI.1.2. Hãy vẽ tất cả các đồng phân của phức [Co(NSSN)Cl 2 ] + . Biết

phối tử NSSN là phối tử 4 càng và có công thức cấu tạo bên:

S

S

CH 2 CH 2 NH 2

CH 2 CH 2 NH 2

101

Phát hành PDF bởi Ths Nguyễn Thanh Tú

Đăng ký Word doc qua Zalo 0905779594 Email thanhtuqn88@gmail.com


Hướng dẫn:

N

Cl

Cl

S

S

Co

Cl

Cl

S

S

Co

Cl

N

S

S

Co

N

N

N

N

Cl

bÊt ®èi bÊt ®èi

VI.1.3. HSGQG - 2015

Phối tử (2-aminoetyl)photphin là phối tử hai càng. Viết các đồng phân hình học và đồng

phân quang học của phức chất đicloro bis(2-aminoetyl)photphin niken(II).

Hướng dẫn:

Các đồng phân hình học và đồng phân quang học của phức chất đicloro bis (2-aminoetyl)

photphin) niken(II)

Phối tử (2-aminoetyl)photphin là phối tử 2 càng:

CH 2 CH 2

Đồng phân trans: 2 đồng phân

H 2 N PH 2

Đồng phân cis: có 3 đồng phân, mỗi đồng phân lại có thêm đồng phân quang học

VI.2. Dạng bài tập thuyết VB và thuyết trường tinh thể trong phức chất:

VI.2.1. Ion [Mn(CN) 6 ] 3- có 2 electron độc thân, ion [MnBr 4 ] 2- có 5 electron độc thân, ion

[Ni(CN) 4 ] 2- không có electron độc thân. Dựa vào thuyết liên kết hoá trị (thuyết VB), hãy viết

cấu hình electron (dưới dạng ô lượng tử) của các ion phức trên, cho biết kiểu lai hoá và cấu

trúc hình học của chúng.

Phát hành PDF bởi Ths Nguyễn Thanh Tú

Đăng ký Word doc qua Zalo 0905779594 Email thanhtuqn88@gmail.com

102


Hướng dẫn:

[Mn(CN) 6 ] 3- : d 2 sp 3 , bát diện

[Mn(CN) 6 ] 3- d 2 sp 3

[MnBr 4 ] 2- : sp 3 , tứ diện

[Ni(CN) 4 ] 2- : dsp 2 , vuông phẳng

[Ni(CN) 4 ] 2- dsp 2

6CN -

4CN -

VI.2.2. Niken (II) có cấu hình electron là 3d 8 . [Ni(CN) 4 ] 2- là phức nghịch từ còn [NiCl 4 ] 2- là

phức thuận từ với hai electron độc thân. Sắt (III) có cấu hình electron là 3d 5 . Phức

[Fe(CN) 6 ] 3- có một electron độc thân, còn phức [Fe(H 2 O) 6 ] 3+ có năm electron độc thân.

a. Hãy giải thích các hiện tượng trên theo thuyết VB.

b. Hãy giải thích các hiện tượng trên theo thuyết trường tinh thể.

Hướng dẫn:

a. [Ni(CN) 4 ] 2- : dsp 2 , không có electron độc thân, nghịch từ.

[NiCl 4 ] 2- : sp 3 , có 2 electron độc thân, thuận từ.

[Fe(CN) 6 ] 3- : d 2 sp 3 , có 1 electron độc thân, thuận từ.

[Fe(H 2 O) 6 ] 3+ : sp 3 d 2 , có 5 electron độc thân, thuận từ.

b.

Phát hành PDF bởi Ths Nguyễn Thanh Tú

Đăng ký Word doc qua Zalo 0905779594 Email thanhtuqn88@gmail.com

103


d x2-y2

d xy

d z2 d x2-y2

d xy d xz d yz

d z2 d x2-y2

d xz

d yz

d z2

d x2-y2

d z2

[NiCl 4 ] 2- [Fe(H 2 O) 6 ] 3+

d xy d xz d yz

d xy d xz d yz

[Ni(CN) 4 ] 2-

[Fe(CN) 6 ] 3+

VI.2.3.a. [Fe(CN) 6 ] 4- có: P = 210 kJ/mol; o = 395 kJ/mol

b. [Fe(H 2 O) 6 ] 2+ có: P = 210 kJ/mol; o = 124 kJ/mol

Hãy viết cấu hình e của ion trung tâm? Cho biết phức thuận từ hay nghịch từ.

Hướng dẫn:

Sự tách mức năng lượng trong thuyết trường tinh thể đối với phức bát diện như sau:

a. Do P < o nên các e của trung tâm dễ ghép đôi => cấu hình e của trung tâm Fe 2+ là (3d 6 )

là (d ) 6 . Đây là phức nghịch từ.

b. Do P > o nên các e của trung tâm khó ghép đôi => cấu hình e của trung tâm Fe 2+ là (3d 6 )

là (d ) 4 (d ) 2 . Đây là phức thuận từ (có 4 e độc thân).

VI.2.4. Thí nghiệm chứng minh rằng cả 2 phức của Fe là [Fe(H 2 O) 6 ] 3+ , [Fe(CN) 6 ] 3- đều

thuận từ và đều có cấu tạo bát diện đều. Hãy cho biết chất nào có tính nghịch từ nhiều hơn.

Giải thích. Cho biết cấu hình e của Fe là [Ar]3d 6 4s 2 .

Phát hành PDF bởi Ths Nguyễn Thanh Tú

Đăng ký Word doc qua Zalo 0905779594 Email thanhtuqn88@gmail.com

104


Hướng dẫn:

Trong cả 2 hợp chất Fe đều có số oxh là +3

d count = 8 - 3 = 5 electron

Vì có cấu tạo bát diện đều nên có giản đồ năng lượng như sau

∆ o nhỏ (phối tử trường yếu) ∆ o lớn (phối tử trường mạnh)

∆ o < PE

cấu hình e: t 3 2

2g e g

Fe(H 2 O) 6 ] 3+ [Fe(CN) 6 ] 3-

∆ o > PE

Vì [Fe(H 2 O) 6 ] 3+ có số electron độc thân nhiều hơn nên có tính thuận từ nhiều hơn.

VI.2.5.1. Thực nghiệm đã xác nhận rằng, trong phức [Ru(NH 3 ) 6 ] 2+ có bốn electron độc thân.

Cho cấu hình electron Ru: [Xe] 4d 7 5s 1 .

a. Theo lí thuyết VB, hãy giải thích sự hình thành của phức trên.

b. Theo thuyết trường tinh thể, hãy vẽ giản đồ phân bố năng lượng của obitan d.

c. Tại sao trong phức [Ru(NH 3 ) 6 ] 2+ , nguyên tử trung tâm có 4 electron độc thân còn trong

[Ru(NH 3 ) 6 ] 3+ thì nguyên tử trung tâm chỉ có 1 electron độc thân.

VI.2.5.2. Cùng là phức bát diện, người ta nhận thấy rằng, nếu phối tử là nhiều càng thì phức

dễ tạo hơn các phối tử ít càng hơn và đơn càng (cùng một loại nguyên tử của phối tử liên kết

với nguyên tử trung tâm). Ở đây, chúng ta sẽ xét tới 3 phức sau đó là [Ni(NH 3 ) 6 ] 2+ ,

[Ni(en) 3 ] 2+ , [Ni(pen)] 2+ .

Ni 2+ + 6NH 3 ⇌ [Ni(NH 3 ) 6 ] 2+ K f = 3,2.10 8 .

Ni 2+ + 3en ⇌ [Ni(en) 3 ] 2+ K f = 1,6.10 18

Ni 2+ + pen ⇌ [Ni(pen)] 2+ K f = 2,0.10 19

Hãy giải thích tại sao lại có sự khác biệt về các hằng số cân bằng của phản ứng tạo thành

các phức trên.

Phát hành PDF bởi Ths Nguyễn Thanh Tú

Đăng ký Word doc qua Zalo 0905779594 Email thanhtuqn88@gmail.com

t 2g

5

105


Hướng dẫn:

1.a. Ru 2+ : [Xe] 4d 6 . Phối tử NH 3 là phối tử trường trung bình, lai hóa trong, không đủ năng

lượng để ghép cặp các electron ở 3d của Ru 2+ . Phức nghịch từ, spin = 0.

b.

c. Điều này đã dẫn tới một câu hỏi rằng tại sao trong phức [Ru(NH 3 ) 6 ] 2+ , NH 3 lại là phối tử

trường trung bình còn ở phức [Ru(NH 3 ) 6 ] 3+ , NH 3 lại là phối tử trường mạnh. Ta có thể lí giải

dễ dàng do khi tăng điện tích hạt nhân lên, phối tử càng gần nguyên tử trung tâm hơn dẫn tới

tăng lực tương tác. Như vậy NH 3 lại trở thành phối tử trường mạnh.

2. Thứ nhất, K phản ứng quyết định rất lớn bởi ΔG.ΔG = ΔH – T.ΔS

Ở cả ba phản ứng trên đều là sự tạo liên kết giữa phối tử và ion Ni 2+ . Do vậy ΔH ở đây

có thể coi là chênh nhau không nhiều. Không phải là yếu tố quyết định độ lớn của K.

Tuy nhiên, chúng ta có thể thấy, ở phản ứng (1) từ 7 chất ban đầu ra một sản phẩm, ở (2)

là từ 4 chất ra 1 còn ở (3) là từ 2 ra 1 sản phẩm.

Như vậy ta có thể kết luận rằng ΔS (1) < ΔS (2) < ΔS (3) (chênh lệch nhiều)

Do đó: K (1) < K (2) < K (3).

VI.3. Bài tập phản ứng trong phức chất:

VI.3.1. Phản ứng oxi hoá cộng

VI.3.1.1. Phản ứng oxi hoá cộng sử dụng rộng rãi trong việc tổng hợp phức Pt (IV) từ Pt (II)

(phức vuông phẳng), sản phẩm thu được là phức bát diện mà tác nhân cộng ở vị trí trans.

Hoàn thành các phản ứng sau:

Phát hành PDF bởi Ths Nguyễn Thanh Tú

Đăng ký Word doc qua Zalo 0905779594 Email thanhtuqn88@gmail.com

106


Hướng dẫn:

O 2 N

Pt

Br

+ Cl 2

O 2 N

Cl

Pt

Br

O 2 N

Br

O 2 N

Cl

Br

H 3 N

Pt

Br

+ H 2 O 2

H 3 N

OH

Pt

Br

H 3 N

Br

H 3 N

OH

Br

VI.3.1.2. Hòa tan muối của một kim loại vào nước thu được dung dịch A chứa phức của ion

kim loại có momen từ bằng 5,2 M.B. Thêm lượng dư CN - vào dung dịch A thu được phức

chất nghịch từ. Oxi hóa ion kim loại trong A để số oxi hóa của kim loại tăng thêm 1 đơn vị

thu được dung dịch B chứa một phức có momen từ bằng 5,9 M.B. Thêm lượng dư CN - vào

B thu được phức chất có momen từ bằng 1,8 M.B. Biết kim loại thuộc dãy đầu tiên của

nhóm các kim loại chuyển tiếp. Hãy xác định kim loại và giải thích các giá trị momen từ đo

được.

Hướng dẫn:

Phức [M(H 2 O) 6 ] n+ có 4 electron độc thân cấu hình electron của M n+ là 3d 4 hoặc 3d 6

Phức [M(CN) 6 ] 6-n có 0 electron độc thân cấu hình electron của M n+ là 3d 6 => M n+ có thể

là Fe 2+ , Co 3+ .

Do ion Co 3+ không thể bị oxi hóa lên ion Co 4+ nên M n+ chỉ có thể là Fe 2+ .

[Fe(H 2 O) 6 ] 2+ t 4 2

2g e g 4 electron độc thân

4(2

4) 4,9 M . B 5,2M . B

107

Phát hành PDF bởi Ths Nguyễn Thanh Tú

Đăng ký Word doc qua Zalo 0905779594 Email thanhtuqn88@gmail.com


[Fe(CN) 6 ] 4- t 2g

6

[Fe(H 2 O) 6 ] 3+ t 2g 3 e g

2

[Fe(CN) 6 ] 3- t 2g

5

0 electron độc thân 0M . B

5 electron độc thân 5(2

5) 5,9 M. B

1 electron độc thân

1(2

1)

1,7

M . B 1,8

M.

B

VI.3.2. Phản ứng khử phức chất

VI.3.2.1. Phản ứng khử tách: thường dùng để điều chế các phức chất mà kim loại ở trạng

thái oxi hoá thấp không đặc trưng, khi đó số phối trí thường giảm. Hoàn thành các phản ứng

sau:

K 2 [PtCl 6 ] + K 2 C 2 O 4

Hướng dẫn:

Na 2 [PtCl 6 ] + C 2 H 5 OH ?

KCl

?

K 2 [PtCl 6 ] + K 2 C 2 O 4 K 2 [PtCl 4 ] + 2CO 2

Na/Hg

Mn 2 (CO) 10 2Na[Mn(CO) 5 ]

KCl

K[Pt(C 2 H 4 )Cl 3 ]

Na 2 [PtCl 6 ] + C 2 H 5 OH H[Pt(C 2 H 4 )Cl 3 ]

VI.3.2.2. Phản ứng khử không giảm số phối trí. (số phối trí không giảm mà có thể tăng).

Các chất khử thường dùng là kim loại, hỗn hống. Hoàn thành các phản ứng sau:

[Pt(Ph 2 PCH 2 CH 2 PPh 2 ) 2 ]Cl 2 + 2Na[BH 4 ]

Hướng dẫn:

[MCl 2 Cp 2 ] 2 + 4CO

Na/Hg

THF 2[M(CO) 2 Cp 2 ]

(M là Zr hoặc Hf)

[Pt(Ph 2 PCH 2 CH 2 PPh 2 ) 2 ]Cl 2 + 2Na[BH 4 ] [Pt(Ph 2 PCH 2 CH 2 PPh 2 ) 2 ] +2NaCl +B 2 H 6 + H 2

VI.3.3. Dạng bài tập thay thế phối tử

VI.3.3.1. Dự đoán sản phẩm của các phản ứng sau, giải thích?

Phát hành PDF bởi Ths Nguyễn Thanh Tú

Đăng ký Word doc qua Zalo 0905779594 Email thanhtuqn88@gmail.com

108


a. [PtCl 4 ] 2- +

-

NO2

A

A +

-

NO2

B

b. [PtCl 4 ] 2- + NH 3 C

C +

-

Br D

Hướng dẫn:

a. [PtCl 4 ] 2- +

-

NO2

A, A + NH 3 B

A = [PtCl 3 (NO 2 )] 2- , B = trans-[PtCl 2 (NO 2 ) 2 ] 2- ; Do

b. [PtCl 4 ] 2- + NH 3 C, C + Br - D

-

NO

2

có hiệu ứng trans mạnh hơn Cl -

C = [PtCl 3 (NH 3 )] - , D = cis-[PtCl 2 Br(NH 3 )] + ; Cl - có hiệu ứng trans mạnh hơn NH 3

VI.3.3.2. Tại sao cộng Cl - vào [Pt(NH 3 ) 4 ] 2+ là phương pháp thích hợp để điều chế trans-

[Pt(NH 3 ) 2 Cl 2 ]?

Cis-[Pt(NH 3 ) 2 Cl 2 ] là thuốc điều trị ung thư, hãy đưa ra cách điều chế cis-[Pt(NH 3 ) 2 Cl 2 ] từ

2-

phức PtI 4

Hướng dẫn:

[Pt(NH 3 ) 4 ] 2+ nếu phản ứng với Cl - sẽ thu được [PtCl(NH 3 ) 3 ] + , tiếp theo do hiệu ứng trans

mạnh của Cl - so với NH 3 nên sản phẩm thu được là trans.

- Để thu được đồng phân cis thì làm ngược lại tức là cho [PtCl 4 ] 2- phản ứng với NH 3 .

- Hoặc đi từ PtI 4 2- tác dụng với NH 3 để thu được đồng phân cis, sau đó cho phản ứng với

AgNO 3 để tạo ra [Pt(NH 3 ) 2 (H 2 O) 2 ] 2+ , rồi cho phản ứng với dung dịch HCl, hiệu suất của

phương pháp này cao hơn phương pháp trên do I - có hiệu ứng trans mạnh hơn Cl - .

VI.3.3.3. Phức chất [PtCl 2 (NH 3 ) 2 ] được xác định là đồng phân trans. Nó phản ứng chậm với

Ag 2 O cho phức chất [PtCl 2 (NH 3 ) 2 (OH 2 ) 2 ] 2+ (kí hiệu là X). Phức chất X không phản ứng

được với etylenđiamin (en) khi tỉ lệ mol phức chất X:en = 1:1. Hãy giải thích các sự kiện

trên và vẽ cấu trúc của phức chất X.

Hướng dẫn:

[PtCl 2 (NH 3 ) 2 ] + Ag 2 O + 3H 2 O [PtCl 2 (NH 3 ) 2 (H 2 O) 2 ] 2+ + 2OH - + 2Ag

Etylenđiamin là phối tử hai càng mạch ngắn. Khi phối trí với các ion kim loại nó chỉ có thể

chiếm hai vị trí phối trí cạnh nhau (vị trí cis). Hiện tượng en không thể phản ứng với X

Phát hành PDF bởi Ths Nguyễn Thanh Tú

Đăng ký Word doc qua Zalo 0905779594 Email thanhtuqn88@gmail.com

109


chứng tỏ rằng hai phân tử H 2 O nằm ở hai vị trí trans đối với nhau. Như vậy cấu trúc của

phức X là:

OH 2

2+

Cl NH 3

Pt

H 3 N

Cl

OH 2

Phần 3. KẾT LUẬN

Trên cơ sở tổng hợp những tài liệu khác nhau, chuyên đề đã hệ thống hóa được kiến thức

về các kim loại nhóm VIIB, VIIIB, đồng thời nêu ra một số câu hỏi, bài tập vận dụng kiến

thức trong chuyên đề để học sinh tham khảo. Qua đó giúp cho các em có thể hệ thống lại

kiến thức của mình về phần này, giúp các em phát huy được tính chủ động, tích cực.

Mặc dù đã có nhiều cố gắng, nhưng không thể tránh khỏi những hạn chế, thiếu sót. Tôi rất

mong nhận được sự đóng góp ý kiến của các đồng nghiệp để chuyên đề hoàn thiện hơn.

Tôi xin chân thành cảm ơn!

Phát hành PDF bởi Ths Nguyễn Thanh Tú

Đăng ký Word doc qua Zalo 0905779594 Email thanhtuqn88@gmail.com

110

Hooray! Your file is uploaded and ready to be published.

Saved successfully!

Ooh no, something went wrong!